You are on page 1of 307

B

arbosu D. (coordonator),
Berinde V., Coroian I., Horvat-Marc A.,
Kozma L., Kovacs G., Pis coran L., Pop S.M.,
Pop A., Sass I.H.A., Tas cu I.
Matematica
de
baza
Universitatea de Nord
Baia Mare
Cuprins
I Algebra 7
1 Grupuri 8
1.1 Grupuri. Denit ie. Proprietat i. Exemple. . . . . . . . . . . . . . . . . . . . . . . . . . . . 8
1.2 Subgrupuri ale unui grup . . . . . . . . . . . . . . . . . . . . . . . . . . . . . . . . . . . . 11
1.3 Ordinul unui grup si al unui element al grupului. Grupuri ciclice. . . . . . . . . . . . . . . 14
1.4 Relat ii de echivalent a induse de un subgrup pe elementele unui grup . . . . . . . . . . . . 15
1.5 Subgrupuri normale . . . . . . . . . . . . . . . . . . . . . . . . . . . . . . . . . . . . . . . 17
1.6 Omomorsme de grupuri . . . . . . . . . . . . . . . . . . . . . . . . . . . . . . . . . . . . . 18
2 Inele 24
2.1 Inele. Subinele. Denit ii. Exemple. . . . . . . . . . . . . . . . . . . . . . . . . . . . . . . . 24
2.2 Ideale . . . . . . . . . . . . . . . . . . . . . . . . . . . . . . . . . . . . . . . . . . . . . . . 27
2.3 Omomorsme de inele . . . . . . . . . . . . . . . . . . . . . . . . . . . . . . . . . . . . . . 30
2.4 Corpuri. Subcorpuri. Morsme de corpuri. . . . . . . . . . . . . . . . . . . . . . . . . . . . 32
3 Probleme de algebra 36
3.1 Enunt uri . . . . . . . . . . . . . . . . . . . . . . . . . . . . . . . . . . . . . . . . . . . . . . 36
Grupuri . . . . . . . . . . . . . . . . . . . . . . . . . . . . . . . . . . . . . . . . . . . . . . . . . 36
Inele . . . . . . . . . . . . . . . . . . . . . . . . . . . . . . . . . . . . . . . . . . . . . . . . . . . 39
3.2 Solut ii . . . . . . . . . . . . . . . . . . . . . . . . . . . . . . . . . . . . . . . . . . . . . . . 41
Grupuri . . . . . . . . . . . . . . . . . . . . . . . . . . . . . . . . . . . . . . . . . . . . . . . . . 41
Inele . . . . . . . . . . . . . . . . . . . . . . . . . . . . . . . . . . . . . . . . . . . . . . . . . . . 57
3.3 Probleme propuse . . . . . . . . . . . . . . . . . . . . . . . . . . . . . . . . . . . . . . . . . 70
II Geometrie 75
1 Spat iul vectorial al vectorilor liberi 76
1.1 Segmente orientate si vectorii liberi. . . . . . . . . . . . . . . . . . . . . . . . . . . . . . . 76
1.2 Repere carteziene . . . . . . . . . . . . . . . . . . . . . . . . . . . . . . . . . . . . . . . . . 80
1.3 Produse de vectori . . . . . . . . . . . . . . . . . . . . . . . . . . . . . . . . . . . . . . . . 83
2 Elemente de geometrie analitica liniara 90
2.1 Reprezentarile analitice ale dreptei n plan si n spat iu. Ecuat ia planului. . . . . . . . . . 90
2.2 Pozit iile relative ale punctelor, dreptelor si planelor din spat iu . . . . . . . . . . . . . . . . 94
2.3 Distant a de la un punct la o dreapta si de la un punct la un plan. Distant a dintre doua
plane din spat iu. . . . . . . . . . . . . . . . . . . . . . . . . . . . . . . . . . . . . . . . . . 97
3 Conice 100
3.1 Denit ia conicelor. Conice studiate pe ecuat iile lor reduse . . . . . . . . . . . . . . . . . . 100
3.2 Reducerea conicelor date prin ecuat ia generala la forma canonica . . . . . . . . . . . . . . 103
3
4 CUPRINS
4 Probleme de geometrie 109
4.1 Enunt uri . . . . . . . . . . . . . . . . . . . . . . . . . . . . . . . . . . . . . . . . . . . . . . 109
Vectori liberi . . . . . . . . . . . . . . . . . . . . . . . . . . . . . . . . . . . . . . . . . . . . . . 109
Dreapta si planul n spat iu . . . . . . . . . . . . . . . . . . . . . . . . . . . . . . . . . . . . . . 109
Conice . . . . . . . . . . . . . . . . . . . . . . . . . . . . . . . . . . . . . . . . . . . . . . . . . . 111
4.2 Solut ii . . . . . . . . . . . . . . . . . . . . . . . . . . . . . . . . . . . . . . . . . . . . . . . 111
Vectori liberi . . . . . . . . . . . . . . . . . . . . . . . . . . . . . . . . . . . . . . . . . . . . . . 111
Dreapta si Planul n spat iu . . . . . . . . . . . . . . . . . . . . . . . . . . . . . . . . . . . . . . 113
Conice . . . . . . . . . . . . . . . . . . . . . . . . . . . . . . . . . . . . . . . . . . . . . . . . . . 117
4.3 Probleme propuse . . . . . . . . . . . . . . . . . . . . . . . . . . . . . . . . . . . . . . . . . 120
III Analiza matematica 124
1 Siruri de numere reale 125
1.1 Siruri de numere reale . . . . . . . . . . . . . . . . . . . . . . . . . . . . . . . . . . . . . . 125
1.2 Criterii de convergent a . . . . . . . . . . . . . . . . . . . . . . . . . . . . . . . . . . . . . . 129
2 Serii de numere 134
2.1 Serii de numere . . . . . . . . . . . . . . . . . . . . . . . . . . . . . . . . . . . . . . . . . . 134
2.2 Serii alternante. . . . . . . . . . . . . . . . . . . . . . . . . . . . . . . . . . . . . . . . . . . 136
2.3 Serii cu termeni pozitivi. Criterii de comparat ie . . . . . . . . . . . . . . . . . . . . . . . 137
3 Funct ii f : D R R continue 141
3.1 Elemente de topologie . . . . . . . . . . . . . . . . . . . . . . . . . . . . . . . . . . . . . . 141
3.2 Limita unei funct ii f : D R R ntr-un punct . . . . . . . . . . . . . . . . . . . . . . . 142
3.3 Funct ii f : D R R continue ntr-un punct . . . . . . . . . . . . . . . . . . . . . . . . . 147
3.4 Funct ii f : D R R continue pe o mult ime . . . . . . . . . . . . . . . . . . . . . . . . . 150
4 Funct ii f : D R R derivabile 157
4.1 Derivata funct iei ntr-un punct. Derivabilitatea funct iei ntr-un punct . . . . . . . . . . . 157
4.2 Funct ii derivabile pe o mult ime . . . . . . . . . . . . . . . . . . . . . . . . . . . . . . . . . 166
4.3 Derivate de ordin superior . . . . . . . . . . . . . . . . . . . . . . . . . . . . . . . . . . . . 174
4.4 Funct ii convexe . . . . . . . . . . . . . . . . . . . . . . . . . . . . . . . . . . . . . . . . . . 176
4.5 Formula lui Taylor . . . . . . . . . . . . . . . . . . . . . . . . . . . . . . . . . . . . . . . . 182
4.6 Caracterizarea punctelor de optim ale unei funct ii cu ajutorul derivatelor . . . . . . . . . 185
5 Funct ii integrabile Riemann si primitive 187
5.1 Funct ii integrabile. Integrala Riemann. . . . . . . . . . . . . . . . . . . . . . . . . . . . . . 187
5.2 Criteriul de integrabilitate al lui Darboux. Clase de funct ii integrabile . . . . . . . . . . . 192
5.3 Operat ii cu funct ii integrabile Riemann. Proprietat ile funct iilor integrabile si ale integralei
Riemann . . . . . . . . . . . . . . . . . . . . . . . . . . . . . . . . . . . . . . . . . . . . . . 204
5.4 Primitive. Integrala nedenita. Primitivabilitatea funct iilor continue . . . . . . . . . . . . 210
5.5 Calculul unor primitive si metode de integrare . . . . . . . . . . . . . . . . . . . . . . . . . 216
5.6 Integrarea funct iilor rat ionale si a altor clase de funct ii . . . . . . . . . . . . . . . . . . . . 229
5.7 Aplicat ii geometrice ale integralei Riemann . . . . . . . . . . . . . . . . . . . . . . . . . . 243
5.7.1 Aria unei mult imi plane . . . . . . . . . . . . . . . . . . . . . . . . . . . . . . . . . 243
5.7.2 Lungimea unui arc de curba . . . . . . . . . . . . . . . . . . . . . . . . . . . . . . . 246
5.7.3 Volumul uni corp de rotat ie . . . . . . . . . . . . . . . . . . . . . . . . . . . . . . . 249
5.7.4 Aria suprafet elor de rotat ie . . . . . . . . . . . . . . . . . . . . . . . . . . . . . . . 252
CUPRINS 5
6 Probleme de analiza matematica 255
6.1 Enunt uri . . . . . . . . . . . . . . . . . . . . . . . . . . . . . . . . . . . . . . . . . . . . . . 255
Siruri de numere reale . . . . . . . . . . . . . . . . . . . . . . . . . . . . . . . . . . . . . . . . . 255
Serii de numere reale . . . . . . . . . . . . . . . . . . . . . . . . . . . . . . . . . . . . . . . . . . 256
Funct ii continue . . . . . . . . . . . . . . . . . . . . . . . . . . . . . . . . . . . . . . . . . . . . 257
Funct ii derivabile . . . . . . . . . . . . . . . . . . . . . . . . . . . . . . . . . . . . . . . . . . . . 258
Funct ii integrabile Riemann si primitive . . . . . . . . . . . . . . . . . . . . . . . . . . . . . . . 260
6.2 Solut ii . . . . . . . . . . . . . . . . . . . . . . . . . . . . . . . . . . . . . . . . . . . . . . . 263
Siruri de numere reale . . . . . . . . . . . . . . . . . . . . . . . . . . . . . . . . . . . . . . . . . 263
Serii de numere reale . . . . . . . . . . . . . . . . . . . . . . . . . . . . . . . . . . . . . . . . . . 267
Funct ii continue . . . . . . . . . . . . . . . . . . . . . . . . . . . . . . . . . . . . . . . . . . . . 271
Funct ii derivabile . . . . . . . . . . . . . . . . . . . . . . . . . . . . . . . . . . . . . . . . . . . . 280
Funct ii integrabile Riemann si primitive . . . . . . . . . . . . . . . . . . . . . . . . . . . . . . . 289
6.3 Probleme propuse . . . . . . . . . . . . . . . . . . . . . . . . . . . . . . . . . . . . . . . . . 313
Prefat a
Aceast a lucrare cuprinde n principal temele din programa analitic a necesare preg atirii exa-
menului de licent a la disciplina matematic a pentru absolvent ii specializ arilor Matematic a
Fizic a si Matematic a Informatic a din cadrul Universit at ii de Nord Baia Mare, Facultatea de
S tiint e. Cartea poate util a pentru preg atirea examenelor pentru denitivarea n nv at am ant si
obt inerea gradului didactic II.
Dorim ca aceast a lucrare s a vin a n sprijinul viitorilor absolvent i, drept urmare ecare din cele
trei p arti ale lucr arii cont ine capitole cu rezumate teoretice nsot ite de exemple concludente. La
nele ec arei p art i exist a c ate un capitol cu probleme rezolvate integral si probleme propuse
spre rezolvare.
Partea I, Algebr a, a fost conceput a de c atre conferent iar univ. dr. Maria S. Pop mpreun a cu
asistent univ. drd. Adina Pop.
La partea a II-a, Geometrie, au contribuit conferent iar univ. dr. I.H.A. Sass, asistent univ. dr.
L. Piscoran si asistent drd. A. Horvat-Marc.
Colectivul care a realizat partea a III-a, Analiz a Matematic a, este format din conferent iar univ.
dr. Lidia Kozma pentru capitolele Siruri de numere reale si Serii de numere; lector univ. drd.
Gabriella Kovacs pentru capitolele Funct ii f : D R R continue si Funct ii f : D R R
derivabile; profesor univ. dr. I. Coroian pentru capitolul Funct ii integrabile Riemann si primitive.
Problemele date spre rezolvare, din cadrul acestei p art i, au fost selectate si propuse de c atre
conferent iar univ. dr. D. B arbosu, profesor univ. dr. V. Berinde, profesor univ. dr. I. Coroian,
lector univ. dr. Ioana Tascu si asistent univ. A. Horvat-Marc.
Conf. univ. D. B arbosu
Partea I
Algebra
7
Capitolul 1
Grupuri
1.1 Grupuri. Denit ie. Proprietat i. Exemple.
1.1.1 Denit ie Fie A o mult ime, n N si A
n
=
_
(a
1
, . . . , a
n
) ; a
i
A; i = 1, n
_
.
1
0
. Aplicat ia : A
n
A care asociaza ecarui n-uplu de elemente din A un element din A, adica
(a
1
, . . . , a
n
) (a
1
, . . . , a
n
) se numeste operat ie n-ara pe A sau lege de compozit ie interna;
2
0
. Daca este o operat ie n-ara pe A, submult imea B Acu proprietatea (B) B se numeste
parte stabila a lui A relativ la , iar restrict ia aplica t iei la B
n
, deci [
B
n se numeste operat ie
indusa de n B;
3
0
. Perechea (A, ) unde este o mult ime de operat ii pe A se numeste structura algebrica cu
suportul A sau algebra universala;
1.1.2 Observat ii. 1) Exista si operat ii part iale care nu sunt denite pentru orice sistem ordonat
de n-elemente din A. Ele se numesc part iale. Asa sunt, spre exemplu, scaderea n mult imea numerelor
naturale N sau mpart irea n mult imea Z a ntregilor.
2) Pentru n = 0 avem A
0
= si operat ia : A se numeste nulara; ea xeaza un anumit
element din A.
Pentru n = 1, operat ia : A A se numeste unara; ea asociaza ecarui element din A un element
si numai unul din A.
Pentru n = 2 operat ia : A
2
A se numeste binara. Vom nota (a, b)
not
a b sau aditiv, a + b,
respectiv multiplicativ, a b ori simplu ab.
1.1.3 Exemple. 1)

In N existent a elementului unitate 1 cu proprietatea x N; x 1 = 1 x = x
deneste o operat ie nulara.
2)

In Z aplicat ia : Z Z, (a) = a, care asociaza ecarui element opusul sau este o operat ie
unara.

In continuare studiem structuri algebrice nzestrate cu o operat ie interna binara pe care o vom nota
prin si vom deni cateva proprietat i posibile ale unei astfel de operat ii:
1.1.4 Denit ii. O operat ie binara : A
2
A se numeste:
1
0
. asociativa (as) daca x, y, z A; (x y) z = x (y z);
2
0
. comutativa (com) daca x, y A; x y = y x;
3
0
. cu element neutru (ne) daca e A; x A; x e = e x = x; spunem n acest caz ca avem
o operat ie care are element neutru.
4
0
cu element simetrizabil (si) daca are element neutru si pentru x A; x
t
A; x x
t
=
x
t
x = e; Elementul x
t
se numeste simetricul lui x.

In particular, pentru notat ia multiplicativa simetricul unui element x se numeste inversul lui x si
se noteaza x
1
, iar n cazul notat iei aditive, el se numeste opusul lui x si se noteaza prin x.
5
0
. Elementul a A se numeste regular (re) daca a x = a y si x a = y a implica x = y.

In cazul notat iei aditive, daca din a + x = a + y rezulta x = y respectiv din x + a = y + a rezulta
x = y, spunem ca se poate reduce cu a la stanga respectiv la dreapta.

In cazul notat iei multiplicative,
daca pentru x, y A din ax = ay rezulta x = y respectiv din xa = ya rezulta x = y, spunem ca se poate
simplica cu a la stanga, respectiv la dreapta.
8
1.1 Grupuri. Denit ie. Propriet at i. Exemple. 9
6
0
. O proprietate se numeste ereditara daca din faptul ca B A si proprietatea are loc n A,
rezulta ca ea are loc si n mult imea B.
1.1.5 Exemple. 1
0
. Adunarea, nmult irea numerelor reale este asociativa, dar diferent a sau ridi-
carea la putere nu sunt asociative;
2
0
.

Inmult irea numerelor reale sau complexe este comutativa, dar nmult irea matricilor, ridicarea la
putere sau compunerea funct iilor nu sunt operat ii comutative;
3
0
.

In mult imea matricilor patrate de ordinul n exista element neutru relativ la nmult ire si anume
matricea unitate I
n
= (
ij
)
i,j=1,n
, unde

ij
=
_
1 pentru i = j
0 pentru i ,= j
este simbolul lui Kronecker.
4
0
. Ridicarea la putere n mult imea numerelor reale strict pozitive nu are element neutru deoarece
a
1
= a ,= 1
a
pentru a ,= 1;
5
0
.

In mult imea funct iilor bijective f : R R cu element neutru aplicat ia identica, simetricul lui f
este inversa f
1
:R R.
6
0
. Orice numar natural nenul este regular n raport cunmult irea, dar zero nu este regular: 20 = 30
desi 2 ,= 3;
7
0
. Asociativitatea, comutativitatea, regularitatea sunt proprietat i ereditare, dar existent a elemen-
tului neutru sau simetrizabil nu sunt ereditare. Spre exemplu, n Z ecare element are opus, dar n
mult imea N

Z nici un element nu are simetric.


1.1.6 Denit ii. 1
0
. Numim grupoid perechea (A, ), unde este operat ie binara pe A;
2
0
. Un grupoid (A, ) asociativ se numeste semigrup;
3
0
. Un semigrup cu element neutru se numeste monoid;
4
0
. Un monoid n care ecare element este simetrizabil se numeste grup;
5
0
. Un grupoid (semigrup, monoid, grup) n care operat ia binara este comutativa se numeste co-
mutativ.
Dam n continuare cateva proprietat i generale ale structurilor algebrice:
1.1.7 Propozit ii. 1
0
. Daca ntr-un grupoid (A, ) exista element neutru, atunci acesta este unic;
2
0
. Orice element simetrizabil ntr-un monoid are un unic simetric;
3
0
.

Intr-un monoid orice element simetrizabil este regular;
Reciproca n general nu este adevarata: n monoidul (N

, ), toate numerele naturale nenule sunt


regulare dar nu sunt inversabile (exceptandu-l pe 1).

In caz nit are loc si implicat ia inversa armat iei
3
0
, adica:
4
0
.

Intr-un monoid nit orice element regular este simetrizabil;
5
0
.

Intr-un monoid, daca a este simetrizabil atunci si simetricul lui a, este simetrizabil si avem
(a
t
)
t
= a; Daca a si b sunt simetrizabili, atunci ab este simetrizabil si avem (a b)
t
= b
t
a
t
;
Demonstrat ie. 1
0
. Presupunem ca exista doua elemente neutre e si e
t
. Atunci avem:
e
e

ne
= e e
t
e ne
= e
t
.
2
0
. Daca atat a
t
cat si a
tt
sunt simetricele lui a, atunci:
a
t
ne
= a
t
e = a
t
(a a
tt
) = (a
t
a) a
tt
= e a
tt
= a
tt
.
3
0
Daca a x = a y si a este simetrizabil, atunci nmult ind egalitatea la stanga cu a
t
si folosind
asociativitatea si existent a elementului neutru, avem:
(a
t
a)x = (a
t
a)y e x = e y x = y.
Analog n x a = y a nmult ind la dreapta cu a
t
rezulta x = y, deci a este regular.
4
0
. Fie A nit si anume A = a
1
, ..., a
n
si elementul a
i
A, xat. Aplicat ia f : A a
i
A;
f(x) = a
i
x este o inject ie deoarece din f(x) = f(y) a
i
x = a
i
y, regularitatea lui a
i
conduce la
x = y.

Intrucat mult imea A este nita rezulta ca f este si surject ie, deci exista x A, e acesta elementul
10 1. Grupuri
a
j
, astfel ca f(a
j
) = e, de unde a
i
a
j
= e. Aplicat ia g : A A a
j
este de asemenea inject ie, deci si
surject ie si exista y A astfel ncat g(y) = e. Fie y = a
k
. Asociativitatea operat iei n-are si existent a
elementului neutru conduc, conform armat iei 2
0
, la a
j
= a
k
, deci exista simetricul lui a
i
. Observam ca
n enunt ul propozit iei 1.1.7.4
0
era sucient ca A sa e semigrup nit.
5
0
. Deoarece (ab)(b
t
a
t
)
as
= a (b b
t
) a
t
= a e a
t
= a a
t
= e, exista (ab)
t
= b
t
a
t
. Asociativitatea
permite denirea recursiva a puterilor naturale ntr-un semigrup si ale celor ntregi ntr-un grup.
1.1.8 Denit ii. 1
0
. Daca (A, ) este un semigrup si a A atunci a
1
= a si daca pentru n > 1,
n N, a
n1
este denit, atunci a
n
def
= a
n1
a.
2
0
. Daca (A, ) este un grup si a A, atunci puterile lui a se extind si la numere negative sau
zero. Astfel a
0
= e, elementul neutru al grupului, a
1
= a
t
, simetricul sau, iar pentru n N; n > 1;
a
n
def
= (a
t
)
n
= (a
n
)
t
.
Prin induct ie dupa n N se demonstreaza regulile de calcul cu puteri:
1.1.9 Propozit ie.

Intr-un semigrup (A, ) avem:
a
m
a
n
= a
m+n
; (a
m
)
n
= a
mn
, m, n N

relat ii ce se extind si pentru puteri ntregi negative ntr-un grup. Dupa cum am vazut numim grup
perechea (A, ) unde este o operat ie binara denita pe A, asociativa, cu element neutru, astfel ncat
orice element este simetrizabil. O alta denit ie echivalenta cu aceasta este data de urmatoarea teorema:
1.1.10 Teorema. Semigrupul (A, ) este grup daca si numai daca ecuat iile a x = b si y a = b au
solut ii unice n A pentru orice a, b A.
Demonstrat ie. Daca (A, ) este grup atunci exista simetricul lui a notat a
t
si nmult ind, la
stanga respectiv la dreapta, cu a
t
cele doua ecuat ii obt inem:
a
t
(a x) = a
t
b
as
(a
t
a) x = a
t
b
Ne
x = a
t
b A
si analog y = b a
t
, solut ii unice n A.
Daca orice ecuat ii de tipul din enunt au solut ii unice n A, atunci ecuat ia a x = a are solut ia
x = e
a
, deci a e
a
= a. Observam ca oricare ar b A avem b e
a
= b.

Intr-adevar
b e
a
ipoteza
= (y a) e
a
As
= y (a e
a
) = y a
ipoteza
= b.
Prin urmare
e A, x A; x e = x. (1.1)
Analog, e e

a
solut ia ecuat iei x e = x. Se demonstreaza ca e

a
b = b pentru orice b A(utilizand
existent a solut iei ecuat iei x a = b). Prin urmare
e
t
A, x A; e
t
x = x. (1.2)
Din (1) si (2) rezulta e = e
t
e = e
t
, deci semigrupul are element neutru. Fie acum a
t
solut ia ecuat iei
a x = e si a
tt
solut ia ecuat iei y a = e. Cele doua solut ii coincid, deoarece a
t
= e a
t
= (a
tt
a) a
t
=
a
tt
(a a
t
) = a
tt
e = a
tt
, deci (A, ) este grup.
1.1.11 Exemple. a) (N, +); (N, ); (, ) (R, ) si (C, ) sunt monoizi comutativi fara a grupuri.
b) (Z, +); (, +); (R, +) (C, +) (

, ) (R

, ) si (C

, ) sunt grupuri comutative.


c) Fie M o mult ime si M
M
= f ; f : M M, mult imea aplicat iilor mult imii M n ea nsasi.
Perechea (M
M
, ) este un monoid n raport cu compunerea funct iilor, avand ca element neutru aplicat ia
identica. Grupul elementelor simetrizabile ale monoidului este U(M
M
, ) = f M
M
; f biject ie.
Deoarece o biject ie f : M M se numeste permutare, grupul U(M
M
, ) este grupul permutarilor lui M
numit si grupul simetric al lui M, notat S
M
. Daca M este mult imea nita cu n elemente, atunci S
M
se
noteaza S
n
si se numeste grupul simetric de ordinul n. Daca n 3, atunci (S
n
, ) este grup necomutativ.
1.2 Subgrupuri ale unui grup 11
1.2 Subgrupuri ale unui grup
1.2.1 Denit ie. Fie (G, ) un grup. Submult imea H G se numeste subgrup al grupului (G, )
daca H mpreuna cu operat ia indusa din G pe elementele lui H este grup. Notam H G, iar prin o (G)
mult imea tuturor subgrupurilor lui G, deci o(G) = H; H G.
1.2.2 Observat ii. 1
0
. G G si e G unde e este elementul neutru al grupului G. Aceste doua
subgrupuri se numesc improprii spre deosebire de toate celelalte care se vor numi proprii;
2
0
. Daca H G, atunci e H, deci H ,= . Prin urmare submult imea suport a unui subgrup este
n mod necesar nevida;
3
0
. Daca H G si G H atunci H = G;
4
0
. Daca H H
t
si H
t
H
tt
atunci H H
tt
;
5
0
. Din 1
0
, 3
0
si 4
0
rezulta ca relat ia este subgrup al lui este o relat ie de ordinen mult imea
o (G).
Urmatoarea propozit ie ne da doua denit ii echivalente cu denit ia 1.2.1 a subgrupului unui grup, cu
care se opereaza mai usor n aplicat ii:
1.2.3 Teorema de caracterizare a subgrupurilor unui grup. Daca (G,) este un grup si H o
submult ime a lui G, atunci sunt echivalente urmatoarele armat ii:
1
0
. H G (H este subgrup al lui G);
2
0
. H ,= , x, y H x y H si x H x
1
H;
3
0
. H ,= , x, y H x y
1
H.
Demonstrat ie. Implicat iile 1
0
2
0
si 3
0
sunt evidente.
2
0
3
0
deoarece y H y
1
H si cum x H avem x y
1
H.
3
0
2
0
ntrucat H ,= implica ca exista x H si deci e = x x
1
H. Pentru x = e si y H
rezulta ca e y
1
H, deci y
1
H. Daca x, y H, atunci si y
1
H si avem x y = x (y
1
)
1
H.
2
0
1
0
deoarece H este parte stabila a lui G, iar asociativitatea operat iei din G este o proprietate
ereditara si cum e = x x
1
pentru x H, deducem ca e H. Existent a lui x
1
si apartenent a sa la H
pentru orice x H este asigurata din enunt ul armat iei 2
0
.
1.2.4 Observat ie.

In cazul unui grup notat aditiv (G, +), submult imea nevida H G este subgrup
al lui G daca si numai daca:
1
0
. x, y H x +y H si -x H;
respectiv
2
0
. x, y H x + (y) H.
1.2.5 Exemple. 1
0
Subgrupurile grupului aditiv al ntregilor Z sunt de forma nZ, unde n N. Fie
(Z, +) grupul aditiv al numerelor ntregi si n N xat. Notam prin nZ mult imea tuturor multiplilor lui n,
deci nZ = kn; k Z. Observam ca nZ ,= si k
1
n, k
2
n nZ avem k
1
n + (k
2
n) = (k
1
k
2
)n nZ,
deoarece k
1
k
2
Z. Aceasta demonstreaza ca nZ este subgrup al lui (Z, +).

In particular, pentru
n = 0 obt inem subgrupul impropriu format din elementul neutru (cel nul), deci 0Z =0, iar pentru
n = 1 obt inem grupul Z. Se poate demonstra ca (Z, +) nu are alt fel de subgrupuri, adica daca H Z
atunci exista n N astfel ncat nZ = H.

Intr-adevar, daca H Z si H = 0 atunci H = 0Z. Daca
H ,= 0 atunci H 0 , = si ntrucat mult imea numerelor naturale este bineordonata, n submult imea
nevida (H0 N) exista un cel mai mic element, e acesta n. Deoarece n ,= 0, atunci pentru orice
x H, aplicand teorema mpart irii cu rest, exista un cat q si un rest r, unici, astfel ncat x = qn + r
cu 0 r < n. Dar x, n H implica qn H si r = x + (qn) H. Deoarece r este mai mic decat n si
n este minim cu proprietatea ca este natural nenul din H, rezulta ca nu e posibil decat r = 0. De aici
x = qn, adica H nZ. Incluziunea invers a este imediata, deci H = nZ.
2
0
Mult imea elementelor unui grup, care comuta cu orice element al grupului formeaza un subgrup
al acelui grup, numit centrul grupului, notat Z(G).

Intr-adevar, prin denit ie
Z(G) = z G; zx = xz; x G.
Deoarece ex = xe = x, x G, avem e Z(G), deci Z(G) ,= . Mai mult, oricare ar z
1
, z
2
Z(G)
avem z
1
z
1
2
Z(G) deoarece
_
z
1
z
1
2
_
x = z
1
_
z
1
2
x
_
= z
1
_
x
1
z
2
_
1
=
12 1. Grupuri
= z
1
_
z
2
x
1
_
1
= z
1
_
(x
1
)
1
z
1
2
_
= z
1
_
xz
1
2
_
=
= (z
1
x) z
1
2
= (xz
1
) z
1
2
= x
_
z
1
z
1
2
_
.
1.2.6 Propozit ie. Intersect ia a doua subgrupuri ale unui grup este subgrup al acelui grup. Mai
general, intersect ia unei familii (S
i
)
iI
de subgrupuri ale unui grup este un subgrup a acelui grup, adica:
S
i
G; i I

iI
S
i
G.
Demonstrat ie. Fie S
1
, S
2
subgrupuri ale lui G. Cum elementul neutru a lui G apart ine atat lui S
1
cat
si lui S
2
rezulta ca e S
1
S
2
, deci S
1
S
2
,= . Pentru orice x, y S
1
S
2
avem x, y S
1
si x, y S
2
,
de unde conform propozit iei 1.2.3.3
0
rezulta ca x y
1
S
1
si x y
1
S
2
, deci x y
1
S
1
S
2
.
Aceasta demonstreaza ca S
1
S
2
G.
1.2.7 Exemplu. Subgrupurile 12Z si 18Z ale grupului aditiv al numerelor ntregi au intersect ia
12Z18Z =36Z care este subgrup al lui (Z, +), dar reuniunea lor, 12Z18Z, nu este subgrup a lui Z
deoarece, spre exemplu, 12, 18 12Z18Z dar 12 + 18 / 12Z18Z. Se poate arata ca mZnZ = [m, n]Z
unde prin [m, n] s-a notat cel mai mic multiplu comun al numerelor naturale m si n.

In demonstrat ie
observam mai ntai ca mZ nZ daca si numai daca n [ m.
1.2.8 Denit ii. Fie (G, ) un grup si X G o submult ime a lui G. Prin subgrupul generat de X
nt elegem cel mai mic subgrup al lui G care-l cont ine pe X, adica tocmai intersect ia tuturor subgrupurilor
lui G care-l cont in pe X. Notam subgrupul generat de X prin < X >. Avem prin urmare:
< X >=

H; H G; H X .
Daca mult imea X cont ine un singur element a, deci X = a atunci subgrupul generat de a se noteaza
< a > si se numeste subgrupul ciclic generat de a. Daca grupul G =< X > atunci spunem ca X
este un sistem de generatori pentru G.
1.2.9 Observat ie. Daca X = atunci < >= e
G
, unde e
G
este elementul neutru al grupului.
1.2.10 Exemplu.

In grupul aditiv al numerelor ntregi avem < 12Z18Z >= 6Z.

In general,
< mZnZ >=(m, n)Z, unde (m, n) este cel mai mare divizor comun al numerelor naturale m si n.
Vericarea acestui rezultat este un simplu exercit iu.
1.2.11 Propozit ie. Mult imea subgrupurilor unui grup formeaza o latice (de fapt o latice completa)
n raport cu incluziunea.
Demonstrat ie. Mult imea (o(G), ) este o mult ime ordonata. Fie S
1
, S
2
G subgrupuri ale lui G.
Deoarece avem implicat iile
S
1
S
2
S
1
S
1
S
2
S
2
_
H
_
H S
1
H S
2
[H S
1
S
2
_
,
rezulta ca S
1
S
2
= inf
S(G)
(S
1
, S
2
). Totodata conform denit iei subgrupului generat de S
1
S
2
avem:
S
1
< S
1
S
2
>
S
2
< S
1
S
2
>
_
H
_
S
1
H
S
2
H
[< S
1
S
2
> H
_
,
de unde rezulta < S
1
S
2
>= sup
S(G)
(S
1
, S
2
). Remarcam ca aceasta latice nu este o sublatice a part ilor
grupului (T(G), ), deoarece desi:
inf
1(G)
(S
1
S
2
) = S
1
S
2
= inf
S(G)
(S
1
S
2
),
totusi
sup
1(G)
(S
1
, S
2
) = S
1
S
2
< S
1
S
2
>= sup
S(G)
(S
1
S
2
).
Urmatoarea propozit ie ne da un procedeu de construct ie a subgrupului generat de o submult ime a unui
grup.
1.2.12 Propozit ie. Daca (G, ) este un grup si X G, atunci:
< X >=
_
x
1
x
2
...x
n
; n N; x
i
X sau x
1
i
X; i = 1, n
_
.
1.2 Subgrupuri ale unui grup 13
Demonstrat ie. Notam
S =
_
x
1
x
2
...x
n
; n N; x
i
X sau x
1
i
X; i = 1, n
_
.
Se verica usor ca S G deoarece:
1) pentru orice n, m N, daca x
1
...x
n
si x
n+1
...x
n+m
S atunci, conform legii asociativitat ii
generalizate, avem produsul lor x
1
...x
n
...x
n+m
S (parte stabila);
2) (x
1
...x
n
)
1
= x
1
n
x
1
n1
...x
1
1
S, iar
3) pentru n = 0 conform denit iei operat iei nulare produsul x
1
...x
n
este elementul unitate al grupului.
Subgrupul S cont ine pe X deoarece pentru x X produsul unar (n = 1) apart ine lui S, x
1
= x,
deci x S.
Pe de alta parte, oricare ar H G astfel ncat X H, pentru n N si x
i
X; i = 1, n avem
x
1
x
2
...x
n
H si de asemenea daca x
1
i
X. Deci orice produs n-ar cu factorii x
i
din X sau inversul
x
1
i
X apart ine lui H, ceea ce arata ca S H. Am demonstrat astfel ca S este cel mai mic subgrup
al lui G care-l cont ine pe X, deci S =< X >.
1.2.13 Observat ii. 1
0
. Daca elementele mult imii X comuta doua cate doua ntr-un produs, atunci
factorii de acelasi fel conduc la puteri ntregi, adica
< X >=
_
x
k
1
1
...x
k
n
n

n N; k
1
, ..., k
n
Z; x
i
X ; i = 1, n
_
.
Evident < X > este grup abelian.

In particular, daca X = a
1
, ..., a
m
este o mult ime nita si elementele comuta ntre ele, atunci
subgrup generat de ele este
< a
1
, ..., a
m
>=
_
a
k
1
1
...a
k
m
m
; k
1
, ..., k
m
Z
_
.
2
0
. Daca x = a, atunci grupul abelian generat de a, numit ciclic, este format din puterile ntregi
ale lui a
< a >=
_
a
k
; k Z
_
.
Distingem doua cazuri: grupul ciclic este innit daca toate puterile lui a sunt distincte, respectiv
nit daca p, q Z astfel ncat a
p
= a
q
.
Exemplul 1.2.5.1
0
ne arata ca orice subgrup al grupului aditiv al ntregilor este ciclic si anume
nZ =< n >, unde n N.
3
0
.

In cazul notat iei aditive, daca (G, +) este un grup si X G, atunci:
< X >=
_
x
1
+x
2
+... +x
n
; n N; x
i
X sau x
i
X; i = 1, n
_
.
Daca X = a atunci < a >= ka; k Z .
1.2.14 Exemple. 1
0
. Pentru grupul aditiv al numerelor reale (R, +), subgrupul generat de
X =
_
2,

3
_
este:
<

2,

3 >=
_
k
1

2 +k
2

3; k
1
, k
2
Z
_
.
2
0
. Pentru grupul multiplicativ al numerelor complexe (C

, ), subgrupul generat de
= cos
2
n
+i sin
2
n
este
< >=
_

k
; k Z
_
=
_
1, ,
2
, ...,
n1
_
deoarece
n
= 1. Acesta este subgrupul ciclic al radacinilor de ordinul n al unitat ii, numit si grupul
unitat ilor de ordinul n, notat de obicei prin U
n
.
14 1. Grupuri
1.3 Ordinul unui grup si al unui element al grupului. Grupuri
ciclice.
1.3.1 Denit ii. Fie (G,) un grup. Prin ordinul sau notat ordG nt elegem cardinalul mult imii G,
deci ordG = [ G[.
Fie a G. Prin ordinul elementului a nt elegem cel mai mic numar natural nenul n N

astfel
ncat a
n
= e, unde e este elementul neutru al grupului. Scriem orda = n. Daca nu exista n N

cu
aceasta proprietate, adica a
n
,= e , n N

atunci spunem ca ordinul lui a n grupul G este innit si


scriem ord a = .
Propozit ia de mai jos demonstreaza ca ordinul unui element coincide cu ordinul grupului ciclic generat
de a, deci orda =ord< a >.
1.3.2. Propozit ie. Fie (G, ) un grup multiplicativ, a G si 1 elementul neutru al grupului:
1
0
. daca ordinul lui a este innit atunci a
k
= 1 k = 0;
2
0
. daca orda = n, n N

, atunci < a >= a


k
; k = 0, 1, ..., n 1.
3
0
. daca orda = , atunci i, j Z; i ,= j a
i
,= a
j
si prin urmare, < a > este grup ciclic innit.
Demonstrat ie. 1
0
. Daca a
k
= 1 si k ,= 0 atunci, a
km
= 1 pentru orice m Z

. Fie n = minm
N

, a
km
= 1. Avem atunci: < a >= a
0
= 1, a
1
, ..., a
n1
, deci ord< a > este nit, contradict ie. Daca
k = 0, atunci evident a
0
= 1.
2
0
. Daca ord a = n, e H = a
k
; k = 0, 1, ..., n 1. Evident H < a >= a
k
; k Z. Deoarece
ord< a >= n, elementele lui H sunt doua cate doua distincte.

Intr-adevar, daca exista i, j N

;
0 i < j < n astfel ncat a
i
= a
j
, atunci nmult ind egalitatea cu a
i
avem a
ji
= 1 si cum 0 i < j < n
aceasta contrazice minimalitatea lui n din denit ia ordinului lui a. Deci H are efectiv n elemente.
Pentru orice element a
k
< a > avem a
k
H deoarece aplicand lui k Z teorema mpart irii cu rest
prin n, exista q, r Z astfel ncat k = q n +r cu 0 r < n. De aici avem a
k
= (a
n
)
q
a
r
= 1 a
r
H.
Am demonstrat astfel ca < a > H si cum H < a > rezulta < a >= H =
_
a
0
, a
1
, ..., a
n1
_
.
3
0
. Presupunem ca a
i
= a
j
pentru anumit i i, j Z si i < j. Atunci a
ji
= 1 si cum orda =
rezulta j i = 0, contradict ie.
1.3.3 Observat ie.

In cazul unui grup notat aditiv (G, +) ordinul lui a G se deneste ca cel mai
mic numar natural nenul n astfel ncat na = 0 unde na = a +... +a, nsumarea lui a efectuandu-se de n
ori.
1.3.4 Exemple. 1
0
.

In (C, ) elementul i are ordinul 4 deoarece < i >=
_
i; i
2
= 1; i
3
= 1; i
4
= 1
_
,
iar elementul = cos
2
n
+i sin
2
n
are ordinul n ntrucat < >=
_
1, ,
2
, ...,
n1
_
. Tot n acest grup
elementul 1 are ordinul 1, iar 1 are ordinul doi, < 1 >= 1, 1. Elementul 2 are ordinul innit
deoarece 2
k
= 1 k = 0.
2
0
.

In (R, +) elementul 1 are ordin innit deoarece n1 = 0 n = 0. Evident < 1 >= Z si ordinul
grupului (Z, +) este innit, [Z[ =
0
.
3
0
.

In grupul aditiv al claselor de resturi modulo n, (Z
n
, +), unde Z
n
=
_

0,

1, ...,

n 1
_
avem
Z
n
=
_
k

1; k Z
_
=<

1 >, deci acest grup este ciclic si ord <

1 >= n. Daca n este numar compus
si m divide pe n, atunci exista k N astfel ca n = mk, iar ordinul lui m este k deoarece m k =

0.
Spre exemplu, n Z
12
avem: ord

0 = 1; ord

6 = 2; ord

4 =ord

8 = 3; ord

3 =ord

9 = 4; ord

2 =ord

10 = 6 si
ord

1 =ord

5 =ord

7 =ord

11 = 12. Mai mult, Z


12
=<

1 >=<

5 >=<

7 >=<

11 >.
1.3.5 Propozit ie. Daca (G, ) este un grup, atunci :
1
0
. unicul element de ordin 1 n G este elementul sau neutru e;
2
0
. ()x G, ordx
1
=ordx;
3
0
. daca x G, iar ordx = n si m N, atunci x
m
= 1 n [ m.
Demonstrat ie. Armat ia 1
0
este evidenta.
2
0
. Fie n N. Pentru orice x G avem (x
1
)
n
= e x
n
= e x
n
= e
1
= e. De aici rezulta
ca ordx
1
=ordx, cu precizarea ca daca ordinul lui x este innit atunci si ordinul lui x
1
este innit si
reciproc.
3
0
. Daca x
m
= e si ordx = n, atunci conform teoremei mpart irii cu rest exista si sunt unici q, r N
1.4 Relat ii de echivalent a induse de un subgrup pe elementele unui grup 15
astfel ncat m = nq +r; 0 r < n. De aici
x
r
= x
mnq
= x
m
(x
n
)
q
= e,
de unde, cum r < n si ordx = n rezulta ca r = 0, deci m = nq si ordn [ m. Reciproc, daca n [ m atunci
exista q N astfel ncat m = nq si
x
m
= x
nq
= (x
n
)
q
= e
q
= e.
1.3.6 Propozit ie. Orice subgrup al unui grup ciclic este ciclic.
Demonstrat ie. Fie (G, ) un grup ciclic generat de elementul a, deci G =< a > si H G. Daca
H = e, atunci H =< e > este ciclic. Presupunem n continuare ca H ,= e. Atunci H cont ine
puteri pozitive ale lui a deoarece daca k < 0 si a
k
H, atunci si (a
k
)
1
= a
k
H unde k N.
Alegem n = mink N

, a
k
H si demonstram ca H =< a
n
>.

Intr-adevar, deoarece a
n
H avem
< a
n
> H.
Pentru a demonstra incluziunea inversa, observam ca daca a
m
H pentru un anumit m Z, atunci
aplicand teorema mpart irii cu rest exista si sunt unici q, r Z astfel ncat m = nq + r, 0 r < n. De
aici avem
a
r
= a
mnq
= a
m
(a
n
)
q
H,
deoarece a
m
, a
n
H. Din alegerea lui n si faptul ca 0 r < n rezulta r = 0, de unde avema
m
(a
n
)
q
= e,
deci a
m
= (a
n
)
q
< a
n
>.
1.3.7 Propozit ie. Daca (G, ) este un grup ciclic nit de ordinul n si G =< a >, atunci
G =< a
k
>, k N daca si numai daca (n, k) = 1.
Demonstrat ie. Presupunem ca G =< a >=< a
k
> si (n, k) = d ,= 1. Prin urmare exista numerele
naturale n
t
, k
t
N, (n
t
, k
t
) = 1, astfel ncat n = n
t
d si k = k
t
d. Atunci (a
k
)
n

= (a
k

d
)
n

= (a
dn

)
k

=
(a
n
)
k

= e
k

= e. De aici orda
k
n
t
< n ceea ce conduce la

< a
k
>

strict mai mic decat [< a >[ ,


contradict ie, deci (n, k) = 1.
Reciproc, daca (n, k) = 1 atunci exista u, v Z astfel ca nu + kv = 1, de unde a = a
nu+kv
=
(a
n
)
u
(a
k
)
v
= (a
k
)
v
, ceea ce demonstreaza ca a < a
k
> si deci G =< a >< a
k
>. Incluziunea inversa
ind evidenta, avem < a >=< a
k
>.
1.4 Relat ii de echivalent a induse de un subgrup pe elementele
unui grup
Fie (G, ) un grup si H un subgrup al sau (H G) cu ajutorul caruia denim urmatoarele doua
relat ii pe G:
x
H
y x
1
y H; (1)
x
t
H
y y x
1
H. (2)

In particular, pentru H = e, elementul neutru n G, avem x


e]
y x
1
y = e deci y = x, adica

e]
si la fel
t
e]
sunt tocmai relat ia de egalitate pe G.
Daca H = G, atunci
G
=
t
G
reprezinta relat ia universala pe G.
1.4.1 Propozit ie. Relat iile
H
si
t
H
anterior denite sunt relat ii de echivalent a pe G.
Demonstrat ie. Relat ia
H
este reexiva deoarece x G avem x
1
x = e
G
H deci x
H
x.
Totodata
H
este relat ie simetricantrucat daca x
H
y, deci x
1
y H, atunci, deoarece H este subgrup al
lui G rezulta ca
_
x
1
y
_
1
H. Cum
_
x
1
y
_
1
= y
1
(x
1
)
1
= y
1
x avem y
1
x H, adica y
H
x.
De asemenea din x
H
y si y
H
z x
1
y H si y
1
z H.

Intrucat H este parte stabila a lui G, rezulta
(x
1
y)(y
1
z) H. Conform asociativitat ii operat iei binare de grup si existent ei elementului neutru avem
x
1
(y y
1
) z = x
1
z H. Aceasta demonstreaza ca x
H
z, deci relat ia
H
este tranzitiva. Prin
urmare
H
este o relat ie de echivalent a pe G.
Analog se verica ca
t
H
este de asemenea o relat ie de echivalent a pe G.
16 1. Grupuri
1.4.2 Teorema. Daca (G, ) este grup si H G, atunci relat iile
H
si
t
H
determina doua partit ii
ale lui G n clase de echivalent a de forma xH respectiv Hx cu x G, toate clasele ind echipotente (deci
de acelasi cardinal) cu H, iar mult imile factor
G/
t
H
= xH, x G si G/
t
H
= Hx, x G
au de asemenea acelasi cardinal.
Demonstrat ie. Prin denit ie clasa de echivalent a n raport cu
H
avand reprezentantul x este

H
< x >= y G; x
H
y = y G; x
1
y H = y G; y xH = xH
Analog se arata ca
t
H
< x >= Hx.
Aplicat ia f
x
: H xH; f
x
(h) = xh este injectiva deoarece daca f
x
(h
1
) = f
x
(h
2
), deci xh
1
= xh
2
,
nmult ind la stanga aceasta egalitate cu x
1
obt inem h
1
= h
2
. De asemenea aplicat ia este surjectiva. Din
bijectivitatea lui f
x
rezulta egalitatea cardinalilor [H[ = [xH[ pentru orice x G. Analog se demonstreaza
ca [H[ = [Hx[, x G.
Fie g : G/
H
G/
t
H
; g(xH) = Hx
1
. Aplicat ia g este bine denita deoarece nu depinde de
alegerea reprezentant ilor.

Intr-adevar daca xH = yH, atunci (xH)
1
= (yH)
1
deci H
1
x
1
=
H
1
y
1
si, cum H este subgrup avem H
1
= H. Rezulta de aici Hx
1
= Hy
1
, adica g(xH) = g(yH).
Analog, aplicat ia g
t
: G/
t
H
G/
H
; g
t
(Hx) = x
1
H este bine denita.

Intrucat
(g
t
g)(xH) = g
t
(g(xH)) = g
t
(Hx
1
) = (x
1
)
1
H = xH
si
(g g
t
)(Hx) = Hx, x G,
rezulta ca g este inversabila, g
t
= g
1
si g este biject ie. Aceasta ne arata ca mult imile factor de mai sus
sunt echipotente, deci au acelasi cardinal [G/
H
[ = [G/
t
H
[.
1.4.3 Denit ie. Daca (G, ) este un grup si H G un subgrup al sau, atunci cardinalul mult imii
factor G/
H
se numeste indicele lui H n G si se noteaza [G : H[.
Observam ca, n particular, [G : e[ = [G[ si [G : G[ = 1.
1.4.4 Teorema lui Lagrange. Daca H este un subgrup al lui G atunci [G[ = [G : H[ [H[.
Demonstrat ie. Fie x
i
G; i I un sistem de reprezentant i pentru mult imea factor G/
H
. Deci
[I[ = [G/
H
[ = [G : H[. Cum G/
H
= x
i
H; i I, din denit ia partit iei unei mult imi n clase de
echivalent a rezulta ca G =

iI
x
i
H si toate clasele x
i
H sunt doua cate doua disjuncte. Aceasta mpreuna
cu denit ia adunarii cardinalilor si cu faptul ca toate clasele au acelasi cardinal, [H[ , iar numarul claselor
este [G : H[ conduce la
[G[ =

iI
[x
i
H[ =

iI
[H[ = [I[ [H[ = [G : H[ [H[ .
1.4.5 Corolare. 1
0
.

Intr-un grup nit ordinul ecarui subgrup si indicele lui divid ordinul grupului,
adica [H[ [ [G[ si [G : H[ [ [G[ .
2
0
. Ordinul ecarui element al grupului divide ordinul grupului.

Intr-adevar, daca ordx = m, adica x


m
= e si m este un minim cu aceasta proprietate, atunci
subgrupul ciclic generat de x este < x >= e, x, . . . , x
m1
deci [< x >[ = m. Corolarul 1
0
aplicat
acestui subgrup conduce la m [ [G[.
3
0
. Un grup de ordin prim nu are subgrupuri proprii, el este ciclic.

Intr-adevar, din [G[ = numar prim si corolarele 2


0
si 1
0
rezulta ca G este generat de orice element
al sau diferit de elementul neutru, deci G =< x >; x G e.
Observam ca, n general, ind dat un subgrup H G , clasele xH ,= Hx si deci G/
H
,= G/
t
H
.
1.4.6 Exemplu. Fie (S
3
, ) grupul permutarilor de trei elemente,
S
3
= e, (12); (13); (23); (231); (312),
1.5 Subgrupuri normale 17
unde
e =
_
1 2 3
1 2 3
_
; (12) =
_
1 2 3
2 1 3
_
; (13) =
_
1 2 3
3 2 1
_
;
(23) =
_
1 2 3
1 3 2
_
; (231) =
_
1 2 3
2 3 1
_
si (321) =
_
1 2 3
3 1 2
_
.
Fie subgrupul H = e; (12). Pentru x = (13) spre exemplu avem
xH = (13); (231)
Hx = (13); (321),
iar
G/
H
= H, (13); (231).(23); (321)
G/
t
H
= H, ((13); (321), (23); (231) .
1.4.7. Aplicat ie. Fie G un grup si x G; ordx = n Demonstrat i ca pentru orice k Z; ordx
k
divide
ordinul lui x
Solut ie. Deoarece x
k
< x >, subgrupul ciclic generat de x, si ordinul unui element al grupului
divide ordinul grupului, rezulta ca ordx
k
divide pe [< x >[. Dar [< x >[ =ordx = n, de unde avem ordx
k
divide ordx = n.
1.5 Subgrupuri normale

Intrebarea, n ce caz clasele de echivalent a avand acelasi reprezentant, n raport cu cele doua relat ii
de echivalent a ale unui grup
H
si
t
H
induse de un subgrup H, ne conduce la un tip special de subgrupuri,
numit subgrupuri normale.

In acest caz avem
(xH = Hx, x G) (G/
H
= G/
t
H
) (
H
=
t
H
) .
1.5.1 Denit ie. Daca (G, ) este un grup, subgrupul N G se numeste normal (sau invariant
sau divizor normal al lui G) daca
xN = Nx; x G.
Notam N G.
1.5.2 Exemple. 1
0
. Orice subgrup al unui grup abelian este normal;
2
0
. Grupul nsusi si subgrupul format doar din elementul neutru al grupului sunt subgrupuri normale
(improprii);
3
0
. Centru Z(G) al unui grup (G, ) este subgrup normal.

Intr-adevar deoarece Z(G)


def
z G; xz = zx; x G este subgrup al lui G (vezi exemplu 1.2.5.2
0
)
si xZ(G) = Z(G)x pentru orice x G.
4
0
. Daca H G si [G : H[ = 2 atunci H G.

Intr-adevar din [G : H[ = 2 rezulta G/


H
= H, G H = G/
t
H
.
Urmatoarea teorema da mai multe caracterizari ale subgrupurilor normale, utile n aplicat ii.
1.5.3 Teorema. Daca (G, ) este un grup si N G un subgrup al sau, atunci sunt echivalente
urmatoarele armat ii:
1
0
. N G;
2
0
.
N
=
t
N
;
3
0
. G/
N
= G/
t
N
notat
= G/N;
4
0
. x G, n N n
t
N; xn = n
t
x;
5
0
. x G, n N; x
1
nx N;
6
0
. x G, x
1
Nx N;
7
0
. x G, x
1
Nx = N.
Demonstrat ie. Primele trei echivalent e sunt evidente conform Teoremei 1.4.2. Observam ca

N
=
t
N
daca si numai daca
N
< x >=
t
N
< x > adica x N = N x, x G, ceea ce este
18 1. Grupuri
echivalent cu aceea ca x G si n N exista n
t
N astfel ncat x n
t
= n x. De aici nmult ind ultima
egalitate cu x
1
obt inem x
1
nx = n
t
, deci x
1
nx N si totodata x
1
Nx N.
De asemenea avem x
1
Nx = N si implicat iile inverse.
1.5.4 Propozit ie. Intersect ia oricarei familii de subgrupuri normale ale unui grup este subgrup
normal al acelui grup.
Demonstrat ie. Fie N
i
G; i I o familie de subgrupuri normale ale grupului G si N =

iI
N
i
. S-a
demonstrat n paragraful 1.2 ca intersect ia oricarei familii de subgrupuri ale unui grup este subgrup al
acelui grup. Totodata, cum N
i
este subgrup normal al lui G, pentru orice x G si pentru orice n

iI
N
i
avem n N
i
si x
1
nx N
i
, oricare ar i I. Atunci x
1
nx

iI
N
i
= N.
Se demonstreaza usor armat ia
1.5.5 Propozit ie. Daca H
1
, H
2
sunt subgrupuri ale grupului (G, ), atunci mult imea
H
1
H
2
= h
1
h
2
; h
i
H
i
; i = 1, 2 este subgrup al lui G daca si numai daca H
1
H
2
= H
2
H
1
.
1.5.6 Corolare. 1
0
Daca N
1
, N
2
sunt subgrupuri normale ale lui G atunci N
1
N
2
este de asemenea
subgrup normal al lui G;
2
0
Daca N
1
N
2
G atunci n mult imea subgrupurilor normale ale lui G exista sup(N
1
, N
2
) = N
1
N
2
;
3
0
(o
n
(G), ), mult imea subgrupurilor normale ale lui G, mpreuna cu relat ia de incluziune formeaza
o latice.
1.6 Omomorsme de grupuri
1.6.1 Denit ii. 1
0
. Fie (G, ) si (G
t
, ) doua grupuri. Aplicat ia f : G G
t
se numeste omomor-
sm de grupuri daca x, y G avem f(x y) = f(x) f(y).
Notam prin Hom (G, G
t
) mult imea tuturor omomorsmelor de la G la G
t
;
2
0
. Omomorsmul f Hom(G, G
t
) se numeste izomorsm daca exista omomorsmul g : G
t
G
astfel ncat g f = id
G
si f g = id
G
. Notam izomorsmul grupurilor G si G
t
prin G G
t
;
3
0
. Un omomorsm de la G la G se numeste endomorsm. Mult imea endomorsmelor grupului G
o vom nota prin End G;
4
0
. Un izomorsm de la G la G se numeste automorsm. Mult imea automorsmelor grupului G o
vom nota prin Aut G.
1.6.2 Exemple. 1. Incluziunea canonica a unui subgrup S G, i : S G; i (x) = x; x S, este
un omomorsm injectiv de grupuri;
2. Aplicat ia identica a unui grup n el nsusi, id
G
: G G; id
G
(x) = x, este un automorsm al
acelui grup;
3. Aplicat ia f : (R, +) (R

, ); f(x) = e
x
este un omomorsm de grupuri deoarece x, y G
avem:
f(x +y) = e
x+y
= e
x
e
y
= f(x) f(y);
4. Aplicat ia f : (Z, +) (Z, +) ; f(x) = nx, unde n N este un numar natural xat, este un
endomorsm al lui Z deoarece:
f(x +y) = n(x +y) = nx +ny = f(x) +f(y).
5. Daca (G, ) este un grup si a G este element xat, atunci aplicat ia
i
a
: (G, ) (G, ) ; i
a
(x) = a
1
x a
este un automorsm al lui G numit automorsm interior.

Intr-adevar, aplicat ia i
a
este endomorsm al lui G, deoarece x, y G
i
a
(xy) = a
1
(xy) a = a
1
x(aa
1
)ya = (a
1
xa)(a
1
xa) = i
a
(x)i
a
(y).
Mai mult, exista j
a
: (G, ) (G, ) ; j
a
(x) = axa
1
de asemenea endomorsm al lui G, iar
i
a
j
a
(x) = a
1
(axa
1
)a = x = j
a
i
a
(x) pentru orice x G,
1.6 Omomorsme de grupuri 19
ceea ce demonstreaza ca i
a
este izomorsm.
1.6.3 Propozit ie. Daca f Hom(G, G
t
) atunci f(e
G
) = e
G
si f(x
1
) = [f(x)]
t
, simetricul lui
f(x) n (G
t
, ).
Deomonstrat ie. Din f(x) = f(x e
G
) = f(x) f(e
G
) rezulta f(e
G
) = e
G
, iar e
G
= f(e
G
) =
f(x x
1
) = f(x) f(x
1
) implica f(x
1
) = [f(x)]
t
.
1.6.4 Propozit ie. Compunerea a doua omomorsme de grupuri este de asemenea un omomorsm
de grupuri, adica f Hom(G, G
t
) si g Hom(G
t
, G
tt
) implica g f Hom(G, G
tt
).
Demonstrat ia este imediata.
1.6.5 Corolare. 1
0
Mult imea endomorsmelor unui grup nzestrata cu operat ia de compunere
formeaza un monoid, iar cea a automorsmelor acelui grup formeaza un grup n raport cu compunerea.
Avem deci (EndG, ) = monoid si (AutG, ) = grup.
2
0
Mult imea automorsmelor lui G formeaza un subgrup al grupurilor tuturor aplicat iilor bijective
ale lui G n el nsusi, adica (AutG, ) (B
ij
G, ).
1.6.6 Aplicat ie. Mult imea Int G = i
a
: G G; a G a automorsmelor interioare ale grupului
G este un subgrup normal al grupului tuturor automorsmelor lui G, deci Int G Aut G.
Demonstrat ie. Deoarece i
a
i
b
= i
ba
si (i
a
)
1
= i
a
1, mult imea Int G este subgrup al lui Aut G.

Intr-adevar
i
a
i
b
(x) = i
a
(i
b
(x)) = i
a
(b
1
xb) = a
1
(b
1
xb)a =
= (a
1
b
1
)x(ba) = (ba)
1
x(ba) = i
ba
(x)
si i
a
1 i
a
= i
e
= i
a
i
a
1, unde i
e
(x) = x.
Mai mult, pentru orice f Aut G avem
(f
1
i
a
f)(x) = f
1
(i
a
(f(x))) = f
1
(a
1
f(x)a) =
= f
1
(a
1
)f
1
(f(x))f
1
(a) = [f
1
(a)
]
1
xf
1
(a)
= i
f
1
(a)
(x),
deci f
1
i
a
f = i
f
1
(a)
IntG ceea ce demonstreaza ca subgrupul Int G este normal.
Urmatoarea propozit ie da o alta caracterizare pentru izomorsmele de grupuri, adica o denit ie
echivalenta care se aplica practic. Precizam ca n continuare vom nota multiplicativ ambele operat ii de
grup.
1.6.7 Propozit ie. Daca (G, ) si (G
t
, ) sunt grupuri, aplicat ia f : G G
t
este izomorsm de
grupuri daca si numai daca f este un omomorsm bijectiv.
Demonstrat ie. Presupunem ca omomorsmul f este bijectiv. Rezulta ca exista inversa
f
1
: G
t
G astfel ncat f f
1
= id
G
si f
1
f = id
G
. Sa demonstram ca f
1
este omomor-
sm.

Intr-adevar x
t
, y
t
G
t
, cum f este aplicat ia surjectiva, exista x, y G; f(x) = x
t
; f(y) = y
t
si
f
1
(x
t
y
t
) = f
1
(f(x) f(y))
f. omom.
= f
1
(f(xy)) =
= id
G
(xy) = x y = f
1
(x
t
) f
1
(y
t
).
Aceasta ne arata ca f
1
este omomorsm si deci f este izomorsm.
Daca f : G G
t
este izomorsm atunci din denit ia 1.6.1.2
0
rezulta ca exista g : G
t
G
care este tocmai inversa lui f si f ind inversabila este bijectiva.
1.6.8 Denit ie. Prin nucleul unui omomorsm f Hom (G,G
t
), notat Kerf nt elegem
mult imea elementelor lui G ale caror imagine prin f este elementul neutru din G
t
, deci:
Ker f = x G; f(x) = e
G
.
Evident Ker f ,= deoarece e
G
Ker f.
1.6.9 Propozit ie. Daca (G, ) si (G
t
, ) sunt grupuri, f Hom(G, G
t
), atunci:
a, b G; f(a) = f(b) ab
1
Ker f.
20 1. Grupuri
Demonstrat ie. Daca f(a) = f(b), atunci nmult ind ambii membri ai egalitat ii cu [f(b)]
1
si t inand
seama de Propozit ia 1.6.3 avem f(a) [f(b)]
1
= e
G
de unde f(a) f(b
1
) = e
G
sau f(ab
1
) = e
G
,
adica ab
1
Kerf. Implicat ia n sens invers este imediata.
1.6.10 Denit ie. Prin imaginea unui grup G printr-un omomorsm f Hom(G, G
t
)
nt elegem mult imea:
Imf = f(G) = y G
t
[ x G; f(x) = y.
Daca X G si Y G
t
atunci f(X) = f(x); x G este imaginea lui X prin f, iar
f
1
(Y ) = x G; f(x) Y noteaza contra imaginea lui Y prin f.
Relativ la act iunea omomorsmelor de grupuri asupra subgrupurilor unui grup avem urmatorul
rezultat:
1.6.11 Teorema corespondent ei pentru subgrupuri. Daca f Hom(G, G
t
) atunci:
1
0
. S G f(S) G
t
;
2
0
. S
t
G
t
f
1
(S
t
) G.
Demonstrat ie. 1
0
. Pentru orice y, y
t
f(S) exista x, x
t
S astfel ncat f(x) = y si f(x
t
) = y
t
, de
unde
y (y
t
)
1
= f(x)[f(x
t
)]
1
= f(x) f(x
t
1
) = f(x x
t
1
) f(S).
Deoarece x, x
t
S si cum S este subgrup al lui G rezulta ca xx
/ 1
S.
2
0
. x, x
t
f
1
(S
t
) f(x), f(x
t
) S
t
.

Intrucat S
t
G
t
rezulta f(x) [f(x
t
)]
1
S
t
. Dar
f(x) [f(x
t
)]
1
f omom.
= f(x) f(x
t
1
), deci f(x x
t
1
) S
t
adica x x
t
1
f
1
(S
t
), ceea ce demonstreaza
ca f
1
(S
t
) este subgrup al lui G.
1.6.12 Corolare. Daca (G, ) si (G
t
, ) sunt grupuri, f Hom(G, G
t
) atunci:
1
0
. Ker f G;
2
0
. Imf G
t
;
3
0
. f surjectiv Imf = G
t
;
4
0
. f injectiv Ker f = e
G
;
5
0
. f izomorsm Imf = G
t
si Kerf = e
G
.
Demonstrat ie. 1
0
Din denit ia nucleului lui f ca si contraimaginea subgrupului format numai din
elementul neutru din G
t
, Ker f = f
1
(e
G
), cum e
G
G , conform teoremei anterioare avem ca si
Ker f este subgrup al lui G.
4
0
Daca Ker f = e
G
si f(a) = f(b), atunci conform Propozit iei 1.6.9 avem ab
1
Ker f, deci
ab
1
= e
G
, de unde nmult ind egalitatea cu b la dreapta rezulta a = b, deci f este injectiva.
Reciproc, daca f este injectiva si x Ker f, atunci f(x) = e
G
= f(e
G
) implica x = e
G
, deci
Kerf = e
G
.
Armat iile 2
0
si 3
0
rezulta din faptul ca G este propriul sau subgrup, respectiv din denit ia surjec-
tivitat ii.
1.6.13 Teorema corespondent ei pentru subgrupurile normale. Daca (G, ) si (G
t
, ) sunt
doua grupuri si f Hom(G, G
t
), atunci:
1) N
t
G
t
f
1
(N
t
) G unde f
1
(N
t
) = x G; f(x) N
t

2) (N G si f este surject ie) f(N) G


t
Demonstrat ie. 1) Conform teoremei 1.6.11 din N
t
G
t
rezulta ca f
1
(N
t
) este subgrup al lui G.
Totodata x G si n f
1
(N
t
), deci f(n) N
t
avem f(x
1
nx) = [f(x)]
1
f(n) f(x) N
t
, adica
x
1
nx f
1
(N
t
). Am demonstrat astfel ca f
1
(N
t
) G
t
.
2) Din proprietatea analoaga pentru subgrupuri, din faptul ca N G rezulta f(N) G
t
. Pentru
orice y G
t
, cum f este surject ie, rezulta ca exista x G astfel ncat y = f(x). Atunci k f(N) exista
n N astfel ncat k = f(n). Prin urmare y
1
ky = [f(x)]
1
f(n) f(x) = f(x
1
nx) f(N), deoarece
N G x
1
nx N. Am demonstrat astfel ca f(N) G
t
.
1.6.14 Corolare.
1) Nucleul omomorsmului f Hom(G, G
t
), Ker f este subgrup normal al lui G.

Intr-adevar, aplicand teorema precedenta pentru N


t
= e
G
G obt inem f
1
(e
G
) = Ker f.
2) Daca omomorsmul f este surjectiv, atunci aplicat ia : o

n
(G) o

n
(G), unde
o

n
(G) = N; N G; Ker f N
1.6 Omomorsme de grupuri 21
este bijectiva.
Observam ca daca f este omomorsm injectiv de grupuri, f Hom(G, G
t
), atunci G

f(G) G
t
.
1.6.15 Denit ie. Spunem ca grupul G se scufunda izomorf n grupul G
t
daca exista un omo-
morsm injectiv de la G la G
t
.
1.6.16 Exemplu. Grupul aditiv al ntregilor, (Z, +), se scufunda izomorf n grupul aditiv al nu-
merelor rat ionale, (, +), deoarece exista omomorsmul injectiv f : Z ; f(k) =
k
1
.
Urmatoarea teorema ne arata ca studiul grupurilor se reduce la cel al grupurilor de permutari, adica
la cel al aplicat iilor bijective a unei mult imi n ea nsasi
1.6.17 Teorema lui Cayley. Scufundarea unui grup ntr-un grup de permutari. Orice
grup se scufunda izomorf ntr-un grup de permutari (grupul translat iilor sale stangi).
Demonstrat ie. Fie (G, ) un grup, a G un element xat si aplicat ia t
a
: G G; t
a
(x) = ax
numita translat ie la stanga. Sa notam prin T
G
= t
a
; a G mult imea tuturor translat iilor stangi ale
grupului G. Se verica ca (T
G
, ) formeaza un grup n raport cu compunerea deoarece a, b G
(t
a
t
b
)(x) = t
a
(t
b
(x)) = a(bx) = (ab)x = t
ab
(x) (t
a
t
b
) = t
ab
.
Mai mult, pentru orice t
a
T
G
, (t
a
)
1
= t
a
1 T
G
si n T
G
exista element neutru, aplicat ia identica
pe G, care este tocmai t
e
G
. Deoarece t
a
(x) = t
a
(y) implica ax = ay, de unde nmult ind la stanga cu
a
1
, rezulta x = y, avem ca aplicat ia t
a
este injectiva. De asemenea y G, x = a
1
y G astfel ncat
t
a
(a
1
) = y, deci t
a
este surject ie, si aplicat iile t
a
sunt biject ii. Prin urmare, (T
G
, ) este un subgrup al
grupului tuturor aplicat iilor bijective denite pe G.
Fie acum f : G T
G
; f(a) = t
a
: G G. Aplicat ia f este omomorsm injectiv deoarece:
f(a) = f(b) t
a
= t
b
ax = bx; x G
x=e
a = b
si
f(ab) = t
ab
= t
a
t
b
= f(a) f(b).
Grupul factor al unui grup n raport cu un subgrup al sau. Am vazut n paragraful 1.5 ca
daca N este subgrup normal al grupului G, atunci mult imile factor G/
N
si G/
t
N
coincid si le notam prin
G/N. Prin urmare G/N = xN; x G. Pe aceasta mult ime denim o operat ie binara multiplicativa:
(x
1
N) (x
2
N)
def.
= (x
1
x
2
)N.
Observam ca operat ia este bine denita nedepinzand de alegerea reprezentant ilor.

Intr-adevar, daca
x
t
1
, x
t
2
G si x
1
N = x
t
1
N iar x
2
N = x
t
2
N, adica x
1
1
x
t
1
N si x
1
2
x
t
2
N, atunci
(x
1
x
2
)
1
(x
t
1
x
t
2
) = (x
1
2
x
1
1
)(x
t
1
x
t
2
) = x
1
2
(x
1
1
x
t
1
)x
t
2
x
1
2
Nx
t
2
=
= x
1
2
(Nx
t
2
) = x
1
2
(x
t
2
N) = (x
1
2
x
t
2
)N N.
Se verica usor ca G/N mpreuna cu aceasta operat ie este un grupn care elementul neutru este N = eN,
iar simetricul clasei xN este clasa x
1
N.

Intr-adevar,
N (xN) = (eN)(xN) = (ex)N = xN
si
(xN)(x
1
N) = (xx
1
)N = eN = N
pentru orice xN G/N.
1.6.18 Denit ie. Daca N este un subgrup normal al grupului G, atunci grupul (G/N, ) anterior
denit se numeste grupul factor al lui G prin N.
1.6.19 Propozit ie. Aplicat ia p : G G/N; p(x) = xN este omomorsm surjectiv de grupuri si
Kerp = N.
1.6.20 Exemple. Pentru orice n natural subgrupul nZ este normal n (Z, +), grupul factor Z [
nZ
notat
=
Z
n
se numeste grupul claselor de resturi modulo n. El este ciclic, un generator al sau ind

1 = 1+nZ.
Completam cele spuse n paragraful 1.3 relativ la grupuri ciclice cu urmatoarele:
22 1. Grupuri
1.6.21 Teorema. 1
0
.Toate grupurile ciclice de acelasi ordin sunt izomorfe ntre ele.
2
0
. Oricare doua grupuri ciclice innite sunt izomorfe.
Demonstrat ie. 1
0
. Fie < a > si < b > doua grupuri ciclice de acelasi ordin n, si f :< a >< b >;
f(a
k
) = b
k
; k = 0, 1, . . . n1. Deoarece k, l 0, 1, . . . , n1 avem f(a
k
a
l
) = f(a
k+l
) = b
k+l
=
b
k
b
l
= f(a
k
)f(a
l
), f este omomorsm.
Aplicat ia g :< b >< a >; g(b
k
) = a
k
; k = 0, 1, . . . n 1 este de asemenea omomorsm si mai mult:
(f g)(b
k
) = f(g(b
k
)) = f(a
k
) = b
k
, deci f g = id
<b>
si analog g f = id
<a>
. Am demonstrat astfel
ca f este izomorsm.
2
0
. Presupunem ca G =< a > si G
t
=< b > sunt grupuri ciclice innite. Aplicat ia f : G G
t
;
f(a
k
) = (b
k
) este un izomorsm.
1.6.22 Corolare.
1
0
.Orice grup ciclic innit este izomorf cu grupul aditiv al numerelor ntregi (Z, +);
2
0
. Orice grup ciclic de ordin n este izomorf cu grupul aditiv al claselor de resturi modulo n, (Z
n
, +)
(si cu grupul unitat ilor de ordinul n, U
n
= 1, , . . . ,
n1
, vezi exemplul 1.2.13.2
0
).
Aplicand teoremele de corespondent a pentru subgrupuri obt inem
1.6.23 Teorema. Orice subgrup al unui grup ciclic este ciclic. Imaginea omomorfa a unui grup
ciclic este grup ciclic.
Teoremele de izomorsm au fost date de Emily Noether, teorema fundamentala numita si prima
teorema de izomorsm stand la baza celorlalte.
1.6.24 Prima teorema de izomorsm. Daca f : G G
t
este un omomorsm de grupuri, atunci
G/Ker f

Imf.
Demonstrat ie. Am vazut, conform corolarului 1.6.14.1) ca nucleul Ker f G. Pe grupul factor
G [
Ker f
denim aplicat ia f : G/Ker f Imf; f (xKer f) = f(x), x G. Aplicat ia este bine denita
nedepinzand de alegerea reprezentant ilor.

Intr-adevar, daca x
1
, x
2
G astfel ncat x
1
Ker f = x
2
Ker f,
atunci
x
1

Ker f
x
2
x
1
1
x
2
Ker f f(x
1
) = f(x
2
)
conform propozit iei 1.6.9.
Aplicat ia f este un omomorsm de grupuri deoarece x
1
, x
2
G avem:
f ((x
1
Ker f)(x
2
Ker f)) = f ((x
1
x
2
)Ker f) = f(x
1
x
2
) =
= f(x
1
)f(x
2
) = f (x
1
Ker f) f (x
2
Ker f).
Mai mult, daca x
1
, x
2
G astfel ncat f (x
1
Ker f) = f (x
2
Ker f), deci f(x
1
) = f(x
2
), atunci
[f(x
1
)]
1
f(x
2
) = e. De aici avem f(x
1
1
x
2
) = e
G
, deci x
1
1
x
2
= Ker f, adica x
1
Ker f = x
2
Ker f.
Aplicat ia f este evident surjectiva deci f este izomorsm.
1.6.25 Aplicat ii. 1) Fie (R, +) grupul aditiv al numerelor reale, (C

, ) grupul multiplicativ al
numerelor complexe nenule si aplicat ia f : R C

; f(x) = cos 2x +i sin2x. Deoarece conform regulii


de nmult ire a numerelor complexe scrise sub forma trigonometrica avem:
f(x
1
)f(x
2
) = (cos 2x
1
+i sin2x
1
)(cos 2x
2
+i sin2x
2
) =
= cos 2(x
1
+x
2
) +i sin2(x
1
+x
2
) = f(x
1
+x
2
)
aplicat ia f este omomorsm de grupuri. Imf este subgrupul lui C

format din toate numerele complexe de


modul 1 ntrucat [cos 2x +i sin2x[ = 1 si oricare ar z C

astfel ncat [z[ = 1 avem z = cos t+i sint,


deci exista, x =
t
2
astfel nvat f(x) = z. De asemenea
Ker f = x R; f(x) = 1 = x R; cos 2x +i sin2x = 1 =
= x R; cos 2x = 1si sin2x = 0 = x = k Z = Z.
Aplicand prima teorema de izomorsm, rezulta ca R/Z este izomorf cu grupul multiplicativ al numerelor
complexe de modul 1.
2) Fie U = x C; n N; z
n
= 1. Sa se arate ca:
a) U este subgrup al lui (C

, );
1.6 Omomorsme de grupuri 23
b) Grupul cat (/Z, +) este izomorf cu (U, ).
Solut ie. a) Mult imea U este nevida pentru ca exista 1 U. Mai mult, z
1
, z
2
U m, n N

;
z
m
1
= 1, z
m
2
= 1 p = [m, n] cel mai mic multiplu comun al numerelor m si n astfel ncat (z
1
z
1
2
)
p
= 1,
de unde z
1
z
1
2
U.
b) Se aplica prima teorema de izomorsm omomorsmului Q/Ker f Imf, deci Q [
Z

U.
Fie f : C

; f
_
k
n
_
= cos 2
k
n
+ i sin2
k
n
. Aplicat ia f este omomorsm al grupurilor (, +) si
(C

, ), deoarece
k
n
,
p
m
; k, p Z; n, m N

avem:
f
_
k
n
+
p
m
_
= cos 2
_
k
n
+
p
m
_
+i sin2
_
k
n
+
p
m
_
=
=
_
cos 2
k
n
+i sin2
k
m

_
_
cos 2
p
m
+i sin2
p
m

_
= f
_
k
n
_
f
_
p
m
_
.
Mai mult,
Ker f =
_
k
n
; f
_
k
n
_
= 1
_
=
_
k
n
; cos
2k
n
= 1; sin
2k
n
= 0
_
=
=
_
k
n
;
2k
n
= 2k
t
; k
t
Z
_
=
_
k
n
;
k
n
= k
t
Z
_
= Z.
si ntrucat
_
f
_
k
n
_
n
= 1 avem Imf U. Totodata, oricare ar z U, exista n N

, astfel ncat
z
n
= 1, deci este o radacina de ordinul n a unitat ii. Prin urmare exista k 0, 1, . . . n 1, astfel ca
z = cos
2k
n
+i sin
2k
n
= f
_
k
n
_
, adica z Imf, ceea ce demonstreaza ca U Imf, deci U = Imf.
Capitolul 2
Inele
2.1 Inele. Subinele. Denit ii. Exemple.
2.1.1 Denit ii. Fie R o mult ime nevida nzestrata cu doua legi de compozit ie interne, una notata
aditiv si alta multiplicativ +, : R
2
R
a) Tripletul (R, +, ) se numeste inel daca
1
0
(R, +) este un grup abelian, numit grupul subiacent inelului;
2
0
(R, ) este un semigrup;
3
0
operat ia notata multiplicativ este distributiva fat a de cea notata aditiv, adica
()a, b, c R; a(b +c) = ab +ac; (a +b)c = ac +bc;
b) Dacantr-un inel (R, +, ) exista element neutru fat a de operat ia multiplicativa ., notat prin 1, numit
element unitate, adica (R, ) este un monoid, atunci inelul se numeste unitar sau cu unitate;
c) Daca ntr-un inel (R, +, ) operat ia este comutativa atunci inelul se numeste comutativ.
Ment ionam ca, mai general, inelul se deneste ca un triplet (R, +, ) n care (R, +) este grup abelian,
(R, ) este grupoid si operat ia este distributiva fat a de operat ia +, inelul denit de noi n 2.1.1a.)
numindu-se inel asociativ.

In cele ce urmeaza noi vom trata doar inelele asociative.


Ca si consecint e directe ale denit iei inelului avem urmatoarea
2.1.2 Propozit ie. Reguli de calcul n inele. Daca (R, +, ) este un inel, 0 R elementul sau
neutru fat a de adunare, numit zero, atunci:
1
o
. a 0 = 0 a = 0; a R;
2
o
. a, b R; a(b) = (a)b = ab si (a)(b) = ab;
3
o
. a R si n N avem
(a)
n
=
_
a
n
, daca n este par
a
n
, daca n este impar
4
o
. a, b R; i = 1, n; n N; a(b
1
+... +b
n
) = ab
1
+... +ab
n
;
5
o
. Daca a, b R si ab = ba, atunci avem
(a +b)
n
=
n

k=0
C
k
n
a
nk
b
k
;
24
2.1 Inele. Subinele. Denit ii. Exemple. 25
Demonstrat iile sunt simple exercit ii, precizam ca proprietatea 2.1.2.5
o
se face prin induct ie dupa n.
2.1.3 Denit ii. a) Fie (R, +, ) un inel. Un element a R0 se numeste divizor al lui zero la
stanga (la dreapta) daca exista b R0, astfel ncat ab = 0 (c R0; ca = 0);
b) Fie (R, +, ) un inel unitar. Elementul a R se numeste inversabil daca exista b R astfel ncat
ab = ba = 1;
c) Numim domeniu de integritate un inel comutativ, unitar cu 1 ,= 0 si fara divizori ai lui zero.
Se demonstreaza usor ca au loc urmatoarele proprietat i:
2.1.4 Propozit ii. a) Mult imea elementelor inversabile ale unui inel unitar (R, +, ), notata |(R),
formeaza un grup multiplicativ numit grupul unitat ilor lui R;
b) Un element inversabil al unui inel nu este divizor al lui zero (nici la stanga, nici la dreapta).
Demonstrat ie. a) Fie |(R) =
_
x R; x
1
R; xx
1
= x
1
x = 1
_
. Daca a, b |(R) atunci
ab |(R) deoarece exista (ab)
1
= b
1
a
1
si daca a |(R), atunci (a
1
)
1
= a, de unde avem ca si
a
1
|(R). De aici rezulta ca |(R) formeaza un grup multiplicativ.
b) Fie elementul a |(R) astfel ncat a este divizor al lui zero la stanga n R. Deci exista b R0
astfel ncat ab = 0. Deoarece exista a
1
R avem b = (a
1
a)b = a
1
(ab) = a
1
0 = 0, deci b = 0, ceea
ce este fals.
2.1.5 Exemple. a) (Z, +, ) este domeniul de integritate; (2Z, +, ) este inel comutativ fara unitate;
b) (Z
n
, +, ), unde Z
n
=

0,

1, . . . ,

n 1 sunt clasele de resturi modulo n este inel unitar. Elementele
inversabile ale lui Z
n
sunt clasele de resturi r. unde r este prim cu n, deci |(Z
n
) = r; (r, n) = 1;
c) Fie X o mult ime nevida si R un inel. Pe mult imea R
X
a funct iilor denite pe X cu valori n R
denim operat iile de adunare si nmult ire astfel
f, g : X R; (f +g)(x) = f(x) +g(x); (fg)(x) = f(x) g(x); x X.
Tripletul (R
X
, +, ) este un inel cu elementul zero aplicat ia : X R; (x) = 0
R
; x X. Daca R este
inel cu unitate atunci R
X
este inel unitar avand ca unitate aplicat ia : X R; (x) = 1
R
; x R.
d) Fie R o mult ime si m, n N. Aplicat ia
A : 1, . . . , m 1, . . . , n R
se numeste matrice de tipul (m, n) cu elemente din R.
Notam A(i, j) = a
ij
R. Scriem
A =
_
_
a
11
. . . a
1n
. . .
a
m1
. . . a
mn
_
_
sau pe scurt (a
ij
)
i=1,m
j=1,n
. Pentru m = n , matricea A se numeste matrice patrata.
Vom nota prin /
m,n
(R) mult imea matricelor de tip m, n cu elemente din R. Daca (R, +, ) este un
inel, atunci operat iile din R induc doua operat ii pe mult imea /
m,n
(R) n raport cu care /
m,n
(R) este
un inel si anume: daca A = (a
ij
)
i=1,m
j=1,n
; B = (b
ij
)
i=1,m
j=1,n
, atunci
A+B = (a
ij
)
i=1,m
j=1,n
si A B = (a
ij
b
ij
)
i=1,m
j=1,n
.
Se verica usor ca (/
m,n
(R), +, ) este inel (unitar daca R este inel unitar, unitatea lui ind matricea
n care toate elementele sunt egale cu unitatea).
Dupa cum se stie din liceu, acest tip de produs nu este utilizat, ci se deneste nmult irea unei
matrici A = (a
ij
) /
m,n
(R) cu matricea B = (b
ij
) /
n,p
(R) astfel ncat AB = (c
ij
) /
m,p
(R),
unde c
ij
=
n

k=1
a
ik
b
kj
(asa zisa nmult ire linii pe coloane).

In mult imea /
n
(R) a matricilor patrate de ordinul n cu elemente dintr-un inel R pot nmult ite
linii pe coloane oricare doua matrici.
26 2. Inele
Se verica usor ca (/
n
(R), +, ) este un inel (necomutativ, chiar daca R este inel comutativ). Daca
R este inel cu unitate atunci si /
n
(R) este inelul cu unitatea I
n
= (
ij
)
ij=1,n
, unde
ij
este simbolul lui
Kronecker, adica
ij
=
_
1 pentru i = j
0 pentru i ,= j
. Mai mult, inelul /
n
(R) are divizori ai lui zero.

Intr-adevar
matricele A = (a
ij
)
ij=1,n
cu a
11
,= 0 si a
ij
= 0 pentru (i, j) ,= (1, 1) si B = (b
ij
)
ij=1,n
cu b
nn
,= 0 si
b
ij
= 0 pentru (i, j) ,= (n, n), sunt diferite de matricea nula, dar produsul lor AB = O
n
.
e) Fie (G, +) un grup abelian si EndG mult imea endomorsmelor grupului. Ea poate organizata
ca un inel cu unitate n raport cu adunarea si compunerea endomorsmelor. Astfel, daca f, g EndG
atunci denind f +g : G G; (f +g)(x) = f(x) +g(x) si f g : G G; (f g)(x) = f(g(x)) se verica
usor ca f +g, f g EndG si ca (EndG, +, ) este un inel cu unitate (endomorsmul identic). Grupul
unitat ilor (a elementelor inversabile) acestui inel este AutG, adica cel al automorsmelor grupului G.
f) Pe o mult ime cu un singur element A = a exista o singura operat ie, pe care considerand-o atat
+ cat si obt inem un inel comutativ cu unitate n care 1 = 0 = a, numit inel nul.
g) Fie (G, +) un grup abelian G ,= 0. Denind : G
2
G; x y = 0
G
, obt inem un inel comutativ
fara unitate n care orice element este divizor al lui zero, (G, +, ) numit inel de patrat nul, caracterizat
de egalitatea G
2
= G G = 0.
2.1.6 Denit ii. Fie (R, +, ) un inel si S o mult ime nevida a lui R. S se numeste subinel al lui R
(notam S R ) daca operat iile denite pe R induc pe S operat ii algebrice n raport cu care la randul S
este inel.
Daca R este un inel unitar, S un subinel al lui R si unitatea inelului apart ine lui S, deci 1 S, atunci
spunem ca S este un subinel unitar al lui R. Practic, n aplicat ii se utilizeaza urmatoarea propozit ie
2.1.7 Teorema de caracterizare a subinelelor unui inel. Daca (R, +, ) este un inel si
, = S R, atunci sunt echivalente armat iile
1
0
S este un subinel al lui R;
2
0
i) () x, y S x y S (S, +) (R, +) si
ii) () x, y S xy S.
Demonstrat ia este simplu exercit iu, ea se bazeaza pe teorema de caracterizare a subgrupurilor unui
grup.
2.1.8 Exemple. 1
0
. R si 0 sunt subinele ale lui R numite subinele improprii;
2
0
. Subinelele lui (Z, +, ) sunt de forma nZ = nk; k Z unde n N, singurul inel unitar ind Z.
Vericarea este simplu exercit iu;
3
0
. Subinelele lui (Z
n
, +, ) coincid cu subgrupurile ciclice ale grupului subiacent (Z
n
, +) prin urmare
<

d >= k

d ; k Z, unde d divide pe n;
4
0
. Daca R este un inel atunci mult imea elementelor din inel care comuta cu orice element al lui R
deci, Z(R) = z R; x R; zx = xz este un subinel numit centrul lui R.

Intr-adevar, deoarece 0 Z(R) avem Z(R) ,= . Daca a, b Z(R), atunci pentru orice x R avem
(a +b)x = ax +bx = xa +xb = x(a +b), deci a +b Z(R) . Totodata
(ab)x = a(bx) = a(xb) = (ax)b = (xa)b = x(ab),
deci ab Z(R) si
(a)x = (ax) = (xa) = x(a),
deci a Z(R), ceea ce demonstreaza ca Z(R) R.
Se demonstreaza usor urmatoarea armat ie:
2.1.9 Propozit ie. Daca (R, +, ) este un inel, atunci oricare ar familia de subinele (S
i
, i I) ale
inelului, intersect ia lor,

iI
S
i
, este de asemenea subinel al lui R.
2.1.10 Denit ie. Fie (R, +, ) un inel si X R o submult ime a inelului. Prin subinelul generat de
X, notat < X >, nt elegem intersect ia tuturor subinelelor lui R care l cont in pe X, deci
< X >=

(S; S R; X S) .
Are loc urmatoarea
2.2 Ideale 27
2.1.11 Teorema. Mult imea subinelelor unui inel formeaza o latice completa n raport cu incluziunea.

Intr-adevar, data ind familia de subinele (S


i
)
iI
ale inelului R avem
inf(S
i
)
iI
=

iI
S
i
,
iar
sup(S
i
)
iI
=<
_
iI
S
i
> .
2.2 Ideale
2.2.1 Denit ie. Fie (R, +, ) un inel. O submult ime nevida H a lui R se numeste ideal la stanga
(la dreapta) lui R daca
1
0
. () x, y H x y H;
2
0
. ()a R, x H ax H (xa H).
Notam H
s
R (H
d
R) .
Un ideal drept care este n acelasi timp si un ideal stang se numeste ideal bilateral sau simplu
ideal n R; notam H R.
2.2.2 Observat ii. a) Din condit ia 1
0
a denit iei rezulta ca (H, +) este un subgrup normal al
grupului subiacent inelului, deci 0 Hsi ()x H x H;
b) Orice ideal la stanga (dreapta, bilateral) este subinel al lui R. Reciproca nu este adevarata. Ca
un contraexemplu: Inelul numerelor ntregi Z, este subinel al inelului numerelor rat ionale (, +, ) dar
nu este ideal n .
2.2.3 Exemple. a) R si 0 sunt ideale n R;
b) Idealele lui Z coincid cu nZ deci coincid cu subinelele lui Z;
c) Idealele lui Z
n
coincid cu subinelele lui Z
n
;
d) Daca R este un inel si a R un element xat n R, atunci mult imea
aR := ax [ x R,
respectiv
Ra := xa [ x R,
respectiv
RaR :=
_
n

i=1
x
i
ay
i
[ x
i
, y
i
R; i = 1, n; n N
_
este ideal la dreapta, respectiv la stanga, respectiv ideal bilateral al lui R numite idealul principal (la
dreapta, respectiv la stanga, respectiv bilateral) generat de a R.
2.2.4 Observat ii. a)

In inelul numerelor ntregi Z si n cel al claselor de resturi modulo n, toate
idealele sunt principale.
b)

In inelele comutative exista un singur concept de ideal.
2.2.5 Propozit ie. Fie R un inel unitar si H un ideal al lui R la stanga, la dreapta respectiv
bilateral. Avem H = R daca si numai daca H cont ine un element inversabil din R. Pe scurt
H R; |(R) H ,= H = R.
Demonstrat ie. Fie H R si H = R atunci 1 H si 1 |(R).
Reciproc, daca H R si exista u |(R) H, atunci exista v R astfel ncat uv = vu = 1. Oricare
ar x R avem x = x 1 = x(v u) = (xv) u H deci R H. Cum H R, rezulta R = H.
2.2.6 Propozit ie. Daca R este inel si (H
i
)
iI
o familie de ideale la stanga (dreapta, bilateral) ale
lui R, atunci intersect ia

iI
H
i
este ideal de acelasi tip al inelului.
Demonstrat ie. Demonstrat ia este imediata.
28 2. Inele
2.2.7 Denit ie. Fie R un ideal si submult imea X R. Intersect ia tuturor idealelor la stanga ale
lui R care-l cont in pe X se numeste idealul stang al lui R generat de X, notat prin (X [. Deci
(X [= H; H
s
R; X H.
Analog se denesc idealul drept generat de X, notat prin [ X),
[ X) = H; H
d
R; X H
si idealul bilateral generat de X, notat (X)
(X) = H; H R; X H.
Mult imea X se numeste mult ime de generatori ai idealului (X [, [ X) respectiv (X).
Daca X este o mult ime nita atunci idealele generate de ea se numesc nit generate.
2.2.8 Observat ii. 1
0
. Daca X = atunci avem ( [=[ ) = () = 0.
2
0
. Idealele (X [, [ X), (X) sunt cele mai mici ideale stangi, drepte, respectiv bilatere care cont in
mult imea X.
2.2.9 Propozit ie. Fie R un inel unitar si X o submult ime nevida a lui R. Atunci avem
(X [=
_
n

i=1
a
i
x
i

n N; a
i
R; x
i
X; i = 1, n
_
[ X) =
_
n

i=1
x
i
a
i

n N; a
i
R; x
i
X; i = 1, n
_
(X) =
_
n

i=1
a
i
x
i
b
i

n N; a
i
, b
i
R; x
i
X; i = 1, n
_
adica, idealul stang, drept, respectiv bilater al unui inel unitar R, generat de X sunt formate din mult imea
sumelor nite de elemente din RX, XR, respectiv RXR.
Demonstrat ie. Fie H
t
=
_
n

i=1
a
i
x
i

n N; a
i
R; x
i
X; i = 1, n
_
. Oricare ar u, v H, deci
u =
n

i=1
a
i
x
i
; v =
m

j=1
b
j
x
t
j
; n, m N; a
i
, b
j
R; x
i
, x
t
j
X; i = 1, n; j = 1, m,
diferent a u v = H
t
, ind o suma de acelasi tip si anume u v =
n+m

i=1
c
k
x
tt
k
, unde
c
k
=
_
a
k
pentru k = 1, n
b
kn
pentru k = n + 1, n +m
;
x
tt
k
=
_
x
k
pentru k = 1, n
x
t
kn
pentru k = n + 1, n +m
.
De asemenea, () a R si ()u H
t
avem au =
n

i=1
(aa
i
)x
i
H
t
deci H
t
este ideal stang n R. Cum R
este inel unitar pentru orice x R avem x = 1 x H
t
rezulta X H
t
si prin urmare
(X [ H. (1)
Daca H este un alt ideal stang al lui R astfel ncat X H, atunci rezulta ca orice suma de forma
n

i=1
a
i
x
i
din H
t
apart ine de asemenea si lui H (deoarece x
i
X H) si prin urmare H
t
H.

Intrucat H este
oarecare, rezulta ca
H
t
H; H
s
R; X H = (X [ .
Din (1) si (2) rezulta ca H
t
= (X [.
2.2 Ideale 29
2.2.10 Observat ie. Idealele stangi, drepte, bilatere generate de un element a R al unui inel
unitar R, notate prin (a [, [ a) respectiv (a) sunt tocmai idealele principale ale lui R si anume, aR, Ra
respectiv RaR denite n exemplul 2.2.3.d). Deci
(a [= aR; [ a) = Ra ; (a) = RaR.

In particular, daca R este comutativ si unitar, atunci avem


(a) = ra +na [ r R; n Z.
2.2.11 Denit ie. Fie (H)
iI
o familie de ideale ale lui R de acelasi tip. Idealul generat de reuniunea
lor,

iI
H
i
, se numeste suma idealelor si se noteaza prin

iI
H
i
.
Avem

iI
H
i
= x
i
1
+x
i
2
+. . . +x
i
n
[ n N; x
i
k
H
i
k
; k = 1, n; i
k
I.
Daca mult imea de indici I este nita, I = 1, 2, . . . , n atunci
n

i=1
H
i
= x
1
+x
2
+. . . x
n
[ x
i
H
i
; i = 1, n.
Sa notam prin L
S
(R), L
D
(R) respectiv L
B
(R) mult imea idealelor stangi, drepte, respectiv bilatere ale
lui R. Din cele mai nainte demonstrate rezulta ca oricare ar o familie (H
i
)
iI
de ideale de acelasi tip
ale idealului R, n mult imea (L(R), ) exista
inf (H
i
)
iI
=

iI
H
i
si
inf (H
i
)
iI
=
_
_
iI
H
i
_
=

iI
H
i
.
Avem prin urmare
2.2.12 Teorema. Mult imile idealelor de acelasi tip ale unui inel R, stangi, drepte, bilatere
(L
S
(R), ); (L
D
(R), ), respectiv (L
B
(R), ) sunt latici complete. Laticea idealelor unui inel R este
sublatice a laticei subgrupurilor grupului subiacent (R, +).
Ultima armat ie rezulta din faptul ca orice ideal a lui R este subgrup normal al grupului (R, +).
2.2.13 Corolar. Daca H
1
, H
2
sunt ideale ale inelului R, atunci
H
1
+H
2
= x
1
+x
2
; x
1
H
1
; x
2
H
2

este un ideal al lui R si


supH
1
, H
2
= H
1
+H
2
.
2.2.14 Observat ie. Prin calcul direct se verica ca daca S este subinel al inelului R, iar H ideal al
lui R, atunci
S +H = a +u; a S; u H
este subinel al lui R.
2.2.15 Exemple.

In inelul M
2
(R) al matricilor patrate de ordinul doi cu elemente dintr-un inel
comutativ si unitar, mult imile
H
1
=
__
0 a
0 b
_
; a, b R
_
; H
2
=
__
a 0
b 0
_
; a, b R
_
sunt ideale la stanga dar nu sunt ideale la dreapta ale lui M
2
(R). Suma lor H
1
+H
2
este tocmai M
2
(R),
iar produsele lor sunt H
1
H
2
= H
2
si H
2
H
1
= H
1
.
30 2. Inele
2.3 Omomorsme de inele
2.3.1 Denit ie. Fie (R, +, ) si (R
t
, +, ) inele. Aplicat ia f : R R
t
se numeste morsm (sau
omomorsm) de inele daca pentru orice x, y R avem
f(x +y) = f(x) +f(y) ; f(xy) = f(x)f(y).
Daca R si R
t
sunt inele unitare si n plus avem f(1
R
) = 1
R
atunci f se numeste morsm unital de
inele.
Un morsm al inelului R n el nsusi se numeste endomorsm.
Fie Hom(R, R
t
) mult imea morsmelor de la inelul R la R
t
.
2.3.2 Observat ii. 1
0
. Exista cel put in un morsm de la R la R
t
si anume aplicat ia : R R
t
;
(x) = 0
R
este un morsm de inele numit morsmul nul.
Mai mult, chiar daca R si R
t
sunt inele unitare exista morsme neunitare de la R la R
t
, un astfel de
exemplu ind tocmai cel nul.
Daca R ,= 0 atunci exista cel put in doua morsme de la R la el nsusi si anume cel nul si cel
identic id
R
: R R; id
R
(x) = x.
2
0
. Daca f este morsm de inele, atunci f este un morsm al structurilor de grup, subiacente inelelor.
De aici avem
f(0
R
) = 0
R
si x R; f(x) = f(x).
3
0
. Daca R si R
t
sunt inele unitare si f : R R
t
este morsm unital de inele, atunci el induce un
morsm de la grupul elementelor inversabile din R, U(R) la cel al elementelor inversabile din R
t
, U(R
t
).

Intr-adevar, daca a U(R) atunci exista a


t
R astfel ncat a a
t
= a
t
a = 1
R
, de unde avem
f(a) f(a
t
) = f(a
t
) f(a) = 1
R
si deci f(a) U(R
t
). Mai mult, notand prin a
1
respectiv [f(a)]
1
simetricele lui a, respectiv f(a) n raport cu operat ia multiplicativa denita n cele doua inele avem
f(a
1
) = [f(a)]
1
, a U(R).
Se verica imediat urmatoarea armat ie
2.3.3 Propozit ie. Compunerea a doua morsme de inele este tot un morsm de inele,adica daca
f Hom(R, R
t
) si g Hom(R
t
, R
tt
) atunci g f Hom(R, R
tt
).
2.3.4 Denit ii. Un morsm de inele f : R R
t
se numeste morsm injectiv sau monomorsm
daca aplicat ia f este injectiva. El se numeste omomorsm surjectiv daca f este surjectiva. Un morsm
de inele f : R R
t
se numeste izomorsm al inelelor R si R
t
(notam R R
t
) daca exista morsmul
g : R
t
R astfel ncat g f = id
R
si f g = id
R
. Un izomorsm f : R R se numeste automorsm
al inelului R.
Prin nucleul unui morsm de inele f : R R
t
nt elegem mult imea
Kerf = x R; f(x) = 0
R
.
Urmatoarea teorema da o alta caracterizare a izomorsmelor de inele, mai usor de aplicat.
2.3.5 Propozit ie. Un morsm f : R R
t
de inele, este izomorsm daca si numai daca este un
morsm bijectiv, deci aplicat ia f este bijectiva.
Demonstrat ie. Daca f este un izomorsm atunci conform denit iei 2.3.4, f este inversabil ceea ce
implica ca f este bijectiv.
Reciproc, daca f : R R
t
este o biject ie atunci aplicat ia f este inversabila, deci exista f
1
: R
t
R
astfel ncat f
1
f = 1
R
si f f
1
= 1
R
.
Sa demonstram ca f
1
este un morsm de inel.

Intr-adevar, f ind surject ie, oricare ar x
t
, y
t
R
t
exista x, y R astfel ncat f(x) = x
t
si f(y) = y
t
si deci x = f
1
(x
t
) si y = f
1
(y
t
). Avem atunci
f
1
(x
t
y
t
) = f
1
(f(x) f(y)) = f
1
(f(xy)) =
= (f
1
f)(xy) = xy = f
1
(x
t
) f
1
(y
t
),
ceea ce demonstreaza ca f
1
este morsm de inele.
2.3 Omomorsme de inele 31
Deoarece f este un izomorsm al grupurilor subiacente (R, +) si (R
t
, +), atunci f
1
este izomorsm
al acestor grupuri si deci f
1
(x
t
+y
t
) = f
1
(x
t
) +f
1
(y
t
).
2.3.6 Teorema de corespondent a pentru subinele si ideale. Daca R si R
t
sunt inele si
f Hom(R, R
t
) atunci
1
0
. Daca S este un subinel al lui R, atunci f(S) este subinel al lui R
t
;
2
0
. Daca S
t
este subinel al lui R
t
atunci contraimaginea lui S
t
, f
1
(S
t
) = x R [ f(x) S
t
este
deasemenea subinel al lui R;
3
0
. Daca H este un ideal stang, (drept respectiv bilateral) al lui R si f este un morsm surjectiv de
inele atunci f(H) este un ideal de acelasi tip al lui R
t
;
4
o
. Daca H
t
este un ideal stang, drept respectiv bilater al lui R
t
atunci contraimaginea lui H
t
prin
f, f
1
(H
t
), este un ideal de acelasi tip al lui R.
Demonstrat ie. Vom demonstra armat ia 3
0
spre exemplu. Deoarece (R, +) este un grup abelian
si (H, +) este un subgrup normal al lui (R, +), iar f : (R, +) (R
t
, +) este un omomorsm de grupuri,
rezulta ca (f(H), +) este subgrup normal al lui (R
t
, +).
Fie H un ideal stang n R. Oricare ar b R
t
, ntrucat f este surject ie, exista x H astfel ncat
b = f(a). Prin urmare, pentru orice y f(H) exista x H astfel ncat y = f(x), deci by = f(a) f(x) =
f(ax) f(H) deoarece ax H.

In cazurile particulare S = R, respectiv H


t
= 0
R
notand prin Imf = f(R), respectiv Ker f =
f
1
(0
R
) teorema 2.3.6 aliniatele 1
0
si 4
0
ne conduc la urmatorul
2.3.7 Corolar. Daca R si R
t
sunt inele si f Hom(R, R
t
) atunci f(R) este subinel al lui R
t
, iar
nucleul lui f, Kerf, este deasemenea ideal al lui R.
Fie R un inel, H un ideal al sau si o relat ie denita pe elementele lui R astfel xy x y H.

Intrucat H este subgrup normal al grupului (R, +), este relat ie de echivalent a si mult imea factor R/

n raport cu notata prin R/H este R/H = x = x +H; x R.


Aplicand teoremele corespunzatoare de la grupuri, pe mult imea R/H se deneste adunarea astfel
ncat (R/H, +) este grup abelian si anume: daca x+H, y+H R/H atunci (x+H)+(y+H) = (x+y)+H.
Denind si o operat ie multiplicativa (x +H)(y +H) = xy +H (operat ia este bine denita, nedepinzand
de alegerea reprezentant ilor ceea ce rezulta prin simpla vericare), se arata ca (R/H, +, ) este un inel cu
elementul nul H.
Notand simplicat clasa de echivalent a x+H prin x si y +H prin y, operat iile din inelul factor sunt
x + y =

x +y si x y = xy.
2.3.8 Denit ie. Daca R este un inel, H un ideal al sau, atunci inelul (R/H, +, ) se numeste inelul
factor al lui R n raport cu idealul H.
Aplicat ia canonica
H
: R R/H;
H
(x) = x+H este un morsm surjectiv de inele si Ker
H
= H.
Cum nucleul Ker f al unui omomorsm de inele f Hom(R, R
t
) este ideal n R, putem vorbi de inelul
factor R/
Ker f
, iar aplicat ia canonica
Ker f
are nucleul tocmai Ker f.
2.3.9 Observat ii. 1
0
Daca R este un inel comutativ atunci inelul factor n raport cu orice ideal al
sau este de asemenea comutativ.
2
0
Daca R este inel unitar atunci orice inel factor al sau este cu unitate.
2.3.10 Exemplu. Inelul claselor de resturi modulo n. Fie (Z, +, ) inelul numerelor ntregi.
Se stie ca (Z, +) este grup ciclic generat de 1 si daca H este un subgrup al lui (Z, +) atunci el este de
asemenea ciclic si anume daca n este cel mai mic numar natural apart inand lui H, atunci H = nZ. De
aici rezulta ca H este ideal al lui Z, tocmai idealul principal generat de n. Orice ideal al lui Z este de
forma nZ. Inelul factor al lui Z n raport cu idealul nZ, deci Z/
nZ
se noteaza prin Z
n
si elementele lui
sunt clasele de echivalent a modulo n:
Z
n
= nZ, 1 +nZ, 2 +nZ, . . . , (n 1) +nZ.
Notand clasa k +nZ prin

k scriem Z
n
=

0,

1, . . . ,

n 1. Se verica usor ca daca n este numar compus,
deci n = p q cu 1 < q, p < n, atunci p q =

0, deci inelul factor Z


n
are divizori ai lui zero. Clasa

k este
element inversabil n Z
n
daca si numai daca k si n sunt prime ntre ele.

Intr-adevar, daca (k, n) = 1,
atunci exista u, v Z astfel ncat uk + vn = 1, de unde rezulta ca u

k + v n =

1 deci u

k =

1 (cum
n = 0). Prin urmare exista (

k)
1
= u. Invers, daca

k este element inversabil n Z
n
atunci u astfel ncat
32 2. Inele

k u =

1 si

k u 1 =

0 si deci n divide pe (ku 1). Prin urmare v Z astfel ncat ku 1 = vn, de


unde ku + (v)n = 1. Aceasta demonstreaza ca (n, k) = 1.
Ca o consecint a a celor demonstrate mai sus relativ la elementele inversabile din Z
n
deducem ca
ordinul grupului U(Z
n
) este egal cu (n), indicatorul lui Euler, adica cardinalul mult imii numerelor
naturale mai mici ca n, prime cu n.

In particular, daca n este numar prim, atunci toate elementele
nenule ale lui Z
n
sunt inversabile, deci (Z
n
, +, ) este corp.
2.3.11 Teorema fundamentala de izomorsm pentru inele. Daca R si R
t
sunt inele si
f Hom(R, R
t
) atunci inelul factor al lui R n raport cu nucleul lui f, Ker f si f(R) sunt izomorfe:
R/
Ker f
Imf.
Demonstrat ie. Din prima teorema de izomorsm pentru grupuri rezulta ca exista si este bine
denit omomorsmul bijectiv al grupurilor f : (R/
Ker f
, +) (Imf, +); f ( x) = f (x +Ker f) = f(x).
Deoarece pentru orice x, y R/
Ker f
avem f ( x y) = f ( xy) = f(xy) = f(x) f(y) = f ( x) f ( y)
rezulta ca f este un omomorsm bijectiv de inele si putem scrie
R/
Ker f
Imf.
2.4 Corpuri. Subcorpuri. Morsme de corpuri.
2.4.1 Denit ie. Un inel unitar (K, +, ) cu 1 ,= 0 se numeste numeste corp daca x K0
notat
K

,
x este inversabil. Un corp (K, +, ) n care operat ia multiplicativa este comutativa se numeste corp co-
mutativ sau camp.
2.4.2 Observat ii a)

Intr-un corp exista doua grupuri (K, +) si (K

, );
b) Un corp are cel put in doua elemente;
c) Un corp nu are divizori ai lui zero.

Intr-adevar, daca ab = 0 si a ,= 0 atunci ntrucat exista
a
1
R avem
b = (a
1
a)b = a
1
(ab) = a
1
0 = 0.
Reciproca acestei armat ii nu este adevarata. Spre exemplu (Z, +, ) este un inel integru dar nu e
corp.

In caz nit dupa cum se va demonstra n continuare, orice domeniu de integritate este corp.
2.4.3 Exemple 1
0
. Mult imile numerelor rat ionale, reale respectiv complexe nzestrate cu adunarea
si nmult irea obisnuite sunt corpuri comutative.
2
0
.Daca d N astfel ncat ori care ar p numar prim, p
2
,= d, atunci d se numeste liber de patrate.
Mult imea (

d) = a + b

d, a, b nzestrata cu adunarea si nmult irea din R, deci ((

d), +, )
este un corp comutativ.
3
0
. Inelul claselor de resturi modulo n, (Z
n
, +, ) este un corp nit comutativ daca si numai daca n
este numar prim.
2.4.4 Propozit ie. Orice domeniu de integritate nit este corp.
Demonstrat ie. Fie (R, +, ) un domeniu de integritate nit si a R. Aplicat ia t
a
: R R, t
a
(x) =
ax (translat ia la stanga) este injectiva deoarece t
a
(x
1
) = t
a
(x
2
) implica a(x
1
x
2
) = 0 si cum a ,= 0 si
n R nu exista divizori ai lui zero, rezulta x
1
x
2
= 0, deci x
1
= x
2
.

Intrucat R este inel nit si aplicat ia t


a
este injectiva, rezulta ca ea este si surjectiva, deci exista
b R astfel ncat t
a
(b) = 1, adica ab = 1.
Analog, translat ia la dreapta t
t
a
: R R, t
t
a
(x) = xa este injectiva, deci surjectiva, deci exista
c R astfel ncat t
t
a
(c) = 1, adica ca = 1.
Dar c = c 1 = c (ab) = (ca) b = 1 b = b, ceea ce ne arata ca n R exista inversul oricarui element
nenul (a
1
= b = c) si (R, +, ) este corp.
2.4.5 Denit ie. Daca K este un corp si F o submult ime nevida a sa, F se numeste subcorp al
lui K daca aplicat iile denite pe K induc pe F operat ii algebrice n raport cu care F este corp. Notam
F K.
O caracterizare a subcorpurilor este data de propozit ia de mai jos, a carei demonstrat ie este un
simplu exercit iu.
2.4 Corpuri. Subcorpuri. Morsme de corpuri. 33
2.4.6 Propozit ie. Daca K este un corp , F K , F ,= , F este subcorp al lui K daca si numai
daca
1
0
. x, y F x y F;
2
0
x, y F; y ,= 0; xy
1
F.
2.4.7 Exemple. 1
0
.Corpul numerelor rat ionale este subcorp al corpului numerelor reale care la
randul sau este subcorp al corpului numerelor complexe R C;
2
0
. Daca d este liber de patrate atunci avem (

d) R.
2.4.8 Propozit ie. Singurele subcorpuri ale corpului numerelor rat ionale si ale lui Z
p
, unde p
este numar prim, sunt ele nsele.
Demonstrat ie. 1
0
. Daca F este un subcorp al lui atunci 0, 1 F, deci avem si 1 F. Pentru
orice n N se demonstreaza prin induct ie ca avem n = n 1 F, deci si n F. Prin urmare avem
Z F. Fie acum x , deci x =
m
n
, unde n ,= 0, m, n Z. Deoarece Z F conform Propozit iei 2.4.6
avem x = m n
1
F, deci F si deoarece F , avem F = .
2
0
. Daca p este numar prim si F este un subcorp al lui Z
p
atunci ntrucat (F, +) este un subgrup al
grupului subiacent (Z
p
, +), conform teoremei lui Lagrange, rezulta ca ordinul lui F divide ordinul lui Z
p
,
deci divide pe p. Cum p este numar prim si orice corp are cel put in doua elemente, rezulta ca [ F [= p
de unde F = Z
p.
Precizam ca un corp pentru care singurul subcorp este el nsusi se numeste corp prim.
Utilizand Propozit ia 2.4.6. se demonstreaza imediat urmatoarea armat ie:
2.4.9 Propozit ie. Intersect ia subcorpurilor oricarei familii de subcorpuri ale unui corp este un
subcorp al acelui corp.
Not iunea de ideal poate denita si ntr-un corp, privit ca si inel cu unitate, dar conform propozit iei
2.2.5 ntr-un corp singurele ideale ale sale sunt cele improprii, adica 0 si el nsusi.
2.4.10 Denit ie. Daca (K, +, ) si (K
t
, +, ) sunt corpuri, aplicat ia f : K K
t
se numeste
omomorsm de corpuri daca pentru orice x, y K avem f(x + y) = f(x) + f(y) , f(xy) = f(x)f(y)
si f(1
K
) = 1
K
.
2.4.11 Observat ie. Un omomorsm de corpuri f : K K
t
este omomorsm al grupurilor (K, +)
n (K
t
, +), respectiv (K

, ) n (K
/
, ) de unde avem:
f(0
K
) = 0
K
; f(x) = f(x), x K; f(x
1
) = [f(x)]
1
, x K

.
2.4.12 Propozit ie. Orice omomorsm de corpuri este injectiv.
Demonstrat ie. Fie f : K K
t
un morsm de corpuri. Pentru orice x, y K, x ,= y avem xy ,= 0,
deci exista (x y)
1
= z ,= 0 si (x y)z = 1. De aici avem f(x y)f(z) = 1
K
si cum f(z) ,= 0 rezulta
f(x y) ,= 0, deci f(x) ,= f(y), ceea ce demonsreaza ca f este injectiv.
2.4.13 Observat ie. Imaginea Imf a unui omomorsm de corpuri f : K K
t
este un subcorp a
lui K
t
.
2.4.14 Exemple. 1
0.
Corpul numerelor complexe. Mult imea numerelor complexe C = a +
bi, a, b R, i
2
= 1 este un corp comutativ n raport cu adunarea si nmult irea:
(a +bi) + (a
t
+b
t
i) = (a +a
t
) + (b +b
t
)i
(a +bi)(a
t
+b
t
i) = (aa
t
bb
t
) + (ab
t
+a
t
b)i.
Aplicat ia : R C, (a) = a+0i ind un omomorsm de corpuri ne indica ca R, corpul numerelor
reale se scufunda izomorf n C.
Fie M =
__
a b
b a
_
, a, b R
_
o submult ime a inelului a matricilor patrate denite pe R. Se
verica usor ca n raport cu adunarea si nmult irea matricilor (M, +, ) este un corp. Mai mult, el este
izomorf cu corpul numerelor complexe, deoarece aplicat ia f : C M; f(a + bi) =
_
a b
b a
_
este un
omomorsm surjectiv de corpuri.
Vom generaliza acest rezultat obt inand:
2
0
. Corpul cuaternionilor. Fie K = a
1
+a
2
i +a
3
j +a
4
k ; a
1
, a
2
, a
3
, a
4
R, unde
34 2. Inele
ij = ji = k; ki = ik = j; jk = kj = i,
relat ii care se ret in din urmatoarea tabla a nmult irii
i j k
i 1 k j
j k 1 i
k j i 1
Daca pe mult imea K denim adunarea pe componente si nmult irea ca si cea a polinoamelor
utilizand tabla de nmult ire de mai sus, adica
(a
1
+a
2
i +a
3
j +a
4
k) + (b
1
+b
2
i +b
3
j +b
4
k) = (a
1
+b
1
) + (a
2
+b
2
)i + (a
3
+b
3
)j + (a
4
+b
4
)k
(a
1
+a
2
i +a
3
j +a
4
k)(b
1
+b
2
i +b
3
j +b
4
k) = (a
1
b
1
a
2
b
2
a
3
b
3
a
4
b
4
)+
+i(a
1
b
2
+a
2
b
1
a
3
b
4
+a
4
b
3
) +j(a
1
b
3
+a
3
b
1
a
2
b
4
+a
4
b
2
) + (a
1
b
4
+a
4
b
1
+a
2
b
3
a
3
b
2
),
se verica ca (K, +, ) este un corp numit corpul cuaternionilor n care corpul numerelor complexe se
scufunda izomorf.
Aceasta construct ie nu este articiala, ntrucat se demonstreaza ca urmatoarea submult ime a inelului
matricilor patratice de ordinul doi cu elemente din C,
H =
__


_
; , C
_
este un corp n raport cu adunarea si nmult irea obisnuita a matricilor.

Intr-adevar H este parte stabila
n raport cu adunarea si nmult irea, opusa oricarei matrici din H este element al lui H, si pentru orice
matrice nenula
_


_
H avem determinantul ei =[ [
2
+ [ [
2
,= 0 ceea ce arata ca exista
inversa ei,
_
_
_

_
_
_ H.
Daca = a +bi si = a
t
+b
t
i putem scrie
_


_
= a
_
1 0
0 1
_
+b
_
i 0
0 i
_
+a
t
_
0 1
1 0
_
+b
t
_
0 i
i 0
_
ceea ce ne arata ca aplicat ia f : K H;
f(a
1
+a
2
i +a
3
j +a
4
k) = a
1
_
1 0
0 1
_
+a
2
_
i 0
0 i
_
+a
3
_
0 1
1 0
_
+a
4
_
0 i
i 0
_
=
=
_
a
1
+a
2
i a
3
+a
4
i
(a
3
a
4
i) a
1
a
2
i
_
este un omomorsm de corpuri, rezultat care-l generalizeaza pe cel de la numere complexe.
Un rezultat fundamental relativ la corpuri pe care-l prezentam fara demonstrat ie este
2.4.15 Teorema lui Wedderburn. Orice corp nit este comutativ.

Incheiem acest capitol cu cateva considerat iuni privind not iunile de caracteristica a unui inel, respec-
tiv corp.
2.4.16 Denit ie. Fie (R, +, ) un inel cu unitate. Ordinul elementului unitate 1
R
n grupul subiacent
inelului (R, +) se numeste caracteristica inelului R.
Notam prin charR caracteristica inelului R.
2.4 Corpuri. Subcorpuri. Morsme de corpuri. 35
2.4.17 Observat ie. Din denit ia de mai sus rezulta ca daca charR = n atunci n este cel mai mic
numar natural nenul cu proprietatea ca n 1
R
= 0
R
, deci 1
R
+1
R
+. . . +1
R
= 0
R
, nsumarea facandu-se
de n ori. Daca ord1
R
= atunci spunem ca R este inel de caracteristica innita sau dupa alt i autori,
de caracteriastica zero.
2.4.18 Exemple. 1) Inelul Z si corpurile , R si C au caracteristica ;
2) Inelele Z
n
si Z
n
[X], cel al polinoamelor n nedeterminata X cu coecient i n Z
n
, au caracteristica
n (n 2).
2.4.19 Propozit ie.

Intr-un inel cu unitate fara divizori ai lui zero, caracteristica inelului este
sau un numar prim.
Demonstrat ie. Presupunem ca avem charR = n, numar compus, deci n = mk unde 1 < m, k < n.
Din ord1
R
= n rezulta ca 0
R
= n1
R
= (mk)1
R
= (m1
R
)(k1
R
) si ntrucat m, k < n rezulta ca m1
R
= 0
R
sau k1
R
= 0
R
, contradict ie, deoarece inelul n-are divizori ai lui zero.
2.4.20 Corolar. Caracteristica unui domeniu de integritate sau corp este un numar prim sau innit.
Se verica usor urmatoarele armat ii
2.4.21 Teorema. Daca R este un inel comutativ cu unitate avand caracteristica un numar prim p,
atunci
(a +b)
p
= a
p
+b
p
pentru orice a, b R.
2.4.22 Teorema. Pentru orice inel cu unitate R exista un singur omomorsm de inele f : Z R,
f(m) = m1
R
. Daca charR = atunci f este injectiv, iar daca charR = n; n N

, atunci Kerf = nZ.


Demonstrat ie. Demonstram mai ntai unitatea lui f.

Intr-adevar daca f : Z R este omomorsm


unital de inele, atunci k N

avem:
f (k) = f (1 +... + 1) = f (1) +... +f (1) = 1
R
+... + 1
R
= k1
R
.
Deci f este denit de relat ia din enunt ul Teoremei 2.4.22. Se verica usor ca f este un omomorsm unital
de inele.
Daca caracteristica inelului este , atunci Kerf = 0
Z
deci f este injectiv. Atunci Z se scufunda
izomorf n inelul R de caracteristica innit.
Daca caracteristica inelului este n atunci
Kerf = m Z; k Z; m = kn = nZ.
Conform primei teoreme de izomorsm pentru inele avem
Z
n
= Z/
nZ
Imf R.
Prin urmare inelul R are un subinel izomorf cu Z
n
.
Am demonstrat astfel urmatoarele
2.4.23 Corolare. 1) Inelul ntregilor este cel mai mic inel cu unitate de caracteristica ;
2) Z
n
este cel mai mic inel cu unitate de caracteristica n.
Capitolul 3
Probleme de algebra
3.1 Enunt uri
Grupuri
3.1.1 a) Sa se arate ca (G, ), unde G = (1, 1) si operat ia * se deneste x y =
x +y
1 +xy
este un grup
abelian.
b) Aratat i ca funct ia f : (0, ) (1, 1) , f(x) =
x 1
x + 1
, () x (0, ) este un izomorsm de la
grupul (R

+
, ) la grupul (G, ).
3.1.2 Pentru a R

, b R denim funct ia f
a,b
: R R, , f
a,b
= ax+b. Notam A = f
a,b
[a R

; b R;
T = f
1,b
[b R si S = f
a,0
[a R

. Demonstrat i ca:
a) (A, ) este grup ind operat ia de compunere a funct iilor. Este (A, ) subgrup normal n grupul
(o
R
, ) al permutarilor lui R?
b) (T, ) este un subgrup abelian al lui A, izomorf cu (R, +).
c) (S, ) este un subgrup abelian al lui A, izomorf cu (R

, ). Este S subgrup normal n A?


d) T este un subgrup normal n A si A/
T
este izomorf cu (R

, ). Este T subgrup normal si n (o


R
, )?
3.1.3 Sa se arate ca Z(S
3
) = e unde e este permutarea identica din grupul permutarilor de ordinul
trei, (S
3
, ), iar Z(S
3
) este centrul grupului. Generalizare pentru S
n
, n N, n 3.
3.1.4 Aratat i ca orice subgrup al unui grup ciclic este ciclic.
3.1.5 Fie (G, ) un grup si x, y G astfel ca xy = yx, ordx = m si ord y = n. Sa se arate ca:
a) ord(xy) este nit si ord(xy) [ [m, n];
b) daca < x > < y >= e atunci ord(xy) = [m, n];
c) daca (m, n) = 1 atunci ord(xy) = m n si < x, y >=< x y >.
3.1.6 Fie G un grup si H, K subgrupuri ale lui G. Sa se arate ca H K este subgrup al lui G daca si
numai daca H K sau K H.
3.1.7 a) Daca G este un grup, sa se arate ca G nu se poate scrie ca reuniune a doua subgrupuri proprii
ale sale.
b) Sa se arate ca exista grupuri care se pot scrie ca reuniune a trei subgrupuri proprii.
3.1.8 Sa se precizeze ordinul ecarui element din grupul radacinilor de ordinul 6 ale unitat ii, (U
6
, ), si
sa se construiasca diagrama laticii subgrupurilor sale. Care dintre subgrupuri sunt subgrupuri normale?
36
3.1 Enunt uri 37
3.1.9 a) Fie f End(Z, +). Sa se arate ca f(n) = f(1) n, () n Z.
b) Fie f End(, +). Sa se arate ca f(r) = f(1) r , () r .
3.1.10 Sa se determine endomorsmele si automorsmele grupului (Z, +) si (, +).
3.1.11 Sa se determine grupurile cat ale lui (Z, +) si sa se arate ca acestea sunt ciclice.
3.1.12 Sa se determine subgrupurile grupului aditiv Z
n
.
3.1.13 Sa se determine subgrupurile si grupurile cat ale urmatoarelor grupuri:
(Z
4
, +); (Z
6
, +); (Z
12
, +); (Z
15
, +); (Z
24,+)
.
3.1.14 Fie H = z C[ [ z [ = 1. Sa se arate ca:
a) H este un subgrup al lui (C

, );
b) Grupul cat al lui (R, +) n raport cu Z este izomorf cu (H, ).
c) Grupul cat C

/
H
este izomorf cu (R

+
, ) si cu (R, +).
d) Grupul cat C

/
R

+
este izomorf cu (H, ).
3.1.15 Sa se arate ca urmatoarele grupuri nu sunt izomorfe:
a) (, +) si (R, +);
b) (

, ) si (R

, ).
3.1.16 Fie p un numar prim. Sa se arate ca orice grup de ordinul p este ciclic si este generat de orice
element al sau diferit de elementul neutru.
3.1.17 Sa se determine toate grupurile neizomorfe de ordinele 1, 2, 3, 4 si 5.
3.1.18 Fie H = 1, 1, i, i, j, j, k, k cu operat ia data n tabla urmatoare:
1 1 i i j j k k
1 1 1 i i j j k k
1 1 1 i i j j k k
i i i 1 1 k k j j
i i i 1 1 k k j j
j j j k k 1 1 i i
j j j k k 1 1 i i
k k k j j i i 1 1
k k k j j i i 1 1
a) Sa se arate ca (H, ) este un grup necomutativ numit grupul cuaternionilor;
b) Sa se precizeze ordinul ecarui element din grupul (H, );
c) Sa se determine subgrupurile lui (H, ) si sa se gureze subgrupul laticii subgrupurile lui (H, );
d) Care dintre subgrupurile lui (H, ) sunt subgrupuri normale?
3.1.19 Fie H = 1, 1, i, i, j, j, k, k grupul cuaternionilor. Sa se descrie clasele la stanga ale lui H
n raport cu echivalent a denita de
a) subgrupul generat de i;
b) subgrupul generat de j;
c) subgrupul generat de k;
d) subgrupul generat de 1;
e) subgrupul generat de (1);
f) subgrupul generat de i, j.
3.1.20 Sa se construiasca grupul cat al grupului 2
x
3
y
5
z
[x, y, z Z prin grupul 2
z
[z Z.
3.1.21 Fie (G, ) un grup, H si N subgrupuri ale lui G astfel ncat N sa e subgrup normal. Aratat i ca:
a) NH = HN si, mai mult, aceasta mult ime este un subgrup al lui G.
b) N H este subgrup normal al lui H.
38 3. Probleme de algebr a
c) exista un izomorsm de grupuri: H/
N H
NH/
N
.
3.1.22 Fie G un grup, H si N subgrupuri normale ale lui G astfel ncat H N. Aratat i ca N/
H
este
subgrup normal al lui G/
H
si exista un izomorsm de grupuri (G/
H
) / (N/
H
) G/
N
.
3.1.23 Fie K unul dintre corpurile , R, C si /
n
(K) mult imea matricilor patrate de ordinul n cu
elemente din K si
GL
n
(K) = A /
n
(K) [ det A ,= 0,
SL
n
(K) = A /
n
(K) [ det A = 1.
Sa se arate ca:
a) (GL
n
(K), ) este un grup, numit grupul liniar general de gradul n peste K (pentru n 1).
b) (GL
n
(K), ) are un subgrup izomorf cu (K

, ).
b) (GL
2
(R), ) are un subgrup izomorf cu (C

, ).
d) SL
n
(K) are un subgrup normal al lui (GL
n
(K), ).
e) Aratat i izomorsmul de grupuri multiplicativ: GL
n
(K)/
SL
n
(K)
K

.
3.1.24 Sa se determine ordinele elementelor
A =
_
1 0
0 1
_
; B =
_
1 1
0 1
_
; C =
_
1 0
0 i
_
n (GL
n
(C), ) si grupurile ciclice generate de acestea.
3.1.25 Fie (G, ) un grup si x, y G elemente permutabile, adica xy = yx. Fie m =ordx, n =ordy,
m, n N

. Sa se arate ca:
a) ord(xy) este nit si ord(xy) divide pe [m, n], unde [m, n] este c.m.m.m.c. al lui m si n.
b) daca < x > < y >= e atunci ord(xy) = [m, n].
c) daca (m, n) = 1 atunci ord(xy) = m n si < x, y >=< x y > .
3.1.26 Fie G = Z Z 1, 1 si operat ia denita n G astfel:
(m
1
, m
2
, m
3
) (n
1
, n
2
, n
3
) =
_
(m
1
+n
1
, m
2
+n
2
, n
3
), daca m
3
= 1
(m
1
+n
2
, m
2
+n
1
, n
3
), daca m
3
= 1
Sa se arate ca
1) (G, ) este un grup;
2) Subgrupul H
1
=< (1, 0, 1), (0, 1, 1) > este normal n G;
3) Subgrupul H
2
=< (1, 0, 1) > este normal n H, dar nu este normal n G, adica relat ia nu este
tranzitiva.
3.1.27 Fie n Z si nZ = nk [ k Z. Sa se arate ca:
a) nZ mZ daca si numai daca m[n (m divide pe n);
b) mZ nZ = [m, n]Z si < mZ nZ >= mZ +nZ = (m, n)Z, unde (m, n) si [m, n] sunt c.m.m.d.c.
si respectiv c.m.m.m.c. al lui m si n.
3.1.28 Sa se dea cate un exemplu de ciclu (permutare circulara) de lungime 4, de transpozit ie si de
permutare care nu este ciclu.
3.1.29 a) Sa se descompuna n produs de cicluri disjuncte permutarea
=
_
1 2 3 4 5 6 7 8 9 10 11 12
9 12 8 11 6 7 5 3 2 4 10 1
_
.
b) Sa se scrie n doua moduri ca produs de transpozit ii permutarea
=
_
1 2 3 4 5 6 7 8
2 3 1 5 6 8 7 4
_
.
3.1 Enunt uri 39
3.1.30 Fie =
1

2
. . .
k
descompunerea n cicluri disjunte a permutarii S
n
. Sa se arate ca
ord coincide cu cel mai mic multiplu comun al ordinelor ciclurilor
i
(i = 1, 2, . . . , k).
3.1.31 Aat i permutarea
100
stiind ca:
a) =
_
1 2 3 4 5 6 7 8 9 10
4 6 2 7 8 3 10 9 5 1
_
;
b) =
_
1 2 3 4 5 6 7 8 9 10 11 12
3 4 6 5 9 1 12 8 10 2 11 7
_
.
3.1.32 Fie
1
= (1, 2, 3) (4, 5, 6, 8),
2
= (3, 4) (5, 2, 6, 1, 8),
3
= (1, 3, 4) (2, 3, 5, 7) (1, 8, 4, 6),

4
= (8, 2, 1, 4, 3) (1, 2) (1, 5) si
5
= (8, 7, 4, 3, 1, 2) (5, 6) permutari din S
8
. Sa se gaseasca
3
1
,
2
2

1
,

3

4

5
,
4
3

2
4
si
5

4

3
.
3.1.33 a) Fie = (i
1
, i
2
, . . . , i
k
) S
n
. Sa se arate ca pentru orice S
n
avem
1
=
((i
1
), (i
2
), . . . , (i
k
)).
b) Fie
1
= (1, 2, 3) (4, 5, 6, 8),
2
= (3, 4) (5, 2, 6, 1, 8),
3
= (1, 3, 4) (2, 3, 5, 7) (1, 8, 4, 6),

4
= (8, 2, 1, 4, 3)(1, 2)(1, 5),
5
= (8, 7, 4, 3, 1, 2)(5, 6) permutari din S
8
. Sa se determine:
1

1
1
,

2
5

3

2
5
,
5
2

4

5
2
.
c) Fie
1
= (1, 2, 3) (4, 5, 6) (7, 8, 9),
2
= (1, 4, 7) (2, 5, 8) (3, 6, 9),
3
= (4, 5, 6) (7, 8, 9). Sa
se arate ca
1

2
=
2

1
si
1

3
=
3

1
.
3.1.34 a) Sa se precizeze ordinul ecarui element al grupului S
3
.
b) Sa se determine toate subgrupurile lui (S
3
, ). Care dintre ele sunt normale?
c) Sa se construiasca diagrama laticii subgrupurilor sale.
d) Sa se determine grupurile cat ale lui (S
3
, ).
3.1.35 Aratat i ca subgrupul generat de permutarile (1, 2) (3, 4) si (1, 3) (2, 4) din S
4
este izomorf cu
grupul lui Klein.
Inele
3.1.36 Pe mult imea Z denim operat iile
x y = x +y +a
x y = (x +a)(y +a) a,
unde a Z este un numar xat.
a) Sa se arate ca tripletul (Z, , ) este un domeniu de integritate;
b) Sa se determine grupul elementelor inversabile din acel inel.
3.1.37 Fie * operat ia denita pe C astfel
(a
1
+b
1
i) (a
2
+b
2
i) = a
1
a
2
+ (a
1
b
2
+a
2
b
1
)i , a
1
, a
2
, b
1
, b
2
R.
Sa se arate ca (C, +, ) este un inel comutativ cu unitate care are divizori ai lui zero.
3.1.38 Fie A un inel n care orice element este idempotent, adica x
2
= x, pentru orice x A (un
asemenea inel se numeste inel boolean). Sa se arate ca
a) x +x = 0 , () x A;
b) A este comutativ.
c) Daca A ,= 0 este inel boolean fara divizori a lui zero, atunci el este izomorf cu (Z
2
, +, ) .
3.1.39 Pe mult imea A = Z Z denim legile de compozit ie
(a
1
, b
1
) + (a
2
, b
2
) = (a
1
+a
2
, b
1
+b
2
)
(a
1
, b
1
) (a
2
, b
2
) = (a
1
a
2
+ 3b
1
b
2
, a
1
a
2
+a
2
b
1
).
40 3. Probleme de algebr a
Aratat i ca aceste legi de compozit ie confera mult imii A o structura de inel comutativ cu unitate si fara
divizori ai lui zero.
3.1.40 Fie + si operat iile denite pe astfel:
(a
1
, b
1
) + (a
2
, b
2
) = (a
1
+a
2
, b
1
+b
2
)
(a
1
, b
1
) (a
2
, b
2
) = (a
1
a
2
b
1
b
2
, a
1
b
2
+a
2
b
1
+b
1
b
2
).
a) Sa se arate ca (, +, ) este un corp comutativ.
b) Aratat i ca funct ia f : (

3) K, f(z) = (a b, 2b) , z (

3), z = a +b

3 cu a, b
este un izomorsm de corpuri.
3.1.41 Sa se arate ca ntr-un inel cu unitate, comutativitatea adunarii este o consecint a a celorlalte
axiome.
3.1.42 Sa se dea exemplu de inel nit necomutativ. Exista corpuri nite necomutative?
3.1.43 Fie (R, +, ) un inel cu a, b R. S a se arate ca:
a) (a +b)
2
= a
2
+ 2ab +b
2
a b = b a a
2
b
2
= (a b)(a +b).
b) Daca ab = ba atunci pentru orice n N

avem
(a +b)
n
= C
0
n
a
n
+C
1
n
a
n1
b +. . . +C
n1
n
ab
n1
+C
n
n
b
n
. (*)
a
n
b
n
= (a b)(a
n1
+a
n2
b +. . . a b
n2
+b
n1
).
a
2n+1
+b
2n+1
= (a +b)(a
2n
a
2n1
b +. . . a b
2n1
+b
2n
).
3.1.44 Sa se arate ca daca ntr-un inel (R, +, ) avem x
6
= x pentru orice x R, atunci x
2
= x pentru
orice x R.
3.1.45 Sa se rezolve n Z
12
ecuat iile:

4x +

5 =

9 ;

5x +

5 =

9 si n /
2
(C) ecuat ia
_
1 2
1 2
_
X =
_
1 2
1 2
_
.
3.1.46

In Z
12
sa se determine divizorii lui zero si sa se rezolve sistemul
_

3x +

4y =

11

4x +

9y =

10
.
3.1.47 Fie Z[

2] =
_
a +b

2 [ a, b Z
_
si (

2) =
_
a +b

2 [ a, b
_
, adica Z[

2] = Z + Z

2 si
(

2) = +

2. Sa se arate ca:
i) Z[

2] este un subinel al lui (R, +, ) care cont ine pe 1 si acest subinel este generat de 1,

2.
ii)
_
2
_
este un subcorp al lui (R, +, ) si acest corp este generat de

2.
iii) S
1
=
_
a +b
3

2 [ a, b Z
_
nu este subinel al lui (R, +, ).
iv) S
2
=
_
a +b
3

2 [ a, b
_
nu este subcorp al lui (R, +, ).
3.1.48 Fie d Z un ntreg liber de patrate (adica d ,= 1 si d nu se divide prin patratul nici unui
numar prim) si funct ia : Z[

d] N, (z) = [z z[ unde cu z = a b

d s-a notat conjugatul lui


z = a +b

d Z[

d]. Sa se arate ca z este inversabil n Z[

d] daca si numai daca (z) = 1.


3.1.49 Sa se arate ca inelul Z[

2] are o innitate de elemente inversabile.


3.1.50 a) Sa se arate ca
R =
__
a b
2b a
_

a, b Z
_
este subinel al lui (/
2
(Z), +, ) si ca R este domeniu de integritate n raport cu operat iile induse.
3.2 Solut ii 41
b) Sa se arate ca inelul Z[

2] este izomorf cu inelul R.


3.1.51 Sa se arate ca (Z, +, ) si inelul (Z, , ) din problema 3.1.30 pentru a = 3 sunt izomorfe.
3.1.52 Fie A un inel cu element unitate cu 0 ,= 1, n care x
2
= 1, pentru orice x A0. Sa se arate ca
A este izomorf cu unul din corpurile Z
2
sau Z
3
.
3.1.53 Daca f : C este un morsm de corpuri, atunci f(x) = x, () x , adica singurul
omomorsm nenul de corpuri de la (, +, ) la (C, +, ) este omomorsmul de incluziune. Determinat i
apoi automorsmele corpului (

2).
3.1.54 Sa se determine endomorsmele si automorsmele inelelor (Z, +, ) si (, +, ).
3.1.55 Fie R un inel comutativ si unitar. Sa se arate ca mult imea N(R) = a R[ () m Z

+
astfel
ncat a
m
= 0 formeaza un ideal al lui R. Este esent iala comutativitatea?
3.1.56 Fie (R, +, ) un inel si e A, B doua ideale ale lui R.
a) Fie A : B = r R[ () b B, r b A. Sa se arate ca A : B este un ideal bilateral al lui R.
Idealul A : B se numeste catul perechii (A, B) .
b)

In (Z, +, ) avem nZ : mZ =
[m, n]
m
Z(m, n N, m ,= 0).
3.1.57 Fie m si n numere ntregi pozitive astfel ncat (m, n) = 1. Aratat i ca Z
mn
si Z
m
Z
n
sunt
izomorfe.
3.1.58 Aratat i ca daca I si J sunt doua ideale ale unui inel (R, +, ) cu proprietatea ca I +J = R atunci
R/
IJ
este omomorf cu R/
I
R/
J
.
3.1.59 Considerand inelul polinoamelor cu coecient i reali n nedeterminata X, R[X] si aplicat ia f :
R[X] R denita prin f(P) = P(1) , P R[X], aratat i ca R[X]/
(X1)
R unde (X 1) este idealul
principal generat de polinomul X 1.
3.1.60 Determinat i idealele si inelele cat ale inelului (Z, +, ).
3.1.61 Fie r Z
n
si n > 1. Aratat i ca urmatoarele armat ii sunt echivalente:
a) clasa r nu este divizor al lui zero n Z
n
.
b) r este un element inversabil n Z
n
.
c) numerele r si n sunt relativ prime.
3.1.62 Determinat i idealele lui Z
21
inelul claselor de convergent a modulo 21 si inelele cat n raport cu
aceste ideale.
3.1.63 Fie M o mult ime nevida si T(M) mult imea part ilor lui M. Tripletul (T(M), , ) este un inel cu
unitate n raport cu diferent a simetrica respectiv intersect ia submult imilor, izomorf cu inelul (Z
M
2
, +, )
al funct iilor denite pe M cu valori n inelul claselor de resturi modulo 2.
3.2 Solut ii
Grupuri
3.2.1 a) Vom verica axiomele grupului aratand mai ntai ca pentru orice x, y (1, 1) avem
x y (1, 1). Fie x, y G. Avem
1 < x < 1 [ x[ < 1 x
2
< 1 x
2
1 < 0
1 < y < 1 [ y[ < 1 y
2
< 1 y
2
1 < 0
_
(x
2
1)(y
2
1) > 0.
Prin urmare de aici rezulta
x
2
y
2
x
2
y
2
+ 1 > 0 x
2
y
2
+ 1 > x
2
+y
2
x
2
y
2
+ 2xy + 1 > x
2
+ 2xy +y
2

42 3. Probleme de algebr a

(xy + 1)
2
> (x +y)
2
x, y G xy ,= 1 xy + 1 ,= 0

_
x +y
1 +xy
_
2
< 1

x +y
1 +xy

< 1
x y G, () x, y G.
Asociativitatea: (x y) z = x (y z) , () x, y, z G.
(x y) z =
x +y
1 +xy
z =
x +y
1 +xy
+z
1 +
x +y
1 +xy
z
=
x +y +z +xyz
1 +xy +xz +yz
x (y z) = x
y +z
1 +xy
=
x +
y +z
1 +yz
1 +x
y +z
1 +yz
=
x +xyz +y +z
1 +yz +xy +xz
=
x +y +z +xyz
1 +xy +xz +yz

(x y) z = x (y z) , () x, y, z G este asociativa.
Comutativitatea: () x, y G x y = y x.
x y =
x +y
1 +xy
=
y +x
1 +xy
= y x, () x, y G este comutativa.
Element neutru: Demonstram ca () e G astfel ncat e x = x e = x, () x G. Ecuat ia
x e = x
x +e
1 +xe
= x x +e = x +x
2
e , () x G.
x
2
e e = 0
e(x
2
1) = 0
x
2
,= 1

e = 0 G e = 0 element neutru.
Element simetrizabil. Aratam ca () x G, () x
t
G astfel ncat x x
t
= x
t
x = e.
x x
t
= 0
x +x
t
1 +x x
t
= 0 x
t
= x.
Din () x G 1 < x < 1 1 > x > 1 x (1, 1) x
t
(1, 1) = G.

In concluzie (G, ) este grup abelian.


b) Pentru a demonstra izomorsmul cerut vericam ca f este un omomorsm bijectiv.
Injectivitatea:
f(x) = f(y)
x 1
x + 1
=
y 1
y + 1
xy +x y 1 = xy x +y 1.
De aici avem 2x = 2y x = y, deci f este injectiva. Fie y = f(x); y (1, 1). Avem
y =
x 1
x + 1
xy +y = x 1 x(1 y) = y + 1 x =
1 +y
1 y
(0, ).
Am aratat ca () y (1, 1) , () x (0, ) astfel ncat f(x) = y, deci aplicat ia f este surjectiva.

In
concluzie f este bijectiva.
Omomorsm: f(x y) = f(x) f(y) , () x, y (0, )
f(x y) =
xy 1
xy + 1
. (1)
f(x) f(y) =
x 1
x + 1

y 1
y + 1
=
x 1
x + 1
+
y 1
y + 1
1 +
x 1
x + 1

y 1
y + 1
=
xy +y y 1 +xy x +y 1
xy +x +y + 1 +xy x y + 1
=
3.2 Solut ii 43
=
2xy 2
2xy + 2
=
xy 1
xy + 1
. (2)
Din (1) si (2) rezulta f(x y) = f(x) f(y). Prin urmare avem (R

+
, ) (G, ).
3.2.2 a) Reamintim ca prin permutari ale unei mult imi nevide M nt elegem elementele inversabile ale
monoidului (M
M
, ), adica aplicat iile bijective denite pe M cu valori n M. Cum compusa a doua
biject ii este tot o biject ie, rezulta ca (o
M
, ) este grup.

In particular, (o
R
, ) este grup si dac a a R

rezulta ca f
a,b
o
R
. Avem A ,= si
(f
a,b
f
c,d
)(x) = f
a,b
(cx +d) = acx +ad +b = f
ac,ad+b
(x) () x R.
De aici
f
a,b
f
c,d
= f
ac,ad+b
A (a ,= 0, c ,= 0 si ac ,= 0). (*)
Daca ax +b = y , a ,= 0 x =
1
a
y
b
a
atunci f
1
a,b
(x) =
1
a
x
b
a
, () x R, deci
f
1
a,b
= f1
a
,
b
a
. (**)
Din (*) si (**) rezulta ca A este subgrup al lui (o
R
, ).

In particular, (A, ) este grup.
Fie g : R R, g(x) = x
3
, g o
R
Ea are inversa g
1
(x) =
3

x, () x R. Pentru ca A sa e
subgrup normal n grupul (o
R
, ) ar trebui ca
g
1
f
a,b
g A, () f
a,b
A.
Avem
(g
1
f
a,b
g)(x) = g
1
(f
a,b
(g(x))) = g
1
(f
a,b
(x
3
)) = g
1
(ax
3
+b) =
3
_
ax
3
+b / A,
pentru b ,= 0. Deci A nu este subgrup normal n o
R
.
b) Conform punctului a) avem f
1,b
f
1,c
= f
1,c+b
= f
1,b+c
= f
1,c
f
1,b
, f
1
1,b
= f
1,b
ceea ce ne arata
ca T este subgrup abelian al lui (A, ).
Fie F : T R, F(f
1,b
) = b. Aplict ia F este un omomorsm bijectiv al lui (T, ) pe (R, +).

Intr-
adevar () f
1,b
; f
1,c
T, avem F(f
1,b
f
1,c
) = F(f
1,c+b
) = c + b = b + c = F(f
1,b
) + F(f
1,c
), de unde
rezulta ca F este omomorsm de grupuri.
Bijectivitatea. Daca F(f
1,b
) = F(f
1,c
) b = c f
1,b
= f
1,c
F este injectiva. Surjectivitatea
rezulta imediat.
c) Conform punctului a) avem: f
a,0
f
c,0
= f
ac,0
S si f
1
a,0
= f1
a
, 0
S ceea ce arata ca S este
subgrup al lui (A, ). Mai mult,
(f
a,0
f
c,0
)(x) = f
ac,0
(x) = acx = cax = f
ca,0
(x) = (f
c,0
f
a,0
)(x), () x R.
f
a,0
f
c,0
= f
c,0
f
a,0
, () f
a,0
; f
c,0
S.
Rezulta ca S este subgrup abelian al lui A.
Totodata () f
e,d
A, () f
a,0
S : f
1
c,d
f
a,0
f
c,d
= f1
c
,
d
c
f
ac,ad
= f
a,
ad
c

d
c
/ S daca d ,= 0, deci
S nu este subgrup normal n (A, ).
Izomorsmul cerut se arata foarte usor.
d) S-a aratat la punctul b) ca (T, ) este subgrup abelian al lui A. Mai trebuie aratat ca este subgrup
normal al lui A.

Intr-adevar
() f
c,d
A si () f
1,b
T avem
f
1
c,d
f
1,b
f
c,d
= f1
c
,
d
c
f
1,b
f
c,d
= f1
c
,
d
c
f
c,d+b
= f
1,
d+b
c
,
d
c
= f
1,
b
c
T.
Deci subgrupul T este normal n (A, ).
Aplict ia : A R

, (f
a,b
) = a este un omomorsm surjectiv al lui (A, ) pe (R

, ).

Intr-adevar (f
a,b
f
c,d
) = (f
ac,ad+b
) = ac = (f
a,b
) (f
c,d
), deci este un omomorsm de
grupuri.
44 3. Probleme de algebr a
Surjectivitatea este imediata.
Conform primei teoreme de izomorsm avem A/
Ker
R

.
Ker = f
a,b
A[ (f
a,b
) = 1 = f
a,b
A[ a = 1 = f
1,b
[ b R = T.

In concluzie A/
T
(R

, ).
Din considerente similare cu cele de la punctul a) rezulta ca T nu este subgrup normal n (o
R
, ).
3.2.3 Prin denit ie centrul grupului este Z(S
3
) = S
3
[ = , () S
3
. Evident
e Z(S
3
). Fie Z(S
3
) , ,= e. Atunci exista i, j 1, 2, 3 astfel ncat i ,= j si (i) = j. Fie
k 1, 2, 3 , k ,= i, k ,= j. Fie permutarea S
3
denita astfel (i) = i , (j) = k si (k) = j.
Deoarece Z(S
3
) avem ((i)) = ((i)) (i) = (j) j = k. Fals! Rezulta Z(S
3
) = e. Analog
se arata ca Z(S
n
) = e, () n N

, n 3.
3.2.4 Fie G =< a > grup ciclic generat de a, ord a = +, deci G < a >= a
k
[ k Z. Deorece G Z
si subgrupurile lui Z sunt de forma nZ =< n >, deci ciclice rezulta ca subgrupurile lui G sunt ciclice.
Fie G =< a > grup ciclic nit, ord a = n si e H un subgrup al lui G.
Daca H = e atunci evident H este ciclic, H =< e >.
Daca H ,= e atunci exista x H, x ,= e. Dar x G, adica x = a
k
, k ,= 0.
x H
H G

x
1
H (a
k
)
1
= a
k
H () r > 0 astfel ncat a
r
H.
Este sucient sa luam r =
_
k pentru k > 0
k pentru k < 0
.
Mult imea numerelor naturale M = n[ a
n
H, n > 0 este nevida si cum mult imea N a numerelor
naturale este bine ordonata rezulta ca M are cel mai mic element m. Vom arata ca H =< a
m
>.
Fie x < a
m
>, deci x = (a
m
)
k
, k Z. Dar H este subgrup al lui G si a
m
H. Rezulta ca
x = (a
m
)
k
H, adica
< a
m
> H. ()
Fie y H y G adica y = a
t
, [ t[ m, t Z.
Dupa teorema mpart irii cu rest pentru numere ntregi avem t = mq +r , unde q, r Z, 0 r < m.
Deci
y = a
t
= a
mq+r
= (a
m
)
q
a
r
a
r
= (a
m
)
q
y H
Observat ie. Daca G =< a >, [G[ = n atunci G Z
n
, izomorsmul ind realizat de aplicat ia
f : Z
n
G, f <

k > a
k
. Avem
Z
n
=<

k > (k, n) = 1.
Rezulta ca
< a
k
>= G (k, n) = 1.
Deoarece m este cel mai mic numar al lui M rezulta ca r = 0. Deci t = mq si
y = a
mq
= (a
m
)
q
< a
m
>
H < a
m
> ()
Din () si () rezulta ca H =< a
m
> .
3.2.5 a) Fie k = [m, n] si d = (m, n). Daca m = dk
1
, n = dk
2
, k
1
, k
2
N, (k
1
, k
2
) = 1, atunci
k = k
1
k
2
d = mk
2
= k
1
m. Deorece xy = yx avem
(xy)
k
= x
k
y
k
= (x
m
)
k
2
(y
n
)
k
1
= e e = e ord(x, y) [ k.
Rezulta ca ord (xy) este nit si ord (xy) [ [m, n].
b) Daca ord (xy) = a, din faptul ca xy = yx rezulta x
a
y
a
= (xy)
a
= e ceea ce implica x
a
= y
a
.
Dar x
a
< x > si y
a
< y >, de unde x
a
= y
a
< x > < y > .
3.2 Solut ii 45
Deoarece
< x > < y >< y >ord (< x > < y >) [ m
< x > < y >< y >ord (< x > < y >) [ n

rezulta ca ord(< x > < y >) = 1.


Daca
< x > < y >= e x
a
= y
a
= e
m[ a
n[ a

[m, n] [ a,
ceea ce mpreuna cu punctul a) conduce la a = [m, n].
c) Din teorema lui Lagrange avem[< x > < y >[ divide pe m = [< x >[ si pe n = [< y >[ . Dar cum
(m, n) = 1 avem [< x > < y >[ = 1.

In consecint a < x > < y >= e. Asadar ord(xy) [m, n] = mn.


Avem ca < xy >< x, y > . Deoarece (m, n) = 1 rezulta ca exista u, v Z. Astfel ncat um + vn = 1.
Deci
x = x
um+vn
= (x
m
)
n
x
vn
= x
vn
= x
vn
(y
n
)
v
= (xy)
vn
si analog, y = (xy)
um
. Deci x, y < x, y >, ceea ce implica < x, y >< xy > .

In concluzie, daca (m, n) = 1 avem < x, y >=< x, y > .


3.2.6 Presupunem ca H K este subgrup al lui G, H _ K si K _ H. Rezulta ca exista h HK si
exista k KH. Rezulta h, k H K. Dar H K este subgrup al lui G si deci
h k H K h k H h k K k H h K.Fals!
(k = h
1
(h k) H h = (h k) k
1
K).

In concluzie, H K K H.
Implicat ia inversa este evidenta.
3.2.7 a) Presupunem ca exista H, K subgrupuri proprii ale lui G astfel ncat H K = G, H ,= G,
K ,= G. Daca H K K K G = H G = K. Fals! Rezulta ca H _ K si K _ H, adica exista
x, y G astfel ncat x H K si y K H, xy G = KH. Avem xy Kxy H. Presupunem ca
xy H, x H de unde y = x
1
(x y) H. Fals! Analog xy K, y K de unde x = (x y) y
1
K.
Fals!

In concluzie, nu exista H, K subgrupuri proprii ale lui G astfel ncat H K = G.


b) Fie (K, ) grupul lui Klein K = e, a, b, c, a
2
= b
2
= c
2
= e. K
1
= e, a, K
2
= e, b, K
3
= e, c.
K
1
, K
2
, K
3
sunt subgrupuri ale lui K si K = K
1
K
2
K
3
.
3.2.8 Avem U
6
= z C[ z
6
= 1 adica U
6
este mult imea radacinilor de ordinul 6 al unitat ii
z
6
= 1 z
k
= cos
2k
6
+i sin
2k
6
, k = 0, 5.
U
6
= (z
0
= 1, z
1
, z
2
, z
3
, z
4
, z
5
.
Avem z
k
= z
k
1
, k = 0, 5, U
6
=< z
1
>=< z
5
1
> . Pentru () k, l 0, 1, 2, 3, 4, 5 avem z
k
z
l
= z
r
, unde r
este restul mpart irii lui k +l la 6, deci tabla operat iei este urmatoarea
1 z
1
z
2
z
3
z
4
z
5
1 1 z
1
z
2
z
3
z
4
z
5
z
1
z
1
z
2
z
3
z
4
z
5
1
z
2
z
2
z
3
z
4
z
5
1 z
1
z
3
z
3
z
4
z
5
1 z
1
z
2
z
4
z
4
z
5
1 z
1
z
2
z
3
z
5
z
5
1 z
1
z
2
z
3
z
4
ord z
0
=ord1 = 1; ord z
1
=ord z
5
= 6
ord z
2
=ord z
4
= 3; (z
2
z
2
= z
4
si z
3
2
= z
4
z
2
= 1)
ord z
3
= 2.
Deoarece orice subgrup al unui grup ciclic este ciclic, subgrupurile lui U
6
sunt ciclice si sunt date de
< z
k
>, k = 0, 1, 2, 3, 4, 5.
46 3. Probleme de algebr a
Avem:
H
1
=< z
0
>=< 1 >= 1
H
2
=< z
1
>=< z
5
>= U
6
H
3
=< z
2
>= 1, z
2
, z
2
2
= z
4
=< z
4
>
H
4
=< z
3
>= 1, z
3
= 1, z
3
1
.
Laticea
S(U
6
) = 1, 1, z
3
, 1, z
2
, z
4
, U
6

a subgrupurilor lui U
6
are diagrama:
H
2
= U
6
,
H
3
H
4
,
H
1
3.2.9 a) Daca f End(Z, +) rezulta ca oricare ar n N avem:
f(n) = f(1 + 1 +... + 1) = f(1) +f(1) +. . . f(1) = n f(1)
f(0) = 0 = 0 f(1),
nsumarile facandu-se de nori.
Fie n Z, n < 0. Atunci n > 0 si ntrucat f este endomorsm avem
f(n) = f((n)) = f(n) = (n) f(1) = nf(1).
De aici avem f(n) = n f(1) , () n Z.
b) T inand seama de punctul a) avem f(0) = 0 f(1). Mai mult, pentru orice numar rat ional, e
acesta r

, r =
m
n
, m, n Z, n ,= 0 , m > 0, scriind numarul ca o suma de m termeni egali cu
1
n
,
din faptul ca f End(, +) avem
f(r) = f
_
m
n
_
= f
_
1
n
+
1
n
+. . . +
1
n
_
= m f
_
1
n
_
() k N

, f(1) = f
_
k
k
_
= k f
_
1
k
_
f
_
1
k
_
=
1
k
f(1)
Pentru k Z, k < 0, f
_
1
k
_
= f
_

1
k
__
= f
_

1
k
_
=
_

1
k
_
f(1) =
1
k
f(1)
f
_
1
k
_
=
1
k
f(1) () k Z

f(r) = m f
_
1
n
_
=
m
n
f(1) = r f(1) , () r .
3.2.10 Conform problemei 3.2.9 am vazut ca daca f End(Z, +) atunci
f(x) = x f(1) () x Z,
adica f este o translat ie a lui (Z, ) si deci f = t
f(1)
. () a Z, (Z, +) grup t
a
: Z Z, t
a
(x) = ax
translat ia lui (Z, )
t
a
(x +y) = (x +y)a = xa +ya = t
a
(x) +t
a
(y) t
a
End(Z, +).
Rezulta ca End(Z, +) = t
a
[ a Z.
Analog se arata ca endomorsmele grupului (, +) coincid cu translat iile lui (, ).
End(, +) = t
a
[ a ;
Aut(Z, +) = t
1
.t
1
(Z
2
, +);
Aut(, +) = t
a
[ a

.
3.2 Solut ii 47
3.2.11 Z =< 1 >=< 1 >. Rezulta ca oricare subgrup al sau este ciclic. H = nZ =< n >=< n > .
x
nZ
y x +y nZ y x nZ y x +nZ.
Mult imea cat Z/

H
= Z/

nZ
not
= Z
n
; [Z : nZ[ = n. Subgrupul nZ are n Z indicele n si 0, 1, 2, . . . n 1
este un sistem de reprezentant i pentru Z
n
.

Intr-adevar, 0 i < j n 1 j i / nZ i + nZ ,= j + nZ numerele 0, 1, . . . , n 1 sunt


situate n clase diferite. Daca k Z si [ k[ > n () q, r Z astfel ncat k = nq +r , 0 r < n, k +nZ =
nq +r +nZ = r +nZ. Prin urmare 0, 1, 2, . . . , n 1 este un sistem de reprezentant i pentru Z
n
, adica
Z
n
= nZ, 1 +nZ, 2 +nZ, . . . , (n 1) +nZ =

0,

1,

2, . . . ,

n 1, i +nZ
not.
=

i.

In grupul cat (Z
n
, +) operat ia + este denia astfel
(i +nZ +j +nZ) = (i +j) +nZ sau

i +

j =

i +j.
Elementul nul al acesteui grup este nZ =

0.
Daca i Z atunci opusa clasei i +nZ este i +nZ.
Grupul (Z
n
, +) se numeste grupul aditiv al ntregilor modulo n; Z
n
=< 1 +nZ >=<

1 > .
3.2.12

Intrucat Z
n
= Z/
nZ
rezulta ca subgrupurile lui Z
n
sunt de forma H/
nZ
unde H este subgrup al
lui (Z, +) cu proprietatea Ker f = nZ H = dZ d[n.

In concluzie subgrupurile lui Z
n
sunt generate
de clasa d unde d[n.
3.2.13 Fie (Z
12
, +) , d
12
= 1, 2, 3, 4, 6, 12. Subgrupurile lui (Z
12
, +) sunt:
H
1
= 1Z/
12Z
= Z/
12Z
= Z
12
=<

1 >=<

5 >=<

7 >=<

11 >
H
2
= 2Z/
12Z
=

2 Z
12
=<

2 >=

0,

2,

4,

6,

8,

10
H
3
= 3Z/
12Z
=

3 Z
12
=<

3 >=

0,

3,

6,

9
H
4
= 4Z/
12Z
=

4 Z
12
=<

4 >=

0,

4,

8
H
5
= 6Z/
12Z
=

6 Z
12
=

0,

6
H
6
= 12Z/
12Z
=

0 Z
12
=<

0 >=

0
Grupurile cat au urmatoarele mult imi suport:
Z
12
/H
1
= Z
12
/Z
12
= Z
12
;
Z
12
/H
2
= a +H
2
[a Z
12
= H
2
,

1 +H
2
=

0,

2,

4,

6,

8,

10,

1,

3,

5,

7,

9,

11 Z
2
Z
12
/H
3
= a +H
3
[a Z
12
= H
3
,

1 +H
3
,

2 +H
3
=
=

0,

3,

6,

9,

1,

4,

7,

10,

2,

5,

8,

11 Z
3
Z
12
/H
4
= a +H
4
[a Z
12
= H
4
,

1 +H
4
,

2 +H
4
,

3 +H
4
=
=

0,

4,

8,

1,

5,

9,

2,

6,

10,

3,

7,

11 Z
4
Z
12
/H
5
= a +H
5
[a Z
12
= H
5
,

1 +H
5
,

2 +H
5
,

3 +H
5
,

4 +H
5
,

5 +H
5
=
=

0,

6,

1,

7,

2,

8,

3,

9,

4,

10,

5,

11 Z
6
Z
12
/H
6
=

i[

i Z
12
=

0,

1,

2,

3,

4,

5,

6,

7,

8,

9,

10,

11 Z
12
.
3.2.14 a) Mult imea este nevida H ,= deoarece 1 H. Totodata () z
1
, z
2
H [ z
1
[ = 1 ; [ z
2
[ = 1,
[ z
1
z
2
[ = [ z
1
[ [ z
2
[ = 1 1 = 1 z
1
z
2
H. Mai mult oricare ar z H si z
1
C

, z z
1
= 1 =

z z
1

= 1 [ z[

z
1

= 1 1

z
1

= 1. Deci

z
1

= 1 z
1
H, adica H este subgrup al lui
(C

, ).
b) Fie f : R C

, f(x) = cos 2x +i sin2x. Deoarece


f(x
1
) f(x
2
) = (cos 2x
1
+i sin2x
1
) (cos 2x
2
+i sin2x
2
) =
= (cos 2x
1
cos 2x
2
sin2x
1
sin2x
2
) +i(cos 2x
1
sin2x
2
+ sin2x
1
cos 2x
2
) =
= cos 2(x
1
+x
2
) +i sin2(x
1
+x
2
) = f(x
1
+x
2
), () x
1
, x
2
R
48 3. Probleme de algebr a
funct ia este omomorsm al lui (R, +) n (C

, ).
Ker f = x R[ f(x) = 1 = x R[ cos 2x +i sin2x = 1
= x R[ cos 2x = 1 si sin2x = 0 = Z.
Avem f(R) = H. Conform primei teoreme de izomorsm rezulta ca R/
Ker f
f(R) adica R/
Z
H.
c) Aplicat ia g : C

+
, g(z) = [ z [ este omomorsm surjectiv al lui (C

, ) n (R

+
, ).

Intr-adevar
g (z
1
z
2
) = [z
1
z
2
[ = [z
1
[ [z
2
[ = g (z
1
) g (z
2
) , () x
1
, x
2
C

,
iar
Ker g = z C

[ g(z) = 1 = z C

[ [ z [ = 1 = H.
Se aplica prima teorema de izomorsm si se obt ine C

/H R

+
.
Aplicat ia g
t
: R

+
R, g
t
(x) = lnx este un izomorsm al lui (R

+
, ) pe (R, +).
d) Aplicat ia h : C

, h(z) =
z
[ z [
este un endomorsm al lui (C

, ) , h(C

) = H.
Ker h = z C

[ h(z) = 1 =
_
z C

z
[ z[
= 1
_
= z C

[ z = [ z[ = R

+
.
Aplicand prima teorema de izomorsm rezulta C

/
R

+
H.
3.2.15 a) Presupunem ca exista un izomorsm f : R. Atunci () x, y

astfel ncat f(x) = 1 si


f(y) =

3. Daca m, n Z astfel ncat mx = ny avem f(mx) = f(ny) de unde conform rezultatului din
Problema 3.2.10 avem mf(x) = nf(y) de unde m = n

3 =
m
n
. Fals!
b) Presupunem ca exista un izomorsm f :

. Atunci exista y R

astfel ca y
3
= f(5).
Deoarece f este surjectiva, exista x

astfel ncat f(x) = y. Deoarece f este omomorsm avem


y
3
= f(x
3
) de unde f(x
3
) = f(5). Conform injectivitat ii lui f de aici avem x
3
= 5
3

5 . Fals!
3.2.16 Fie (G, ) un grup de ordin p si x G, x ,= e unde eelemetul neutru al lui G. Din teorema lui
Lagrange rezulta ca ordx = p < x >= G.
3.2.17 Daca G = e este o mult ime cu un singur element atunci pe G se poate deni o singura operat ie
binara
e
e e
si G n raport cu aceasta operat ie este grup. Abstract ie facand de un izomorsm, exista un singur grup
de ordinul 1. Daca G este un grup de ordinul 2, 3 sau 5 rezulta conform problemei anterioare ca este
ciclic si deci izomorf cu (Z
2
, +), (Z
3
, +) sau (Z
5
, +).
Fie un grup G de ordinul 4. Daca G este ciclic atunci este izomorf cu (Z
4
, +). Presupunem ca exista
un grup de ordin 4 neciclic. Fie G = e, a, b, c, e elementul sau neutru si grupul (G, ) neciclic. Din
ipoteza ca G nu este ciclic urmeaza ca nici un element din G nu are ordinul 4.

Intrucat e este singurul
element de ordinul 1 si ordinul unui element divide ordinul grupului rezulta ca orda =ordb =ordc = 2
a
2
= b
2
= c
2
= 2.

In tabela de operat ie a lui (G, ) pe ecare linie si pe ecare coloana trebuie sa gureze
toate elementele lui G o singura data. Din acest motiv tabela de operat ie poate completata ntr-un
mod unic
e a b c
e e a b c
a a e c b
b b c e a
c c b a e
Astfel s-a demonstrat ca exista cel mult un grup de ordin 4 neciclic.
Consideram urmatoarele transformari ale punctelor planului x0y:
1
aplicat ia identica,
2
simetria
n raport cu axa 0y,
3
simetria n raport cu axa 0x,
4
simetria n raport cu originea 0 a axelor de
coordonate. Considerand mult imea K =
1
,
2
,
3
,
4
n raport compunerea transformarilor obt inem
urmatoarea tabla de operat ie
3.2 Solut ii 49

1

2

3

4

1

1

2

3

4

2

2

1

4

3

3

3

4

1

2

4

4

3

2

1
Din acest tabel deducem ca mult imea K n raport cu compunerea transformarilor este un grup comutativ
de ordin 4 neciclic; ord
2
=ord
3
=ord
4
= 2 si ord
1
= 1. Asadar abstract ie facand de un izomorsm,
exista un singur grup de ordin 4 neciclic numit grupul lui Klein.
3.2.18 b) ord 1 = 1, ord (1) = 2, ordi =ord (i) =ordj =ord (j) =ordk =ord (k) = 4
(ex: ((1)
2
= 1);i
2
= i i = 1; i
3
= i
2
i = i; i
4
= i
3
i = (1) i = i
2
= 1)
c) Subgrupurile ciclice ale lui H sunt
H
1
=< 1 >= 1; H
2
=< 1 >= 1, 1, H
3
=< i >= 1, 1, i, i =< i >
H
4
=< j >= 1, 1, j, j =< j >, H
5
=< k >= 1, 1, k, k =< k >, H
6
= H.
Din teorema lui Lagrange rezulta ca ordinul unui subgrup al lui H poate 1, 2, 4, si 8. Singurul
subgrup de ordin 1 este H
1
.

Intrucat orice grup de ordin 2 este ciclic rezulta ca singurul subgrup de
ordin 2 este H
2
. Grupul H neavand trei elemente de ordinul 2, rezulta ca nu are subgrupuri de ordinul 4
neciclice (vezi problema 3.1.17). Deci subgrupurile de ordin 4 sunt H
3
, H
4
si H
5
.

In concluzie mult imea
subgrupurilor lui H este o(H) = H
1
, H
2
, H
3
, H
4
, H
5
, H.
c) Subgrupurile H
1
si H sunt normale deoarece subgrupurile triviale ale oricarui grup sunt normale.
Elementele H
2
comuta cu toate elementele lui H (H
2
= Z(H)) rezulta ca H
2
este normal (H
2
H).
Conform Teoremei lui Lagrange avem
[H : H
3
[ =
[H[
[H
3
[
=
8
4
= 2 , [H : H
4
[ = 2 , [H : H
5
[ = 2.
Deoarece orice subgrup de indice doi este normal rezulta ca H
3
, H
4
, H
5
sunt subgrupuri normale.
Observat ie. Un grup necomutativ care are toate subgrupurile normale se numeste grup hamilto-
nian.

In concluzie, (H, ) este un grup hamiltonian.
3.2.19 Din problema 3.1.18, subgrupurile lui H sunt toate subgrupuri normale, deci clasele la stanga
coincid cu clasele la dreapta n raport cu ecare subgrup. Prin urmare, vom descrie chiar grupurile cat
ale lui H.
H
1
=< 1 >= 1; H/
H
1
= xH
1
[x H = 1, 1, i, i, j, j, k, k;
H
2
=< 1 >= 1, 1; H/
H
2
= xH
2
[x H = H
2
, i, i, j, j, k, k;
H
3
=< i >= 1, 1, i, i; H/
H
3
= xH
3
[x H = H
3
, j, j, k, k;
H
4
=< j >= 1, 1, j, j; H/
H
4
= xH
4
[x H = H
4
, i, i, k, k;
H
5
=< k >= 1, 1, k, k; H/
H
5
= xH
5
[x H =)H
5
, i, i, j, j.
Subgrupul generat de i, j cont inandu-l si pe k va include pe H
3
, H
4
, H
5
, deci coincide cu H.
H/
i,j]
= H/
H
= H.
3.2.20 Fie H = 2
z
[z Z, 2
0
= 1 H H ,= . Daca a = 2
z
1
, b = 2
z
2
H atunci a b = 2
z
1
2
z
2
=
2
z
1
+z
2
H. Daca a = 2
z
si a
t
= 2
z
atunci a a
t
= 2
z
2
z
= 2
zz
= 1 a
t
= 2
z
= a
1
H. Prin
urmare H este subgrup al grupului comutativ G = 2
x
3
y
5
z
[x, y, z Z, G/
H
= xH [ x G. Daca
a, b H atunci ele se aa n aceeasi clasa daca si numai daca a
1
b H.
Fie a = 2
x
1
3
y
1
5
z
1
; b = 2
x
2
3
y
2
5
z
2
. Avem a
1
b = 2
x
2
x
1
3
y
2
y
1
5
z
2
z
1
.
a
1
b H y
2
y
1
= 0, z
2
z
1
= 0 y
2
= y
1
siz
2
= z
1
.
50 3. Probleme de algebr a

In concluzie, G/
H
= 2
x
3
y
5
z
[ x, y, z Z [ (y, z) Z Z.
3.2.21 a) NH = n h[ n N, h H; HN = h n[ h H, n N. Fie x NH, adica x = n h, n N,
h H. Deoarece N este subgrup normal, rezulta ca N h = h N. Rezulta ca x = n h = h n
t
, n
t
N
adica x H N. Asadar N H H N. Analog se arata ca H N N H, adica HN = NH.
Sa aratam acum ca NH G.

Intr-adevar, e x, y NH x = nh, y = n
t
h
t
; n, n
t
N; h, h
t
H.
Atunci, x y
/1
= (n h)(n
t
h
t
)
1
= (n h)(h
t
1
n
t
1
) = n h
tt
n
t
1
, unde h
tt
= h h
t
1
H.
Cum N este subgrup normal, atunci N h
tt
= h
tt
N si deci n h
tt
= h
tt
n
tt
, n
tt
N. Rezulta ca
x y
1
= h
tt
n
tt
n
t
1
= h
tt
n
ttt
, n
ttt
= n
tt
n
t
1
N.

In concluzie xy
1
HN = NH.
b) Cum N G iar N NH (x N, x = x e NH) rezulta ca N N H si deci putem construi
grupul factor NH/
N
. Denim aplicat ia f : H NH/
N
; f(x) = xN (x = e x NH). Aratam ca f este
omomorsm de grupuri.

Intr-adevar, f(xy) = (xy)N = xN yN = f(x) f(y). Conform primei teoreme
de izomorsm avem ca H/
Ker f
Imf. Sa calculam Ker f si Imf. Ker f = x H[ f(x) = N. Vom
arata ca Ker f = N H si astfel s-a aratat punctul b).
Fie x Ker f f(x) = N sau xN = N x N. Deoarece x H si x N x H N adica
Kerf H N.
Reciproc, e x N H x N si x H. Cum x N xN = N f(x) = xN = N x
Ker f. Rezulta ca N H Kerf.
Sa aratam acum ca f este omomorsm surjectiv iar n cazul acesta Imf = N H/
N
.
Fie zN NH/
N
zN = nh N, n N, h H. Dar n h N = h N deoarece (nh) h
1
= n H
si deci zN = hN = f(h). Deci f este surjectiva.

In concluzie H/
N H
NH/
N
.
3.2.22 Denim aplicat ia : G/
H
G/
N
prin (xH) = xN. Funct ia este bine denita nedepinzand
de alegerea reprezentant ilor.

Intr-adevar, daca xH = yH x
1
y H. Dar H N si deci rezulta ca
x
1
y N, adica xN = yN si n concluzie (xH) = (yH). Funct ia este morsm de grupuri deoarece
(xH yH) = (xyH) = xyN = xN yN = (xH) (yH).
Din denirea aplicat iei avem ca este omomorsm surjectiv si deci Im = G/
N
. Rezulta, conform
primei teoreme de izomorsm, ca (G/H) /
Ker f
G/
N
.
Sa aratam ca Ker = N/
H
ceea ce ar arata ca N/
H
G/
H
. Ker = xH G/
H
[ (xH) = N.
Daca xH Ker (xH) = N xN = N x N x H N/
H
, adica Ker N/
H
.
Reciproc, daca xH N/
H
x N (xN) = xN = N xH Ker adica Ker Kerf.

In concluzie (G/
H
) /(N/
H
) G/
N
.
3.2.23 a) Fie A, B GL
n
(K) det A ,= 0, det B ,= 0 det(AB) = det Adet B ,= 0 AB GL
n
(K).
Din asociativitatea nmult irii matricilor rezulta ca operat ia indusa de aceasta n GL
n
(K) este asociativa.
Daca A GL
n
(K) det A ,= 0 ()A
1
/
n
(K) astfel ncat A A
1
= A
1
A = I
n
.
det(A A
1
) = det I
n
= det(A
1
A) det A det(A
1
) = det(A
1
) det A = 1 det(A
1
) ,= 0
A
1
GL
n
(K).
det(A
1
) = (det A)
1
.

In concluzie, (GL
n
(K), ) este grup.
b) Aplicat ia : K

GL
n
(K) ; (x) =
_
_
_
_
_
_
_
x 0 0 0
0 x 0 0
0 0 x 0
.
.
.
.
.
.
.
.
.
.
.
.
0 0 0 x
_
_
_
_
_
_
_
= xI
n
este un omomorsm injectiv.

Intr-adevar, (x y) = (x) (y), ()x, y K

si daca (x) = (y) rezulta


_
_
_
_
x 0 0
0 x 0

0 0 x
_
_
_
_
=
_
_
_
_
y 0 0
0 y 0

0 0 y
_
_
_
_
x = y K

(K

) GL
n
(K).
c) Aplicat ia: : C

GL
2
(R), (a +bi) =
_
a b
b a
_
este un omomorsm injectiv.
3.2 Solut ii 51

Intr-adevar, (z
1
z
2
) = (z
1
)(z
2
) , () z
1
, z
2
C

si daca (z
1
) = (z
2
) (a+bi) = (c+di)
_
a b
b a
_
=
_
c d
d c
_

_
a = c
b = d
a +bi = c +di z
1
= z
2
.

In concluzie rezulta ca C

(C

) GL
2
(R).
d) det I
n
= 1 I
n
SL
n
(K) SL
n
(K) ,= .
Daca A, B SL
n
(K) det A = det B = 1 det(A B) = det A det B = 1 A B SL
n
(K).
Daca A SL
n
(K) atunci det A = 1 deci ()A
1
/
n
(K) astfel ncat A A
1
= I
n
de unde
det A det(A
1
) = 1. Prin urmare det(A
1
) = 1 adica A
1
SL
n
(K).
Am demonstrat astfel ca SL
n
(K) este un subgrup al lui (GL
n
(K), ). Pentru orice A SL
n
(K) si
orice X GL
n
(K) avem
det(X
1
A X) = det(X
1
) det A det X = det(X
1
) det X = det(X
1
X) = det I
n
= 1
X
1
A X SL
n
(K) SL
n
(K) GL
n
(K).
e) Observat ie. Acest fapt se poate dovedi si aratand ca aplicat ia g : GL
n
(K) K

, g(A) = det A
este un omomorsm surjectiv al lui (GL
n
(K), ) pe (K

, ) si Kerg = SL
n
(K). Conform primei teoreme
de izomorsm GL
n
(K)/
Ker g
K

, adica GL
n
(K)/
SL
n
(K)
K

.
3.2.24
A
2
=
_
1 0
0 1
_

_
1 0
0 1
_
=
_
1 0
0 1
_
= I
2
ord A = 2
B
2
=
_
1 1
0 1
_

_
1 1
0 1
_
=
_
1 2
0 1
_
B
3
=
_
1 2
0 1
_

_
1 1
0 1
_
=
_
1 3
0 1
_
Prin induct ie matematica se arata ca B
n
= (1)
n

_
1 n
0 1
_
. B
n
= I
2
n = 0 adica ordB = .
C
2
=
_
i 0
0 i
_

_
i 0
0 i
_
=
_
i
2
0
0 i
2
_
=
_
1 0
0 1
_
C
4
=
_
1 0
0 1
_

_
1 0
0 1
_
=
_
1 0
0 1
_
= I
2
ordC = 4
< A >= I
2
, A; < B >= B
k
[ k Z; < C >= I, C, C
2
, C
3
.
3.2.25 Fie M = [m, n] si d = (m, n). Daca m = m
t
d, n = n
t
d (m
t
, n
t
N

), din M d = m n rezulta
ca M = m
t
n
t
d = m
t
n = m n
t
.
a) Din xy = yx rezulta ca (x y)
M
= x
M
y
M
= x
mn

y
m

n
= (x
m
)
n

(y
n
)
m

= e e = e. De aici
rezulta ca ord(xy) este nit si ord(xy) [ M.
b) Daca b =ord(xy) avem (xy)
b
= x
b
y
b
= e. Rezulta ca x
b
= y
b
. Dar, x
b
< x > si y
b
< y >.
Din faptul ca < x > < y >= e rezulta ca x
b
= y
b
= e. Cum ordx = m si ordy = n avem m[b si n[b.
Rezulta ca [m, n] [ b adica M [ b. T inand seama de punctul a) rezulta ca avem si b [ M, adica M = b.
c) Din teorema lui Lagrange avem[< x > < y >[ divide [< x >[ =ordx = msi deasemenea divide si
pe [< y >[ =ordy = n. Deoarece (m, n) = 1 rezulta ca [< x > < y >[ = 1 si deci < x > < y >= e.

In concluzie ord(x y) = [m, n] = m n. Avem < xy >< x, y > . Deoarece (m, n) = 1 exista u, v Z
astfel ncat um + vn = 1. De aici x = x
um+vn
= (x
m
)
n
x
vn
= e x
vn
= x
vn
(y
n
)
v
= (xy)
vn
. Analog
y = (xy)
um
. Deci x, y < xy > ceea ce implica < x, y >< x y >.
3.2.26 1) Se verica asociativitatea legii de compozit ie. Elementul neutru este (0, 0, 1) G. Daca
m
3
= 1, atunci (m
1
, m
2
, m
3
) (x, y, z) = (m
1
+x, m
2
+y, z) = (0, 0, 1). Adica vom avea
_
_
_
m
1
+x = 0
m
2
+y = 0
z = 1

_
_
_
x = m
1
y = m
2
z = 1
; (x, y, z) = (m
1
, m
2
, 1) G.
Simetricul lui (m
1
, m
2
, 1) este (m
1
, m
2
, 1). Analog, simetricul lui (m
1
, m
2
, 1) este (m
2
, m
1
, 1).
52 3. Probleme de algebr a
2) Avem (1, 0, 1) (0, 1, 1) = (1, 1, 1) = (0, 1, 1) (1, 0, 1). Deci
H =< (1, 0, 1), (0, 1, 1 >=< (1, 0, 1) > < (0, 1, 1) > .
Avem
< (1, 0, 1) >= (1, 0, 1)
k
[ k Z = (0, 0, 1), (1, 0, 1), (2, 0, 1), (3, 0, 1), . . . = Z 0 1.
< (0, 1, 1) >= (0, 1, 1)
k
[ k Z = 0 Z 1.
Rezulta H = Z Z 1 ceea ce ne arata ca H
1
este comutativ, deci H
1
G.
3) H
2
H
1
,H
1
comutativ ne conduce la faptul ca H
2
H
1
.

Intrucat H
2
= Z 0 1 si
(1, 0, 1)
1
(1, 0, 1) (1, 0, 1) = (0, 1, 1) (1, 0, 1) (1, 0, 1) = (0, 0, 1) (1, 0, 1) = (0, 1, 1) / H
2
,
rezulta ca H
2
nu este normal n G.
3.2.27 a) Fie nZ mZ n mZ ()k Z astfel ncat n = mk m[n.
Reciproc, daca m[n si y nZ ()k
1
Z astfel ncat n = m k
1
si ()k
2
astfel ncat y = nk
2
.
Avem y = (mk
1
)k
2
= m (k
1
k
2
) = mk, unde k = k
1
k
2
Z.

In concluzie, y mZ si deci nZ mZ.


b) mZ nZ Z. Pe M Z astfel ncat mZ nZ = MZ. Vom arata ca M = [m, n], adica M este
cel mai mic multiplu comun al lui m si n. Din MZ mZ m[M si MZ nZ n[M, rezulta ca M
este multiplu comun al lui m si n. Deci [m, n][M si conform punctului a) avem MZ [m, n]Z.
Daca M
t
este un alt multiplu comun al lui m si n, atunci avem
m[M
t
M
t
Z mZ
n[M
t
M
t
Z nZ
_
M
t
Z mZ nZ M
t
Z MZ M[M
t
,
adica M este cel mai mic multiplu comun al lui m si n.

In concluzie mZ nZ =[m, n] Z.
c) (Z, +) ind grup abelian, mZ + nZ = nZ + mZ mZ + nZ Z. Fie d Z astfel ncat
mZ +nZ = dZ. Vom arata ca d = (m, n), adica d este cel mai mare divizor comun al lui m si n.
mZ < mZ nZ > si nZ < nZ mZ >
adica
mZ dZ si nZ dZ.
Conform punctului a) rezulta ca d [ m si d [ n, adica d este un divizor comun al lui m si n.
Daca d
t
este un alt divizor comun al lui m si n atunci
d
t
[m mZ d
t
Z
d
t
[n nZ d
t
Z
_
mZ +nZ d
t
Z dZ d
t
Z d
t
[d,
adica d este c.m.m.d.c. al lui m si n, (m, n) = d.

In concluzie, mZ +nZ = < mZ nZ >= (m, n) Z.


Exemplu.
6Z 8Z = [6, 8]Z adica 6Z 8Z = 24Z
6Z + 8Z =< 6Z 8Z >= (6, 8)Z adica 6Z + 8Z = 2Z.
3.2.28

1
=
_
1 2 3 4 5 6
2 4 3 6 5 1
_
= (1, 2, 4, 6) n S
6
,

2
=
_
1 2 3 4 5 6
1 2 5 4 3 6
_
= (3, 5) n S
6
,

3
=
_
1 2 3 4 5 6
3 1 2 6 4 5
_
= (1, 3, 2) (4, 6, 5).
3.2.29 a)
=
_
1 2 3 4 5 6 7 8 9 10 11 12
9 12 8 11 6 7 5 3 2 4 10 1
_
= (1, 9, 2, 12) (3, 8) (4, 11, 10) (5, 6, 7).
3.2 Solut ii 53
b) =
_
1 2 3 4 5 6 7 8
2 3 1 5 6 8 7 4
_
.
Cum (1) = 2 atunci (1) ,= 1 si consideram transpozit ia (1, 2). Facem produsul
1
= (1, 2) .
Avem

1
=
_
1 2 3 4 5 6 7 8
2 1 3 4 5 6 7 8
_

_
1 2 3 4 5 6 7 8
2 3 1 5 6 8 7 4
_
=
_
1 2 3 4 5 6 7 8
1 3 2 5 6 8 7 4
_
.
Cum
1
(2) = 3, atunci
1
(2) ,= 2 si consideram transpozit ia (2, 3). Facem produsul

2
= (2, 3)
1
=
_
1 2 3 4 5 6 7 8
1 2 3 5 6 8 7 4
_
.
Cum
2
(4) = 5, atunci
2
(4) ,= 4 si consideram transpozit ia (4, 5). Facem produsul

3
= (4, 5)
2
=
_
1 2 3 4 5 6 7 8
1 2 3 4 6 8 7 5
_
.
Cum
3
(5) = 6, atunci consideram transpozit ia (5, 6)

4
= (5, 6)
3
=
_
1 2 3 4 5 6 7 8
1 2 3 4 5 8 7 6
_
= (6, 8).
Deci
(6, 8) = (5, 6)
3
= (5, 6) (4, 5)
2
= (5, 6) (4, 5) (2, 3)
1
= (5, 6) (4, 5) (2, 3) (1, 2)
de unde obt inem
= (1, 2) (2, 3) (4, 5) (5, 6) (6, 8)
sau
=
_
1 2 3 4 5 6 7 8
2 3 1 5 6 8 7 4
_
= (1, 2, 3) (4, 5, 6, 8).
T inand seama ca orice ciclu este un produs de transpozit ii rezulta ca
= (1, 3) (1, 2) (4, 8) (4, 6) (4, 5).
3.2.30 Fie ord
1
= l(
1
) = m
1
, ord
2
= l(
2
) = m
2
, . . . ,ord
k
= l(
k
) = m
k
. Ciclurile
1
,
2
, . . . ,
k
ind
disjuncte rezulta ca pentru i ,= j, <
i
> <
j
>= e unde e este permutarea identica. T inand seama
de problema 25 rezulta ca ord =ord(
1

2
. . .
k
) = [ord
1
,ord
2
, . . . ,ord
k
] = [m
1
, m
2
, . . . , m
k
].
3.2.31 Descompunem ntai permutarea n cicluri disjuncte:
a) = (1, 4, 7, 10)(2, 6, 3)(5, 8, 9). Ciclul
1
= (1, 4, 7, 10) are ord
1
= l(
1
) = 4; ciclul
2
= (2, 6, 3)
are ord
2
= l(
2
) = 3; ciclul
3
= (5, 8, 9) are ord
3
= l(
3
) = 3. T inand seama de problema 29 rezulta
ca ord = [4, 3, 3] = 12.

100
=
128+4
= (
12
)
8

4
= e
4
=
4

4
= (1, 4, 7, 10)
4
(2, 6, 3)
4
(5, 8, 9)
4
= e (2, 6, 3) (5, 8, 9)

100
= (2, 6, 3) (5, 8, 9) =
_
1 2 3 4 5 6 7 8 9 10
1 6 2 4 8 3 7 9 5 10
_
.
b) = (1, 3, 6) (2, 4, 5, 9, 10) (7, 12); ord = [3, 5, 2] = 30
30
= e;
100
=
330+10
=
(
30
)
3

10
=
10
;
10
= (1, 3, 6)
10
(2, 4, 5, 9, 10)
10
(7, 12) = (1, 3, 6).

In concluzie,

100
= (1, 3, 6) =
_
1 2 3 4 5 6 7 8 9 10 11 12
3 2 6 4 5 1 7 8 9 10 11 12
_
.
54 3. Probleme de algebr a
3.2.32 Daca = (i
1
, i
2
, . . . , i
m
) S
n
, atunci
k
(i
j
) =
_
i
j+k
, daca j +k m
i
j+km
, daca j +k > m
.

3
1
= (1, 2, 3)
3
(4, 5, 6, 8)
3
= (4, 5, 6, 8)
3
= (4, 8, 6, 5)

2
2
= (3, 4)
2
(5, 2, 6, 1, 8)
2
= (5, 2, 6, 1, 8)
2
= (5, 6, 8, 2, 1)

2
2

1
= ((5, 6, 8, 2, 1) (1, 2, 3) (4, 5, 6, 8) = (2, 3, 5, 8, 4, 6)

3

4

5
= (1, 3, 4) (2, 3, 5, 7) (1, 8, 4, 6) (8, 2, 1, 4, 3) (1, 2) (1, 5) (8, 7, 4, 3, 1, 2) (5, 6) =
= (1, 8) (2, 4, 6, 3, 5, 7) (8, 2, 1, 4, 3) (1, 5, 2) (8, 7, 4, 3, 1, 2) (5, 6) =
= (1, 6, 8, 2, 4) (3, 7, 5)

4
3

2
4
= [(1, 3, 4) (2, 3, 5, 7) (1, 8, 4, 6)]
4
[(8, 2, 1, 4, 3) (1, 2) (1, 5)]
2
=
= [(1, 8) (2, 4, 6, 3, 5, 7)]
4
[(8, 2, 1, 4, 3) (1, 5, 2)]
2
=
= (1, 8)
4
(2, 4, 6, 3, 5, 7)
4
[(1, 5) (2, 4, 3, 8)]
2
=
= (2, 4, 6, 3, 5, 7)
4
(1, 5)
2
(2, 4, 3, 8)
2
= (2, 5, 6) (4, 7, 3) (2, 3) (4, 8) = (2, 4, 8, 7, 3, 5, 6)

5

4

3
= [(8, 7, 4, 3, 1, 2) (5, 6)] [(8, 2, 1, 4, 3) (1, 2) (1, 5)] [(1, 3, 4) (2, 3, 5, 7) (1, 8, 4, 6)] =
= [(8, 7, 4, 3, 1, 2) (5, 6)] [(8, 2, 1, 4, 3) (1, 5, 2)] [(1, 8) (2, 4, 6, 3, 5, 7))] =
= (8, 7, 4, 3, 1, 2) ((5, 6) (1, 5) (2, 4, 3, 8) (1, 8) (2, 4, 6, 3, 5, 7) = (1, 8, 6, 7, 3, 2) (4, 5).
3.2.33 Daca i (i
1
), (i
2
), . . . , (i
k
), atunci
(
1
)(i) = (
1
)((i
m
)) = ( )(i
m
) = ((i
m
)) = (i
m+1
)
pentru m 1, 2, 3, . . . , k 1. Daca i = i
k
avem (
1
) (i
k
) = ( (i
k
)) = (i
1
). Daca i /
(i
1
), (i
2
), . . . , (i
k
) adica
1
(i) / i
1
, i
2
, . . . , i
k
. atunci (
1
(i)) =
1
(i) (
1
)(i) = i.
b) Folosind punctul a) avem

1

4

1
1
=
1
[(8, 2, 1, 4, 3) (1, 2) (1, 5)]
1
1
=
= [
1
(8, 2, 1, 4, 3)
1
1
] [
1
(1, 2)
1
1
] [
1
(1, 5)
1
1
] =
= (
1
(8),
1
(2),
1
(1),
1
(4),
1
(3)) (
1
(1),
1
(2)) (
1
(1),
1
(5)) =
= (4, 3, 2, 5, 1) (2, 3) (2, 6).

2
5

3

2
5
= (
2
5
)
3
(
2
5
)
1

2
5
= (
2
5
)
1

2
5
= [(8, 7, 4, 3, 1, 2) (5, 6)]
2
= (8, 7, 4, 3, 1, 2)
2
= (8, 4, 1) (7, 3, 2)

2
5
= [(8, 4, 1) (7, 3, 2)]
1
= (8, 4, 1)
1
(7, 3, 2)
1
= (1, 4, 8) (2, 3, 7)

2
5

3

2
5
= (
2
5
(1),
2
5
(3),
2
5
(4)) (
2
5
(2),
2
5
(3),
2
5
(5),
2
5
(7))
(
2
5
(1),
2
5
(8),
2
5
(4),
2
5
(6)) = (4, 7, 8) (3, 7, 5, 2) (4, 1, 8, 6)
Analog, pentru
5
2

4

5
2
=
2
5

4
(
5
2
)
1

5
2
= [(3, 4) (5, 2, 6, 1, 8)]
5
= (3, 4)
5
(5, 2, 6, 1, 8)
5
= (3, 4);
5
2
= (4, 3)

5
2

4

5
2
= (
5
2
(8),
5
2
(2),
5
2
(1),
5
2
(4),
5
2
(3)) (
5
2
(1),
5
2
(2)) (
5
2
(1),
5
2
(5)) =
= (8, 2, 1, 3, 4) (1, 2) (1, 5).
3.2 Solut ii 55
c) Avem

1

2
=
2

1

1

2

1
1
=
2
,

1

3
=
3

1

1

3

1
1
=
3
.
Trebuiesc vericate ultimele egalitat i

1

2

1
1
= (
1
(1),
1
(4),
1
(7)) (
1
(2),
1
(5),
1
(8)) (
1
(3),
1
(6),
1
(9)) =
= (2, 5, 8) (3, 6, 9) (1, 4, 7) = (1, 4, 7) (2, 5, 8) (3, 6, 9 =
2
(Fiind cicluri disjuncte ele comuta doua c ate doua).

1

3

1
1
= (
1
(4),
1
(5),
1
(6)) (
1
(7),
1
(8),
1
(9)) =
(5, 6, 4) (8, 9, 7) = (4, 5, 6) (7, 8, 9) =
3
.
3.2.34 a) Subgrupul simetric de ordinul 3 este
S
3
=
_
e;
1
=
_
1 2 3
2 1 3
_
= (1, 2);
2
=
_
1 2 3
3 2 1
_
= (1, 3) ;

3
=
_
1 2 3
1 3 2
_
= (2, 3);
4
=
_
1 2 3
2 3 1
_
= (1, 2, 3);
5
=
_
1 2 3
3 1 2
_
= (1, 3, 2)
_
ord
1
= l(
1
) = 2; ord
2
= l(
2
) = 2; ord
3
= l(
3
) = 2; orde = 1, ord
4
= l(
4
) = 3; ord
5
=
l(
5
) = 3.
b) S
3
are subgrupurile improprii: H
1
= e, H
2
= S
3
. Conform teoremei lui Lagrange ordinul unui
subgrup divide ordinul grupului. Rezulta ca S
3
poate avea subgrupurile proprii de ordinul 2 sau 3 care
sunt ciclice.
H
3
=<
1
>= e,
1

H
4
=<
2
>= e,
2

H
5
=<
3
>= e,
3

H
6
=<
4
>= e,
4
,
2
4
= e,
4
,
5
=<
5
>= A
3
(permutarile pare).
H
1
, S
3
sunt subgrupuri normale.
[S
3
: H
6
[ =
[S
3
[
[H
6
[
=
6
3
= 2 H
6
S
3
Verticam daca
2
H
3
= H
3

2
avem

2
H
3
=
2
,
2

1
=
1
,
4

2

1
= (1, 3) (1, 2) = (1, 2, 3)

1

2
= (1, 2) (1, 3) = (1, 3, 2) =
5
H
3

2
=
2
,
1

2
=
2
,
5

2
H
3
,= H
3

2
H
3
nu este subgrup normal n S
3
.
Analog se arata ca nici H
4
si H
5
nu sunt subgrupuri normale n S
3
.
d) S
3
/
H
1
= H
1
[ S
3
= e.
1
,
2
,
3
,
4
,
5

S
3
/
H
2
= S
3
[ S
3
= S
3

S
3
/
H
6
= H
6
[ S
3
= H
6
, (1, 2) < (1, 2, 3) > = e, (1, 2, 3), (1, 3, 2), (1, 2), (2, 3), (1, 3) =
A
3
,
2
A
3

iar operat iile sunt denite astfel:


e (
1

2

3

4

5

e e (
1

2

3

4

5

(
1

1
e
5

4

3

2

2

2

4
e
5
(
1

3

3

3

5

4
e
2
(
1

4

4

2

3
(
1

5
e

5

5

3
(
1

2
e
4

S
3
S
3
S
3
A
3

2
A
3
A
3
A
3

2
A
3

2
A
3

2
A
3
A
3
56 3. Probleme de algebr a
Deci grupul S
3
/
H
1
S
3
; S
3
/
A
3
Z
2
.
3.2.35 Fie
1
= (1, 2) (3, 4);
2
= (1, 3) (2, 4)
1

2
= (1, 2) (3, 4) (1, 3) (2, 4) = (1, 4) (2, 3) =
3

2

1
= (1, 3) (2, 4) (1, 2) (3, 4) = (1, 4) (2, 3) =
3
ord
1
= [2, 2] = 2
2
1
=
2
2
=
2
3
= e
ord
2
= [2, 2] = 2
ord
3
= [2, 2] = 2
H =< (1, 2) (3, 4), (1, 3) (2, 4) >=< (1, 2) (3, 4) > < (1, 3) (2, 4) >
H = e, (1, 2) (3, 4), (1, 3) (2, 4), (1, 4) (2, 3)
Completand tabla de operat ie avem
e
1

2

3
e e
1

2

3

1

1
e
3

2

2

2

3
e
1

3

3

2

1
e

1

3
= (1, 2) (3, 4) (1, 4) (2, 3) = (1, 3) (2, 4) =
2

2

3
= (1, 3) (2, 4) (1, 4) (2, 3) = (1, 2) (3, 4) =
1
Constatam ca este la fel structurata cu cea a grupului lui Klein.
Inele
3.2.36 a) Vericam axiomele inelului comutativ cu element unitate: i) (Z, ) este un grup abelian

Intr-adevar, operat ia este asociativa, adica ()x, y, z Z avem


(x y) z = (x +y +a) z = x +y +a +z +a = x +y +z + 2a,
x (y z) = x (y +z +a) = x +y +z +a +a = x +y +z + 2a,
deci (x y) z = x (y z).
Operat ia este comutativa, adica x y = y x, ()x, y Z
x y = x +y +a = y +x +a = y x.
Elementul neutru: ()e

Z astfel ncat e

x = x e

= x, () x Z.
Din ecuat ia x e

= x x +e

+a = x e

= a Z. Deci elementul neutru este e

= a.
Element simetric: () x Z, () x
t
Z astfel ncat x x
t
= e

= x
t
x.
Din ecuat ia x x
t
= e

avem x +x
t
+a = a x
t
= x 2a Z. Deci elementul simetric al unui
element x Z este x
t
= x 2a.
ii) (Z, ) este monoid comutativ.

Intr-adevar, nmult irea este asociativa, ntrucat pentru x, y, z Z arbitrare, putem scrie
(x) z = [(x +a)(y +a) a] z = (x +a)(y +a)(z +a) a.
x (y z) = x [(y +a)(z +a) a] = (x +a)[(y +a)(z +a) a +a] a = (x +a)(y +a)(z +a) a.
Deci (x y) z = x (y z).
Aratam ca legea de compozit ie este comutativa.

Intr-adevar, daca x, y Z arbitrare avem
x y = (x +a)(y +a) a = (y +a)(x +a) a = y x.
Elementul neutru fat a de operat ia va : () e
_
Z astfel ncat xe
_
= e
_
x = x, () x Z.
Rezolvand ecuat ia x e
_
= x avem
(x +a)(e
_
+a) a = x (x +a)(e
_
+a) = x +a, () x Z.
Pentru x ,= a avem e
_
+ a = 1 e
_
= 1 a Z. Pentru x = a avem a (1 a) = (a + a)(1
a +a) a = a. Deci elementul neutru este e
_
= 1 a.
3.2 Solut ii 57
iii)

Inmult irea este distributiva fat a de adunarea .

Intr-adevar, pentru x, y, z Z arbitrare,
avem
x (y z) = x (y +z +a) = (x +a)(y +z +a +a) a = (x +a)(y +z + 2a) a
x y x z = [(x +a)(y +a) a] [(x +a)(z +a) a] =
= (x +a)(y +a) a + (x +a)(z +a) a +a = (x +a)(y +z + 2a) a.
Deci x (y z) = x y x z. Aceasta nseamna ca nmult irea este distributiva la stanga fat a de
adunarea dar cum nmult irea este comutativa, este asigurata si distributivitatea la dreapta. Prin
urmare, (Z, , ) este un inel comutativ cu element unitate e
_
.
Pentru a arata ca n acest inel nu exista divizori ai lui zero, revine la a arata ca xy = e

x = e

sau y = e

Intr-adevar, x y = e

(x + a)(y + a) a = a (x + a)(y + a) = 0 x + a = 0 sau


y +a = 0 x = a sau y = a adica x = e

sau y = e

.
b) Determinam grupul (U(Z), ) al elementelor inversabile. Avem x U(Z) ()x
1
Z cu
x x
1
= x
1
x = e
_
(x +a)(x
1
+a) a = 1 a (x +a)(x
1
+a) = 1

_
x +a 1, 1
x
1
+a 1, 1

_
x 1 a, 1 a = e
_

x
1
(1 a, 1 a = e
_

.
Asadar, U(Z) = 1 a, 1 a = e
_
este un grup ciclic de ordinul 2. Sa remarcam ca pentru a = 0
inelul (Z, , ) coincide cu inelul obisnuit (Z, +, ).
3.2.37 (C, +) este un grup comutativ. Aratam ca (C, ) este un monoid.
Vericam asociativitatea operat iei *. ()z
1
, z
2
, z
3
C avem
(z
1
z
2
) z
3
= [(a
1
+b
1
i) (a
2
+b
2
i)] (a
3
+b
3
i) = [a
1
a
2
+ (a
1
b
2
+a
2
b
1
)i] (a
3
+b
3
i) =
= a
1
a
2
a
3
+ (a
1
a
2
b
3
+a
1
a
3
b
2
+a
2
a
3
b
1
)i.
z
1
(z
2
z
3
) = (a
1
+b
1
i) [(a
2
+b
2
i) (a
3
+b
3
i)] = (a
1
+b
1
i) [a
2
a
3
+ (a
2
b
3
+a
3
b
2
)i] =
= a
1
a
2
a
3
+ (a
1
a
2
b
3
+a
1
a
3
b
2
+a
2
a
3
b
1
)i.
Deci, (z
1
z
2
) z
3
= z
1
(z
2
z
3
), () z
1
, z
2
, z
3
C.
Vericam comutativitatea operat iei *. Daca z
1
, z
2
C sunt elemente arbitrare, atunci
z
1
z
2
= (a
1
+b
1
i) (a
2
+b
2
i) = a
1
a
2
+ (a
1
b
2
+a
2
b
1
)i = a
2
a
1
+ (a
2
b
1
+a
1
b
2
)i = z
2
z
1
.
Element neutru: () e

C astfel ncat e z = z e = z pentru z C ales arbitrar. Rezolvand


ecuat ia z e = z vom avea
(a +bi) (x +yi) = a +bi sau ax + (ay +bx)i = a +bi
_
ax = a
ay +bx = b

_
x = 1
ay = 0
_
x = 1
y = 0
.
Elementul neutru este e = 1.
Elementul nul este e

= 0 si daca operam i i = 0 rezulta ca i este divizor al lui zero.


3.2.39 Fie x A. Deoarece x +x A rezulta ca x +x este idempotent si vom avea
(x +x)
2
= x +x (1)
Pe de alta parte, putem scrie
(x +x)
2
= (x +x)(x +x) = x
2
+x
2
+x
2
+x
2
= x +x +x +x.
Asadar,
(x +x)
2
= x +x +x +x. (2)
Din (1) si (2) rezulta x +x = x +x +x +x, de unde considerand opusul lui x si anume pe x vom
avea x +x = 0. Daca A este inel cu element unitate, rezulta ca n particular 1 +1 = 0, adica charA = 2.
58 3. Probleme de algebr a
b) Fie x, y A. Din ipoteza avem (x+y)
2
= x+y. Dar (x+y)
2
= (x+y)(x+y) = x
2
xy+yx+y
2
=
= x + xy + yx + y, adica x + y = x + xy + yx + y, de unde rezulta ca xy + yx = 0, adica xy = (yx).
Din x +x = 0 rezulta ca x = x, adica orice element din A este egal cu opusul sau si atunci rezulta ca
xy = (yx) = yx.
Cum x, y A au fost arbitrare, rezulta ca inelul A este comutativ.
c) Daca R ,= 0 este integru si a, b R0, atunci ab(a + b) = a
2
b + ab
2
= ab + ab = 0 conform
celor demonstrate la punctul 1
0
. Cum R este domeniu de integritate si a, b ,= 0 rezulta ca a +b = 0, deci
a = b = b si prin urmare toate elementele nenule ale lui R sunt egale ntre ele. Aceasta demonstreaza
ca R are doar doua elemente si este izomorf cu Z
2
.
3.2.39 Se arata relativ usor ca (A, +) este un grup comutativ. Elementul zero ind (0, 0) si pentru ()
a, b Z opusul lui (a, b) A este (a, b) A.
Sa aratam, n continuare ca (A, ) este un grup comutativ cu unitate. Fie (a
1
, b
1
), (a
2
, b
2
), (a
3
, b
3
) A
alese arbitrar. Vom avea
[(a
1
, b
1
) (a
2
, b
2
)] (a
3
, b
3
) = (a
1
a
2
+ 3b
1
b
2
, a
1
b
2
+a
2
b
1
) (a
3
, b
3
) =
= a
1
a
2
a
3
+ 3a
3
b
1
b
2
+ 3a
1
b
2
b
3
+ 3a
2
b
1
b
3
, a
1
a
2
b
3
+ 3b
1
b
2
b
3
+a
1
a
3
b
2
+a
2
a
3
b
1
); (1)
(a
1
, b
1
) [(a
2
, b
2
) (a
3
, b
3
)] = (a
1
, b
1
) (a
2
a
3
+ 3b
2
b
3
, a
2
b
3
+a
3
b
2
) =
= a
1
a
2
a
3
+ 3a
1
b
2
b
3
+ 3a
2
b
1
b
3
+ 3a
3
b
1
b
2
, a
1
a
2
b
3
+ 3b
1
b
2
b
3
+a
1
a
3
b
2
+a
2
a
3
b
1
). (2)
Comparand (1) cu (2) obt inem [(a
1
, b
1
) (a
2
, b
2
)] (a
3
, b
3
) = (a
1
, b
1
) [(a
2
, b
2
) (a
3
, b
3
)].
Comutativitatea nmult irii:
(a
1
, b
1
) (a
2
, b
2
) = (a
1
a
3
+ 3b
1
b
2
, a
1
b
2
+a
2
b
1
) = (a
3
a
2
+ 3b
2
b
1
, a
2
b
1
+a
1
b
2
) =
= (a
2
, b
2
) (a
1
, b
1
) , () (a
1
, b
1
), (a
2
, b
2
) A.
Elementul unitate: () (e
1
, e
2
) A astfel ncat (e
1
, e
2
) (a
1
, b
1
) = (a
1
, b
1
) (e
1
, e
2
) = (a
1
, b
1
), ()
(a
1
, b
1
) A.
Rezolvam ecuat ia (a
1
, b
1
)(e
1
, e
2
) = (a
1
, b
1
) (a
1
e
1
+3b
1
e
2
, a
1
e
2
+b
1
e
1
) = (a
1
, b
1
) , () (a
1
, b
1
) A.
Avem sistemul:
_
a
1
e
1
+ 3b
1
e
2
= a
b
1
e
1
+a
1
e
2
= b

(b)
a
(a
2
1
3b
2
1
)e
2
= 0; a
2
1
,= 3b
2
1
, ()(a
1
, b
1
) A cu
_
a
1
,= 0
b
1
,= 0
. Rezulta ca e
2
= 0. Din
_
a
1
e
1
= a
b
1
e
1
= b
e
1
= 1. Deci elementul unitate este (1, 0) A.
Distributivitatea:
[(a
1
, b
1
) + (a
2
, b
2
)] (a
3
, b
3
) = (a
1
+a
2
, b
1
+b
2
) (a
3
, b
3
) =
= (a
1
a
3
+a
2
a
3
+ 3b
1
b
3
+ 3b
2
b
3
, a
1
b
3
+a
2
b
3
+a
3
b
1
+a
3
b
2
)
(a
1
, b
1
) (a
3
, b
3
) + (a
2
, b
2
) (a
3
, b
3
) = (a
1
a
3
+ 3b
1
b
3
, a
1
b
3
+a
3
b
1
) + (a
2
a
3
+ 3b
2
b
3
, a
2
b
3
+a
3
b
2
) =
= (a
1
a
3
+a
2
a
3
+ 3b
1
b
3
+ 3b
2
b
3
, a
1
b
3
+a
2
b
3
+a
3
b
1
+a
3
b
2
)
Rezulta ca
[(a
1
, b
1
) + (a
2
, b
2
)] (a
3
, b
3
) = (a
1
, b
1
) (a
3
, b
3
) + (a
2
, b
2
) (a
3
, b
3
), ()(a
1
, b
1
), (a
2
, b
2
), (a
3
, b
3
) A.

In concluzie, (A, +, ) este inel comutativ.


Sa aratam, n continuare, ca inelul A este fara divizori ai lui zero. Fie (a
1
, b
1
), (a
2
, b
2
) A, a
1
, b
1
,= 0
cu proprietatea ca (a
1
, b
1
) (a
2
, b
2
) = (0, 0), adica
_
a
1
a
2
+ 3b
1
b
2
= 0
a
1
b
2
+a
2
b
1
= 0

b
2
(a
2
)
3.2 Solut ii 59
3b
1
b
2
2
a
2
2
b
1
= 0 b
1
(3b
2
2
a
2
2
) = 0, ()a
1
, b
1
Z, a
2
2
,= 3b
2
2
b
1
= 0,
a
1
b
1
,= 0, b
1
= 0 a
1
,= 0 si
_
a
1
a
2
= 0
a
1
b
2
= 0

_
a
2
= 0
b
2
= 0.

In concluzie nu exista divizori ai lui zero.


3.2.40 a) Se arata relativ usor ca operat ia + este asociativa, comutativa, elementul neutru fat a de
adunare este e = (0, 0) si oricare ar (a, b) simetricul lui este (a, b) , adica (, +)
este grup comutativ.
Din calcule, rezulta ca operat ia este asociativa si comutativa. Sa vedem, mai departe, existent a
elementului neutru fat a de operat ia : () e
t
= (e
1
, e
2
) astfel ncat
(a, b) (e
1
, e
2
) = (a, b) = (e
1
, e
2
) (a, b).
Rezolvand ecuat ia (a, b) (e
1
, e
2
) = (a, b) vom avea
(ae
1
be
2
, ae
2
+be
1
+be
2
) = (a, b)
_
ae
1
be
2
= a
be
1
+ (a +b)e
2
= b

(b)
(+a)

(a
2
+ab +b
2
)e
2
= 0 , () a, b e
2
= 0

_
ae
1
= a
be
1
= b
() (a, b) e
2
= 1.
Elementul neutru fat a de operat ia este e
t
= (1, 0) .
Sa vedem, mai departe, daca () (a, b) , (a, b) ,= (0, 0), () (a
t
, b
t
) astfel ncat
(a, b)(a
t
, b
t
) = (1, 0) = (a
t
, b
t
)(a, b).
Rezolvand ecuat ia (a, b)(a
t
, b
t
) = (1, 0) avem: (aa
t
bb
t
, ab
t
+a
t
b +bb
t
) = (1, 0)

_
aa
t
bb
t
= 1
ba
t
+ (a +b)b
t
= 0

(b)
a

(b
2
+a
2
+ab)b
t
= b b
t
=
b
a
2
+b
2
+ab
, a ,= 0 sau b ,= 0.
a
t
=
a +b
b
b
t
=
a +b
b

_

b
a
2
+ab +b
2
_
=
a +b
a
2
+ab +b
2
.
Deci () (a, b) , unde (a, b) ,= (0, 0) este inversabil si
(a
t
, b
t
) =
_
a +b
a
2
+ab +b
2
,
b
a
2
+ab +b
2
_
.
Distributivitatea se verica prin calcul. Deci (, +, ) este corp comutativ.
b) Fie funct ia f : (

3) K, f(z) = (a b, 2b) , z (

3), z = a +b

3 cu a, b .
Sa vericam ca f este omomorsm bijectiv.
f(z
1
+z
2
) = f(z
1
) +f(z
2
)
f(z
1
z
2
) = f(z
1
)f(z
2
),
pentru z
1
, z
2
Q(

3), z
1
= a
1
+b
1

3, z
2
= a
2
+b
2

3. Avem
f(z
1
+z
2
) = f(a
1
+a
2
+ (b
1
+b
2
)

3) = (a
1
+a
2
b
1
b
2
, 2b
1
+ 2b
2
) =
= (a
1
b
1
+a
2
b
2
, 2b
1
+ 2b
2
) = (a
1
b
1
, 2b
1
) + (a
2
b
2
, 2b
2
) = f(z
1
) +f(z
2
).
f(z
1
z
2
) = (a
1
a
2
3b
1
b
2
a
1
b
2
a
2
b
1
, 2a
1
b
2
+ 2a
2
b
1
) (1)
f(z
1
)f(z
2
) = (a
1
b
1
, 2b
1
)(a
2
b
2
, 2b
2
) =
= (a
1
a
2
a
1
b
2
a
2
b
1
+b
1
b
2
4b
1
b
2
, 2a
1
b
2
2b
1
b
2
+ 2a
2
b
1
2b
1
b
2
+ 4b
1
b
2
) =
= (a
1
a
2
3b
1
b
2
a
1
b
2
a
2
b
1
, 2a
1
b
2
+ 2a
2
b
1
) (2)
60 3. Probleme de algebr a
Din (1) si (2) rezulta ca f(z
1
z
2
) = f(z
1
)f(z
2
), adica f este omomorsm.
Sa vericam injectivitatea. Daca f(z
1
) = f(z
2
) (a
1
b
1
, 2b
1
) = (a
2
b
2
, 2b
2
)

_
a
1
b
1
= a
2
b
2
2b
1
= 2b
2

_
a
1
= a
2
b
1
= b
2
, deci z
1
= z
2
f este injectiva.
Surjectivitatea: Fie (a, b) , avem de rezolvat ecuat ia f(z) = (a, b), unde z = a
1
+b
1

3
(

3) adica (a
1
b
1
, 2b
1
) = (a, b) =
_
a
1
b
1
= a
2b
1
= b

_

_
a
1
= a +
b
2

b
1
=
b
2

, deci f este surjectiva.
Prin urmare f este bijectiva si deci (

3) .
3.2.41. Fie (R, +, ) un inel cu 1unitate. Pentru orice a, b R avem
(1 + 1) (a +b) = 1 (a +b) + 1 (a +b) = 1 a + 1 b + 1 a + 1 b = a +b +a +b.
Folosind distributivitatea nmult irii fat a de adunare
(1 + 1) (a +b) = (1 + 1) a + (1 + 1) b = a +a +b +b,
de unde deducem ca a +a +b +b = a +b +a +b. Adunand la stanga opusul lui a, iar la dreapta opusul
lui b obt inem b +a = a +b. Deoarece a, b R au fost alese arbitrar rezulta ca + este comutativa.
3.2.42 Daca consideram inelul /
2
(Z
2
) =
__
a b
c d
_

a, b, c, d Z
2
_
al matricilor patratice de ordin
2 cu coecient i din Z
2
, (/
2
(Z
2
), +, ) are 16 elemente, este un inel necomutativ.

Intr-adevar, daca
consideram elementele
_

1

1

1

0
_
si
_

1

1

0

1
_
prin nmult ire vom avea
_

1

1

1

0
_

_

1

1

0

1
_
=
_

1

0

1

1
_
.
_

1

1

0

1
_

_

1

1

1

0
_
=
_

0

1

1

0
_
.
Deci
_

1

1

1

0
_

_

1

1

0

1
_
,=
_

1

1

0

1
_

_

1

1

1

0
_
, adica /
2
(Z
2
) este inel necomutativ. Nu exista
corpuri nite necomutative conform teoremei lui Wedderburn.
3.2.43 a)Din
(a +b)
2
= (a +b)(a +b) = a
2
+a b +b a +b
2
(a +b)
2
= a
2
+ 2ab +b
2

a
2
+ab +ba +b
2
= a
2
+ab +ab +b
2
.
Cum n grupul (R, +) se poate simplica cu orice element rezulta ca ba = ab.
Reciproc, daca ab = ba, atunci
(a +b)
2
= (a +b)(a +b) = a
2
+ab +ba +b
2
= a
2
+ab +ab +b
2
= a
2
+ 2ab +b
2
.
Din a
2
b
2
= (a b)(a +b) a
2
b
2
= a
2
+ab ba b
2
ab ba = 0 ab = ba.
Reciproc, daca ab = ba, atunci (a b) (a +b) = a
2
+ab ba b
2
= a
2
+ab ab b
2
= a
2
b
2
.
b) Demonstram prin induct ie dupa n.
Pentru n = 1, (a +b)
1
= C
0
1
a
1
b
0
+C
1
1
a
0
b. Deci egalitatea se verica.
Pentru n = 2, (a +b)
2
= C
0
2
a
2
+C
1
2
a b +C
2
2
b
2
.
Daca egalitatea este adevarata pentru n atunci
(a +b)
n+1
= (a +b)
n
(a +b) = (C
0
n
a
n
+C
1
n
a
n1
b +. . . +C
n1
n
ab
n1
+C
n
n
b
n
)a+
+(C
0
n
a
n
+C
1
n
a
n1
b +. . . +C
n1
n
ab
n1
+C
n
n
b
n
)b =
3.2 Solut ii 61
= C
0
n
a
n+1
+ (C
1
n
+C
0
n
)a
n
b +. . . + (C
n
n
+C
n1
n
)ab
n
+C
n
n
b
n+1
.
Cum C
0
n
= C
n
n
= 1 si C
k
n
+C
k1
n
= C
k
n+1
pentru orice n N

si 1 k n, avem
(a +b)
n+1
= C
0
n+1
a
n+1
+C
1
n+1
a
n
b +. . . +C
n
n+1
ab
n
+C
n+1
n+1
b
n+1
.
Deci egalitatea (*) din enunt este adevarata pentru () n N

.
Din faptul ca ab = ba rezulta a
m
b
n
= b
n
a
m
, () m, n N

(se arata prin induct ie dupa m respectiv


n). (ab)(a
n1
+a
n2
b+. . . +ab
n2
+b
n1
) = a
n
+a
n1
b+. . . +a
2
b
n2
+ab
n1
ba
n1
ba
n2
b. . .
bab
n2
b
n
= a
n
+a
n1
b +. . . +a
2
b
n2
+ab
n1
a
n1
b a
n2
b
2
. . . ab
n1
b
n
= a
n
b
n
Analog, pentru ultima egalitate.
3.2.44 () x R = x
6
= x x = (x)
6
= x
6
= x x +x = 0 2 x = 0, adica carR = 2. Atunci
(x +x
2
)
6
= x
6
+ 6x
7
+ 15x
8
+ 20x
9
+ 15x
10
+ 6x
11
+x
12
= (1)
= x
6
+x
8
+x
10
+x
12
= x +x
3
+x
5
+x
2
.
Dar
(x +x
2
)
6
= x +x
2
. (2)
Din (1) si (2) rezulta ca x + x
2
= x + x
3
+ x
5
+ x
2
adica x
3
+ x
5
= 0 sau x
5
= x
3
= x
3
. Rezulta
x = x
6
= x
2
x
4
= x
2
x = x
3
= x
2
.
3.2.45

4x +

5 =

4x =

4x =

4 x

1,

4,

7,

10


1

2

3

4

5

6

7

8

9

10

11

4

4

8

0

4

8

0

4

8

0

4

8

5x +

5 =

5x =

4. Deoarece (5, 7) = 1 exista



5
1
Z
12
si anume

5
1
=

5
Vom avea, x =

5
1
4 =

4 =

8 x =

8
Fie
_
a b
c d
_
/
2
(C) astfel ncat
_
1 2
1 2
_

_
a b
c d
_
=
_
1 2
1 2
_

_
a + 2c = 1
b + 2d = 2

_
a = 1 2c
b = 2 2d
.
Mult imea solut iilor ecuat iei matriceale este
S =
__
1 2c 2 2d
c d
_
, c, d C
_
.
3.2.46 U(Z
12
) =

1,

5,

7,

11. Divizorii lui zero sunt elemente neinversabilen Z


12
, adica

2,

3,

4,

6,

8,

9,

10.
Coecient ii lui x si y ind divizorii lui zero, aplicarea metodei reducerii poate conduce la sisteme
neechivalente. Adunand cele doua ecuat ii ale sistemului dat obt inem

7x + y =

9 y =

5x +

9.
Vom avea
_

3x +

4y =

11
y =

5x +

9

_

3x +

4(

5x +

9) =

11
y =

5x +

9

_
3x + 8x = 11
y =

5x +

_

11x =

11
y =

5x +

9

_
x =

1
y =

5 +

9 =

2.
Deci sistemul are solut ie unica pe (

1,

2).
3.2.47 Avem 1 = 1 + 0

2 Z[

2] deci Z[

2] ,= . Pentru orice u = a + b

2 Z[

2], v = a
t
+ b
t

2
Z[

2]:
u v = (a a
t
) + (b b
t
)

2 Z[

2]
u v = (aa
t
+ 2bb
t
) + (ab
t
+a
t
b)

2 Z[

2].
62 3. Probleme de algebr a
Deci, Z[

2] este un subinel al lui (R, +, ) care cont ine unitatea 1.


Aratam n continuare ca subinelul Z[

2] este generat de 1,

2. Evident ca
1,

2 Z[

2]. (1)
Fie A subinel al lui (R, +, ) si 1,

2 A. Sa aratam ca Z[

2] A, adica Z[

2] este cel mai mic


subinel al lui (R, +, ) care cont ine pe 1,

2. Din faptul ca A este subgrup al lui (R, +) si 1 A rezulta


ca 1 + 1 A 2 A; 2 + 1 A 3 A s.a.m.d. k A, () k N. Din 1 A rezulta ca 1 A si
1 1 A s.a.m.d. k A, adica Z A. Analog, din faptul ca

2 A si ca A este subgrup al lui
(R, +) se arata ca Z[

2] A. Din faptul ca Z, Z[

2] A si din faptul ca (A, +) este grup rezulta ca


Z +Z

2 A adica
Z[

2] A (2)
Din (1) si (2) rezulta ca Z[

2] este cel mai mic subinel al lui R care include pe 1,

2 adica Z[

2] este
subinelul generat de 1,

2.
ii) 1 = 1 + 0

2 (

2), (

2 , = ,

2)

2.
Pentru orice x = a +b

2 (

2), y = a
t
+b
t

2 (

2) avem
x y = (a a
t
) + (b b
t
)

2 (

2)
x y = (aa
t
+ 2bb
t
) + (ab
t
+a
t
b)

2 (

2)
Fie x = a +b

2 (

2), a ,= 0 sau b ,= 0, adica x ,= 0. Avem (a +b

2)(a b

2) = a
2
2b
2
,= 0 (daca
a
2
2b
2
= 0 atunci a
2
= 2b
2
si pentru b ,= 0 am avea
a
2
b
2
= 2 adica
a
b
=

2 ceea ce este fals).


Avem x
1
=
1
a +b

2
=
a b

2
a
2
2b
2
=
a
a
2
2b
2

b
a
2
2b
2

2 (

2).
Deci (

2) este subgrup al lui (R, +).


Aratam n continuare ca (

2) este generat de

2. Evident,

2 = 0 + 1

2 (

2). Fie A
subcorp al lui (R, +, ) cu proprietatea ca

2 A. Se arata, n continuare, ca (

2) A, adica (

2)
este cel mai mic subcorp al corpului (R, +, ) care cont ine pe

2. A ind subcorp al lui (R, +, ) rezulta
ca 0, 1 A.
Deoarece (A, +) este subgrup al lui (R, +) rezulta, analog ca la punctul i) ca Z A.
Din faptul ca (A, +, ) este subgrup al lui (R, +, ) rezulta ca (A

, ) este subgrup al lui (R

, ).
Deoarece 1 A rezuta ca < 1 >= Z A adica pentru orice k Z

avem k A

si din faptul ca
(A

, ) este subgrup rezulta ca si k


1
=
1
k
A

. Pentru orice
m
n

avem
m
n
=
1
n
+
1
n
+. . . +
1
n
. .
m ori
A

In concluzie

si deci A.
Din faptul ca

2 A, rezulta ca si +

2 A, adica (

2) A. (

2) ind cel mai mic


subcorp al lui R care l cont ine pe

2, rezulta ca

2 genereaza acest subcorp.
iii) Fie =
3

2 S
1
. Rezulta
2
=
3

4. Presupunem ca
2
S
1
. Rezulta ca exista x, y Z astfel
ncat
2
= x+y
3

2 = x+y, ceea ce implica


3
= (x+y) = x+y
2
. Rezulta ca 2 = x+y(x+y).
Deci 2 = xy + (x +y
2
) si deci xy = 2 si x +y
2
= 0. Rezolvam sistemul:
_
xy = 2
x +y
2
= 0
.
Aceasta conduce la
I.
_
x = 1; y = 2
4 = 1 Fals!
II.
_
x = 2; y = 1
1 = 2 Fals!
III.
_
x = 1; y = 2
4 = 1 Fals!
IV.
_
x = 2; y = 1
1 + 4 = 0 Fals!
Rezulta ca sistemul nu are solut ii ntregi si deci
2
/ S
1
.

In concluzie S
1
nu este stabila n raport cu
si deci S
1
nu este subinel al lui (R, +, ).
3.2 Solut ii 63
iv) Analog punctului iii) trebuie rezolvat sistemul
_
x y = 2
x +y
2
= 0
unde x, y . Pentru x < 0 avem
_
y
2
y = 2
y
2
= x

_
y
3
= 2
x = y
2

_
y =
3

2 /
x = y
3
si deci sistemul nu are solut ii rat ionale.

In concluzie =
3

2 S
2
dar
2
/ S
2
si deci S
2
nu este stabila n raport cu adica S
2
nu este
subcorp cu (R, +, ).
Observat ii.
Z[

2] = < Z

2 > .
(

2) = <

2 > .
3.2.48 Fie z
1
= a
1
+b
1

d , z
2
= a
2
+b
2

d din Z[

d] =
_
a +b

d [ a, b Z
_
.
z
1
z
2
= (a
1
+b
1

d)(a
2
+b
2

d) = a
1
a
2
+db
1
b
2
+ (a
1
b
2
+a
2
b
1
)

d.
z
1
z
2
= a
1
a
2
+b
1
b
2
d (a
1
b
2
+a
2
b
1
)

d.
(z
1
z
2
) (z
1
z
2
) = (a
1
a
2
+b
1
b
2
d)
2
d(a
1
b
2
+a
2
b
1
)
2
= a
2
1
a
2
2
+ 2a
1
a
2
b
1
b
2
d +b
2
1
b
2
2
d
2

a
2
1
b
2
2
2a
1
a
2
b
1
b
2
d a
2
2
b
2
1
d = a
2
1
a
2
2
a
2
1
b
2
2
d a
2
2
b
2
1
d + 4b
2
1
b
2
2
d
(z
1
z
2
) =

a
2
1
a
2
2
a
2
1
b
2
2
d a
2
2
b
2
1
d + 4b
2
1
b
2
2
d

(1)
(z
1
) (z
2
) =

a
2
1
db
2
1

a
2
2
db
2
2

(a
2
1
b
2
1
d)(a
2
2
b
2
2
d)

= (2)
=

a
2
1
a
2
2
a
2
1
b
2
2
d a
2
2
b
2
1
d + 4b
2
1
b
2
2
d

Din (1) si (2) rezulta ca (z


1
z
2
) = (z
1
) (z
2
) pentru orice z
1
, z
2
Z[

d].
Observat ii. Omomorsmul se poate arata astfel:
(z
1
z
2
) = [z
1
z
2
z
1
z
2
[ = [z
1
z
2
z
1
z
2
[ = [(z
1
z
1
) (z
2
z
2
[ =
= [z
1
z
1
[ [z
2
z
2
[ = (z
1
) (z
2
) , pentru orice z
1
, z
2
Z[

d].
Fie z Z[

d] inversabil. Atunci exista z


1
astfel ncat z z
1
= z
1
z = 1. Avem (z z
1
) =
(1) = 1 si (z) (z
1
) = 1 n N ceea ce implica (z) = 1.
Reciproc, daca (z) = 1 atunci [z z [ = 1 si deci z z = 1 sau z z = 1, adica z
1
= z sau
z
1
= z .
3.2.49 Fie funct ia : Z[

2] N, (z) = [z z [ . Daca z = a +b

2 atunci (z) =

a
2
2b
2

. Conform,
problemei anterioare z este inversabil daca si numai daca (z) = 1, adica z U(Z[

2])

a
2
2b
2

= 1.
Deci z U(Z[

2]) a
2
2b
2
= 1. Aceasta egalitate este satisfacuta pentru z
1
= 1, z
2
= 1, z
3
=
1 +

2, z
4
= (1 +

2)
2
= 3 + 2

2, z
5
= (1 +

2)
3
= 7 + 5

2 si n general pentru orice putere a lui


1 +

2. Cum sirul puterilor lui 1 +

2 este innit rezulta ca exista o innitate de elemente inversabile


n Z[

2].
3.2.50 Evident R ,=
__
1 0
0 1
_
R
_
. Fie A =
_
a
1
b
1
2b
1
a
1
_
, B =
_
a
2
b
2
2b
2
a
2
_
.
AB =
_
a
1
a
2
b
1
b
2
2(b
1
b
2
) a
1
a
2
_
R.
A B =
_
a
1
b
1
2b
1
a
1
_

_
a
2
b
2
2b
2
a
2
_
=
_
a
1
a
2
+ 2b
1
b
2
a
1
b
2
+a
2
b
1
2(a
1
b
2
+a
2
b
1
) a
1
a
2
+ 2b
1
b
2
_
R.
Deci R este subinel n (/
2
(Z), +, ).
Avem [R[ 2 si ca A B =
_
a
1
a
2
+ 2b
1
b
2
a
1
b
2
+a
2
b
1
2(a
1
b
2
+a
2
b
1
) a
1
a
2
+ 2b
1
b
2
_
.
B A =
_
a
2
b
2
2b
2
a
2
_

_
a
1
b
1
2b
1
a
1
_
=
_
a
2
a
1
+ 2b
2
b
1
a
2
b
1
+a
1
b
2
2(b
2
a
1
+a
2
b
1
) 2b
2
b
1
+a
2
a
1
_
=
64 3. Probleme de algebr a
=
_
a
1
a
2
+ 2b
1
b
2
a
1
b
2
+a
2
b
1
2(a
1
b
2
+a
2
b
1
) a
1
a
2
+ 2b
1
b
2
_
= A B,
adica operat ia din R este comutativa.
Avem I
2
=
_
1 0
0 1
_
=
_
1 0
2 0 1
_
R este elementul unitate n (R, ).
Mai trebuie aratat ca (R, +, ) nu are divizori ai lui zero. Presupunem ca A ,=
_
0 0
0 0
_
, adica
a
1
,= 0 sau b
1
,= 0 si ca A B =
_
0 0
0 0
_
ceea ce este echivalent cu
_
a
1
a
2
+ 2b
1
b
2
= 0
a
1
b
2
+a
2
b
1
= 0

_
a
1
a
2
+ 2b
1
b
2
= 0
b
1
a
2
+a
1
b
2
= 0
.
Avem un sistem liniar si omogen n necunoscutele a
2
, b
2

2
=

a
1
2b
1
b
1
a
1

= a
2
1
2b
2
1
,= 0 deoarece

2 R .
Deci singura solut ie a sistemului este a
2
= b
2
= 0 ceea ce arata ca
A B =
_
0 0
0 0
_
si A ,=
_
0 0
0 0
_
B =
_
0 0
0 0
_
adica (R, +, ) nu are divizori ai lui zero.

In concluzie (R, +, ) este un domeniu de integritate.


b) Fie funct ia f : Z[

2] R, f
_
a +b

2
_
=
_
a b
2b a
_
.
Aratam ca f este izomorsm de inele. Daca z
1
= a
1
+b
1

2 si z
2
= a
2
+b
2

2 doua elemente oarecare


din Z[

2], atunci
z
1
+z
2
= (a
1
+a
2
) + (b
1
+b
2
)

2siz
1
z
2
= a
1
a
2
+ 2b
1
b
2
+ (a
1
b
2
+a
2
b
1
)

2.
Avem
f(z
1
+z
2
) =
_
a
1
+a
2
b
1
+b
2
2(b
1
+b
2
) a
1
+a
2
_
=
_
a
1
b
1
2b
1
a
1
_
+
_
a
2
b
2
2b
2
a
2
_
= f(z
1
) +f(z
2
).
f(z
1
z
2
) =
_
a
1
a
2
+ 2b
1
b
2
a
1
b
2
+a
2
b
1
2(a
1
b
2
+a
2
b
1
) a
1
a
2
+ 2b
1
b
2
_
=
_
a
1
b
1
2b
1
a
1
_

_
a
2
b
2
2b
2
a
2
_
= f(z
1
) f(z
2
)
pentru orice z
1
, z
2
Z[

2], adica f este omomorsm de inele. Mai trebuie aratata bijectivitatea funct iei
f.
Daca f(z
1
) = f(z
2
), adica
_
a
1
b
1
2b
1
a
1
_
=
_
a
2
b
2
2b
2
a
2
_
a
1
= b
1
, a
2
= b
2
, atunci a
1
+ b
1

2 =
a
2
+b
2

2 z
1
= z
2
si deci f este injectiva.
Surjectivitatea este evidenta.
3.2.51 Reamintim ca elementul neutru n (Z, ) este e

= 3 iar n (Z, ) este e

= 2. Daca x Z
simetricul lui n (Z, ) este x
t
= 6 x. Presupunem ca f : Z Z este un izomorsm, rezulta ca
f(0) = e

si f(1) = e

adica f(0) = 3 si f(1) = 2. Pentru orice n N

avem:
f(n) = f(1 + 1 +. . . 1)
. .
n ori
= (f (1) f (1) . . . f (1))
. .
n ori
=
= ((2) (2) . . . (2))
. .
n ori
= (2) n + 3(n 1) = n 3
f(n) coincide cu f(n)
t
adica simetricul lui f(n) n (Z, ), adica
f(n) = f(n)
t
= 6 f(n) = 6 (n 3) = n 3.
3.2 Solut ii 65
Deci f(x) = x 3 pentru orice x Z.
Aratam, acum, ca funct ia f : Z Z, f(x) = x 3 este un izomorsm. Pentru orice x, y Z avem
f(x +y) = x +y 3 = x 3 +y 3 + 3 = f(x) f(y)
f(x y) = x y 3 = (x 3 + 3) (y 3 + 3) 3 = f(x) f(y)
adica f este omomorsm de inele.
Pentru orice y Z ecuat ia f(x) = y x 3 = y are o solut ie unica x = y + 3 Z adica f este
bijectiva.

In concluzie, f este izomorsm.


3.2.52 Din ipoteza, rezulta ca orice x A0 este inversabil avand inversul x
1
= x, deci A este corp.
Fie x A0 arbitrar. Egalitatea x
2
1 = 0 se mai scrie (x 1)(x + 1) = 0 si deoarece un corp nu
are divizori ai lui zero, rezulta ca x = 1 sau x = 1, adica x 1, 1. Rezulta A0 1, 1, deci
A 1, 0, 1. Dar 1
2
= 1, g
2
= 1 rezulta ca 1, 0, 1 A, deci A = 1, 0, 1. Distingem doua
cazuri dupa cum 1 = 1 respectiv 1 ,= 1.
a) 1 = 1 1 + 1 = 0 adica corpul A are caracteristica 2. Rezulta A = 0, 1 si deci A Z
2
.
b) 1 ,= 1. Atunci n grupul (A, +) avem 1 + 1 ,= 0 dar 1 + 1 + 1 = 0 caci ordinul grupului este 3.
Deci corpul A are caracteristica 3.
Atunci, 1 = 1 + 1 si A = 0, 1, 1 + 1. Rezulta A

Z
3
.
3.2.53 Fie f : C omomorsm de corpuri.
f(0) = 0 , f(1) = 1
f(2) = f(1 + 1) = f(1) +f(1) = 1 + 1 = 2
f(n) = f(1 + 1 +. . . + 1)
. .
n ori
= (f (1) +f (1) +. . . +f (1))
. .
n ori
= nf (1) = n, iar
f(n) = f(n) = n () n N f(k) = k , () k Z.

Intrucat 1 = f
_
n
1
n
_
= n f
_
1
n
_
= n f(n
1
) f(n
1
) =
1
n
, () n N.
Pentru orice x exista a, b Z, b ,= 0 astfel ncat x =
a
b
= a b
1
si deci
f(x) = f(a b
1
) = a b
1
=
a
b
= x, () x .
Fie g : (

2) (

2) un automorsm. Avem g(x) = x, () x . Fie z = a + b

2, atunci
g(z) = g(a +b

2) = g(a) +g(b) g(

2) = a +b g(

2). Din (

2)
2
= 2 rezulta ca
z = g(2) = g[(

2)
2
] = [g(

2)]
2
g(

2)

2,

2.
Daca g
1
(

2) =

2, atunci g
1
(a +b

2) = a +b

2, iar daca g
2
(

2) =

2, atunci
g
2
(a +b(

2) = a

2.
Din g
1
= 1
Q(

2)
rezulta ca g
1
este automorsm. Avem
(g
2
g
2
)(z) = g
2
(a b

2) = a (b)

2 = a +b

2 = 1
Q(

2)
(z),
adica g
2
g
2
= 1
Q(

2)
ceea ce implica g
2
biject ie si g
1
2
= g
2
. Se verica usor ca g
2
este omomorsm.
Deci si g
2
este automorsm.

In concluzie, automoesmele lui (

2) sunt g
1
si g
2
.
3.2.54 Morsmele de inele sunt morsme de grupuri ntre grupurile aditive ale inelelor.
f : Z Z, f(k) = f(1) k , () k Z.
Dar f(1) = f(1 1) = f(1) f(1) si atunci f(1) 0, 1.

In concluzie f(k) = 0 adica f = omomorsmul
nul sau f(k) = 1 k = f adica f = 1
Z
. Deci End(Z, +, ) = , 1
Z
si Aut(Z, +, ) = 1
Z
. Analog
End(, +, ) = , 1
Q
si Aut(, +, ) = 1
Q
.
3.2.55 Avem 0 N(R) si deci N(R) ,= . Trebuie sa aratam ca pentru orice a, b N(R) avem a b
N(R) si pentru orice r R si orice a N(R) avem r a N(R). Daca a, b N(R) rezulta ca exista
m, n Z
+
astfel ncat a
m
= 0 si b
n
= 0. Trebuie sa aratam ca exista o putere k astfel ncat (a b)
k
= 0.
66 3. Probleme de algebr a
Observat ie. Comutativitatea este esent iala deoarece binomul lui Newton funct ioneaza numai n
acest caz.
(a b)
k
= a
k
C
1
k
a
k1
b +C
2
k
a
k2
b
2
+. . . + (1)
k1
C
k1
k
ab
k1
+ (1)
k
C
k
k
b
k
=
=
m

p=0
(1)
p
C
p
k
a
kp
b
p
+
k

p=m+1
(1)
p
C
p
k
a
kp
b
p
.
Pentru ca tot i termenii primei sume sa e zero impunem condit ia ca kp m adica p km. Pentru ca
tot i termenii celei de-a doua sume sa se anuleze datorita puterii lui b impunem condit ia ca k m+1 n,
adica k n +m1. Deci pentru orice k m+n 1 avem (a b)
k
= 0.

In particular, pentru k = m+n 1 avem


(a b)
m+n1
= a
m+n1
C
1
m+n1
a
m+n1
b +C
2
m+n1
a
m+n2
b
2
+. . .
+ (1)
n1
C
n1
m+n1
a
m
b
n1
+ (1)
n
C
n
m+n1
a
m1
b
n
+
+ (1)
n+1
C
n+1
m+n1
a
m2
b
n+1
+. . . + (1)
m+n1
b
m+n1
= 0.
Din cei m+n termeni ai sumei din membrul drept, n primii n (deci n cei pentru care p n 1) apare
ca factor a
m
, iar n ceilalt i apare ca factor b
n
. Asadar (a b)
m+n1
= 0 si deci a b N(R).
Fie a N(R). Rezulta ca exista n Z

+
astfel ncat a
m
= 0. Din comutativitatea lui R avem pentru
orice r R ca (ra)
n
= r
n
a
n
= r
n
0 = 0. Deci ra N(R).
3.2.56 0 A : B si deci A : B ,= . Pentru ecare r
1
, r
2
A : B trebuie sa aratam ca r
1
r
2
A : B si
ca oricare ar x R si pentru orice r A : B avem x r A : B si r x A : B.
Daca r
1
, r
2
A : B, atunci pentru orice b B avem r
1
b, r
2
b A. Prin urmare (r
1
r
2
)b = r
1
br
2
b
A, A ind ideal n R deci, n particular, subgrup n (R, +).
Pentru orice x R si pentru orice b B avem xb, bx B, B ind ideal al lui R.
Pentru r A : B avem
(xr) b = x (rb) A, (rb Asi A ideal n R) respectiv (rx) b = r (xb) A
Deci A : B este ideal bilateral n R.
b) nZ : mZ = r Z[ rmZ nZ = r Z[ n[rm. Deoarece n [ rm rezulta ca exista x Z astfel
ncat rm = nx, m = m
t
d , n = n
t
d unde d = (m, n) si [m, n] d = m n. Rezulta ca r m
t
d = n
t
d x
deci n
t
[r m
t
; (n
t
, m
t
) = 1, de unde n
t
[r
n
d

r
[m, n]
m

r.

In concluzie nZ : mZ =
_
r Z

[m, n]
m
[ r
_
=
[m, n]
m
Z.
3.2.57. Fie Z
m
=

0,

1,

2, . . . ,

m1, Z
n
=

0,

1,

2, . . . , n 1 si Z
mn
=

0,

1,

2, . . . ,

mn 1, Z
m
Z
n
=
(a,

b)[a Z
m
,

b Z
n
. Denim funct ia f : Z
mn
Z
m
Z
n
, f(a) = (a, a). Sa aratam ca f este bine
denita si stabileste izomorsmul de inele cautat.
Daca a =

b, atunci a b(mod mn) si deci mn[ab. Rezulta ca m[ab si n[ab, adica a b(mod m)
si a b(mod n). Deci a =

b si a =

b respectiv (a, a) = (

b,

b), de unde rezula ca f(a) = f(

b).
f(a +

b) = f(

a +b) = (

a +b, a b) = (a +

b, a +b) = (a, a) + (

b, b) = f(a) +f(

b).
Analog, f(

ab) = f(a) f(

b), adica f este omomorsm de inele.


Daca f(a) = f(

b) atunci (a, a) = (

b, b) si deci a =

b, respeciv a = b. De unde rezulta ca m[a b,


n[a b si deoarece (m, n) = 1 avem ca m n[a b, adica a =

b.

In concluzie f este surjectiva.
[Z
m
[ = m, [Z
n
[ = n si [Z
mn
[ = m n. Deoarece Z
mn
si Z
m
Z
n
au acelasi numar de elemente rezulta
ca f este bijectiva.

In concluzie Z
mn
Z
m
Z
n
.
3.2 Solut ii 67
Observat ie. Daca m
1
, m
2
, . . . , m
k
sunt numere ntregi pozitive si (m
i
, m
j
) = 1 pentru orice i ,= j
atunci inelele Z
m
1
,m
2
,...,m
k
Z
m
1
Z
m
2
. . . Z
m
k
.
3.2.58. Fie aplicat ia f : R R/
I
R/
J
, f(a) = (a +I, a +J). Aceasta este bine denita nedepinzand
de alegerea reprezentant ilor. T inand seama de denit ia adunarii si nmult irii n inele factor avem:
f(a +b) = [(a +b) +I, (a +b) +J] = [(a +I) + (b +I), (a +J) + (b +J)] =
= (a +I, a +J) + (b +I, b +J) = f(a) +f(b) pentru orice a, b R.
f(a b) = (ab +I, ab +J) = [(a +I) (b +I), (a +J) (b +J)] =
= (a +I, a +J) (b +I, b +J) = f(a) f(b) pentru orice a, b R.
Deci f este omomorsm de inele. Conform teoremei fundamentale de izomorsm avem ca R/
Ker f
Imf.
Ker f = a R[ f(a) = (0
R/I
, 0
R/J
) = a R[ (a +I, a +J) = (I, J) =
= a R[ a I, a J = I J
Fie b, c R astfel ncat (b + I, c + J) R/
I
R/
J
.

Intrucat R = I + J putem scrie b = i + j si
c = i
t
+j
t
unde i, i
t
I si j, j
t
J.
Fie a = i
t
+ j R. Avem a + I = i
t
+ j + I = j + I = b i + I = b + I si a + J = i
t
+ j + J =
i
t
+J = c j
t
+J = c +J. Rezulta ca (a +I, a +J) = (b +I, c +J), adica f(a) = (b +I, c +J), unde
a = i
t
+j.

In concluzie f este surjectiva si Imf = R/
I
R/
J
.
3.2.59 Fie P, Q R[X] doua polinoame.
f(P +Q) = (P +Q)(1) = P(1) +Q(1) = f(P) +f(Q)
f(P Q) = (P Q)(1) = P(1) Q(1) = f(P) f(Q)
adica f este omomorsm de inele.
Surjectivitatea este evidenta.
Rezulta din teorema fundamentala de izomorsm ca R[X]/
Ker f
R. Avem
Ker f = P R[X] [ f(P) = 0 = P R[X] [ P(1) = 0
Aratam ca Ker f = (X 1) = (X 1) Q, Q R[X

]. Pentru orice P (X 1) rezulta ca


P = (X 1) Q, Q R[X

], f(P) = P(1) = f(X 1) f(Q) = (1 1)f(Q) = 0 P Ker f


adica (X 1) Ker f.
Reciproc, daca P Ker f atunci f(P) = P(1) = 0. Dar P R[X] si conform teoremei mpart irii cu
rest rezulta ca exista Q, R R[X] astfel ncat P = (X 1)Q+R, grad R < 1 adica grad R = 0. Rezulta
ca P = (X 1)Q+a si vom avea 0 = f (P) = f (a) = a, adica P = (X 1)Q si deci P (X 1).

In concluzie R[X]/
(X1)
R.
3.2.60 Mult imea idealelor este inclusa n mult imea subinelelor inelului (Z, +, ). Mult imea subinelelor lui
(Z, +, ) este o(Z, +, ) = nZ[ n Z. Sa aratam ca nZ este ideal al lui Z.

Intr-adevar, nZ este subgrup
n (Z, +) si n plus pentru orice k Z si orice x = na nZ avem xk = (na) k = n k
t
nZ. Prin
urmare, mult imea idealelor inelului Z este
o(Z, +, ) = nZ[ n N.
Toate idealele lui Z sunt ideale principale. Idealele factor ale inelului Z se construiesc prin factorizare
cu ideale care au forma nZ, n N. Plecand de la grupul cat al lui (Z, +, ) si anume (Z
n
, +, ) si t inand
seama de operat ia de nmult ire din Z se poate completa si structura lui Z
n
.
Operat ia x y = x y sau (x + nZ)(y + nZ) = xy + nZ mpreuna cu operat ia de adunare induc pe
nZ o structura de inel comutativ si unitar (Z
n
, +, ), unitatea ind

1 = 1 +nZ.
3.2.61 Aratam echivalent a dintre a) si b).
68 3. Probleme de algebr a
Observat ie.

In general, daca R este un inel nit cu unitate, atunci un element r R, r ,= 0 este
inversabil daca si numai daca r nu este divizor al lui zero.

Intr-adevar, e r R inversabil. Rezulta ca exista r


1
R astfel ncat r r
1
= r
1
r = 1.
Daca exista b R astfel ncat r b = 0 rezulta ca r
1
r b = r
1
0 = 0 adica b = 0 si deci r nu este
divizor al lui zero.
Reciproc, daca r nu este divizor al lui zero, e t
r
: R R, t
r
(x) = rx. Aplicat ia t
r
este injectiva.

Intr-adevar, t
r
(x
1
) = t
r
(x
2
) implica rx
1
= rx
2
adica r(x
1
x
2
) = 0. r neind divizor al lui zero rezulta
ca x
1
x
2
= 0 daca x
1
= x
2
. Aplicat ia t
r
ind injectiva si R ind inel nit rezulta ca t
r
este surjectiva
deci bijectiva, adica exista r
t
R astfel ncat t
r
(r
t
) = 1. Deci r r
t
= 1 si r
1
= r
t
.

In concluzie, a) b) n orice inel nit cu unitate si n particular, echivalent a are loc si n (Z


n
, +, ).
Sa aratam ca b) c).
Daca r este element inversabil n Z
n
atunci exista a Z
n
astfel ncat ra =

1 sau ra = 1+nk, a, k Z.
Deci ra +n(k) = 1 ceea ce arata ca r si n sunt relativ prime.
Reciproc, daca r, n sunt relativ prime atunci exista u, v Z astfel ncat ru+nv = 1 sau ru = 1 nv
adica ru =

1 sau r u =

1 adica u
1
= r .
Exemplu: n Z
6
elementele inversabile sunt

1 si

5.
3.2.62 Idealele lui Z
n
= Z/
nZ
sunt de forma J/
nZ
cu proprietatea ca nZ J unde J este ideal n Z,
adica J = mZ. Deci idealele lui Z
n
sunt de forma mZ/
nZ
= m Z
n
= ( m) cu proprietatea m[ n.

In Z
21
avem mult imea divizorilor lui 21, D
21
= 1, 3, 7, 21 deci
I
1
= 1Z/
21Z
= Z
21
.
I
2
= 3Z/
21Z
= (

3) =

0,

3,

6,

9,

12,

15,

18.
I
3
= 7Z/
21Z
= (

7) =

0,

7,

14.
I
4
= 21Z/
21Z
= (

0) =

0.
Inelele factor vor : Z
21
/
I
1
Z/
Z
, inelul nul
Z
21
/
I
2
= a +I
2
[ a Z
21
=
_
I
2
;

1 +I
2
;

2 +I
2
_
=
=
_

0,

3,

6,

9,

12,

15,

18,

1,

4,

7,

10,

13,

16,

19,

2,

5,

8,

11,

14,

17,

20
_
Z
3
Z
21
/
I
3
= a +I
3
[ a Z
21
=
_
I
3
;

1 +I
3
;

2 +I
3
;

3 +I
3
;

4 +I
3
;

5 +I
3
;

6 +I
3
_
=
=
_

0,

7,

14,

1,

8,

15,

2,

9,

16,

3,

10,

17,

4,

11,

18,

5,

12,

19,

6,

13,

20
_
Z
7
.
Z
21
/
I
4
= a +I
4
[ a Z
21
=
_

0,

1,

2,

3, ...,

19,

20
_
Z
21
.
3.2.63 Prin calcul direct, folosind proprietat ile operat iilor cu mult imi se arata ca n raport cu diferent a
simetrica:
A B = (AB) (BA); A, B T(M),
(T(M), ) este un grup abelian, cu elementul neutru, mult imea vida si opusul oricarui element, el nsusi;
(T(M), ) este un nonoid comutativ cu elementul unitate, mult imea M, iar intersect ia este distributiva
fat a de diferent a simetrica. Cum A A = A, acest inel este inel Boole.
Fie aplicat ia : T(M) Z
M
2
care asociaza ecarei submult imi A M, funct ia sa caracteristica
f, deci (A) = f
A
: M Z
2
unde f
A
(x) =
_
0 daca x M A
1 daca x A
. Aplicat ia este bijectiva. Pentru
orice A, B T(M) avem (A B) = f
AB
: M Z
2
si f
AB
(x) =
_
0 daca x M A B
1 daca x A B
,iar
(A) +(B) = f
A
+f
B
.
Deoarece
(f
A
+f
B
)(x) = f
A
(x) +f
B
(x) =
_
0 daca x / Asi x / B sau (x Asi x B)
1 daca (x Asi x / B) sau (x Bsi x / A)
=
=
_
0 daca x (A B) (M A B)
1 daca x (A B) (B A)
= f
AB
(x),
3.2 Solut ii 69
rezulta (A B) = (A) +(B). Avem, de asemenea, (A) (B) = f
A
f
B
.
Cum (f
A
f
B
)(x) = f
A
(x) f
B
(x) =
_
0 daca x M A B
1 daca x A B
= f
AB
(x)
de aici rezulta ca (A B) = (A) (B), aceasta demonstreaza izomorsmul celor doua inele.
70 3. Probleme de algebr a
3.3 Probleme propuse
Grupuri
3.3.1 Sa se arate ca mult imea H = z C [[z[ = 1 este un subgrup n (C

, ) dar nu n (C, +).


3.3.2 Marcat i valoarea de adevar a urmatoarelor propozit ii justicand raspunsul:
1) Orice grup cu cel mult patru elemente este abelian.
2) Orice doua grupuri cu patru elemente sunt izomorfe.
3) Orice element al unui grup genereaza un subgrup ciclic al grupului.
4) (S
3
, ) unde S
3
este grupul permutarilor de ordinul 3 este comutativ.
5) Orice subgrup al unui grup comutativ este comutativ.
6) (, +) este ciclic.
7) (Z, +) este ciclic.
8) (S
3
, ) este ciclic.
9) Ordinul unui subgrup al unui grup nit divide ordinul grupului.
10) Indicele unui subgrup al unui grup nit divide ordinul grupului.
11) Ordinul unui element al unui grup nit divide ordinul grupului.
12) Se poate vorbi despre cat G/
N
daca si numai daca N este subgrup normal al grupului G.
13) Orice grup cat al unui grup nit este nit.
14) Orice grup cat al unui grup comutativ este comutativ.
15) Orice grup cat al unui grup necomutativ este necomutativ.
3.3.3 Fie G =
__
a b
b a
_

a, b R, a ,= 0 sau b ,= 0
_
. Sa se arate ca:
a) Mult imea G este grup abelian relativ la nmult irea matricilor.
b) Aplicat ia f : C

G, f(a +bi) =
_
a b
b a
_
este un izomorsm de la grupul (C

, ) la grupul
(G, ).
c)Pentru ecare n N

exista un unic subgrup de ordin n al grupului G.


Indicat ie. c) Cum C

are un unic subgrup de ordinul n si anume U


n
rezulta ca G are un unic
subgrup de ordin n si anume f(U
n
). (U
n
ind grupul radacinilor de ordinul n al unitat ii).
3.3.4 Fie f : C

, f(z) = [z[
2
.
a) Aratat i ca f este un omomorsm de grupuri multiplicative.
b) Determinat i Ker f si Imf. Studiat i injectivitatea si surjectivitatea lui f.
c) Aplicat i prima teorema de izomorsm.
3.3.5 Fie f : C R, f(x + iy) = x 2y. Sa se arate ca f este morsm surjectiv de grupuri aditive si
sa se determine Ker f.
3.3.6 Fie x, y R si x y = xy 5x 5y + 30. Este (R, ) grup? Dar (R 5, )?
3.3.7 Fie GL
n
(Z) = A /
n
(Z) [ det A este inversabil n (Z, ) = A /
n
(Z) [ det A 1, 1 si
SL
n
(Z) = A /
n
(Z) [ det A = 1. Sa se arate ca
a) GL
n
(Z) este un subgrup al lui (GL
n
(), ).
b) subgrupul lui (GL
n
(Z), ) generat de
_
1 1
0 1
_
este izomorf cu (Z, +).
c) SL
n
(Z) este un subgrup al lui GL
n
(Z).
3.3.8 Fie A =
_
0 1
1 0
_
, B =
_
0 1
1 1
_
. Sa se arate ca n (GL
n
(Z), ) avem ordA = 4, ordB = 3
si ord(AB) = .
3.3.9 Fie U
2
= 1, 1 grupul radacinilor de ordinul 2 al unitat ii. Sa se arate ca au loc urmatoarele
izomorsme de grupuri:
a) (C/
R
, +) (R, +); b) (/
U
2
, ) (

+
, ); c) (R

/
U
2
, ) (R

+
, ).
3.3 Probleme propuse 71
Indicat ie. Se arata ca funct iile urmatoare sunt omomorsme de grupuri si se aplica prima teorema
de izomorsm
a) f : C R, f(a +bi) = b; b) f :

+
, f(x) = [x[; c) f : R

+
, f(x) = [x[ .
3.3.10 Fie K = , R, C. Sa se arate ca SL
n
(K) este un subgrup normal al lui (GL
n
(K), ). Ce se
ntampla pentru K = Z?
Indicat ie. Se arata ca f : GL
n
(K) K

, f(A) = det A si g : GL
n
(Z) U
2
. g(A) = det A sunt
omomorsme de grupuri si se aplica prima teorema de izomorsm.
3.3.11 Sa se determine subgrupurile si grupurile cat ale lui (Z
8
, +) si (Z
15
, +).
3.3.12 Fie (G, ) un grup. Pentru ecare a G denim funct ia i
a
: G G, i
a
(x) = a a
1
. Sa se arate
ca
a) i
a
este un automorsm al lui G, numit automorsmul interior determinat de a.
b) Mult imea I(G) = i
a
[ a G este un subgrup normal al grupului (Aut(G), ) al tuturor auto-
morsmelor lui G.
c) Aplicat ia : (G, ) (I(G), ), (a) = i
a
este un morsm surjectiv de grupuri.
d) G este abelian daca si numai daca I(G) = i
e
.
3.3.13 Fie H
1
=
__
0 a
0 b
_

a, b Z
_
si H
2
=
__
2a 3b
4c 5d
_

a, b, c, d Z
_
. Aratat i ca H
1
si H
2
sunt subgrupuri ale grupului (/
2
(Z), +).
3.3.14 a) Daca H
1
si H
2
sunt subgrupuri ale unui grup G aratat i ca H
1
H
2
este subgrup al lui G. Cine
este subgrupul determinat de reuniunea lui H
1
si H
2
?
b) Aratat i ca pentru subgrupurile 4Z si 6Z ale grupului (Z, +) avem 4Z6Z = 12Z si 4Z+6Z = 2Z.
3.3.15 Fie n > 1 ntreg impar si e operat ia denita pe R prin x y =
n

x
n
+y
n
. Sa se arate ca (R, )
este un grup izomorf cu (R, +).
Indicat ie. f : R R, f(x) =
n

x este un izomorsm al lui (R, +) pe (R, ).


3.3.16 Fie (G, ) un grup abelian cu proprietatea ca pentru orice n N

mult imea U
n
= x G[ x
n
= e
are n elemente. Sa se arate ca:
a) U
n
este subgrup al lui G.
b) Orice subgrup nit al lui G este un anumit U
n
.
c) U
n
U
m
n[ m.
d) U
n
U
m
= U
d
unde d = (n, m) c.m.m.d.c. al lui m si n.
Inele. Corpuri.
3.3.17 Un numar d Z se numeste ntreg liber de patrate daca d ,= 1 si d nu se divide prin patratul nici
unui numar prim.
Fie d un ntreg liber de patrate. Sa se arate ca:
a)

d / .
b) a, b si a +b

d = 0 a = b = 0.
c) Z[

d] =
_
a +b

a, b Z
_
este un subinel n (C, +, ) care cont ine pe 1 si acest subinel este
generat de
_
1,

d
_
.
d) Q(

d) =
_
a +b

a, b
_
este un subcorp al lui (R, +, ) si acest subcorp este generat de

d.
3.3.18 Sa se determine subinelul si subcorpul lui (R, +, ) generat de
_
1,
3

2
_
.
3.3.19 a) Sa se arate ca mult imea K =
__
a b
2b a
_

a, b
_
este un subcorp al lui (/
2
(), +, ).
b) Sa se arate ca corpul (

2) este izomorf cu corpul K.


3.3.20 Fie A =
__
a b
5b a
_

a, b Z
_
. Aratat i ca A este o parte stabila a lui /
2
(Z) n raport cu
adunarea si nmult irea matricilor si ca formeaza un inel comutativ si fara divizori ai lui zero n raport cu
72 3. Probleme de algebr a
operat iile induse.
3.3.21 Fie d Z un ntreg liber de patrate si K =
__
a b
bd a
_

a, b
_
. Sa se arate ca:
a) K este parte stabila n /
2
() n raport cu adunarea si nmult irea si formeaza corp n raport cu
operat iile induse.
b) Corpurile ((

d), +, ) si (K, +, ) sunt izomorfe.


Indicat ie. a) Se poate arata ca K este subcorp n (/
2
(), +, ).
3.3.22 Sa se dea exemple de part i stabile ale lui Rn raport cu + si , care nu sunt inele n raport
cu operat iile induse de + si , respectiv de part i stabile ale lui C n raport cu + si care
sunt corpuri sau inele n raport cu operat iile induse de + si . Folosind denit ia subinelului si
subcorpului sa se precizeze care din exemplele date mai sus sunt subinele sau subcorpuri ale lui (C, +, ).
3.3.23 Sa se dea exemplu de subinel al unui inel cu unitate care nu cont ine unitatea si de subinel al unui
corp care nu este subcorp.
3.3.24 Sa se dea exemplu de subinel S al unui inel cu unitate, (R, +, ) care este, n raport cu operat iile
induse, un inel cu unitate dar unitatea sa este diferita de unitatea lui R.
Indicat ie. S =
__
a 0
0 0
_

a Z
_
/
2
(Z).
Elementul unitate n S este S =
_
1 0
0 0
_
.
3.3.25 Fie a, b, c R si x y = ax +by 2; x y = xy 2x 2y +c, () x, y R.
a) Sa se determine a, b, c astfel ncat (R, , ) sa e corp.
b) Sa se determine , R astfel ncat funct ia f : R R, f(x) = x + sa stabileasca un
izomorsm de la (R, +, ) la (R, , ).
3.3.26 Fie R un inel si f : R /
2
(R), f(x) =
_
x 0
0 x
_
. Sa se arate ca f este morsm injectiv de
inele.
3.3.27 Care dintre propozit iile urmatoare sunt adevarate:
1
0
. Orice subinel al unui inel comutativ este ideal.
2
0
. Orice ideal al unui inel comutativ este subinel.
3
0
. Daca I este un ideal stang sau drept al unui inel R atunci putem vorbi de un inel factor R/
I
.
4
0
. Daca I este un subinel al unui inel R atunci putem vorbi de un inel factor R/
I
. Justicat i
raspunsul.
3.3.28 Este intersect ia dintre un ideal stang si un ideal drept un ideal (bilateral)? Justicat i raspunsul.
3.3.29 Aratat i ca mult imile M =
__
a b
0 c
_

a, b, c R
_
si N =
__
a 0
0 0
_

a R
_
sunt subinele
cu unitate ale inelului (/
2
(R), +, ). Sunt ele si ideale ale acestui inel?
3.3.30 Aratat i ca S =
__
a 0
b 0
_

a, b Z
_
este subinel al inelului /
2
(Z). Este S ideal n /
2
(Z)
stang? Dar drept?
3.3.31 a) Aat i elementele inversabile si divizorii lui zero n inelul (Z
18
, +, ).
b) Aat i idealele si inelele cat n (Z
18
, +, ).
3.3.32 a) Fie mult imea A =
__
a b
b a
_

a, b Z
_
. Sa se arate ca A este un subinel unitar al inelului
(/
2
(Z), +, ).
b) Inelele (A, +, ) si (Z[i], +, ) sunt izomorfe.
3.3.33 Fie R un inel si n N, n 2. Sa se arate ca inelul /
n
(R) este comutativ daca si numai daca
3.3 Probleme propuse 73
R
2
= 0.
3.3.34 Fie K =
__
0 d
0 0
_

d Z
_
si T
2
(Z) =
__
a b
0 c
_

a, b, c Z
_
. Aratat i ca T
2
(Z)/
K
ZZ.
3.3.35 Fie Z[X] inelul polinoamelor cu coecient i ntregi n nedeterminata X si (X
2
d) idealul principal
generat de polinomul X
2
d. Sa se arate ca Z(

d] Z[X/
(X
2
d)
.
Indicat ie. Se arata ca aplicat ia f : Z[X] Z[

d], f(P) = P(

d) este un omomorsm surjectiv si


se aplica teorema fundamentala de izomorsm.
3.3.36 Fie inelul polinoamelor cu coecient i reali n nedeterminata X si f : R[X] C, f(P) = P(i)
unde P R[X].
a) Calculat i Kerf.
b) Aratat i ca R[X]/
(X
2
+1)
si C sunt izomorfe, unde prin (X
2
+1) nt elegem idealul principal generat
de polinomul X
2
+ 1.
3.3.37 Pe intervalul K = (0, ) denim legile de compozit ie
x y = xy
x y = x
ln y
pentru orice x, y K.
a) Aratat i ca tripletul (K, , ) este corp comutativ.
b) Aratat i ca corpurile (K, , ) si (R, +, ) sunt izomorfe.
Indicat ie. b) Se arata ca funct ia f : K R, f(x) = lnx este izomorsm de corpuri.
Bibliograe
[1] Andrica D., Duca I.D., Purdea I., Pop Ioana, Matematica de baza, Editura Studium, Cluj Napoca
2002
[2] Both N., Crivei S., Culegere de probleme de algebra, Litograa Universitat ii Babes-Bolyai, Cluj
Napoca, 1997
[3] Iancu Lacrimioara, Culegere de probleme de algebra, Litograa Universitat ii de Nord Baia Mare,
1996
[4] Pop S. Maria, Algebra, fascicula 1, Relat ii, Grupuri, Litograa Universitat ii de Nord Baia Mare,
2001
[5] Pop S. Maria, Algebra, fascicula 2, Inele, Litograa Universitat ii de Nord Baia Mare, 2002
[6] Purdea I., Pelea C., Probleme de algebra, Editura Fundat iei pentru Studii Europene, Cluj Napoca,
2005
[7] Purdea I., Pop Ioana, Algebra, Editura Gill, 2003
74
Partea II
Geometrie
75
Capitolul 1
Spat iul vectorial al vectorilor liberi

In natura se cunosc doua feluri de marimi zice:


a) marimi scalare care sunt caracterizabile cu ajutorul unui singur numar real (de exemplu: masa,
arie, volum etc.) si
b) marimi vectoriale care cer trei not iuni pentru a le deni: o valoare numerica numita modul sau
norma, o direct ie si sens (de exemplu: deplasare, viteza, accelerat ie, fort a, etc.).

In ideea de claricare a acestor trei caracteristici vom deni mai ntai not iunea de segment orientat.
1.1 Segmente orientate si vectorii liberi.
1.1.1 Denit ie. Numim segment de dreapta mult imea punctelor cuprinse ntre doua puncte ale unei
drepte.
Fie d o dreapta si punctele A, B d. Segmentul de dreapta cuprins ntre A si B, fara aceste doua
puncte se numeste segmentul deschis AB si se noteaza (AB).
(AB) A = [AB)se numeste segmentul AB nchis n A si deschisa n B.
(AB) A, B = [AB]se numeste segmentul nchis AB.
Drumul cel mai scurt ntre doua puncte este cel parcurs pe segmentul de dreapta care le uneste.
Cum n orice geometrie prin doua puncte diferite trece o singura dreapta, segmentul de dreapta care le
uneste este unic determinat. La o deplasare de la punctul A la punctul B drumul cel mai scurt este pe
[AB] si este sucient sa cunoastem doar punctul de plecare A si cel de sosire B n spat iul euclidian real
cu 3 dimensiuni, E
3
.
1.1.2 Observat ie. E
1
- dreapta, E
2
- planul, E
3
- spat iul.
1.1.3 Denit ie. Numim segment orientat AB perechea ordonata de puncte (A, B) E
3
E
3
,
unde A este originea si B varful (extremitatea) segmentului orientat.
1.1.4 Denit ie. Distant a ntre A si B se numeste norma (modulul) segmentului orientat AB:
d(A, B) = AB =| AB | .
1.1.5 Observat ie. | AB |=| BA | (A, B) E
3
E
3
.
1.1.6 Denit ie. Spunem ca AB si CD au aceeasi direct ie daca dreptele lor de suport sunt egale
sau paralele, adica
AB = CD sau AB | CD.
1.1.7 Denit ie. Doua segmente orientate AB si CD au acelasi sens daca prin translat ia lui CD paralel
cu el nsusi astfel ncat C sa se mute n A si masura unghiului

D
t
AB 90

, unde D
t
este translatatul
lui D.

In caz contrar spunem ca cele doua segmente orientate au sens contrar.
1.1.8 Observat ie. Daca C, D AB, atunci alunecand segmentul orientat CD astfel ncat C sa se
suprapuna peste A si D sa se mute n D
t
, n cazul n care D
t
[AB spunem ca AB si CD au acelasi
sens.
76
1.1 Segmente orientate si vectorii liberi. 77
1.1.9 Denit ie. Segmentele orientate AB si CD sunt echipolente daca au aceeasi norma, direct ie
si sens. Se noteaza AB CD.
Se poate verica usor ca relat ia de echipolent a este o relat ie de echivalent a n E
3
E
3
, adica are
urmatoarele 3 proprietat i:
1. () AB E
3
E
3
AB AB (reexivitate)
2. Daca AB CD atunci si CD AB (simetrie)
3. AB CD si CD EF AB EF (tranzitivitate)
Relat ia de echipolent a
tt
ind o relat ie de echivalent a n E
3
E
3
, nseamna ca mult imea tuturor
segmentelor orientate E
3
E
3
poate mpart ita n clase de echivalent a cu ajutorul acestei relat ii.
1.1.10 Denit ie. Numim vector liber AB (notat si

AB) clasa de echivalent a a segmentelor orientate


echipolente cu AB.
1.1.11 Denit ie. Daca CD AB (Fig. 3.1) atunci CD AB si se scrie
CD = AB (

CD =

AB)
Fig. 3. 1
1.1.12 Observat ie.

In cart i stiint ice vectorii liberi se noteaza cu litere ngrosate. Notat ia cu
sageata deasupra este practica atunci cand utilizam scrierea de mana (notit e).
1.1.13 Denit ie. Mult imea vectorilor liberi V
3
se numeste spat iul vectorial al vectorilor liberi
cu n dimensiuni, daca
AB V
3
si AB E
3
E
3
.
Pentru n = 1, V
1
este mult imea vectorilor situati pe o dreapta, si vectorii n acest caz se numesc vectori
alunecatori. V
2
- vectorii din plan; V
3
- vectorii din spat iu. Vectorii cu origine xata se numesc vectori
legat i.

In continuare vom nzestra mult imea vectorilor liberi cu doua operat ii, una interna, suma vectoriala
alta externa, nmult irea unui vector cu un scalar care i confera o structura de spat iu vectorial.
1.1.14 Denit ie. Numim suma vectoriala o lege de compozit ie interna
+ : V
3
V
3
V
3
unde pentru a AB V
3
si b AC V
3
astfel ncat c = a+b AD, unde ABCD este un paralelogram
din E
3
.(Fig. 3.2)
Fig. 3. 2
Aceasta metoda de nsumare a 2 vectori liberi poarta numele de metoda paralelogramului. Ea
provine din experient a practica a compunerii a doua fort e. Daca consideram
a+b AB +BD = AD,
atunci spunem ca am utilizat metoda triunghiului de nsumare a vectorilor liberi. Metoda triunghiului
se poate generaliza la mai mult i vectori liberi considerand originea vectorilor n varful vectorilor precedent i
si vectorul suma ind vectorul cu originea n originea primului vector si varful n extremitatea ultimului
vector (Fig. 3.3.).

In acest caz avem de a face cu metoda poligonului.
s = a
1
+a
2
+a
3
+a
4
.
Fig. 3. 3
1.1.15 Teorema. (V
3
, +) are structura algebrica de grup abelian, adica:
a) ()a, b V
3
; a +b = b +a;
b) 0 V
3
; ()a V
3
, 0 +a = a +0 = a;
c) () a, b, c V
3
(a+b)+c=a+(b+c);
78 1. Spat iul vectorial al vectorilor liberi
d) () a V
3
, () a V
3
, a + (a) = (a) +a = 0.
Demonstrat ie.
a) Comutativitatea rezulta imediat din nsasi metoda paralelogramului;
b) Vectorul liber nul este 0 = AA, si are norma egala cu 0;
c) Asociativitatea rezulta din metoda poligonului aplicata la 3 vectori liberi (Fig. 3.4)
Fig. 3.4
(a +b) +c (AB +BC) +CD = AC +CD = AD,
a + (b +c) AB + (BC +CD) = AB +BD = AD.
Deci (a +b) +c = a + (b +c) oricare ar a, b, c V
3
.
d) Fie a AB, luand a BA avem
a + (a) AB +BA = AA = 0,
(a) +a BA+AB = BB = 0.
Produsul unui vector cu un scalar
Aceasta lege de compozit ie provine din adunarea repetata a vectorilor liberi. Fie a V
3
si m N,
atunci
a +a +. . . +a
. .
de m ori
= ma.
1.1.16 Denit ie. Prin produsul ntre scalarul k R si vectorul liber a V
m
nt elegem o lege
de compozit ie externa
: R V
3
V
3
astfel ncat () k R si () a V
3
, ka = b V
3
are urmatoarele proprietat i:
a) |ka| = [k[ |a| ;
b) b = ka are aceeasi direct ie cu a;
c) sens ka =
_
sens a, daca k > 0
sens a, daca k < 0
si 0 a = 0.
Se arata usor ca aceasta operat ie are urmatoarele proprietat i: () k, l R si () a, b V
3
avem
a
sv
) 1 a = a;
b
sv
) k (l a) = (k l) a;
c
sv
) (k +l) a = (ka) + (la);
d
sv
) k(a +b) = (ka) + (kb)
Proprietat ile de mai sus denesc produsul unui vector (generalizat) cu un scalar pentru structura
algebrica de spat iu vectorial. Reamintesc din algebra liniara ca un grup abelian (V, +) este un spat iu
vectorial peste campul de scalari K daca exista o lege de compozit ie externa
: K V V,
numit produsul unui vector cu un scalar cu proprietat ile a
sv
) d
sv
).
Prin urmare V
3
este un spat iu vectorial peste campul (corpul comutativ) R.
1.1.17 Teorema. Din proprietat ile produsului unui vector liber cu un scalar rezulta urmatoarele
proprietat i derivate: () k, l R si () a, b V
3
avem
1.) k 0 = 0 a = 0;
2.) 1 a = a;
3.) (ka) = (k)a = k(a);
4.) (k l)a = (ka) (la);
1.2 Repere carteziene 79
5.) k(a b) = (ka) (kb).
Demonstrat ie. 1.) k 0 = k[0 +0] = k 0 +k 0. Notand cu x = k0, avem x = x +x x = 0 si
0 a = (0 + 0)a = 0 a + 0 a, notand cu y = 0 a, rezulta y = y +y y = 0.
2.) Din 1.) rezulta 0 = 0 a = [1 + (1)a] = (1 a) + [(1)a] = a + [(1)a] care ne arata ca
(1)a = a, () a V
3
3.) Din proprietatea b
sv
) avem
(k)a = [(1) k]a = (1) (ka) = (ka)
si
k(a) = k[(1)a] = [k (1)]a = (k)a.
4.) Din c
sv
) si 3.) rezulta
(k l)a = [k + (l)]a = (ka) + [(l)a] = ka + (la) = (ka) (la).
5. Din d
sv
) si relat ia 3.) avem
k(a b) = k(a + (b)] = (ka) + [k (b)] = (ka) + (kb) = (ka) (kb).
1.2 Repere carteziene
Proiect ia unui vector liber pe o axa
1.2.1 Denit ie. Un vector liber cu norma 1 se numeste vector unitate sau versor.
1.2.2 Denit ie. O dreapta pe care s-a xat un punct 0 numit origine, orientata n aceeasi direct ie
si sens cu ale unui versor dat u, se numeste axa.
Un plan este determinat de o dreapta si un punct exterior acesteia, deci ntotdeauna are sens sa
vorbim despre dreapta perpendiculara dusa din punctul respectiv pe dreapta data. Piciorul A
t
al per-
pendicularei duse din punctul A pe dreapta d adica intersect ia perpendicularei respective cu dreapta d,
se numeste proiect ie ortogonala a punctului A pe dreapta d.
1.2.3 Denit ie. Prin vectorul proiect ie al vectorului liber v pe axa 0xnt elegem acel vector liber
care este reprezentat prin segmentul orientat A
t
B
t
, unde A
t
si B
t
sunt proiect iile ortogonale pe axa 0x a
originii A, respectiv a varfului B, daca AB v (Fig. 3.5).
Fig. 3. 5
1.2.4 Denit ie. Prin proiect ia ortogonala a lui v pe u, notata prin pr
u
v se nt elege scalarul
pr
u
v =
_
|A
t
B
t
| , daca sens A
t
B
t
= sens u;
|A
t
B
t
| , daca sens A
t
B
t
= sens u.
si reprezinta masura algebrica a segmentului A
t
B
t
= x
B
x
A
, unde x
A
si x
B
sunt abscisele punctelor A
t
si B
t
pe axa 0x. Din Fig. 3. 5 se poate vedea ca
pr
u
v = |v| cos ,
unde = m(

u, v) este masura unghiului dintre vectorii u si v.


1.2.5 Denit ie. Vectorul de proiect ie al vectorului liber v pe direct ia versorului u este vectorul
(pr
u
v) u
1.2.6 Proprietat i. 1.) Proiect ia ortogonala a sumei de vectori liberi este egala cu suma proiect iilor
vectorilor (Fig. 3.6)
2.) pr
u
(kv) = k pr
u
v, () k R si () v V
3
(Fig. 3.7).
80 1. Spat iul vectorial al vectorilor liberi
Demonstrat ie. 1.)
Fig. 3.6
pr
u
(v
1
+v
2
) = x
3
x
1
pr
u
v
1
+pr
u
v
2
= (x
2
x
1
) + (x
3
x
2
) = x
3
x
1
= pr
u
(v
1
+v
2
).
Prin induct ie matematica rezulta ca aceasta proprietate este aplicabila si la suma mai multor vectori
liberi.
2.)
Fig. 3.7
Fie = m(

u, v), pr
u
v = |v| cos = |AB| cos .
Daca k 0, pr
u
(kv) = |kAB| cos = [k[ |AB| cos = pr
u
v;
Daca k < 0 pr
u
(kv) = |kAB| cos = [k[ |AB| cos = pr
u
v.
Reprezentarea vectorilor liberi din plan
Fie un punct M E
2
si e reperul ortonormat (0, i, j) al spat iului vectorial V
2
al vectorilor liberi
din planul E
2
, unde 0 reprezinta originea axelor ortogonale de coordonate 0x si 0y, iar i si j sunt versorii
acestor axe. Daca v 0M este vectorul de pozit ie al punctului M E
2
, atunci spunem ca x = pr
i
v este
abscisa si y = pr
j
v ordonata punctului M. Perechea ordonata de numere (x, y) R
2
poarta numele
de coordonate carteziene ale punctului M n baza B = i, j. Denumirea acestor coordonate provine
de la numele omului de stiint a francez Descartes (1596-1650) care a pus bazele geometriei analitice si al
carui nume latinizat fusese Cartesius.
Fig. 3.8
Din Fig. 3.8 se observa ca aplicand regula paralelogramului avem
0A+0B=0M,
adica
v = xi +yj,
unde B = i, j este baza ortonormata canonica a spat iului vectorial V
2
.
Cuvantul ortonormat este compus din cuvintele ortogonal = perpendicular si normat deoarece i j
si |i| = |j| = 1.
Norma (modulul) vectorului v reprezinta distant a 0M, care dupa teorema lui Pitagora este egala cu
| v| = | 0M| =
_
x
2
+y
2
Fig. 3.9
Din Fig. 3.9 se observa ca
AB = 0B0A = (x
B
x
A
)i + (y
B
y
A
)j,
deci distant a AB, ca si norma vectorului AB, va egala cu
d(A, B) = AB = | AB| =
_
(x
B
x
A
)
2
+ (y
B
y
A
)
2
.
Reprezentarea vectorilor liberi din spat iu
Sa consideram acum un punct M din spat iul euclidian cu 3 dimensiuni, E
3
. Reperul ortonormat
canonic al spat iului vectorial V
3
este (0, i, j, k) unde i, j si k sunt versorii axelor de coordonate 0x, 0y si
0z.

In acest caz, n locul unui dreptunghi cu diagonala 0M se construieste un paralelipiped dreptunghic
cu diagonala 0M ca n Fig. 3.10.
1.3 Produse de vectori 81
Fig. 3.10
Observam ca
v = 0M = 0N+0C = 0A+0B+0C = xi +yj +zk
si
| v| = d(0M) =
_
x
2
+y
2
+z
2
.
Distant a ntre punctele A(x
A
, y
A
, z
A
) si B(x
B
, y
B
, z
B
) este
d(A, B) = AB =
_
(x
B
x
A
)
2
+ (y
B
y
A
)
2
+ (z
B
z
A
)
2
,
iar AB se poate exprima sub forma
AB = 0B0A = (x
B
x
A
)i + (y
B
y
A
)j + (z
B
z
A
)k.
1.3 Produse de vectori
Produsul scalar a doi vectori liberi

In algebra liniara produsul scalar al vectorilor dintr-un spat iu vectorial V peste campul C al numerelor
complexe se deneste astfel:
1.3.1 Denit ie. Fie un spat iu vectorial V peste campul C. Prin produsul scalar a doi vectori din
V nt elegem o aplicat ie
(, ) : V V C
() v
1
, v
2
V , () (v
1
, v
2
) C astfel ncat pentru () v
1
, v
2
, v
3
V si () k C sa aiba loc:
1.) (v
1
, v
2
) = (v
2
, v
1
), unde (v
2
, v
1
) este conjugatul numarului complex (v
2
, v
1
);
2.) (v
1
, v
2
+v
3
) = (v
1
, v
2
) + (v
1
, v
3
);
3.) k(v
1
, v
2
) = (kv
1
, v
2
);
4.) (v
1
, v
2
) 0 ; (v
1
, v
2
) = 0 v
1
= 0.
1.3.2 Denit ie. Un spat iu vectorial cu un produs scalar se numeste spat iu vectorial euclidian.
1.3.3 Observat ie. Daca scalarii sunt reali, atunci proprietatea 1. 3. 1. 1.) devine
1

.) (v
1
, v
2
) = (v
2
, v
1
).
1.3.4 Teorema. Legea de compozit ie
: V
3
V
3
R,
() v
1
, v
2
V
3
, () k = v
1
v
2
R astfel ncat
k = | v
1
| | v
2
| cos( v
1
, v
2
) = | v
1
| pr
v
1
v
2
= | v
2
| pr
v
2
v
1
(1.3.1)
deneste n spat iul real al vectorilor liberi 3dimensional V
3
un produs scalar.
Demonstrat ie. Fie v
1
, v
2
, v
3
V
3
si () k R si = m(v
1
, v
2
).
1.) v
1
v
2
= | v
1
| | v
2
| cos = | v
2
| | v
1
| cos() = v
2
v
1
;
2.) v
1
(v
2
+v
3
) = | v
1
| pr
v
1
(v
2
+v
3
) = | v
1
| (pr
v
1
v
2
+pr
v
1
v
3
)
= | v
1
| pr
v
1
v
2
+| v
1
| pr
v
1
v
3
= (v
1
v
2
) + (v
1
v
3
);
3.) k(v
1
v
2
) = k (| v
2
| pr
v
2
v
1
) = (k pr
v
2
v
1
) | v
1
| = [pr
v
2
(kv
1
)] | v
2
| = v
2
(kv
1
) = (kv
1
) v
2
;
4.) v
1
v
1
= | v
1
| | v
1
| cos 0 = | v
1
|
2
0 si v
1
v
1
= | v
1
|
2
= 0 | v
1
| = 0 v
1
= 0.
Ultima proprietate ne spune ca patratul unui vector (n sensul produsului scalar) este egal cu patratul
normei sale
v
2
= | v|
2
, () v V
3
. (1.3.2)
82 1. Spat iul vectorial al vectorilor liberi
Din cele 4 proprietat i care denesc un produs scalar aplicate la spat iul vectorial euclidian real al
vectorilor liberi 3dimensionali V
3
rezulta si alte proprietat i.
1.3.5 Teorema Fie spat iul vectorial al vectorilor liberi 3dimensionali V
3
nzestrat cu produsul
scalar denit prin formula (1.3.1). Pentru orice v
1
, v
2
, v
3
V
3
si () k R au loc urmatoarele relat ii:
a) v
1
(kv
2
) = k(v
1
v
2
);
b) (v
1
+v
2
) v
3
= (v
1
v
3
) + (v
2
v
3
);
c) v
1
0 = 0 v
1
= 0;
d) (v
1
v
2
)
2
v
2
1
v
2
2
-inegalitatea Cauchy-Buniacovski-Schwarz
Demonstrat ie. a) v
1
(kv
2
)
1.)
= (kv
2
)v
1
3.)
= k(v
2
v
1
)
1.)
= k(v
1
v
2
);
b) (v
1
+v
2
) v
3
1.)
= v
3
(v
1
+v
2
)
2.)
= (v
3
v
1
) + (v
3
v
2
)
1.)
= (v
1
v
3
) + (v
2
v
3
);
c) v
1
0 = v
1
(0+0)
2.)
= (v
1
0) +(v
1
0). Notand cu l = v
1
0 R, rezulta l = l +l de unde obt inem
l = 0 = v
1
0
1.)
= 0 v
1
;
d) Daca v
2
= 0, atunci 0 = 0 si proprietatea este adevarata.
Fie v
2
,= 0 si scalarul k =
v
1
v
2
v
2
v
2
. Atunci (v
1
v
2
) k(v
2
v
2
) = 0, adica (v
1
kv
2
) v
2
= 0. Din
1.3.1.4.) rezulta ca
(v
1
kv
2
) (v
1
kv
2
) 0
sau
[(v
1
kv
2
) v
1
] k[(v
1
kv
2
)v
2
] 0,
care pentru orice k ales devine (v
1
kv
2
) v
1
0 sau (v
1
v
2
)
v
1
v
2
v
2
v
2
(v
2
v
1
) 0. Cum v
2
v
2
> 0,
rezulta
(v
1
v
1
) (v
2
v
2
) (v
1
v
2
)
2
.
1.3.6 Observat ie. Proprietat ile din teorema 1.3.5 sunt adevarate n orice spat iu vectorial euclidian.
Inegalitatea Cauchy-Buniacovski-Schwarz se poate scrie si sub forma
[v
1
v
2
[ |v
1
| |v
2
| ,
adica
[v
1
v
2
[
|v
1
| |v
2
|
1 pentru orice v
1
, v
2
V
3
0.
Putem deni unghiul a doi vectori liberi 3dimensionali nenuli unghiul = m

(v
1
, v
2
) unde
cos =
[v
1
v
2
[
|v
1
| |v
2
|
, v
1
,= 0 si v
2
,= 0,
care justica formula (1.3.1).
Din formula (1.3.2) putem calcula norma unui vector:
|v| =

v v. (1.3.3)
1.3.7 Denit ie. Spunem ca doi vectori nenuli ai unui spat iu vectorial euclidian real V sunt ortog-
onali daca produsul lor scalar este egal cu 0 :
v
1
,= 0, v
2
,= 0, v
1
v
2
v
1
v
2
= 0.
1.3.8 Observat ie. v
1
v
2
= 0 v
1
v
2
sau v
1
= 0 sau v
2
= 0
1.3 Produse de vectori 83
1.3.9 Denit ie. Numim versor (vector unitate) al unui vector dat v V
3
0 vectorul
u =
v
| v|
.
Aceasta denit ie se justica prin faptul ca
| u| =
_
_
_
_
v
| v|
_
_
_
_
=
| v|
| v|
= 1,
iar sensul si direct ia lui u coincid cu cele ale lui v deoarece scalarul
1
| v|
este strict pozitiv.
Daca baza B nu este ortonormata atunci expresia produsului scalar devine o forma biliniara simetrica,
exprimata cu ajutorul tensorilor.

In cazul vectorilor liberi din plan, cu baza ortonormaa B = i, j, unde a = a


x
i +a
y
j, b = b
x
i +b
y
j
avem
a b = a
x
b
x
+a
y
b
y
.
Pentru vectorii liberi din spat iul V
3
, cu baza ortonormata B = i, j, k si a = a
x
i + a
y
j + a
z
k,
b = b
x
i +b
y
j +b
z
k avem
a b = (a
x
i +a
y
j +a
z
k) (b
x
i +b
y
j +b
z
k) = a
x
b
x
+a
y
b
y
+a
z
b
z
.
Aplicand formula (1.3.3) pentru un vector 3dimenional scris n baza ortonormata B = i, j, k se
obt ine formula de calcul pentru norma vectorului a V
3
, adica
| a| = |a
x
i +a
y
j +a
z
k| =
_
a
2
x
+a
2
y
+a
2
z
. (1.3.4)
Fie acum doua puncte A si B din spat iul euclidian E
3
cu n dimensiuni. Daca pentru spat iul vectorial
V
3
al vectorilor liberi din E
3
alegem ca baza baza ortonormata B = i, j, k, proiect iile vectorilor de pozit ie
0A = a si 0B = b n reperul (0, B) vor coordonatele carteziene ale celor doua puncte A(a
x
, a
y
, a
z
) si
B(b
x
, b
y
, b
z
) si
AB = 0B0A = b a = (b
x
a
x
)i + (b
y
a
y
)j + (b
z
a
z
)k. (1.3.5)
T inand seama de formulele (1.3.4) si (1.3.5) obt inem pentru distant a ntre punctele A si B formula
d(A, B) = AB = | AB| =
_
(b
x
a
x
)
2
+ (b
y
a
y
)
2
+ (b
z
a
z
)
2
.
Pentru unghiul dintre vectorii a si b avem:
n plan, V
2
cos =
a b
|a| |b|
=
a
x
b
x
+a
y
b
y
_
a
2
x
+a
2
y

_
b
2
x
+b
2
y
,
n spat iu, V
3
cos =
a b
|a| |b|
=
a
x
b
x
+a
y
b
y
+a
z
b
z
_
a
2
x
+a
2
y
+a
2
z

_
b
2
x
+b
2
y
+b
2
z
.
1.3.11 Teorema lui Pitagora n V
3
. Fie vectorii b, c V
3
, b c. Are loc relat ia
(b c)
2
= b
2
+c
2
.
Demonstrat ie.
(b c)
2
= (b c)(b c) = b
2
bc cb +c
2
= b
2
+c
2
(Fig.3.11)
Fig. 3.11
84 1. Spat iul vectorial al vectorilor liberi
Notand cu a = b c, vectorul BC al ipotenuzei, rezulta
a
2
= b
2
+c
2
,
adica
| a|
2
= | b|
2
+| c|
2
.
Produsul vectorial a doi vectori liberi din V
3
1.3.12 Denit ie. Prin produsul vectorial a vectorilor liberi v
1
si v
2
din spat iul V
3
, se nt elege o
lege de compozit ie interna a spat iului vectorial V
3
:
: V
3
V
3
V
3
, () v
1
, v
2
V
3
, () v
1
v
2
= v V
3
,
unde
| v
1
v
2
| = | v
1
| | v
2
| [sin[ , = m( v
1
, v
2
)
sensul lui v este dat de regula burghiului, adica este sensul de avansare a burghiului cand se roteste de
la vectorul v
1
spre v
2
.

In natura, momentul unei fort e este un exemplu potrivit pentru produsul vectorial ntre vectorul
fort ei si brat ul fort ei.
Fig. 3.12
Pentru interpretarea geometrica a normei | v
1
v
2
| sa observam ca aria triunghiului 0AB din Fig. 3.12
este egala cu
1
2
| v
1
| | v
2
| [sin[ , si deci
| v
1
v
2
| = /
0ABC
.
1.3.13 Proprietat i.
1. v
1
v
2
= v
2
v
1
; () v
1
, v
2
V
3
;
2. k(v
1
v
2
) = k(v
1
) v
2
= v
1
(kv
2
); () v
1
, v
2
V
3
, si () k R;
3. v
1
(v
2
+v
3
) = (v
1
v
2
) + (v
1
v
3
); () v
1
, v
2
, v
3
V
3
;
4. v
1
v
2
= 0 v
1
si v
2
au aceeasi direct ie sau v
1
= 0 sau v
2
= 0
Demonstrat ie. 1. Sensul de miscare a burghiului cand se roteste de la v
2
la v
1
este invers sensului
de deplasare a acestuia cand se roteste de la v
1
la v
2
.
4. = 0 sau = . Cum sin0 = sin = 0, rezulta ca |v
1
v
2
| = |v
1
| |v
2
| 0 = 0, deci
v
1
v
2
= 0.
Pentru demonstrat ia proprietat ilor 2 si 3 sa determinam formula de calcul a produsului vectorial
cand vectorii sunt exprimat i n baza canonica B = i, j, k:
v
1
v
2
= (x
1
i +y
1
j +z
1
k) (x
2
i +y
2
j +z
2
k).
Fig. 3.13
Cum |i| = |j| = |k| = 1 si i j k i si sin

2
= 1, din Fig. 3.13 se observa ca i j = k etc. si
rezulta urmatoarea tabla de operat ie
i j k
i 0 k -j
i -k 0 i
k j -i 0
1.3 Produse de vectori 85
Obt inem
v
1
v
2
= x
1
y
2
k x
1
z
2
j y
1
x
2
k +y
1
z
2
i +z
2
x
2
j z
1
y
2
i,
adica
v
1
v
2
=

i j k
x
1
y
1
z
1
x
2
y
2
z
2

.
2.
k(v
1
v
2
) = k(x
1
i +y
1
j +z
1
k) (x
2
i +y
2
j +z
2
k) = k

i j k
x
1
y
1
z
1
x
2
y
2
z
2

=
=

i j k
kx
1
ky
1
kz
1
x
2
y
2
z
2

= (kv
1
) v
2
=

i j k
x
1
y
1
z
1
kx
2
ky
2
kz
2

= v
1
(kv
2
).
3.
v
1
(v
2
+v
3
) =

i j k
x
1
y
1
z
1
x
2
+x
3
y
2
+y
3
z
2
+z
3

i j k
x
1
y
1
z
1
x
2
y
2
z
2

+
+

i j k
x
1
y
1
z
1
x
2
y
2
z
2

= (v
1
v
2
) + (v
1
v
3
).
Produsul mixt a trei vectori liberi din V
3
1.3.14 Denit ie. Scalarul (v
1
, v
2
, v
3
) = v
1
(v
2
v
3
) R unde v
1
, v
2
, v
3
V
3
se numeste
produsul mixt al vectorilor v
1
, v
2
si v
3
. Cum
v
1
(v
2
v
3
) = (x
1
i +y
1
j +z
1
k)

i j k
x
2
y
2
z
2
x
3
y
3
z
3

x
1
y
1
z
1
x
2
y
2
z
2
x
3
y
3
z
3

,
unde i, j, k din determinant se nlocuiesc cu x
1
, y
1
si z
1
. Astfel
(v
1
, v
2
, v
3
) = (x
1
i +y
1
j +z
1
k) [(x
2
i +y
2
j +z
2
k) (x
3
i +y
3
j +z
3
k)] =

x
1
y
1
z
1
x
2
y
2
z
2
x
3
y
3
z
3

.
Din proprietat ile determinant ilor rezulta ca produsul mixt este invariant la o permutare circulara a
vectorilor
1. (v
1
, v
2
, v
3
) = (v
2
, v
3
, v
1
) = (v
3
, v
1
, v
2
),
iar prin schimbarea ntre ei a doi vectori n produsul mixt se schimba semnul acestuia.
2. (v
2
, v
1
, v
3
) = (v
1
, v
2
, v
3
).
Pentru interpretarea geometrica a produsului mixt, e Fig. 3.14.
Fig. 3.14
[v
1
, v
2
, v
3
)[ = [v
1
(v
2
v
3
)[ = [|(v
2
v
3
)| pr
v
2
v
3
v
1
[ = /
0ADB

0C

=
= Volumul paralelipipedului cu muchiile v
2
, v
1
si v
3
.
1.3.15 Observat ie. Vectorii coplanari au produsul mixt egal cu 0.
Dublul produs vectorial a trei vectori
86 1. Spat iul vectorial al vectorilor liberi
1.3.16 Denit ie. Prin dublu produs vectorial al vectorilor a, b, c V
3
nt elegem
a (b c) V
3
.
1.3.19 Teorema. Dublul produs vectorial al vectorilor a, b, c V
3
se poate calcula mai usor cu
ajutorul formulei lui Gibbs:
a (b c) =

b c
a b a c

.
Demonstrat ia formulei se face exprim and cele doua expresii n baza ortonormata B = i, j, k.
Capitolul 2
Elemente de geometrie analitica
liniara
2.1 Reprezentarile analitice ale dreptei n plan si n spat iu. Ecuat ia
planului.
2.1.1. Dreapta determinata de un punct si de un vector director

In V
3
alegem pentru exprimarea vectorilor reperul ortonormat (0, B) = (0, i, j, k). Fie M
0
un punct
de coordonate carteziene (x
0
, y
0
, z
0
) si un vector v V
3
de coordonate (l, m, p). Se cer ecuat iile dreptei
paralele cu vectorul v care trece prin punctul M
0
(Fig. 3.15)
Fig. 3.15
Se ia un punct mobil M(x, y, z) pe dreapta (d). Cum v | (d), rezulta ca exista un scalar t R astfel
ncat
M
0
M = tv,
adica
(x x
0
)i + (y y
0
)j + (z z
0
)k = t(li +mj +pk).
Rezulta forma parametrica a ecuat iilor dreptei (d):
(d) :
_
_
_
x = x
0
+tl
y = y
0
+tm
z = z
0
+tp,
(2.1.1)
unde t R se numeste parametru.
Ecuat ia (2.1.1) are si o interpretare geometrica. Daca t reprezinta timpul si v este viteza punctului
M pe dreapta (d), atunci ecuat ia (2.1.1) permite determinarea pozit iei punctului M fat a de punctul
init ial M
0
pentru ecare moment de timp t R. Cand t = 0, atunci M = M
0
.
Se obisnuieste sa se scrie ecuat iile dreptei (d) si sub forma unui sir de rapoarte egale, ecuat iile
carteziene:
(d) :
x x
0
l
=
y y
0
m
=
z z
0
p
, (2.1.2)
cu precizarea ca daca n ecuat iile (2.1.2) vreunul din numitori este nul, atunci si numaratorul core-
spunzator este (prin convent ie) nul.

In planul E
2
, pentru reprezentarea unei drepte, avem ecuat iile
(d) :
x x
0
p
=
y y
0
q
,
unde M(x, y), M
0
(x
0
, y
0
) si v = pi +qj.
87
88 2. Elemente de geometrie analitic a liniar a
2.1.2 Dreapta determinata de doua puncte
Fie M
1
(x
1
, y
1
, z
1
), M
2
(x
2
, y
2
, z
2
) doua puncte din E
3
.

In acest caz vom lua M
0
= M
1
si vectorul
director v = M
1
M
2
. Din (2.1.2) rezulta
(M
1
M
2
) :
x x
1
x
2
x
1
=
y y
1
y
2
y
1

z z
1
z
2
z
1
. (2.1.3)

In plan se obt ine:


(M
1
M
2
) :
x x
1
x
2
x
1
=
y y
1
y
2
y
1
, (2.1.4)
cu M
1
(x
1
, y
1
) si M
2
(x
2
, y
2
).
2.1.3 Observat ie. Daca vectorul director v are norma 1, atunci
v i = cos(

v, i), v j = cos
_

v, j
_
, c k = cos
_

v, k
_
iar numitorii ecuat iilor (2.1.2) se numesc cosinusii directori ai dreptei (d).
2.1.4 Dreapta din plan
Sa consideram dreapta (M
1
M
2
) data de ecuat ia (2.1.4). Daca efectuam toate calculele trecand tot i
termenii n membrul stang se arata usor ca ecuat ia obt inuta este echivalenta cu ecuat ia
(M
1
M
2
) :

x y 1
x
1
y
1
1
x
2
y
2
1

= 0.
Notand cu
m =
y
2
y
1
x
2
x
1
=
y
x
=
_
_
M
1
M
2
_
_
cos(

d, j)
_
_
M
1
M
2
_
_
cos(

d, i)
=
sin
cos
= tg
panta (coecientul unghiular) al dreptei (d). Ecuat ia dreptei care trece prin punctul M
1
(x
1
, y
1
) si
are panta m este:
(d) : y y
1
= m(x x
1
). (2.1.5)
O dreapta paralela cu axa 0x are ecuat ia
y = y
0
,
iar una paralela cu 0y:
x = x
0
.
Din ecuat ia (2.1.5) obt inem
y = mx + (y
1
mx
1
).
Notand cu n = y
1
mx
1
obt inem ecuat ia dreptei cand se cunoaste panta m a acesteia si ordonata
sa n la origine:
(d) : y = mx +n. (2.1.6)
Condit ia ca dreapta (d) sa treaca prin originea 0 a axelor de coordonate este ca n = 0.
Sa observam ca ecuat ia (2.1.6) cuprinde toate dreptele posibile din plan cu except ia celor verticale.
Ecuat ia unei drepte se poate scrie sub forma
(d) : Ax +By +C = 0,
numita ecuat ia generala a dreptei.
Daca A = 0, atunci (d) | 0x.
Daca B = 0, atunci (d) | 0y.
Daca C = 0, atunci 0 (d).
Presupunem ca B ,= 0, atunci y =
A
B
x
C
B
, deci m =
A
B
si n =
C
B
.
Daca consideram punctele A(a, 0) si B(0, b) (Fig. 3.16)
2.1 Reprezent arile analitice ale dreptei n plan si n spat iu. Ecuat ia planului. 89
Fig. 3.16
se obt ine ecuat ia dreptei prin taieturi:
(d) :
x
a
+
y
b
1 = 0.
2.1.5 Planul determinat de un punct si un vector normal
Fig. 3.17
Sa consideram planul (P), punctul M
0
(P) si vectorul n (P) (Fig.3.17). Un punct mobil descrie
planul (P) daca M
0
M (P). Avem astfel condit ia ca
n M
0
M,
adica
n M
0
M = 0.
Daca n = Ai +Bj +Ck , M
0
(x
0
, y
0
, z
0
) si M(x, y, z), atunci
n M
0
M = (Ai +Bj +Ck)[(x x
0
)i + (y y
0
)j + (z z
0
)k] = 0.
Se obt ine ecuat ia planului determinat de un punct M
0
si vectorul normal n:
(P) : A(x x
0
) +B(y y
0
) +C(z z
0
) = 0. (2.1.7)
2.1.6 Ecuat ia generala a planului
Efectuand calculele n formula (2.1.7) obt inem ecuat ia generala a planului
(P) : Ax +By +Cz +D = 0,
unde D = (Ax
0
+By
0
+Cz
0
).
Cazuri particulare:
daca A = 0 atunci (P) | 0x
daca B = 0 atunci (P) | 0y
daca C = 0 atunci (P) | 0z
daca D = 0 atunci 0 (P).
2.1.7 Planul determinat de trei puncte
Sa consideram un plan (P) si punctele M
1
(x
1
, y
1
, z
1
) , M
2
(x
2
, y
2
, z
2
) si M
3
(x
3
, y
3
, z
3
) situate n
planul (P). Fie un punct mobil M(x, y, z) (P) (Fig.3.18).
Fig. 3.18
Vectorii MM
1
, MM
2
si MM
3
ind coplanari, rezulta ca produsul lor mixt este egal cu 0. Deci
(MM
1
, MM
2
, MM
3
) =

x
1
x y
1
y z
1
z
x
2
x y
2
y z
2
z
x
3
x y
3
y z
3
z

= 0.
Completand acest determinant cu linia (x, y, z, 1) si cu coloana (1, 0, 0, 0) si adunand aceasta linie la
cele trei linii se obt ine ecuat ia planului (P) = (M
1
M
2
M
3
) scrisa sub forma:

x y z 1
x
1
y
1
z
1
1
x
2
y
2
z
2
1
x
3
y
3
z
3
1

= 0.
Considerand punctele planului (P) situate pe cele trei axe de coordonate: A(a, 0, 0) , B(0, b, 0) , C(0, 0, c)
obt inem ecuat ia planului prin taieturi
(P) :
x
a
+
y
b
+ +
z
c
1 = 0. (3.43)
Avem astfel o posibilitate pentru reprezentarea planului (P) n spat iu (Fig.3.19).
Fig. 3.19
90 2. Elemente de geometrie analitic a liniar a
2.2 Pozit iile relative ale punctelor, dreptelor si planelor din
spat iu
2.2.1 Intersect ia a doua drepte din plan
Sa consideram doua drepte (d) si (d

) din plan. Prin intersect ia lor se nt elege mult imea punctelor


lor comune, adica solut ia sistemului de ecuat ii formate din ecuat iile celor doua drepte
(d) : Ax +By +C = 0;
(d

) : A

x +B

y +C

= 0.
Notand cu =

A B
A

, pentru existent a solut iei unice trebuie ca ,= 0, adica


AB

BA

,= 0
A
A

,=
B
B

.
Daca = 0, adica
A
A

=
B
B

sau
A
B
=
A

care conform denit iei implica m = m

.

In acest caz
avem doua situat ii:
1. Daca
A
A

=
B
B

=
C
C

drepte confundate (sistem compatibil nedeterminat)


2. Daca
A
A

=
B
B

,=
C
C

drepte paralele (sistem incompatibil).


Dreptele sunt perpendiculare daca
m =
1
m

m m

= 1.
Unghiul cuprins ntre cele doua drepte se poate determina cu ajutorul formulei
tg = tg(

) =
tg tg

1 + tgtg

=
mm

1 +mm

.
2.2.2 Intersect ia a trei plane
Fie acum trei plane (P), (P

) si (P

). Sistemul format din ecuat iile acestora are ca solut ie coordo-


natele punctelor lor comune:
_
_
_
(P) : Ax +By +Cz +D = 0;
(P

) : A

x +B

y +C

z +D

= 0;
(P

) : A

x +B

y +C

z +D

= 0.
Pentru
=

A B C
A

,= 0
sistemul este compatibil determinat, si cele trei plane se intersecteaza ntr-un singur punct. Daca n
plane au un singur punct comun, atunci sistemul format din n ecuat ii cu 3 necunoscute este compatibil
determinat. Daca = 0, atunci rangul sistemului este mai mic ca 3 si sistemul este nedeterminat: e
compatibil (rangul matricii extinse este egal cu rangul sistemului), e incompatibil (cand rangul matricii
extinse este mai mare ca rangul sistemului) conform teoremei lui Kronecker-Capelli.
2.2.3 Unghiul a doua plane
Prin unghiul diedru al celor doua plane
(P) : Ax +By +Cz +D = 0;
(P

) : A

x +B

y +C

z +D

= 0
2.2 Pozit iile relative ale punctelor, dreptelor si planelor din spat iu 91
vom nt elege unghiul format de dreptele perpendiculare pe latura comuna celor doua plane care sunt
situate respectiv n cele doua plane. Unghiul vectorilor lor directori este egal cu unghiul vectorilor
normali la cele doua plane
n = Ai +Bj +Ck;
n

= A

i +B

j +C

k,
deci
cos =
n n

| n| | n

|
=
AA

+BB

+CC

A
2
+B
2
+C
2

A
/2
+B

2
+C

2
.
2.2.4 Unghiul a doua drepte din spat iu
Fie dreptele (d) si (d

) din spat iu:


(d) :
x x
0
p
=
y y
0
q
=
z z
0
r
, si (d

) :
x x

0
p

=
y y

0
q

=
z z

0
r

.
Vectorii lor directori sunt v = pi + qj + rk si v

= p

i + q

j + r

k. Unghiul celor doua drepte este


unghiul celor doi vectori directori
cos(

v, v
t
) =
v v
t
| v| | v
t
|
=
pp
t
+qq
t
+rr
t
_
p
2
+q
2
+r
2

_
p
/2
+q
/2
+r
/2
.
92 2. Elemente de geometrie analitic a liniar a
2.2.5 Unghiul format de o dreapta si un plan
Fig. 3.20
Fie un plan
(P) : Ax +By +Cz +D = 0
si o dreapta
(d) :
x x
0
p
=
y y
0
q
=
z z
0
r
.
Prin unghiul ntre dreapta (d) si planul (P) se nt elege unghiul dintre dreapta (d) si proiect ia ei
(d
t
) din planul (P) (Fig. 3.20). Unghiul ntre vectorul director v al dreptei (d) si vectorul normal n al
planului (P) este egal cu

2
, deci
cos = sin =
v n
| v| | n|
=
Ap +Bq +Cr
_
p
2
+q
2
r
2

A
2
+B
2
+C
2
.
2.2.6 Denit ie. Numim fascicul de plane paralele mult imea tuturor planelor paralele cu un
plan (P) dat
(P) : Ax +By +Cz +D = 0
adica
(F) : Ax +By +Cz + = 0 , () R.
2.2.7 Denit ie. Numim fascicul de plane propriu-zis mult imea tuturor planelor care cont in
aceeasi dreapta (d)
Fig. 3.21
Fie doua plane de baza
(P
1
) : A
1
x +B
1
y +C
1
z +D
1
= 0
(P
2
) : A
2
x +B
2
y +C
2
z +D
2
= 0,
care se intersecteaza dupa dreapta (d). Mult imea planelor
(F) :
1
(A
1
x +B
1
y +C
1
z +D
1
) +
2
(A
2
x +B
2
y +C
2
z +D
2
) = 0, ()
1
,
2
R,
reprezinta ecuat ia fasciculei de plane determinata de cele doua plane (P
1
) si (P
2
). Pentru parametrul

2
= 0 avem planul (P
1
) si pentru parametrul
1
= 0 planul (P
2
). Coordonatele punctelor lor comune
situate pe dreapta (d) verica ecuat iile celor doua plane si implicit ecuat ia fasciculei (F).
Daca
1
,= 0, putem mpart i ecuat ia lui (F) cu , si notand cu =

2

1
obt inem pentru fasciculul de
plane ecuat ia
A
1
x +B
1
y +C
1
z +D
1
+(A
2
x +B
2
y +C
2
z +D
2
) = 0, () R.
2.3 Distant a de la un punct la o dreapta si de la un punct la un
plan. Distant a dintre doua plane din spat iu.
2.3.1 Distant a de la un punct la un plan
Sa consideram un plan
(P) : Ax +By +Cz +D = 0
si un punct M
0
(x
0
, y
0
, z
0
). Daca M
0
(P) atunci distant a d = d(M
0
, (P)) = 0. Fie M
0
/ (P) si un
punct M
1
(x
1
, y
1
, z
1
) (P) (Fig. 3.22)
2.3 Distant a de la un punct la o dreapt a si de la un punct la un plan. Distant a dintre dou a plane din spat iu. 93
Fig. 3.22
Vectorul M
1
M
0
= (x
0
x
1
)i +(y
0
y
1
)j +(z
0
z
1
)k are ca proiect ie dupa vectorul normal la planul (P)
n = Ai +Bj +Ck
pr
n
M
1
M
0
= NM
0
=
n M
1
M
0
| n|
=
A(x
0
x
1
) +B(y
0
y
1
) +C(z
0
z
1
)

A
2
+B
2
+C
2
.
Cum M
1
(x
1
, y
1
, z
1
) (P) rezulta ca Ax
1
+By
1
+Cz
1
+D = 0 si deci NM
0
=
Ax
0
+By
0
+Cz
0
+D

A
2
+B
2
+C
2
.
Distant a de la M
0
(x
0
, y
0
, z
0
) la planul (P) este egala cu [NM
0
[, adica
d(M
0
, (P)) =
[Ax
0
+By
0
+Cz
0
+D[

A
2
+B
2
+C
2
.
2.3.2 Distant a de la un punct la o dreapta din plan
Fie n planul x0y punctul M
0
(x
0
, y
0
, z
0
) si o dreapta (d) : Ax +By +C = 0. Analog cu distant a de
la un punct M
0
la planul (P) avem
d(M
0
, (d)) =
[Ax
0
+By
0
+C[

A
2
+B
2
,
unde vectorul normal la dreapta este n = Ai +Bj si M
1
M
0
= (x
0
x
1
)i + (y
0
y
1
)j.
2.3.3 Distant a de la un punct la o dreapta din spat iu
Fie o dreapta (d) :
x x
0
p
=
y y
0
q
=
z z
0
r
determinata de punctul M
0
(x
0
, y
0
, z
0
) si vectorul
director v = pi +qj +rk si un punct M
1
(x
1
, y
1
, z
1
) (Fig. 3.23)
Fig. 3.23
Aria paralelogramului determinat de vectorii v si M
0
M
1
este egala cu
/ = |v M
0
M
1
| = |v| d
si deci
d =
|v M
0
M
1
|
|v|
.
2.3.4 Distant a ntre doua drepte din spat iu
Fie dreptele
(d
1
) :
x x
1
p
1
=
y y
1
q
1
=
z z
1
r
1
(d
2
) :
x x
2
p
2
=
y y
2
q
2
=
z z
2
r
2
determinate respectiv de punctele M
1
(x
1
, y
1
, z
1
), M
2
(x
2
, y
2
, z
2
) si vectorii directori v
1
= p
1
i +q
1
j +r
1
k,
v
2
= p
2
i +q
2
j +r
2
k.
Daca (d
1
) (d
2
) ,= , atunci distant a ntre cele doua drepte este egala cu 0. Fie cazul n care
(d
1
) (d
2
) = (g.3.24).
Fig 3.24
94 2. Elemente de geometrie analitic a liniar a
Din interpretarea geometrica a produsului mixt a trei vectori ca ind volumul paralelipipedului
determinat de cei trei vectori, rezulta ca
V = [(v
1
, v
2
, M
1
M
2
)[ .
Acest volum este egal si cu aria bazei (a paralelogramului determinat de vectorii v
1
si v
2
ori nalt imea
d a paralelipipedului
V = | v
1
v
2
| d.
Egaland cele doua volume, obt inem distant a dreptelor (d
1
) si (d
2
):
d =
[(v
1
, v
2
, M
1
M
2
)[
| v
1
v
2
|
.
Capitolul 3
Conice
3.1 Denit ia conicelor. Conice studiate pe ecuat iile lor reduse
3.1.1 Denit ia algebrica. Se numeste conica o curba de gradul al doilea, adica o curba plana
care are ca ecuat ie n coordonate carteziene o ecuat ie de gradul al doilea:
(K) : F (x, y) = a
11
x
2
+ 2a
12
xy +a
22
y
2
+ 2a
13
x + 2a
23
y +a
33
= 0, (3.1.1)
unde F : D R
2
R
3.1.2 Denit ia geometrica. Se numeste conica curba de sect iune a unei suprafet e conice de
rotat ie (C) cu un plan (P). Daca varful V al suprafet ei conice (C) nu se gaseste n planul (P) atunci, n
funct ie de unghiul de nclinare fat a de axa suprafet ei conice avem de-a face cu conice nedegenerate:
cerc, elipsa, parabola, hiperbola. Daca V (P) atunci curba de sect iune va o conica degenerata:
un punct, doua drepte secante, doua drepte confundate. Daca varful V este aruncat la innit, atunci
suprafat a conica de rotat ie devine o suprafat a cilindrica de rotat ie. Un plan (P) paralel cu axa de
rotat ie intersecteaza suprafat a cilindrica dupa o conica degenerata: doua drepte paralele, doua drepte
confundate sau .
3.1.3 Denit ie (ca loc geometric). Se numeste conica nedegenerata locul geometric al
punctelor din plan ale caror raport al distant elor la un punct F numit focar si la o dreapta (d) care
nu trece prin punctul F, numita directoare, este constant. Acest raport se numeste excentricitatea
conicii si se noteaza cu e.
Pentru determinarea ecuat iei polare a unei conice cu polul n focarul F sa consideram (Fig. 3.25.)
Fig. 3.25
Pentru o conica trebuie ca
e =
MF
MN
.
Cum MF = si MN = FD FM

= cos , rezulta
e =

cos
,
care dupa un calcul usor devine
=
p
1 +e cos
.
Aceasta este ecuat ia conicei (C) n coordonate polare, cu polul n focarul F, iar
p = e
se numeste parametrul conicei.
95
96 3. Conice
Conicele nedegenerate, dupa excentricitatea e, se clasica astfel:
e = 0 cerc
e (0, 1) eclipsa
e = 1 parabola
e (1, ) hiperbola
3.1.4 Denit ie. Locul geometric al punctelor din plan egal departate de un punct x se numeste
cerc (Fig.3.26)
Fig. 3.26
Ecuat ia cercului cu centrul n (a, b) de raza R este
C(, R) : (x a)
2
+ (y b)
2
= R
2
sau
x
2
+y
2
2ax 2by +p = 0, cu R
2
= a
2
+b
2
p.
Cercul cu centrul n origine are ecuat ia
C(0, R) : x
2
+y
2
= R
2
.
3.1.5 Denit ie. Locul geometric al punctelor din plan egal departate de doua puncte xe F si F

,
numite focare se numeste elipsa (Fig. 3.27).
Fig. 3.27
Ecuat ia elipsei raportata la axele sale de simetrie este
(E) :
x
2
a
2
+
y
2
b
2
1 = 0, cu c
2
= a
2
b
2
,
unde e =
c
a
; p = a(1 e
2
) si ecuat iile directoarelor sunt x =
a
2
c
.
3.1.6 Denit ie. Locul geometric al punctelor din plan ale caror modul al diferent ei distant elor
la doua puncte xe, F si F

este constant, se numeste hiperbola (Fig.3.28). Ecuat ia hiperbolei (H)


raportata la axele de simetrie, avand ca axa 0x axa focarelor este
(H) :
x
2
a
2

y
2
b
2
1 = 0, cu c
2
= a
2
+b
2
.
Hiperbola (H

) care are aceleasi semiaxe ca si (H) dar cu axa focarelor axa 0y, se numeste conjugata
hiperbolei (H).
(H

) :
x
2
a
2
+
y
2
b
2
1 = 0, cu c
2
= a
2
+b
2
.
Hiperbolele (H) si (H

) au aceleasi asimptote y =
b
a
x.
Fig. 3.28
Hiperbola (H) are directoarele x =
a
2
c
, si (H

) pe y =
b
2
c
. Excentricitatea lui (H) este
e =
c
a
si parametrul p = a(e
2
1)
iar hiperbola conjugata (H

)
e =
c
b
si parametrul p = b(e
2
1).
Sa observam ca cercul, elipsa si hiperbola au un centru de simetrie unic determinat.
3.1.7 Denit ie. Locul geometric al punctelor din plan egal departate de un punct x, numit focar,
si de o dreapta xa, numita directoare, se numeste parabola (Fig.3.29).
3.2 Reducerea conicelor date prin ecuat ia general a la forma canonic a 97
Fig. 3.29
Ecuat ia parabolei raportata la axa de simetrie si tangenta la varf este
(P) : y
2
= 2px.
Directoarea (d) are ecuat ia
(d) : x =
p
2
.
Sa observam ca parabola nu are centru de simetrie.
3.2 Reducerea conicelor date prin ecuat ia generala la forma
canonica
Reamintim ecuat ia generala a unei conice (K). Se numeste conica o curba de gradul al doilea, adica
o curba plana care are ca ecuat ie n coordonate carteziene o ecuat ie de gradul al doilea:
(K) : F (x, y) = a
11
x
2
+ 2a
12
xy +a
22
y
2
+ 2a
13
x + 2a
23
y +a
33
= 0,
F : D R
2
R.
Sa observam ca aceasta ecuat ie este compusa dintr-o forma patratica
F(x, y) = a
11
x
2
+ 2a
12
xy +a
22
y
2
,
o forma liniara
g(x, y) = 2a
13
x + 2a
23
y
si o constanta a
33
. Adica
f(x, y) = F(x, y) +g(x, y) +a
33
= 0.
Pe de alta parte, daca notam forma patratica
G(x, y, z) = a
11
x
2
+ 2a
12
xy +a
22
y
2
+ 2a
13
xz + 2a
23
yz +a
33
z
2
,
observam ca G : R
3
R cu G(x, y, 1) = f(x, y). Notam cu
A =
_
a
11
a
12
a
12
a
22
_
matricea formei patratice f si cu

A =
_
_
a
11
a
12
a
13
a
12
a
22
a
23
a
13
a
23
a
33
_
_
matricea lui G. Putem calcula invariant ii metrici (invarian t i fat a de o izometrie)
= det A; = det

A si I = a
11
+a
22
(vezi, de exemplu [3]).
Sa presupunem ca conica (K) are centru de simetrie 0

(a, b). Sa translatam axele de coordonate


astfel ncat originea 0 sa se mute n 0

, transformand astfel reperul x0y n x

(Fig. 3.30).
Fig. 3.30
98 3. Conice
Conform Figurii 3.30 0M = 00

+0

M, care exprimat n coordonate carteziene devine


xi +yj = (ai +bj) + (x

i +y

j),
sau pe componente
_
x = a +x

y = b +y

.
(3.1.2)
Aceasta reprezinta formulele care exprima translat ia ceruta. Matriceal se poate scrie
_
x
y
_
=
_
a
b
_
+
_
x

_
,
care este de forma
X = X

+D,
unde
X =
_
x
y
_
; X

=
_
x

_
; D =
_
a
b
_
.
Aplicand formulele de translat ie (3.1.2) la ecuat ia conicei (3.1.1) obt inem
f(x

, y

) = F(x

, y

) + (2a
11
a + 2a
12
b + 2a
13
)x

+ (2a
12
a + 2a
22
b + 2a
23
)y

+f(a, b) = 0.
Vom determina noua origine 0

(a, b) impunand ca termenii de gradul I n x

si y

sa se anuleze
_
2a
11
a + 2a
12
b + 2a
13
= 0
2a
12
a + 2a
22
b + 2a
23
= 0,
(3.1.3)
sau
_
a
11
a +a
12
b = a
13
;
a
12
a +a
22
b = a
23
.
3.2.1 Observat ie. Sistemul (3.1.3) este acelasi cu
_

_
f
x
(x, y) = 0
f
y
(x, y) = 0.
cu solut ia x = a, y = b.
Sistemul (3.1.3) are solut ie unica (compatibil determinat) daca
=

a
11
a
12
a
12
a
22

,= 0.
Deci daca ,= 0, conica are centru unic.
Daca nsa = 0, atunci conica nu are centru unic, e ca sistemul este compatibil nedeterminat,
e ca este incompatibil.
Dupa translat ia de vector 00

, matricea formei patratice G devine

A =
_
_
a
11
a
12
0
a
12
a
22
0
0 0 f(a, b)
_
_
.
Cum = det

A este invariant metric, rezulta ca
= det

A = f(a, b)
3.2 Reducerea conicelor date prin ecuat ia general a la forma canonic a 99
si deci
f(a, b) =

.
3.2.2 Conice cu centru unic

In acest caz ,= 0 si dupa translat ia de vector 00


t
, obt inem ecuat ia
f(x

, y

) = F(x

, y

) +

= a
11
x

2
+ 2a
12
x

+a
22
y

2
+

= 0. (3.1.4)

In vederea determinarii celei mai simple ecuat ii canonice vom trece la o rotat ie a reperului x

cu
un unghi n jurul originii 0

. Pentru identicarea noilor axe 0

si 0

vom determina versorii e


1
si
e
2
ai acestor axe folosind metoda valorilor si vectorilor proprii.
Valorile proprii
1
si
2
sunt solut iile ecuat iei caracteristice
p() =

a
11
a
12
a
12
a
22

=
2
I + = 0,
unde I = a
11
+a
22
.
Vectorii proprii corespunzatori valorilor proprii
1
si
2
se obt in din sistemul caracteristic
_
(a
11
)x +a
12
y = 0
a
12
x + (a
22
)y = 0
aplicat pe rand valorilor proprii
1
si
2
. Sistemele obt inute sunt compatibile si nedeterminate.
Pentru
1
se obt in vectorii proprii
v
1
= (x
1
, y
1
) = (x

1
, y

1
) = (x

1
, y

1
) = u
1
si
e
1
=
u
1
|u
1
|
=
_
x

1
_
x

1
+y

1
,
y

1
_
x

1
+y

1
_
= (c
1
, d
1
).
Pentru
2
avem
v
2
= (x
2
, y
2
) = (x

2
, y

2
) = (x

2
, y

2
) = u
2
si
e
2
=
u
2
|u
2
|
=
_
x

2
_
x

2
+y

2
,
y

2
_
x

2
+y

2
_
= (c
2
, d
2
).
Formula matriceala a rotat iei este data de
X
t
= CX
tt
, (3.1.5)
sau
_
x

_
=
_
c
1
c
2
d
1
d
2
__
x

_
.
Transformarea (3.1.5) este rotat ie daca
det C =

c
1
c
2
d
1
d
2

= 1.
Daca det C = 1, atunci vom lua e
2
= (c
2
, d
2
) si det C devine egal cu 1. Unghiul de rotat ie se
determina din formula
cos = c
1
.
100 3. Conice
Cand unghiul se determina greu, vom luan planul x

punctele P(x

1
, y

1
), Q(x

2
, y

2
),iar dreptele
0

P, 0

Q vor axele 0

si 0

.
Scriind (3.1.5) sub forma
_
x

= c
1
x

+c
2
y

,
y

= d
1
x

+d
2
y

,
si nlocuind x

, y

n (3.1.4) se obt ine


f(x

, y

) =
1
x

2
+
2
y

2
+

= 0,
care nmult ita cu
_

_
va reprezenta ecuat ia canonica a conicei cu centrul de simetrie n 0

, raportata
la reperul x

0
t
y

.
Daca = 0, conica va degenerata si n acest caz vom evita translat ia si rotat ia, deoarece exista
o alta metoda pentru reprezentarea graca a conicei.
3.2.3 Conice nedegenerate fara centru de simetrie (parabole)
Daca = 0 si ,= 0, vom avea de-a face cu o parabola. Parabola neavand centru de simetrie va
trebui sa facem mai ntai o rotat ie a reperului x0y n jurul originii, folosind metoda valorilor si vectorilor
proprii, determinand noua baza
B

= e
1
, e
2

si valorile proprii
1
si
2
din ecuat ia caracteristica, care n acest caz are forma
p() =

a
11
a
12
a
12
a
22

=
2
I = 0.
Deci
1
= 0 , e
1
= (c
1
, d
1
) si
2
= I , e
2
= (c
2
, d
2
).
Aplicand rotat ia
_
x
y
_
=
_
c
1
c
2
d
1
d
2
__
x

_
sub forma
_
x = c
1
x

+c
2
y

,
y = d
1
x

+d
2
y

,
la ecuat ia conicei (3.1.1) obt inem
f(x

, y

) =
1
x

2
+
2
y

2
+ 2a
13
(c
1
x

+c
2
y

) + 2a
23
(d
1
x

+d
2
y

) +a
33
= 0,
adica
f(x

, y

) = Iy

2
+ (2a
13
c
1
x

+ 2a
13
c
2
y

) + (2a
23
d
1
x

+ 2a
23
d
2
y

) +a
33
= 0,
care este de forma
f(x

, y

) = Iy

2
+ 2a

13
x

+ 2a

23
y

+a
33
= 0. (3.1.6)
Dupa acest moment vom trece la o translat ie de vector 00

, a reperului x

0y

n reperul x

, unde
0

(a, b) este noua origine cu x

= a si y

= b. Translat ia se realizeaza prin formulele


_
x

= a +x

= b +y

si (3.1.6) devine
f(x

, y

) = Iy

2
+ 2a

13
x

+ (2bI + 2a

23
)y

+ (Ib
2
+ 2a

13
a + 2a

23
b +a
33
) = 0.
3.2 Reducerea conicelor date prin ecuat ia general a la forma canonic a 101
Egaland termenii din paranteze cu 0
_
2bI + 2a

23
= 0
Ib
2
+ 2a

13
a + 2a

23
b +a
33
= 0,
obt inem un sistem de ecuat ii din care vom putea determina coordonatele noii origini 0

In nal, se obt ine ecuat ia canonica a parabolei


Iy

2
+ 2a

13
x

= 0,
sau
y

2
= 2
a

13
I
x

,
de forma y

2
= 2px

cu p =
a

13
I
.
3.2.4 Conice degenerate
Dupa determinarea matricilor A,

A si a invariant ilor matricei , , I, daca = 0 avem de-a face cu
o canonica degenerata.

In acest caz ordonam ecuat ia (3.1.1) dupa una din necunoscute, de exemplu dupa
y:
a
22
y
2
+ 2(a
12
x +a
23
)y + (a
11
x
2
+ 2a
13
x +a
33
) = 0
si calculam solut iile y
1
si y
2
ale ecuat iei. Discriminantul ecuat iei,
y
, va :
a) un patrat perfect si ecuat ia se descompune n doua ecuat ii de gradul I (doua drepte);
b) de forma (E(x))
2
, care are solut ie reala cand E(x) = 0 si se obt ine un punct;
c) un numar constant negativ, n acest caz conica va mult imea vida.
3.2.5 Observat ie. Pentru o documentare mai detaliata se recomanda studierea bibliograei.
Capitolul 4
Probleme de geometrie
4.1 Enunt uri
Vectori liberi
4.1.1

In triunghiul ABC, pe latura (AB) alegem punctele E si F astfel ca 3AF = 2AE = AB, iar pe
latura (AC) consideram punctele G si H astfel ca 2AH = 3AG = AC. Fie FHEG = T. Demonstrat i
ca: AB+AC = 5AT.
4.1.2 Fie AB, CD doua coarde perpendiculare ale unui cerc de centru O. Atunci:
MA+MB+MC+MD = 2MO.
4.1.3 Fie punctul A(2, 3) si vectorul u = 3i 4j. Sa se ae ecuat ia unei drepte care trece prin punctul
A si este:
a) paralela cu u;
b) perpendiculara pe u.
4.1.4 Fie punctele A(1, 2), B(3, 1), C(4, 5). Fie M, mijlocul laturii (BC) iar G centrul de greutate al
triunghiului ABC. Determinat i vectorii AB, AC, BC, AM, GM si apoi aat i perimetrul triunghiului
ABC.
Dreapta si planul n spat iu
4.1.5 Sa se studieze pozit ia dreptei de ecuat ie :
x 1
3
=
y
4
=
z 1
6
,
fat a de planul de ecuat ie: 4x 9y + 8z 1 = 0.
4.1.6 Sa se ae distant a dintre dreptele paralele:
(d
1
) :
x 2
3
=
y + 1
2
=
z 1
3
si (d
2
) :
x + 1
3
=
y 4
2
=
z
3
.
4.1.7 Aat i unghiul dintre dreptele de ecuat ii:
(d
1
) :
_
x + 2y +z 1 = 0
x 2y +z + 1 = 0
si (d
2
) :
_
x y + 2z + 1 = 0
x y z 1 = 0
.
102
4.1 Enunt uri 103
4.1.8 Sa se deduca ecuat iile perpendicularei comune a dreptelor de ecuat ii:
(d
1
) :
x 2
2
=
y 1
3
=
z 1
1
si (d
2
) :
x 4
3
=
y 3
1
=
z
4
.
4.1.9 Sa se aduca la forma canonica ecuat iile dreptei:
(d) :
_
3x y +z 1 = 0
2x 6y + 3z 3 = 0.
.
4.1.10 Sa se determine parametrul real astfel ca planele:
(
1
) : x +y + 2z 1 = 0;
(
2
) : 2x 2y 3z + 1 = 0;
(
3
) : 4x + 4y + 8z = 0
sa se intersecteze dupa o dreapta.
4.1.11 Scriet i ecuat ia unui plan care este denit de punctele M
1
(1, 2, 3), M
2
(4, 4, 6) si care este paralel
cu vectorul v = 2i + 4j + 7k.
4.1.12 Scriet i ecuat ia planului care trece prin punctele: A(1, 1, 1), B(0, 1, 2), C(3, 2, 4).
4.1.13 Sa se calculeze distant a dintre planele de ecuat ii:
(
1
) : 2x 7y + 5z 12 = 0 si (
2
) : 2x 7y + 5z + 3 = 0.
4.1.14 Sa se scrie ecuat iile unei drepte care trece prin punctul A(2, 1, 3) si este paralela cu dreapta:
(d) :
_
2x +y z 1 = 0
x 3y + 2z + 2 = 0
.
4.1.15 Sa se ae proiect ia punctului M
1
(1, 2, 2) pe planul de ecuat ie:
() : 3x + 4y 7z + 1 = 0.
4.1.16 Sa se ae distant a de la punctul M(2, 1, 3) la dreapta de ecuat ie:
(d) :
_
x y z 1 = 0
2x +y z + 2 = 0
.
4.1.17 Sa se gaseasca proiect ia dreptei (d) :
_
x +y 2z + 1 = 0
3x + 2y +z 2 = 0
pe planul de ecuat ie:
() : 3x + 5y 6z + 10 = 0.
4.1.18 Stabilit i care este pozit ia relativa a dreptei de ecuat ie
(d)
_
x +y z + 2 = 0
x y + 2 = 0
fat a de planul () : x + 2y 1 = 0.
4.1.19 Un mobil se aa la momentul t = 0 n punctul M(2, 1, 3). El se deplaseaza rectiliniu si uniform
cu viteza data de vectorul v = 5i 7j + 3k. Aat i pozit ia mobilului la momentul t precum si ecuat iile
parametrice ale traiectoriei.
104 4. Probleme de geometrie
4.1.20 Sa se arate ca dreptele
(d
1
) :
x 3
8
=
y + 1
2
=
z 2
4
si (d
2
) :
x 8
3
=
y 1
1
=
z 6
2
sunt concurente si sa se ae ecuat ia planului determinat de cele doua drepte.
Conice
4.1.21 Sa se reduca la forma canonica conica
f(x, y) = 3x
2
+ 10xy + 3y
2
2x 14y 13 = 0
si sa se reprezinte grac n reperul init ial, precizand si formulele de translat ie si de rotat ie aplicate.
4.1.22 Sa se ae ecuat ia canonica a conicei
f(x, y) = 9x
2
6xy +y
2
2x + 4y = 0
si sa se reprezinte grac n reperul x0y.
4.1.23 Sa se reprezinte grac conica de ecuat ie
f(x, y) = 3x
2
+ 4xy +y
2
2x 1 = 0.
4.1.24 Sa se reprezinte grac conica de ecuat ie
f(x, y) = x
2
+ 2xy + 2y
2
+ 6y + 9 = 0.
4.2 Solut ii
Vectori liberi
4.2.1
AF
AB
=
AG
AC
=
1
3
iar
AE
AB
=
AH
AC
=
1
2
FG| BC si EH| BC.
De aici, obt inem:
FT
TH
=
FG
EH
=
BC
3

2
BC
=
2
3
.
Din triunghiul AFH, avem:
AT =
3AF + 2AH
5
=
AB+AC
5
AB+AC = 5AT.
4.2.2
MA = MO+OA
MB = MO+OB
MC = MO+OC
MD = MO+OD
_

_
MA+MB+MC+MD = 4MO+OA+OB+OC+OD
4.2 Solut ii 105
Fie E si F proiect iile punctului O pe laturile AB si CD. Avem
OA+OB = 2OE
OC+OD = 2OF
OE+OF = OM
_
_
_
OA+OB+OC+OD = 2(OE+OF) = 2OM = 2MO
De aici obt inem: MA+MB+MC+MD = 4MO2MO = 2MO.
4.2.3 a) Ecuat ia unei drepte de vector director d(l, m) care trece printr-un punct A(a, b) si este paralela
cu u, se scrie:
x a
l
=
y b
m

x 2
3
=
y + 3
4

ecuat ia dreptei va : 4x + 3y + 1 = 0.
b) Fie v(, ) , astfel ca
vu v u = 0 v u = 3 4 = 0 =
4
3
.
Apoi nlocuind pe n ecuat ia
x 2

=
y + 3

,
obt inem:
x 2
4
3

=
y + 3

x 2 =
4
3
y + 4c
si de aici avem ecuat ia: 3x 4y 18 = 0.
4.2.4 Calculam:
AB = (3 (1)) i + (1 2)j = 4i 3j,
BC = (4 3)i + (5 (1))j = i + 6j,
AC = 5i + 3j
M
_
x
B
+x
C
2
,
y
B
+y
C
2
_
M
_
7
2
, 2
_
.
G
_
x
A
+x
B
+x
C
3
,
y
A
+y
B
+y
C
3
_
G(2, 2) ,
apoi calculam:
AM =
9
2
i, GM =
3
2
i.
|AB| =

16 + 9 = 5
|BC| =

36 + 1 =

37
|AC| =

25 + 9 =

34
_
_
_

perimetrul triunghiului va : P
ABC
= 5 +

34 +

37.
Dreapta si planul n spat iu
4.2.5 Observam ca vectorul director al dreptei este d(3, 4, 6), iar vectorul normal la plan are componentele
n(4, 9, 8). Studiem produsul scalar
n d = l A+m M +n C = 3 (4) + 4 (9) + 6 8 = 0.
Rezulta nd, deci planul este paralel cu dreapta.
4.2.6 Observam ca cele 2 drepte au acelasi vector director de coordonate (3, 2, 3). Calculam distant a de
la un punct M
1
situat pe dreapta ( d
1
) cu vectorul de pozit ie: r
1
= 2i j +k la a doua dreapta (d
2
) unde
avem un punct M
2
cu vectorul de pozit ie r
2
= i + 4j + 0k. Calculam : r
1
r
2
= 3i 5j +k, apoi :
106 4. Probleme de geometrie
(r
1
r
2
) d =

i j k
3 5 1
3 2 3

= 17i 6j + 21k.
Obt inem n nal ca distant a este d =
|(r
1
r
2
) d|
|d|
=

333

11
.
4.2.7 Calculam vectorii directori ai celor doua drepte:
d
1
=

i j k
1 2 1
1 2 1

= 4i 4k, d
2
=

i j k
1 1 2
1 1 1

= 3i + 3j.
Gasim: cos =
d
1
d
2
|d
1
| |d
2
|
=
12

32

18
=
1
2
deci unghiul va =

3
.
4.2.8 Stim ca pentru doua drepte date prin ecuat ie generala:
(d
1
) :
x x
1
l
1
=
y y
1
m
1
=
z z
1
n
1
si (d
2
) :
x x
2
l
2
=
y y
2
m
2
=
z z
2
n
2
, ecuat iile perpendicularei comune sunt:

x x
1
y y
1
z z
1
l
1
m
1
n
1
m
1
n
2
m
2
n
1
n
1
l
2
l
1
m
2
l
2
m
1

= 0
si

x x
2
y y
2
z z
2
l
2
m
2
n
2
m
1
n
2
m
2
n
1
n
1
l
2
n
2
l
1
l
1
m
2
l
2
m
1

= 0
Deci, avem:

x x
1
y 1 z 1
2 3 1
11 1 11

= 0 si

x 4 y 3 z
3 1 4
11 11 11

= 0 Dupa calcule se obt in ecuat iile:


4x +y 5z + 12 = 0si 5x + 7y 2z 41 = 0.
4.2.9 Intersectand cele doua plane care determina ecuat ia dreptei (d) cu axa Oz, obt inem acelasi punct
si anume M(0, 0, 1). Vectorul director al dreptei va d = n
1
n
2
=

i j k
3 1 1
2 6 3

= 3i 7j 16k, deci
ecuat ia dreptei va : (d) :
x
3
=
y
7
=
z 1
16
.
4.2.10 Din ecuat iile celor trei plane formam un sistem de ecuat ii:
_
_
_
x +y + 2z 1 = 0
2x 2y 3z + 1 = 0
4x + 4y + 8z = 0,
care trebuie sa e un sistem compatibil determinat. Consideram matricea sistemului de mai sus:
A =
_
_
1 1 2
2 2 3
4 4 8
_
_
.
4.2 Solut ii 107
Observam ca det(A) = 0, deci consideram un determinant de ordin doi:

1 1
2 2

= 4 ,= 0,
acesta ind luat ca determinant principal. Impunem condit ia ca determinantul caracteristic corespunzator
sa e nul:

1 1 1
2 2 1
4 4

= 0
si din aceasta condit ie obt inem: = 4.
4.2.11 Formam vectorul M
1
M
2
= (4 1)i + (4 2)j + (6 3)k = 3i + 2j + 3k. Vectorul normal la plan
este: n = M
1
M
2
v, adica
n =

i j k
3 2 3
2 4 7

= 2i 15j + 8k
Ecuat ia planului este: A(xx
0
) +B(y y
0
) +C(z z
0
) = 0, unde A, B, C sunt componentele vectorului
normal la plan iar x
0
, y
0
, z
0
sunt coordonatele unui punct din plan.

Inlocuind n ecuat ia de mai sus,
obt inem: 2(x 1) + (15)(y 2) + 8(z 3) = 0 si de aici se obt ine ecuat ia : 2x 15y + 8z + 4 = 0.
4.2.12 Calculam ntai vectorii:
AB = (0 1)i + (1 (1))j + (2 1)k = i + 2j +k
AC = (3 1)i + (2 (1))j + (4 1)k = 2i + 3j + 3k.
Calculam apoi vectorul normal la plan:
n = ABAC =

i j k
1 2 1
2 3 3

= 3i + 5j 7k.
Formam ecuat ia planului folosind ecuat ia generala:
A(x x
0
) +B(y y
0
) +C(z z
0
) = 0.
Obt inem n nal: 3(x 1) + 5(y + 1) 7(z 1) = 0, adica 3x + 5y 7z + 9 = 0.
4.2.13 Coecient ii lui x, y, z sunt proport ionali, deci cele doua plane sunt paralele. Vom calcula distant a
dintre cele doua plane folosind un punct din primul plan si calculand distant a de la acest punct la cel
de-al doilea plan. Intersect ia planului (
1
) cu axa
Oz :
_
_
_
2x 7y + 5z 12 = 0
x = 0
y = 0
,
este (
1
) Oz = M(0, 0,
12
5
). Calculam acum distant a de la punctul M la planul (
2
):
d(M,
2
) =
[Ax
0
+By
0
+Cz
0
+d[

A
2
+B
2
+C
2
=

2 0 + (7) 0 + 5
12
5
+ 3

_
2
2
+ (7)
2
+ 5
2
=
15

78
.
108 4. Probleme de geometrie
4.2.14 Vectorul director al dreptei (d) se obt ine astfel:
d = n
1
n
2
= (2i +j k) (i 3j + 2k) =

i j k
2 1 1
1 3 2

= i 5j 7k.
Folosind ecuat iile parametrice ale dreptei, obt inem n nal ecuat iile:
x 2
1
=
y 1
5
=
z 3
7
.
4.2.15 Fie M
2
(x
0
, y
0
, z
0
) proiect ia punctului M
1
(1, 2, 2) pe planul (). Formam vectorul
M
2
M
1
= (1 x
0
)i + (2 y
0
)j + (2 z
0
)k.
Dar vectorul M
2
M
1
trebuie sa e paralel cu vectorul normal la plan n, deci coecient ii lor trebuie sa e
proport ionali:
1 x
0
3
=
2 y
0
4
=
2 z
0
7
= t,
de aici obt inem ca:
_
_
_
x
0
= 1 3t
y
0
= 2 4t
z
0
= 2 + 7t
.
Dar M
2
(x
0
, y
0
, z
0
) (), deci verica ecuat ia acestui plan, prin nlocuire se obt ine:
3(1 3t) + 4(2 4t) 7(7t 2) + 1 = 0 t =
10
37
.

In nal obt inem : x


0
= 1 3t =
67
37
, y
0
= 2 4t =
34
37
, z
0
= 7t 2 =
4
37
. Astfel proiect ia
punctului M
1
(1, 2, 2) pe plan este punctul
M
2
_

67
37
,
34
37
,
4
37
_
.
4.2.16 Calculam ntai vectorul director al dreptei (d):
d = n
1
n
2
=

i j k
1 1 1
2 1 1

= 2i j + 3k.
Fie punctul A(x
0
, y
0
, z
0
) (d), astfel ca vectorul AM(2 x
0
, 1 y
0
, 3 z
0
) sa e perpendicular
pe vectorul d(2, 1, 3). Dar daca
AMd AM d = 0
2(2 x
0
) + (1)(1 y
0
) + 3(3 z
0
) = 0
2x
0
+y
0
3z
0
6 = 0.
((*))
Dar A(x
0
, y
0
, z
0
) (d), deci coordonatele punctului A trebuie sa verice ecuat ia dreptei (d) si n plus
trebuie sa verice si ecuat ia (), obt inem astfel sistemul:
_
_
_
x
0
y
0
z
0
= 1
2x
0
+y
0
z
0
= 2
2x
0
+y
0
3z
0
= 6.
4.2 Solut ii 109
Determinantul acestui sistem este =

1 1 1
2 1 1
2 1 3

= 2 ,= 0. Calculam apoi:

x
0
=

1 1 1
2 1 1
6 1 3

= 18 x
0
=

x
0

= 9 (1)

y
0
=

1 1 1
2 2 1
2 6 3

= 12 y
0
=

y
0

= 6 (2)

z
0
=

1 1 1
2 1 2
2 1 6

= 20 y
0
=

z
0

= 10 (3)
Din (1), (2), (3) AM = 11i + 7j + 7k , atunci distant a de la punctul A la dreapta (d) este
d(A, d) = |AM| =

219.
4.2.17 Ecuat ia fasciculului de plane care cont ine si dreapta (d) se poate scrie:
x +y 2z + 1 +(3x + 2y +z 2) = 0.
Impunem condit ia ca planul () sa e perpendicular pe un plan al fasciculului, deci avem
3(1 + 3) + 5(1 + 2) 6(2 +) = 0 =
20
13
.
De aici obt inem ecuat ia fasciculului:
x +y 2z + 1
20
13
(3x + 2y +z 2) = 0 47x 27y 46z + 53 = 0.
Deci, ecuat ia dreptei care este proiect ia dreptei (d) pe planul (), se scrie:
(d
1
) :
_
47x 27y 46z + 53 = 0
3x + 5y 6z + 10 = 0.
4.2.18 Din ecuat iile planului si a dreptei formam un sistem de trei ecuat ii cu trei necunoscute pe care l
rezolvam:
_
_
_
x +y z + 2 = 0
x y + 2 = 0
x + 2y 1 = 0
.
Calculam determinantul sistemului:
=

1 1 1
1 1 0
1 2 0

= 3 ,= 0.
Apoi calculam:

x
=

2 1 1
2 1 0
1 2 0

= 3 x =

x

=
3
3
= 1

y
=

1 2 1
1 2 0
1 1 0

= 3 y =

y

=
3
3
= 1

z
=

1 1 2
1 1 2
1 2 1

= 6 z =

z

=
6
3
= 2.
110 4. Probleme de geometrie
Deci, dreapta (d) intersecteaza planul () ntr-un punct de coordonate (1, 1, 2).
4.2.19 Mobilul se misca pe o dreapta care trece prin punctul M si are ca vector director pe v consideram ca
la momentul t mobilul a ajuns n punctul A(x, y, z). Folosind ecuat iile vectoriale ale dreptei: r = r
1
+tv,
obt inem pozit ia mobilului la momentul t
_
_
_
x = 2 + 5t
y = 1 7t
z = 3 + 3t.
De aici obt inem ecuat iile dreptei:
x 2
5
=
y + 1
7
=
z + 3
3
.
4.2.20 Vectorii de pozit ie ai punctelor M
1
(d), respectiv M
2
(d
2
) sunt
r
1
= 3i j + 2k
si
r
2
= 8i +j + 6k r
2
r
1
= 5i + 2j + 4k.
Vectorii directori ai celor doua drepte sunt:
d
1
= 8i + 2j + 4k si d
2
= 3i +j 2k.
Obt inem produsul mixt (r
2
r
1
, d
1
, d
2
) = 0 , deci lungimea perpendicularei comune este nula. Ecuat ia
planului determinat de cele doua drepte va data de:

x 3 y + 1 z 2
5 2 4
3 1 2

= 0 8x + 22y z + 48 = 0.
Conice
4.2.21 A =
_
3 5
5 3
_
,

A =
_
_
3 5 1
5 3 7
1 7 13
_
_
, I = 3 + 3 = 6, = det A = 16 ,= 0,
= det

A = 128 ,= 0, f(a, b) =

= 8. Conica are centru unic ( ,= 0) si este nedegenerata ( ,= 0).


Pentru translat ie avem
_

_
f
x
(x, y) = 6x + 10y 2 = 0
f
y
(x, y) = 10x + 6y 14 = 0
0
t
(a = 2, b = 1) si
_
x = x
t
+ 2
y = y
t
1
.
Dupa translat ia de vector 00
t
, ecuat ia devine :
f(x
t
, y
t
) = 3x
t
2
+ 10x
t
y
t
+ 3y
t
2
8 = 0.
Determinam rotat ia de unghi n jurul originii 0
t
. Ecuat ia caracteristica:
p() =

3 5
5 3

=
2
I + =
2
6 16 = 0,
cu radacinile
1
= 8 si
2
= 2 (valori proprii).
Ecuat ia devine: f(x
tt
, y
tt
) = 8x
tt 2
2y
tt 2
8 = 0 (H) : x
tt 2

y
tt 2
4
1 = 0 (hiperbola).
4.2 Solut ii 111
Pentru determinarea valorilor vectorilor proprii sa consideram sistemul caracteristic
_
(3 )x + 5y = 0
5x + (3 )y = 0.
Pentru = 8 avem
_
5x
t
+ 5y
t
= 0
5x
t
5y
t
= 0,
cu solut iile x
t
= y
t
= si cu vectorii proprii corespunzatori
v
1
= (, ) = (1, 1) = u
1
, | u
1
| =

2 si e
1
=
_
1

2
,
1

2
_
. Pentru = 2 avem
_
5x
t
+ 5y
t
= 0
5x
t
5y
t
= 0
cu solut iile x
t
= y
t
si cu vectorii proprii corespunzatori v
2
= (, ) = (1, 1) = u
1
, cu | u
2
| =

2
si e
1
=
_

2
,
1

2
_
.
Matricea de rotat ie este
C =
_
_
_
1

2

1

2
1

2
1

2
_
_
_,
cu det C = 1 si cos =
1

2
=

2
2
=

4
.
Fig. 3.31
4.2.22 A =
_
9 3
3 1
_
,

A =
_
_
9 3 1
3 1 2
1 2 0
_
_
, = 0 , I = 10 , = 25.
Cum = 0, rezulta ca avem de a face cu o conica fara centru unic. Deoarece ,= 0 este o conica
nedegenerata. Prin urmare, conica este o parabola. Reducerea la forma canonica se ncepe cu o rotat ie
de unghi n jurul originii axelor, 0. Ecuat ia caracteristica este:
p() =

9 3
3 1

=
2
10 = 0,
cu valorile proprii
1
= 0 si
2
= 10. Sistemul caracteristic este:
_
(9 )x 3y = 0;
3x + (1 )y = 0.
Pentru =
1
= 0 avem
_
9x 3y = 0
3x +y = 0

_
x =
y = 3
si v
1
= (, 3) = (1, 3) = u
1
,
| u
1
| =

10 e
1
=
_
1

10
,
3

10
_
.
Pentru =
1
= 10 avem
_
x 3y = 0
3x 9y = 0

_
x = 3
y =
si v
2
= (3, ) = (3, 1) = u
2
,
|u
2
| =

10 e
2
=
_

10
,
1

10
_
.
Matricea de rotat ie este
C =
_
_
_

10
1

10
3

10
1

10
_
_
_
112 4. Probleme de geometrie
si cos =
1

10
. X = CX
t

_

_
x =
x
t
3y
t

10
y =
3x
t
+y
t

10
. Ecuat ia conicei devine
f(x
t
, y
t
) = 10y
t
2
+

10(x
t
+y
t
) = 0.
Translat ia de vector 00
t
, unde 0
t
(x
t
= a, y
t
= b) are formulele:
_
x
t
= a +x
tt
y
t
= b +y
tt
si
f(x
tt
, y
tt
) = y
tt 2
+
1

10
x
tt
+
_
2b +
1

10
_
y
tt
+
_
b
2
+
a +b

10
_
= 0.
Egaland termenii din paranteze cu 0 obt inem a =
1
4

10
, b =
1
2

10
ecuat ia devine
y
tt 2
=
1

10
x
tt
.
Fig. 3.32
4.2.23 A =
_
3 2
2 1
_
,

A =
_
_
3 2 1
2 1 0
1 0 1
_
_
, = 1, = 0, I = 4. Cum = det

A = 0, rezulta ca
avem de a face cu o conica degenerata. Rezolvam ecuat ia y
2
+4xy +(3x
2
2x1) = 0 cu discriminantul

y
= (x + 1)
2
. Se obt in doua drepte secante(d
1
) : y = x + 1, si (d
2
) : y = 3x 1.
Fig. 3.33
4.2.24 A =
_
1 1
1 2
_
,

A =
_
_
1 1 0
1 2 3
0 3 9
_
_
, I = 3, = 1, = 0, este o conica degenerata cu centru
unic. Ecuat ia devine
2y
2
+ 2(x + 3)y + (x
2
+ 9) = 0
cu discriminantul
y
= 4(x 3)
2
0. Ecuat ia are solut ie reala pentru x 3 = 0 si anume un singur
punct 0
t
(3, 3).
Fig. 3.34
4.3 Probleme propuse
4.3.1

In sistemul cartezian de coordonate Oxyz, se considera vectorii u(p, q, q p), v (q p, q, p) si
punctul A(a, a +b, b), unde a, b, p, q R.
a) Sa se determine ecuat ia planelor determinate de vectorii u si v;
b) Sa se arate ca planul determinat de vectorii u si v care trece prin punctul A trece si prin originea
sistemului de axe de coordonate;
c) Sa se determine condit iile n care dreapta (OA) este bisectoare a unghiului format de de dreptele
suport ale vectorilor u, respectiv v.
4.3.2 Se considera planele (P
1
) : 4x3y+z4 = 0, (P
2
) : 2xyz = 0 si dreptele (d
12
) = (P
1
)(P
2
),
(d) :
_
2x y z = 0
y 3z = 0
.
4.3 Probleme propuse 113
a) Sa se scrie ecuat ia dreptei (d
12
);
b) Sa se determine pozit ia dreptelor (d
12
) si (d);
c) Sa se calculeze distant a de la dreapta (d) la planul (P
1
).
4.3.3 Se considera planele (P
1
) : x y +z + 2 = 0, (P
2
) : x +y + 2z 1 = 0 si punctul A(2, 1, 1).
a) Sa se determine ecuat ia dreptei (d) care trece prin punctul A si este paralela cu planele (P
1
), (P
2
);
b) Sa se scrie ecuat ia planului care cont ine dreapta (d);
c) Sa se determine aria trunghiului determinat de punctul A si proiect iile punctului A pe planele (P
1
),
respectiv (P
2
).
4.3.4

In sistemul cartezian de coordonate Oxyz, se considera punctele A(a, a, 2a), B(a, a, 0), M (0, 0, a)
Csi D, unde a (0, ), C este simetricul punctului B fat a de axa Ox, iar D este simetricul punctului B
fat a de punctul M.
a) Sa se scrie ecuat iile dreptelor (AM), respectiv (BM);
b) Sa se arate ca dreapta (AM) intersecteaza planul Oxy n punctul C;
c) Sa se arate ca punctul M este egal departat de punctele A, B, respectiv C;
d) Sa se demonstreze ca punctele A, B, C, D sunt coplanare, iar apoi sa se calculeze aria patrulaterului
ABCD.
4.3.5

In sistemul cartezian de coordonate Oxyz, se considera punctele A(3, 4, 12), B(4, 12, 3), C (12, 3, 4)
si D(4, 3, 12).
a) Sa se arate ca punctele A, B, C sunt egal departate de originea sistemului de axe;
b) Sa se arate ca punctele A, B, C, D sunt coplanare.
4.3.6 Se considera dreapta (d) :
_
_
_
x = 1
y = 2t 1
z = 2
, t R si punctele A(1, 1, 0), B(0, 1, 1).
a) Daca punctele C si D sunt proiect iile punctelor A, respectiv B pe dreapta (d), sa se determine aria
patrulaterului ABDC;
b) Sa se scrie ecuat ia dreptei care trece prin punctul A si este paralela cu dreapta (d);
c) Sa se scrie ecuat ia dreptei care trece prin punctul B si este perpendiculara pe planul determinat de
punctul A si dreapta (d).
4.3.7 Se considera dreptele
(d
1
) :
x
3
=
y 3
1
=
z 4
4
,
(d
2
) :
x 3
1
=
y 4
4
=
z
3
,
(d
3
) :
x 4
4
=
y
3
=
z 3
1
si punctele A, B, C unde (d
1
) (d
3
) = A, (d
2
) (d
1
) = B si (d
3
) (d
3
) = C.
a) Sa se arate ca dreptele (d
1
), (d
2
), (d
3
) sunt coplanare si sa se scrie ecuat ia planului determinat de ele;
114 4. Probleme de geometrie
b) Sa se arate ca puncte A, B, C sunt egal departate de punctul O(0, 0, 0);
c) Sa se determine proiect ia punctului O(0, 0, 0) pe planul (A, B, C).
4.3.8 Fie a R un paramentru real. Se considera vectorii v
1
(1, 1 a, a),
v
2
(2a 1, 1 a, 2 a) si punctul A(1, 1, 1).
a) Sa se scrie ecuat ia planului determinat de vectorii v
1
, v
2
, care trece prin punctul O(0, 0, 0);
b) Sa se determine coordonatele punctului A
1
, simetricul punctului A fat a de planul (O, v
1
, v
2
);
c) Sa se determine valoarea lui a (0, ) pentru care paralelipipedul determinat de vectorii OA, v
1
, v
2
este un paralelipiped dreptunghic cu volumul 6.
4.3.9 Se considera dreptele
(d
1
) : x 1 = y 1 = 1 z,
(d
2
) :
x 1
1
=
y 1
2
=
z 1
1
si punctul A, unde (d
1
) (d
2
) = A.
a) Sa se scrie ecuat ia planului determinat de dreptele (d
1
), (d
2
);
b) Sa se scrie ecuat ia dreptei care reprezinta proiect ia dreptei OA pe planul ((d
1
), (d
2
)).
4.3.10

In sistemul cartezian de coordonate Oxyz, se considera punctele A(2, 1, 0), B(0, 1, 4) si C (0, 2, 1).
a) Sa se determine masurile unghiurilor triunghiului ABC;
b) Sa se scrie ecuat ia planului care trece prin originea sistemului de axe si este perpendicular pe planul
(A, B, C);
Sa se reprezinte grac conicele de ecuat ii:
4.3.11 f(x, y) = 4xy + 3y
2
+ 16x + 12y 36 = 0;
4.3.12 f(x, y) = 7x
2
+ 6xy y
2
+ 28x + 12y + 28 = 0;
4.3.13 f(x, y) = 11x
2
20xy 4y
2
20x 8y + 1 = 0;
4.3.14 f(x, y) = 41x
2
+ 24xy + 34y
2
+ 34x 112y + 129 = 0;
4.3.15 f(x, y) = 5x
2
6xy + 2y
2
2x + 2 = 0;
4.3.16 f(x, y) = x
2
+ 4xy + 3y
2
6x 12y + 9 = 0;
4.3.17 f(x, y) = 3x
2
+ 10xy + 3y
2
+ 4x 4y + 14 = 0;
4.3.18 f(x, y) = x
2
xy 2y
2
+ 4x +y + 3 = 0;
4.3.19 f(x, y) = x
2
+y
2
6x + 4y + 14 = 0.
Bibliograe
[1] Andrica D., Duca I.D., Purdea I., Pop Ioana, Matematica de baza, Editura Studium, Cluj Napoca,
2000
[2] Cot a A., Rado M., Radut iu M., Vornicescu F., Matematica, Geometrie si Trigonometrie, Manual
pentru clasa a IX-a, Editura Didactica si Pedagogica, Bucuresti, 1987
[3] Cot a A., Kurthy E., Popa E.F., Rado M., Radut iu M., Vornicescu F., Matematica, Geometrie si
Trigonometrie, Manual pentru clasa a X-a, Editura Didactica si Pedagogica, Bucuresti, 1987
[4] Galbura Gh., Rado F., Geometrie, Editura Didactica si Pedagogica, Bucuresti, 1973
[5] Pop S. Maria, Algebra liniara, geometrie analitica si diferent iala, Editura Cub Press, Baia Mare,
1998
[6] Vasiu A., Curs de geometrie, Universitatea Babes-Bolyai, Cluj Napoca, 1992
115
Partea III
Analiza matematica
116
Capitolul 1
Siruri de numere reale
1.1 Siruri de numere reale
1.1.1 Denit ie. Se numeste sir de numere reale o funct ie denita pe mult imea numerelor naturale
N cu valori n mult imea R. Daca f : N N, atunci se foloseste notat ia f (n) = f
n
, indicele reprezentand
variabila acestei funct ii.
1.1.2 Exemplu. Consideram
a
n
: N R, a
n
=
n 1
n + 1
.
Mult imea valorilor acestui sir este
a
1
= 0, a
2
=
1
3
, a
3
=
2
4
..., a
n
=
n 1
n + 1
...
Sirul se noteaza prin mult imea valorilor termenilor sau folosind termenul general (a
n
)
nN
. Alte notat ii
a
n,
a
n

nN
, (a
n
)
nN
Toate aceste funct ii reale (siruri de numere reale) sunt obt inute cu ajutorul termenului general a
n
.
1.1.3 Denit ie. Sir monoton. Un sir (a
n
)
nN
se numeste monoton daca este crescator (sau
descrescator).
Fiind data o funct ie, reamintim ca funct ia f : X Y este crescatoare daca
() x
1
< x
2
x
1
, x
2
X
_
f (x
1
) < f (f
2
)
si descrescatoare daca
() x
1
< x
2
x
1
, x
2
X
_
f (x
1
) > f (f
2
) .
1.1.4. Exemplu. Sirul (a
n
) care satisface a
n+1
< a
n
, () n N este descrescator, deoarece
n + 1 > n si valorile a
n+1
< a
n
pentru orice n N.
1.1.5. Denit ie. Sir marginit. Spunem ca un sir (a
n
)
nN
este marginit, daca exista doua numere
m; M R, astfel ncat pentru orice n N
m a
n
M.
1.1.6. Denit ie. Sir convergent. Spunem ca sirul (a
n
)
nN
este convergent spre numarul a R
daca si numai daca pentru orice > 0, ()N () > 0 astfel ncat pentru orice n > N () [a
n
a[ < .
1.1.7. Observat ii. 1) Relat ia [a
n
a[ < este echivalenta cu
< a
n
a <
_
a
n
< a +
a
n
> a .
117
118 1. S iruri de numere reale
2) Denit ia convergent ei se poate formula: sirul (a
n
) a (sau lim
n
a
n
= a) daca pentru orice
vecinatate (a , a +) a numarului a, exista un rang al termenilor notat N () (depinde de ) de la care
ncepand, tot i termeni se aa n aceasta vecinatate (iar nafara acestei vecinatat i vor un numar nit de
termeni).
1.1.8. Aplicat ii. Folosind denit ia convergent ei unui sir, aratat i ca
1) lim
n
1
n
= 0; 2) lim
n
_
1
2
_
n
= 0.
Solut ie. Vom arata convergent a sirurilor propuse, precizand existent a rangului N () pentru orice
> 0.
(1) Din [a
n
a[ =

1
n
0

=
1
n
< rezulta n >
1

, deci exista N () =
_
1

_
, pentru orice > 0.
Astfel lim
n
1
n
= 0.
(2) Din [a
n
a[ =

1
2
n
0

=
1
2
n
< rezulta 2
n
>
1

, () > 0. Aplicam funct ia log


2
x, care are
baza 2 > 1, deci este crescatoare si obt inem n > log
2
1

. Astfel exista N () =
_
log
2
1

_
, deci sirul
1
2
n
0
(cand n ).
1.1.9 Denit ie. Un sir care nu este convergent se numeste divergent.

In aceasta situat ie, sirul
poate avea limita sau nu are limita.
1.1.10 Denit ie. Spunem ca lim
n
a
n
= , daca () M > 0, () N (M) > 0 astfel ncat pentru orice
n > N (M) sa rezulte a
n
> M.
1.1.11 Exemplu. Avem lim
n
3
n
= , deoarece din inegalitatea a
n
= 3
n
> M pentru orice M > 0,
obt inem n = log
3
3
n
> log
3
M , deci exista N (M) = [log
3
M] .
1.1.12 Exemple. Studiat i convergent a sirului cu termenul general
a
n
=
n 1
n + 1
, n 1.
Folosind denit ia convergent ei unui sir de numere reale, vom arata ca lim
n
n 1
n + 1
= 1 . Avem
[a
n
a[ =

n 1
n + 1
1

n 1 (n + 1)
n + 1

=
2
n + 1
<
n + 1
2
>
1

Obt inem n >


2

1si deci () N () =
_
2

.
Comentariu. Daca =
1
10
, atunci
N () =
_
2
1
10
1
_
= [20 1] = 19.
Deci pentru n > 19, adica de la termenul a
20
), tot i termenii se aa la o distant a mai mica decat 0, 10
fat a de 1.
[a
20
1[ =

20 1
20 + 1
1

19
21
1

=
2
21
<
2
20
=
1
10
= 0, 10.
Daca alegem =
1
100
, atunci rangul termenului (care depinde de ) va :
N
_
1
100
_
=
_
2
1
100
1
_
= 199.
De la termenul a
200
, tot i termenii sirului a
n
vor la o distant a mai mica de
1
100
fat a de 1.
1.1 S iruri de numere reale 119
1.1.13 Aplicat ie. Studiat i limita sirului a
n
= (1)
n
+ sin
n
2
, n N. Observam ca
a
n
= (1)
n
+ sin
n
2

_
_
_
0, n = 4k + 1
1, n = 2k
2, n = 4k + 3
Sirul avand trei puncte limita 2, 0, 1, nu este convergent.
1.1.14 Denit ie. Puncte limita ale unui sir. Un numar a
0
se numeste punct limita a unui
sir (a
n
)
nN
daca orice vecinatate a sa V (a
0
) cont ine cel put in un termen al sirului diferit de a
0
(acesta
poate apart ine sau nu sirului)
1.1.15 Observat ie. Daca a
0
este un punct limita al sirului (a
n
)
nN
, atunci orice vecinatatre a sa
cont ine chiar o innitate de termeni ai sirului.
Remarcam ca denit ia si observat ia de mai sus sunt cazuri particulare ale denit iei punctului de
acumulare al unei mult imi, aici mult imea ind formata din termenii sirului (a
n
) .
1.1.16 Exemplu.
a
n
= (1)
n
+ cos
n
4
=
_

_
1 + 1 = 2 n = 8k
1 +

2
2
n = 8k + 1
1 + 0 = 1 n = 8k + 2
1

2
2
n = 8k + 3
1 1 = 0 n = 8k + 4
1

2
2
n = 8k + 5
1 + 0 = 1 n = 8k + 6
1 +

2
2
n = 8k + 7
deci
a
n
=
_

_
1

2
2
pentru n = 8k + 3; n = 8k + 5
1 +

2
2
pentru n = 8k + 1; n = 8k + 7
0 pentru n = 8k + 4;
1 pentru n = 8k + 2;
2 pentru n = 8k.
Sirul (a
n
)
n1
, cu termenul general a
n
= (1)
n
+ cos
n
4
, n 1, are cinci puncte limita. Daca notam
Lmult imea punctelor limita ale sirului (a
n
)
n1
, atunci
L =
_
1

2
2
; 1 +

2
2
; 0; 1; 2
_
1.1.17 Denit ii. Marginea superioara L a mult imii L se numeste limita superioara a sirului,
L = supL = limsup
n
a
n
= lim
n
a
n
.
Marginea inferioara l a mult imii L se numeste limita inferioara a sirului,
l = inf L = liminf
n
a
n
= lim
n
a
n
.

In exemplul anterior L = 2, l = 1

2
2
.
1.1.18 Proprietat ile marginii inferioare si superioare ale unui sir. Fie sirul (a
n
)
nN
si l, L
limitele inferioara, respectiv superioara ale sirului (a
n
)
nN
. Oricare ar > 0, avem:
120 1. S iruri de numere reale
1) La stanga lui l si la dreapta lui l + se aa un numar nit de termeni ai sirului (a
n
)
nN
;
2)

In intervalul (l , L +) se aa un numar nit de termeni ai sirului (a
n
)
nN
.
1.1.19 Teorema. Condit ia necesara si sucienta ca un sir (a
n
)
nN
care are
l = lim
n
a
n
si L = lim
n
a
n
,
sa e convergent este ca l = L.
Demonstrat ia este imediata.
1.1.20 Teorema. Operat ii cu siruri convergente. Fie sirurile (a
n
)
n1
, (b
n
)
n1
convergente,
cu lima
n
= a, respectiv limb
n
= b. Au loc
1) lim
n
(a
n
b
n
) = lim
n
a
n
lim
n
b
n
= a b;
2) lim
n
(a
n
b
n
) = lim
n
a
n
lim
n
b
n
= a b;
3) lim
n
a
n
b
n
=
lim
n
a
n
lim
n
b
n
=
a
b
;
4) lim
n
a
b
n
n
=
_
lim
n
a
n
_
lim
n
b
n
= a
b
.
Except ie: nedeterminarile de forma , 0,

,
0
0
, 1

, 0
0
si
0
.
1.1.21 Limite remarcabile de forma

1).
lim
n
P
(n)
s
Q
(n)
p
= lim
n
a
0
n
s
+a
1
n
s1
+... +a
s1
n +a
s
b
0
n
p
+b
1
n
p1
+... +b
p1
n +b
p
=
= lim
n
n
s
_
a
0
+
a
1
n
+... +
a
s1
n
s1
+
a
s
n
s
_
n
p
_
b
0
+
b
1
n
+... +
b
p1
n
p1
+
b
p
n
p
_ = lim
n
n
sp

a
0
b
0
(deoarece
1
n
,
1
n
2
, ...,
1
n
s
, ...,
1
n
p
au limita zero) si deci
lim
n
P
s
(n)
Q
p
(n)
=
_

_
sign
a
0
b
0
, pentru s > p
a
0
b
0
, pentru s ,= p
0, pentru s < p.
S-a notat sign
a
0
b
0
=( signum = signatura =) semnul lui
a
0
b
0
.
1.1.22 Exemple. Sa se arate ca: 1) lim
n
n 1
n + 1
= 1; 2) lim
n
lnn
n

= 0 pentru > 0;
3) lim
n
a
n
n

= , unde a > 1, > 0; 4) lim


n
a
n
lnn
= , unde a > 1.
Limitele de la 2), 3), 4) compara modul n care se duc spre innit funct iile crescatoare: y = lnn
(si general y = log
a
x, a > 1), y = a
n
, a > 1, y = n

, > 0.
Figuram alaturat gracele funct iilor crescatoare
y = a
x
, a > 1;
y = x

, R;
y = log
a
x, a > 1
Figura 1
Figura 2
1.2 Criterii de convergent a 121
1.2 Criterii de convergent a
1.2.1 Teorema. Criteriul general de convergent a (al lui Cauchy) Condit ia necesara si sucienta
ca sirul de numere reale (a
n
)
n1
sa e convergent este () > 0 exista un numar N () astfel ca pentru
orice p N

are loc
[a
n+p
a
n
[ < .
(Cauchy-matematician francez. A fost ministru al Educat iei, dupa caderea lui Napoleon)
Demonstrat ie.Condit ia este necesara. Sa presupunem ca sirul (a
n
)
n1
este convergent si are limita
a
lim
n
a
n
= a
Folosind denit ia convergent ei unui sir, va rezulta ca pentru orice > 0 exista N () astfel ncat pentru
orice n > N ()
[a
n
a[ <
t
=

2
, deci si [a
n+p
a[ <

2
, p > 1.
Diferent a
[a
n+p
a
n
[ = [a
n+p
a +a a
n
[ [a
n+p
a[ +[a
n
a[ <

2
+ = .
Condit ia este sucienta. Presupunem ca [a
n+p
a
n
[ < n condit iile: () > 0 () N () > 0 astfel
ncat pentru orice n > N () si orice p = 1, 2, 3, ...(ntreg). Cu alte cuvinte inegalitatea [a
n+p
a
n
[ < are
loc pentru valori n+p > N+p si deci termenii a
N+p
(p = 1, 2, 3, ...) se aa n intervalul (a
N
, a
N
+) ,
iar a
1
, a
2
, ..., a
N1
sunt nafara acestui interval.

Intr-adevar inegalitatea [a
N+p
a
N
[ < implica <
a
N+p
< a
N
< sau a
N
< a
N+p
< a
N
+ . Notam a
N
= l, a
N
+ = L Rezulta 0 L l < 2,
oricare ar > 0, iar L si l xe. Diferent a Ll nu poate oricat de mica (oricare ar > 0), numai
daca L = l. Avem deci: de la un rang n > N () , tot i termenii sirului a
N+p
(l, L) , rezulta ca sirul
(a
n
)
n1
este convergent.
1.2.2 Denit ie. Se numeste sir Cauchy (fundamental) un sir cu proprietatea ca pentru orice
> 0 () N () > 0 astfel ncat pentru orice n > N () si orice p N

sa aibe loc
[a
n+p
a
n
[ <
(sirul din criteriul general al lui Cauchy, dat mai sus).
1.2.3 Aplicat ie. Teorema 1.2.1 a fost data de Cauchy cu scopul de a demonstra divergent a sirului
cu termen general
a
n
= 1 +
1
2
+
1
3
+... +
1
n
, n N

,
care reprezinta suma de rang n a inverselor numerelor naturale. Aceasta suma apare n media armonica
a numerelor naturale. Sa se arate ca sirul (a
n
)
n1
este divergent.
Solut ie. Formam
[a
n+p
a
n
[ =
1
n + 1
. .
cel mai mare termen
+
1
n + 2
+... +
1
n +p
. .
cel mai mic termen
> p
1
n +p
= (aleg p = n) =
n
n +n
=
1
2
,
care nu poate oricat de mic.
Observam ca demonstrat ia divergent ei folosind criteriul general al lui Cauchy va trebui sa nege
armat iile criteriului enunt at.
1.2.4 Exemplu. Folosind criteriul general al lui Cauchy, demonstrat i ca sirul (a
n
)
n1
este divergent,
iar sirul (b
n
)
n1
este convergent, unde
a) a
n
=
1

1
+
1

2
+... +
1

n
;
b) b
n
=
1
1 4
+
1
4 7
+... +
1
[1 + 3 (n 1)] [1 + 3n]
.
122 1. S iruri de numere reale
Solut ie. a) Formam
[x
n+p
x
n
[ =
1

n + 1
+
1

n + 2
+... +
1

n +p
> p
1

n +p
=
=
_
Aleg
p = n
_
=
n

2n
=
_
n
2
> 1 pentru n > 2,
deci diferent a nu poate oricat de mica pentru un p ales.
b) Pentru a demonstra armat ia de convergent a va trebui sa aratam existent a lui N () n condit iile
criteriului. Avem diferent a
[x
n+p
x
n
[ =
1
(1 + 3n) [1 + 3 (n + 1)]
+... +
1
[1 + 3 (n +p 1)] [1 + 3 (n +p)]
Dar
1
(1 + 3n) [1 + 3 (n + 1)]
=
1
(1 + 3n) (4 + 3n)
=
1
3
_
1
3n + 1

1
3n + 4
_
.
Deci
[x
n+p
x
n
[ =
1
3

1
3n + 1

1
3n + 4
+
+
1
3n + 4

1
3n + 7
+
+. . . . . . . . . . . . . . . . . . . . . . . . +
+
1
3n + 3p 2

1
3n + 3p + 1

=
=
1
3
_
1
3n + 1

1
3n + 3p + 1
_
<
1
3

1
3n + 1
<
1
n
<
pentru orice > 0. Va rezulta n >
1

.
Am gasit n acest mod N () =
_
1

_
si deci sirul este convergent conform acestui criteriu.
1.2.5 Teorema. Criteriu de convergent a pentru siruri monotone, Criteriul lui Weier-
strass.Un sir monoton si marginit este convergent.
Demonstrat ie. Consideram un sir (a
n
) n 1 marginit, deci nu poate avea limite innite. Vom
arata ca are o singura limita. Fara a restrange generalitatea, consideram sirul crescator avand doua
puncte limita l, L, l ,= L.

Impart im intervalul L l n trei si notam =


L l
3
Intervalele (l , l +) , (L , L +) sunt disjuncte. Consideram un indice n
1
, astfel ncat
a
n
1
(l , l +) . Va exista un indice n
2
,astfel ncat n
2
> n
1
si a
n
2
(L , L +) .

In caz contrar,
intervalul al doilea nu ar mai cont ine o intimitate de termeni si L nu ar mai punct limita. Deoarece
sirul este crescator rezulta
a
n
1
< a
n
2
pentru n
1
< n
2
. (1)
Daca n > n
2
, intervalul al doilea (L , L +) nu poate cont ine tot i termenii sirului, deoarece n
acest caz, n primul intervalar un numar nit de termeni si l nu ar putea punct limita. Rezulta ca
pentru n = n
3
> n
2
a
n
3
>
a
n
2
pentru n
3
> n
2
. (2)
Inegalitat ile (1) si (2) luate mpreuna contrazic monotonia sirului, va rezulta deci ca l = L si deci
sirul este convergent.
1.2.6. Siruri remarcabile
1. Numarul e. Consideram sirul (e
n
)
n1
cu termenul general e
n
=
_
1 +
1
n
_
n
, n 1. Vom arata ca
sirul (e
n
)
n1
este monoton crescator si marginit.
1.2 Criterii de convergent a 123
Solut ie. Folosind binomul lui Newton are loc
x
n
=
_
1 +
1
n
_
n
= 1 +C
1
n
1
n
+C
2
n
1
n
2
+C
3
n
1
n
3
+... +C
n
n
1
n
n
=
= 1 +
n
1

1
n
+
n(n 1)
2!
1
n
2
+
n(n 1) (n 2)
3!
1
n
3
+... +
n!
n!

1
n
n
=
= 1 + 1 +
1
2!
_
1
1
n
_
+
1
3!
_
1
1
n
__
1
2
n
_
+... +
1
n!
_
1
1
n
_

_
1
n 1
n
_
.
Dar 0 < 1
i
n
< 1 pentru i = 1, ..., n 1, deci pentru orice n 1
e
n
< 1 + 1 +
1
2!
+
1
3!
+... +
1
n!
< 2 +
1
2
+
1
2
2
+... +
1
2
n1
= 1 +
1
1
1
2
= 3.
Avem
a
n+1
=
_
1 +
1
n + 1
_
n+1
= 1+
1
1!
+
1
2!
_
1
1
n 1
_
+... +
1
n!
_
1
1
n + 1
__
1
2
n + 1
_
...
_
1
n
n + 1
_
rezulta a
n+1
> a
n
, n 1 deoarece a
n+1
are ecare termen de acelasi rang mai mare decat termenul
corespunzator al din a
n
, n plus mai are un termen pozitiv.

In concluzie, sirul (e
n
)
n1
este monoton crescator si marginit, deci este convergent. Limita acestui
sir se noteaza cu e (2, 3) (n onoarea matematicianului elvet ian Euler)
e = 2, 7182818284...
Este un numar transcendent. Transcendent a sa se datoreaza faptului ca nu se poate obt ine din ecuat ii
care folosesc operat iile algebrice (adunare, scadere, nmult ire...)
Generalizare
lim
n
(1 +u
n
)
1
u
n
= e oricare ar sirul (u
n
)
n1
cu u
n
0, cand n .
1.2.7. Teorema. Criteriul lui Stolz.Daca (x
n
)
n1
este un sir oarecare, iar (y
n
)
n1
este un sir
crescator cu limn y
n
= astfel ncat exista
lim
n
x
n+1
x
n
y
n+1
y
n
= l (nit sau nu),
atunci exista
lim
n
x
n
y
n
= lim
n
x
n+1
x
n
y
n+1
y
n
= l.
Reciproca nu are loc.
1.2.8. Exemple. 1) Fie sirul (a
n
)
n1
si denim
m
aritm
(a
1
, a
2
, ...a
n
) =
a
1
+a
2
+... +a
n
n
, n 1.
Notam x
n
= a
1
+a
2
+... +a
n
, y
n
= n, n 1. Avem
lim
n
x
n+1
x
n
y
n+1
y
n
= lim
n
a
n+1
n + 1 n
= lim
n
a
n+1
.
Constatam ca
lim
n
x
n
y
n
= lim
n
m
aritm
(a
1
, ..., a
n
) = lim
n
a
n+1
= lim
n
a
n
.
2) Fie sirul (a
n
)
n1
. Pentru orice n 1, notam m
aritm
= media armonica (a
1
, a
2
, ..., a
n
) denita prin
relat ia
n
m
arm
=
1
a
1
+
1
a
2
+... +
1
a
n
sau
1
m
arm
=
1
a
1
+
1
a
2
+... +
1
a
n
n
.
124 1. S iruri de numere reale
Aplicam criteriul lui Stolz:
lim
n
1
a
1
+... +
1
a
n
+
1
a
n+1

_
1
a
1
+... +
1
a
n
_
n + 1 n
= lim
n
1
a
n+1
.
Deci
lim
n
1
m
arm
= lim
n
1
a
n+1
sau
lim
n
m
arm
(a
1
, ..., a
n
) = lim
n
a
n+1
= lim
n
a
n
.
Observam ca daca sirul (a
n
)
n1
este convergent (si are limita a) atunci
lim
n
a
n
= lim
n
m
arm
(a
1
, ..., a
n
) = lim
n
m
aritm
(a
1
, ..., a
n
) .
3) Daca termenii sirului (a
n
)
n1
, sunt pozitivi a
n
> 0, denim
m
g
(a
1
, ..., a
n
) =
n

a
1
, a
2
...a
n
, n 1.

Intre mediile aritmetice, geometrica si armonica exista inegalitat ile


m
aritm
m
g
m
arm
,
egalitatea avand loc cand a
1
= a
2
= ... = a
n
.
Demonstrat ia acestor inegalitat i rezulta imediat pentru cazul n = 2. Pentru n > 2 sunt necesare
considerat ii asupra extremului funct iilor de mai multe variabile.
Folosind metoda clestelui, pentru n (sucient de mare), se constata ca cele trei medii tind
spre aceeasi limita. Aceasta observat ie este utila n teoria probabilitat ilor.
1.2.9. Consecint a. Daca a
n
> 0, n N, atunci
lim
n
n

a
n
= lim
n
n
_
a
n
a
n1
a
n1
= ... = lim
n
n
_
a
n
a
n1

a
n1
a
n2

a
2
a
1

a
1
1
=
= lim
n
m
g
_
a
1
a
0
,
a
2
a
1
,
a
3
a
2
, ...,
a
n
a
n1
_
= lim
n
a
n
a
n1
.
cu a
0
= 1.
Va rezulta relat ia utila:
lim
n
a
n+1
a
n
= lim
n
n

a
n
pentrua
n
> 0, a
0
= 1.
1.2.10. Exemple.
1) lim
n
n

n = lim
n+1
n
= 1;
2) lim
n

a = lim
a
a
= 1, a > 0.
1.2.11 Teorema. Criteriul raportului.Fie (a
n
)
n1
un sir de numerele reale strict pozitive cu
proprieatea ca exista lim
n
a
n
= l. Sunt adevarate armat iile de mai jos:
(i) Daca l < 1 rezulta lim
n
a
n
= 0;
(ii) Daca l > 1 rezulta lim
n
a
n
= .
Demonstrat ia teoremei se bazeaza pe teorema de convergent a cu .
1.2.12 Exemplu. Calculat i lim
n
a
n
n!
, unde a R

+
.
Notam a
n
=
a
n
n!
. Rezulta
lim
n
a
n+1
a
n
= lim
n
a
n+1
(n + 1)!

n!
a
n
= lim
n
a
n + 1
= 0 < 1,
deci lim
n
a
n
= 0.
Capitolul 2
Serii de numere
2.1 Serii de numere
Fie sirul de numere reale (a
n
)
n1
. Cea mai simpla operat ie pe care o putem face cu termenii sirului,
adunarea este o suma cu o innitate de termeni. Stim sa adunam doi, trei,...,un milion,..., oricat i termeni
cu condit ia sa stim cat i termeni adunam. Pentru a da un sens acestei sume (innite): a
1
+a
2
+... +a
n
+...
introducem sumele part iale (care formeaza un sir)
s
1
= a
1
s
2
= a
1
+a
2

s
n
= a
1
+a
2
+... +a
n

Sirul (s
n
)
nN
poate convergent, caz n care notam lim
n
s
n
= s si s = a
1
+a
2
+... +a
n
+...
2.1.1 Denit ie. Se numeste serie o suma

n=1
a
n
= a
1
+a
2
+. . . +a
n
+. . .
Daca s = lim
n
n

k=1
a
k
, ea se numeste suma seriei; a
n
se numesc termenii seriei; (s
n
) sirul sumelor
part iale, unde s
n
=
n

k=1
a
k
, n 1.
Notat ie

n=1
a
n
sau

a
n
(fara a specica valorile posibile ale lui n)
2.1.2 Denit ie. Spunem ca seria

a
n
este convergenta (divergenta, oscilanta) dupa cum sirul
sumelor part iale s
n
este convergent (divergent, oscilant).
2.1.3 Observat ie. Daca se suprima un numar nit de termeni ai unei serii, atunci seria obt inuta
are aceeasi natura cu seria init iala.
2.1.4 Exemple. 1) Seria

n=1
1
n
, numita seria armonica este divergenta deoarece sirul sumelor
part iale
s
n
= 1 +
1
2
+... +
1
n
, n 1
este un sir divergent (Criteriul general Cauchy).
2) Fie q R. Seria geometrica este

n=0
q
n
= 1 +q +q
2
+... +q
n
+...
125
126 2. Serii de numere
Pentru q ,= 1, suma primilor n termeni ai unei progresie geometrice

a
0
, a
0
q, ..., a
0
q
n1
este
s
n
=
a
n
q a
1
q 1
=
a
0
q
n1
q a
0
q 1
= a
0
1 q
n
1 q
, n 1.
Deci
s
n

_
a
0
1 q
, [q[ < 1
a
0
, [q[ > 1.
Daca q = 1, atunci a
n
= 1 + 1 + 1 +... + 1 = n, lim
n
s
n
= .
Va rezulta ca seria geometrica

n=0
q
n
este convergenta, pentru [q[ < 1 si este divergenta, pentru
[q[ 1.
2.1.5 Teorema. Criteriul general al lui Cauchy de convergent a a unei serii. Condit ia
necesara si sucienta ca seria

a
n
sa e convergenta este ca pentru () > 0 () N () astfel ncat
pentru orice n > N () si orice p N

sa aiba loc:
[a
n+1
+a
n+2
+... +a
n+p
[ < .
Demonstrat ie. Folosim criteriul general al lui Cauchy pentru sirul sumelor part iale s
n
= a
1
+... +a
n
.
Are loc
[s
n+p
s
n
[ = [a
n+1
+a
n+2
+... +a
n+p
[ < ,
de unde rezulta teorema de mai sus.
2.1.6 Consecint a.

In cazul unei serii convergente termenul ei general are limita zero.

Intr-adevar,
alegand p = 1 si folosind criteriul de mai sus rezulta [a
n+1
[ < care este echivalent cu lim
n
[a
n+1
[ = 0.
Aceasta condit ie este necesara, dar nu este sucienta pentru convergent a unei serii.
2.1.7 Exemplu. Seria
1
n
are
1
n
0, dar este o serie divergenta (seria armonica).
2.1.8 Consecint a Daca la seria

a
n
, lim
n
a
n
,= 0, atunci seria este divergenta, fapt care rezulta
din Consecint a 1).
2.1.9 Exemplu. Fie

n=1
2n 1
3n + 5
=
1
8
+
3
11
+... +
2n 1
3n + 5
+...
este o serie divergenta, deoarece termenul ei general a
n
=
2n 1
3n + 5

2
3
.
2.1.10. Denit ie. O serie

a
n
se numeste absolut convergenta, daca seria

[a
n
[ este o serie
convergenta (notam AC )
2.1.11. Teorema. Daca o serie este absolut convergenta, atunci seria este convergenta.
Demonstrat ie. Presupunem

[a
n
[ , o serie convergenta. Aplicand criteriul general al lui Cauchy:
() > 0 () N () , astfel ncat pentru orice n > N () si orice p N

[a
n+1
[ +[a
n+2
[ +... +[a
n+p
[ < .
Dar
[a
n+1
+... +a
n+p
[ < [a
n+1
[ +[a
n+2
[ +... +[a
n+p
[ <
oricare ar > 0 si oricare ar p N

pentru orice n > N () . Seria



a
n
este o serie convergenta.
Seriile convergente fara a si absolut convergente se numesc serii semiconvergente
2.2 Serii alternante. 127
2.2 Serii alternante.
2.2.1. Denit ie. O serie de forma

n=0
(1)
n
b
n
= b
0
b
1
+b
2
b
3
+... cu b
n
> 0, n N
se numeste serie alternanta.
2.2.2. Teorema. Criteriul lui Abel pentru serii de forma

a
n
b
n
. Daca (b
n
) 0 (este un
sir monoton descrescator de numere pozitive spre zero) si

a
n
este o serie cu sirul sumelor part iale
A
n
= a
1
+a
2
+... +a
n
, marginit, atunci, seria

a
n
b
n
este convergenta
Demonstrat ie. Folosim criteriul al lui Cauchy de convergent a a seriilor, scriem a
n
= A
n
A
n1
.
Avem
[a
n+1
b
n+1
+. . . +a
n+p
b
n+p
[ =

b
n+1
(A
n+1
A
n
)
. .
A
n+1
A
n
+b
n+2
(A
n+2
A
n+1
)
. .
A
n+2
A
n+1
+. . . +b
n+p
(A
n+p
A
n+p1
)
. .
A
n+p
A
n+p1

A
n+1
(b
n+1
b
n+2
)
. .
+
+A
n+2
b
n+2
b
n+3
. .
+
+A
n+p1
(b
n+p1
b
n+p
)

+[b
n+1
(A
n
) +b
n+p
A
n+p
[
M
_
_
_
b
n+1
b
n+2
+b
n+2
. .
0
b
n+3
+. . .
. .
0
+b
n+p1

..
b
n+p
+b
n+1
+b
n+p
_
_
_
= 2b
n+1
M
Deoarece b
n+1
0, folosind denit ia convergent ei spre zero, obt inem ca pentru orice > 0 () N () > 0
astfel ncat pentru orice n > N () sa aiba loc b
n+1
<

2M
, deci
[a
n+1
b
n+1
+... +a
n+p
b
n+p
[ < 2 b
n+1
M <
oricare ar > 0 si pentru orice n > N () si orice p N

.
Seria

a
n
b
n
verica criteriul general al lu Cauchy, deci este convergenta.
Consideram cazul particular al seriei lui Abel

a
n
b
n
, n care a
n
= (1)
n+1
.
2.2.3. Teorema. Criteriul lui Leibniz. Daca ntr-o serie alternanta

(1)
n+1
b
n
sirul
(b
n
)
+
0 (monoton descrescator de numere pozitive spre zero), atunci seria alternanta

(1)
n+1
b
n
este convergenta.
Demonstrat ie. Folosind criteriul lui Abel pentru cazul cand

a
n
=

n=1
(1)
n+1
, deci [a
1
+a
2
+... +a
n
[ < 1
suma part iala
a
1
+a
2
+... +a
n
=
_
0, n = par
1, n = impar
si deci seria alternanta este convergenta.
2.2.4. Exemplu.

n=1
(1)
n+1
1
n
= 1
1
2
+
1
3

1
4
+... + (1)
n+1
1
n
+...
este o serie alternanta convergenta, deoarece sirul
b
n
=
1
n

+
0.
128 2. Serii de numere
Pentru a demonstra monotonia sirului (b
n
)
n1
, avem
b
n+1
b
n
=
1
n + 1

1
n
< 0, n 1.
Atunci b
n+1
< b
n
pentru orice n N, n > 1.
Seria alternanta

n=1
(1)
n+1
1
n
este un exemplu de serie semiconvergenta, deoarece seria valorilor
absolute

n=1
1
n
este o serie divergenta.(seria armonica)
2.3 Serii cu termeni pozitivi. Criterii de comparat ie
2.3.1 Criteriul I de comparat ie Fie

a
n
si

b
n
doua serii cu termeni pozitivi. Daca exista
un numar N astfel ncat () n N avem
a) a
n
b
n
si

b
n
convergenta, atunci

a
n
convergenta;
b) a
n
b
n
si

a
n
divergenta, atunci

b
n
divergenta.
Demonstrat ie. Din a
n
b
n
pentru n > N rezulta
A
n
= a
1
+a
2
+... +a
n
< b
1
+b
2
+... +b
n
= B
n
.
Sirul sumelor part iale B
n
al seriei convergente

b
n
, este un sir convergent, deci marginit, n concluzie
() M > 0 astfel ncat 0 < A
n
B
n
< MA
n+1
= A
n
+ a
n+1
> A
n
[[ Sirul (A
n
)
n1
este un sir
convergent (marginit si monoton). Urmeaza ca si seria

a
n
este convergenta. Punctul (b) al criteriului
I se obt ine prin negarea punctului a)
2.3.2 Exemplu. Studiat i natura seriei
1
n

, 0 < < 1. Deoarece n

< n
1
pentru 0 < < 1;
1
n

>
1
n
si seria
1
n
este divergenta. Va rezulta ca seria
1
n

divergenta pentru (0, 1).


2.3.3 Exemplu. Studiat i natura seriei

n=2
1
lnn
. Deoarece
1
lnn
>
1
n
, n 1 si seria
1
n
este
divergenta, rezulta ca seria
1
lnn
este divergenta.
2.3.4 Criteriul II de comparat ie. Fie

a
n
si

b
n
doua serii cu termeni pozitivi. Daca exista
un numar N N astfel ncat pentru n N sa avem
a)
a
n+1
a
n

b
n+1
b
n
si

b
n
convergenta, atunci

a
n
convergenta;
b)
a
n+1
a
n

b
n+1
b
n
si

a
n
divergenta, atunci

b
n
divergenta.
Demonstrat ie. Putem presupune N = 1. Din inegalitat ile
a
n+1
a
n

b
n+1
b
n
va rezulta
a
1
b
1

a
2
b
2
...
a
n
b
n
. . .
Notam
a
1
b
1
= k (,= 0) si deci
a
1
= kb
1
a
2
kb
2
. . . . . . . . .
a
n
kbn
. . . . . . . . .
Aplicam criteriul I de comparat ie la seriile

a
n
si

kb
n
, de unde vor rezulta armat iile a) si b) din
criteriul al II-lea de comparat ie.
2.3 Serii cu termeni pozitivi. Criterii de comparat ie 129
2.3.5 Criteriul al III-lea de comparat ie. Fie seriile

a
n
si

b
n
cu termeni pozitivi.
a) Daca () lim
n
a
n
b
n
= l (,= 0 nit), atunci seriile

a
n
si

b
n
au aceeasi natura;
b) Daca l = 0 si seria

b
n
este convergenta, atunci

a
n
este convergenta;
c) Daca l = si seria

b
n
este divergenta, atunci

a
n
este divergenta.
2.3.6 Exemplu. Studiat i natura seriilor:
a)

n=1
1
10n + 17
; b)
1
an +b
, a, b R, a ,= 0.
Solut ie. a) Avem
lim
n
1
10n + 17
1
n
=
1
10
.
Seria

n=1
1
10n + 17
este divergenta, deoarece are aceeasi natura cu seria armonica.
b) Avem
lim
n
1
an +b
1
n
=
1
a
.
Seria

1
an+b
este divergenta, are aceeasi natura cu seria armonica.
2.3.7 Criteriul lui DAlembert (raportului) Fie seriua

a
n
, unde a
n
> 0. Daca exista un
numar N astfel ncat pentru orice n > N sa aibe loc:
a)
a
n+1
a
n
l < 1, atunci seria

a
n
este convergenta;
b)
a
n+1
a
n
l > 1, atunci seria

a
n
este divergenta.
Demonstrat ie. a) Presupunem inegalitat ile adevarate pentru n 1, adica
a
2
la
1
a
3
la
2
l
2
a
1
. . . . . . . . .
a
n
la
n1
... l
n1
a
1
obt inem seria

l
n1
a
1
= a
1

l
n1
, serie geometrica cu rat ia l. Daca l < 1, aplicand criteriul I de
comparat ie, rezulta ca si seria

a
n
este convergenta.
b) Din inegalitat ile a
n
l
n1
a
1
si seria a
1

l
n1
este divergenta (serie geometrica cu rat ia l > 1).
Va rezulta ca si seria

a
n
este divergenta (punctul (b) de la criteriul I de comparat ie).
2.3.8 Criteriu practic. Fie seria

a
n
, cu a
n
> 0. Daca lim
a
n+1
a
n
= l, atunci pentru l < 1 seria

a
n
este convergenta, pentru l > 1 seria

a
n
este divergenta, iar pentru l = 1 nu se poate determina
natura seriei

a
n
.
2.3.9 Aplicat ie. Studiat i natura seriei

[n!]
2
a
n
(2n)!
, a > 0.
Solut ie. Avem
a
n+1
=
[(n + 1)!]
2
a
n+1
[2 (n + 1)]!
=
[n!]
2
[(n + 1)]
2
a
n
a
(2n + 2)!
=
[n!]
2
a
n
(2n)!

(n + 1)
2
a
(2n + 1) (2n + 2)
lim
n
a
n+1
a
n
=
a
4
.
130 2. Serii de numere
Pentru
a
4
< 1, adica a < 4 obt inem

a
n
este convergenta, pentru a > 4 seria

a
n
este divergenta.
Daca a = 4, criteriul nu este concludent si vom aplica un alt criteriu (Raabe-Duhamel).
2.3.10 Criteriul lui Raabe-Duhamel Fie seria

a
n
, cu a
n
> 0. Daca exista un numar N astfel
ncat pentru orice n > N
lim
n
n
_
a
n
a
n+1
1
_
= l,
atunci pentru l < 1 seria

a
n
este divergenta, pentru l > 1 seria

a
n
este convergenta, iar pentru
l = 1 criteriul nu e concludent.
2.3.11 Exemplu. Fie seria

[n!]
2
4
n
(2n)!
din exemplul precedent (Aplicat ia 2.3.9), cu a = 4. Avem
a
n+1
a
n
=
4 (n + 1)
2
(2n + 1) (2n + 2)
1,
deci criteriul raportului nu este concludent n determinarea naturii seriei. Deoarece
lim
n
n
_
a
n
a
n+1
1
_
= limn
_
2n + 1
2n + 2
1
_
= lim
n
n
1
2n + 2
= 1 < 1,
obt inem ca pentru a = 4 seria
a
n
(n!)
2
(2n)!
este divergenta.
2.3.12 Criteriul radacinii (Cauchy) criteriul practic. Fie seria

a
n
, cu a
n
> 0. Daca
lim
n
n

a
n
= l,
atunci pentru l < 1 seria

a
n
este convergenta, l > 1 seria

a
n
este divergenta, iar pentru l = 1
criteriul nu este concludent.
2.3.13 Observat ie. O consecinta la Lema lui Stolz arata ca pentru orice sir (a
n
)
n1
, cu a
n
> 0
lim
n
a
n+1
a
n
= lim
n
n

a
n
.
Deci criteriul radacinii este echivalent cu criteriul lui DAlembert.
2.3.14 Exemplu Studiat i natura seriei

n=1
_
n + 1
n
_
n
2
a
n
, a > 0.
Solut ie. Avem
lim
n
n

a
n
= lim
n
a
_
n + 1
n
_
n
= lim
n
a
_
n +
1
n
_
n
= a e.
Daca a <
1
e
, atunci seria este convergenta; daca a >
1
e
, atunci seria este divergenta. Daca a =
1
e
obt inem
a
n
=
_
1 +
1
n
_
n
2

1
e
n
. Deoarece
e
_
1 +
1
n
_
n+1
e
n

_
_
1 +
1
n
_
n+1
_
n
=
_
1 +
1
n
_
n
2

_
1 +
1
n
_
n
1
e
n

1
_
1 +
1
n
_
n
2 _
1 +
1
n
_
n
2.3 Serii cu termeni pozitivi. Criterii de comparat ie 131
rezulta
a
n
=
_
1 +
1
n
_
n
2
1
e
n

1
_
1 +
1
n
_
n
, n 1
Atunci lim
n
a
n

1
e
, deci seria

a
n
nu poate convergenta (termenul ei general nu tinde la zero!)
2.3.15 Criteriul de condensare (Cauchy)Fie seria

a
n
avand (a
n
) avand un sir monoton de-
screscator de numere pozitive. Atunci seriile

n=1
a
n
si

k=1
2
k
a
2
k au aceeasi natura.
2.3.16 Aplicat ie. Fie seria

n=1
l
n

, unde R
+
. Seria

2
k
l
(2
k
)

l
2
k(1)
=

k=1
_
l
2
1
_
k
este o serie geometrica cu rat ia
l
2
1
. Daca > 1, 0 <
l
2
1
< 1 si seria geometrica este convergenta,
deci si seria
1
n

este convergenta.
2.3.17. Observat ie. Seria
1
n

, R
+
se numeste seria armonica generalizata (Riemann).
Concluzie:
_

_
Pentru 1 seria
1
n

este divergenta;
Pentru > 1 seria
1
n

este convergenta.
Capitolul 3
Funct ii f : D R R continue
3.1 Elemente de topologie
Notam R = R , + .
Prin vecinatate (n R) a numarului real x
0
Rnt elegem orice mult ime W R cu proprietatea
ca exista > 0 pentru care (x
0
, x
0
+) W.
Prin vecinatate (n R) a lui +nt elegem orice mult ime W R cu proprietatea ca exista > 0
pentru care (, +] W.
Prin vecinatate (n R) a lui nt elegem orice mult ime W R cu proprietatea ca exista < 0
pentru care [, ) W.
Remarcam ca daca mult imea W este vecinatate pentru x
0
R, atunci x
0
W.
Mai ment ionam ca daca W
1
, W
2
sunt doua vecinatat i pentru x
0
R, atunci mult imea W
1
W
2
este
de asemenea o vecinatate pentru x
0
.
3.1.1 Denit ie. Fie D R, D ,= . Un punct x
0
R , + se numeste punct de
acumulare pentru mult imea D, daca pentru orice vecinatate W a punctului x
0
are loc DWx
0
, = .
Exista posibilitatea ca anumite puncte de acumulare x
0
R pentru o mult ime D R sa nu e
elemente ale mult imii D. De exemplu pentru mult imea D = (a, b) unde a, b R, a < b, ecare punct
x
0
[a, b] este punct de acumulare; pentru mult imea D = (, 0) (0, +) orice x
0
R este punct
de acumulare; pentru D = (2, 1] [2, 3) 7, 8 [10, +) punctele 2, 3 si + sunt puncte de
acumulare, fara a elemente ale lui D.
3.1.2 Denit ie. Fie D R, D ,= . Un punct x
0
D se numeste punct izolat al mult imii D daca
exista o vecinatate W a lui x
0
astfel ncat D W = x
0
.
Este clar ca orice punct x
0
D este ori punct de acumulare pentru mult imea D, ori punct izolat al
mult imii D.
3.1.3 Denit ie. Fie D R, D ,= .
i) Un punct x
0
D se numeste punct interior al mult imii D daca exista o vecinatate W a lui x
0
astfel ncat W D.
ii) Mult imea punctelor interioare ale mult imii D se numeste interiorul lui D.
3.1.4 Denit ie. Mult imea D R se numeste mult ime deschisa daca pentru orice x
0
D exista
o vecinatate W a lui x
0
astfel ncat W D.

In continuare enunt am cateva proprietat i esent iale ale mult imii R, proprietat i care au rol si la studiul
funct iilor f : D R R.
3.1.5 Axioma (Axioma existent ei supremumului) Orice submult ime nevida si majorata a lui R are
supremum (cel mai mic majorant) n R.
3.1.6 Axioma (Axioma existent ei inmumului) Orice submult ime nevida si minorata a lui R are
inmum (cel mai mare minorant) n R.
Ment ionam ca aceste doua axiome sunt echivalente.
3.1.7 Teorema (Teorema lui Cantor) Daca sirurile de numere reale (a
n
)
n1
si (b
n
)
n1
au urmatoarele
proprietat i
i) sirul (a
n
)
n1
este crescator,
132
3.2 Limita unei funct ii f : D R R ntr-un punct 133
ii) sirul (b
n
)
n1
este descrescator,
iii) a
n
b
n
, n N

,
iv) lim
n
(b
n
a
n
) = 0,
atunci cele doua siruri (a
n
)
n1
, (b
n
)
n1
sunt convergente catre o limita comuna si
lim
n
a
n
= lim
n
b
n

nN
[a
n
, b
n
] .
3.1.8 Teorema (Teorema lui Cesaro) Orice sir marginit de numere reale are un subsir convergent.
3.2 Limita unei funct ii f : D R R ntr-un punct
Reamintim pe scurt cateva not iuni legate de limita ntr-un punct a unei funct ii f : D R R.
Formulam doua denit ii, echivalente ntre ele, una pe baza not iunii de vecinatate si cealalta pe baza
not iunii de limita a unui sir de numere reale.
3.2.1 Denit ie. Fie D R, f : D R o funct ie, x
0
R , + un punct de acumulare
pentru D. Fie l R , + .
(i) Spunem ca funct ia f n punctul x
0
are limita l, si notam lim
xx
0
f (x) = l, daca pentru ecare
vecinatate V a lui l exista o vecinatate U a lui x
0
cu proprietatea ca f (D U x
0
) V, ceea ce revine
la
x D U x
0
f (x) V ;
(ii) (Heine) Spunem ca funct ia f n punctul x
0
are limita l, si notam lim
xx
0
f (x) = l, daca oricare
ar (x
n
)
n1
un sir format cu numere reale diferite de x
0
din D, avand ca limita pe x
0
, sirul de numere
reale (f (x
n
))
n1
are limita l.
Alta notat ie: f (x) l, cand x x
0
.
3.2.2 Observat ie. Se arata usor ca daca funct ia f are limita n punctul x
0
, atunci ea are o singura
limita n acest punct. Utilizam denit ia cu siruri a limitei unei funct ii ntr-un punct. Daca f n punctul
x
0
ar avea doua limite l
1
, l
2
, l
1
,= l
2
, atunci, considerand un sir (x
n
)
n1
de numere reale din D x
0
cu
limita x
0
, sirul de numere reale (f (x
n
))
n1
ar avea doua limite diferite, l
1
si l
2
, ceea ce este imposibil.

In urmatoarele noua propozit ii, utilizand caracterizari cu inegalitat i ale vecinatat ilor punctului x
0
,
respectiv l formulam caracterizari ale situat iilor cand funct ia f n punctul x
0
are limita l, concret pe cele
noua cazuri posibile ale Denit iei 3.2.1.
3.2.3 Propozit ie. Fie funct ia f : D R R si x
0
= punct de acumulare pentru D.
Urmatoarele armat ii sunt echivalente.
(i) lim
x
f (x) = ;
(ii) pentru orice numar real < 0 (oricat de mic ar ) exista un numar real () < 0 astfel ncat
x D, x < () f (x) < .
3.2.4 Propozit ie. Fie funct ia f : D R R, x
0
= punct de acumulare pentru D si l R.
Urmatoarele armat ii sunt echivalente:
(i) lim
x
f (x) = l;
(ii) pentru orice numar real > 0 (oricat de mic ar ) exista un numar real () < 0 astfel ncat
x D, x < () [ f (x) l[ < .
3.2.5 Propozit ie. Fie funct ia f : D R R si x
0
= punct de acumulare pentru D.
Urmatoarele armat ii sunt echivalente:
(i) lim
x
f (x) = +;
(ii) pentru orice numar real > 0 (oricat de mare ar ) exista un numar real () < 0, astfel ncat
x D, x < () f (x) > .
134 3. Funct ii f : D R R continue
3.2.6 Propozit ie. Fie funct ia f : D R R si x
0
R punct de acumulare pentru D. Urmatoarele
armat ii sunt echivalente:
(i) lim
xx
0
f (x) = ;
(ii) pentru orice numar real < 0 (oricat de mic ar ) exista un numar real () > 0 astfel ncat
x D x
0
, [x x
0
[ < () f (x) < .
3.2.7 Propozit ie. Fie funct ia f : D R R, x
0
R punct de acumulare pentru D si l R.
Urmatoarele armat ii sunt echivalente:
(i) lim
xx
0
f (x) = l;
(ii) pentru orice numar real > 0 (oricat de mic ar ) exista un numar real () > 0 astfel ncat
x D x
0
, [x x
0
[ < () [ f (x) l[ < .
3.2.8 Propozit ie. Fie funct ia f : D R R si x
0
R punct de acumulare pentru D. Urmatoarele
armat ii sunt echivalente:
(i) lim
xx
0
f (x) = +;
(ii) pentru orice numar real > 0 (oricat de mare ar ) exista un numar real () > 0 astfel ncat
x D x
0
, [x x
0
[ < () f (x) > .
3.2.9 Propozit ie. Fie funct ia f : D R R si x
0
= + punct de acumulare pentru D.
Urmatoarele armat ii sunt echivalente:
(i) lim
x+
f (x) = ;
(ii) pentru orice numar real < 0 (oricat de mic ar ) exista un numar real () > 0 astfel ncat
x D, x > () f (x) < .
3.2.10 Propozit ie. Fie funct ia f : D R R, x
0
= + punct de acumulare pentru D si l R.
Urmatoarele armat ii sunt echivalente:
(i) lim
x+
f (x) = l;
(ii) pentru orice numar real > 0 (oricat de mic ar ) exista un numar real () > 0 astfel ncat
x D, x > () [ f (x) l[ < .
3.2.11 Propozit ie. Fie funct ia f : D R R si x
0
= + punct de acumulare pentru D.
Urmatoarele armat ii sunt echivalente:
(i) lim
x+
f (x) = +;
(ii) pentru orice numar real > 0 (oricat de mare ar ) exista un numar real () > 0 astfel ncat
x D, x > () f (x) > .
Urmatoarele teoreme pun n evident a cateva proprietat i ale funct iei f : D R R care rezulta
direct din existent a limitei funct iei ntr-un punct.
3.2.12 Teorema. Fie D R si x
0
R , + un punct de acumulare pentru D. Daca
funct ia f : D R are n punctul x
0
limita l si c, d R , + , c < l < d, atunci exist a o
vecinatate U a punctului x
0
astfel ncat x D U x
0
c < f (x) < d.
Demonstrat ie. Consideram intervalul V = (c, d) . Din c < l < d rezulta ca V este o vecinatate a
lui l. Cum f are n punctul x
0
limita l, pe baza Denit iei 3.2.1 (i) exista o vecinatate U a lui x
0
astfel
ncat x D U x
0
f (x) (c, d) .
3.2.13 Teorema. Fie D R si x
0
R , + un punct de acumulare pentru D. Daca
funct ia f : D R are n punctul x
0
limita nenula l, atunci exista o vecinatate U a punctului x
0
astfel
ncat
x D U x
0
l f (x) > 0.
3.2 Limita unei funct ii f : D R R ntr-un punct 135
Demonstrat ie. Daca l (, 0) aplicam teorema precedenta cu c = , d = 0; respectiv cu
c = 0, d = + daca l (0, +) . Daca l = sau l = + atunci concluzia rezula direct din Denit ia
3.2.1 (i).
3.2.14 Denit ie. Spunem ca funct ia f : D R R este marginita pe mult imea nevida
D
0
D, daca exista un interval (c, d) R cu proprietatea ca f (D
0
) (c, d) , unde mult imea f (D
0
) =
f (x)[ x D
0
este imaginea mult imii D
0
prin funct ia f.
3.2.15 Teorema. Fie D R si x
0
R , + un punct de acumulare pentru D. Daca
funct ia f : D R are limita l R n punctul x
0
, atunci exista o vecinatate U a punctului x
0
astfel
ncat funct ia f este marginita pe D U.
Demonstrat ie. Fixam arbitrar c, d R astfel ca c < l < d si aplicam Teorema 3.2.12.
Urmatoarea teorema se refera la existent a limitei ntr-un punct a funct iei suma, produs, cat, respectiv
a modulului funct iei.
3.2.16 Teorema. Fie D R si x
0
R, + un punct de acumulare pentru D. Fie funct iile
f, g : D R. Daca exista lim
xx
0
f (x) = l
1
R si lim
xx
0
g (x) = l
2
R, atunci exista limitele
i) lim
xx
0
(f +g) (x) = l
1
+l
2
;
ii) lim
xx
0
(f g) (x) = l
1
l
2
;
iii) lim
xx
0
f
g
(x) =
l
1
l
2
, daca l
2
,= 0 si g (x) ,= 0, x D x
0
;
iv) lim
xx
0
[f[ (x) = [l
1
[.
Demonstrat ie. Utilizam denit ia cu siruri a limitei ntr-un punct a unei funct ii, Denit ia 3.2.1
(ii). Fie un sir (x
n
)
n1
de numere reale din D x
0
cu limita x
0
. Din lim
xx
0
f (x) = l
1
si lim
xx
0
g (x) = l
2
,
pe baza Denit iei 3.2.1 (ii), rezulta lim
n
f (x
n
) = l
1
, respectiv lim
n
g (x
n
) = l
2
. Utilizand proprietat i ale
sirurilor de numere reale obt inem
i) lim
n
(f +g) (x
n
) = lim
n
(f (x
n
) +g (x
n
)) = l
1
+l
2
;
ii) lim
n
(f g) (x
n
) = lim
n
(f (x
n
) g (x
n
)) = l
1
l
2
;
iii) lim
n
f
g
(x
n
) = lim
n
f (x
n
)
g (x
n
)
=
l
1
l
2
;
iv) lim
n
[f[ (x
n
) = lim
n
[f (x
n
)[ = [l
1
[;
de unde, cu privire la funct iile f + g, f g,
f
g
respectiv [f[ , pe baza Denit iei 3.2.1 (ii), urmeaza
concluziile din teorema.

In continuare vom considera not iuni de limite laterale.


3.2.17 Denit ie. Fie D R o mult ime nevida si f : D R o funct ie.
(i) Fie x
0
R un punct de acumulare pentru mult imea D (, x
0
) . Fie l R , + .
Spunem ca funct ia f are limita la stanga n punctul x
0
egala cu l, si notam lim
xx
0
x<x
0
f (x) = l sau
f (x
0
0) = l, daca pentru orice vecinatate V a lui l exista o vecinatate U a lui x
0
cu proprietatea
f (D (, x
0
) U) V, ceea ce revine la
x D (, x
0
) U f (x) V .
(ii) Fie x
0
R un punct de acumulare pentru mult imea D (x
0
, +). Fie l R , + .
Spunem ca funct ia f are limita la dreapta n punctul x
0
egala cu l, si notam lim
xx
0
x>x
0
f (x) = l sau
f (x
0
+ 0) = l, daca pentru orice vecinatate V a lui l exista o vecinatate U a lui x
0
cu proprietatea
f (D (x
0
, +) U) V, ceea ce revine la
x D (x
0
, +) U f (x) V .
3.2.18 Observat ie. Limita la stanga respectiv limita la dreapta se numesc limite laterale.
3.2.19 Observat ie. De fapt lim
xx
0
x<x
0
f (x) este limita n punctul x
0
, n sensul Denit iei 3.2.1 (i),
a restrict iei funct iei f la mult imea D (, x
0
), iar lim
xx
0
x>x
0
f (x) este limita n punctul x
0
a restrict iei
136 3. Funct ii f : D R R continue
funct iei f la mult imea D (x
0
, +) , n sensul Denit iei 3.2.1 (i). Din acest motiv toate rezultatele
legate de not iunea de limita a unei funct ii ntr-un punct se transpun pentru limite laterale. Ca exemplu
ment ionam unicitatea limitei laterale: dac a f are limita la stanga n punctul x
0
, atunci f are o singura
limita la stanga n punctul x
0
; daca f are limita la dreapta n punctul x
0
, atunci f are o singura limita
la dreapta n x
0
.
Urmatoarea teorema exprima legatura dintre limita funct iei ntr-un punct si limitele laterale n acel
punct ale funct iei.
3.2.20 Teorema. Fie D R, f : D R o funct ie, x
0
R un punct de acumulare pentru mult imea
D (, x
0
) si n acelasi timp pentru mult imea D (x
0
, +) .
i) Daca funct ia f are limita n punctul x
0
, atunci funct ia f are limite laterale n punctul x
0
si
lim
xx
0
x<x
0
f (x) = lim
xx
0
x>x
0
f (x) = lim
xx
0
f (x) .
ii) Daca funct ia f n punctul x
0
are limite laterale egale lim
xx
0
x<x
0
f (x) = lim
xx
0
x>x
0
f (x) , atunci funct ia f
are limita n punctul x
0
si lim
xx
0
x<x
0
f (x) = lim
xx
0
x>x
0
f (x) = lim
xx
0
f (x) .
Demonstrat ie. Utilizam denit ia cu vecinatat i a limitei funct iei ntr-un punct adica Denit ia 3.2.1.
(i) si denit ia limitelor laterale.
O clasa importanta de funct ii este cea a funct iilor monotone. Reamintim not iunea de funct ie
monotona.
3.2.21. Denit ie. O funct ie f : D R R se numeste monotona, daca are una din urmatoarele
proprietat i:
i) x
1
, x
2
D, x
1
< x
2
f (x
1
) f (x
2
) ; n acest caz spunem ca funct ia f este crescatoare;
ii) x
1
, x
2
D, x
1
< x
2
f (x
1
) f (x
2
) ; n acest caz spunem ca funct ia f este descrescatoare.
Funct ia f : D R R se numeste strict monotona, daca are una din urmatoarele proprietat i:
iii) x
1
, x
2
D, x
1
< x
2
f (x
1
) < f (x
2
) ; n acest caz spunem ca funct ia f este strict crescatoare;
iv) x
1
, x
2
D, x
1
< x
2
f (x
1
) > f (x
2
) ; n acest caz spunem ca funct ia f este strict descrescatoare.
Urmatoarele teoreme se refera la existent a limitelor laterale n cazul funct iilor monotone pe un
interval.
3.2.22 Teorema. Fie a, b R , + , a < b si f : (a, b) R o funct ie crescatoare. Pentru
ecare c (a, b) are loc:
i) Exista f(c 0), f (c + 0) si f(c 0) R, f (c + 0) R;
ii) f(c 0) = supf (x)[ a < x < c , f (c + 0) = inf f (x)[ c < x < b;
iii) f(c 0) f (c) f (c + 0);
iv) Daca a < c < d < b atunci f(c + 0) f (d 0) .
Demonstrat ie. Deoarece funct ia f este crescatoare, rezulta ca mult imea f (x)[ a < x < c este
majorata (spre exemplu de f(c)) si mult imea f (x)[ c < x < b este minorata (spre exemplu de f (c)).
Mult imea f (x)[ a < x < c ind submult ime nevida a lui R, si majorata, rezulta ca admite supre-
mum (cel mai mic majorant) n R, = supf (x)[ a < x < c R. Mult imea f (x)[ c < x < b ind
submult ime nevida a lui R, si minorata, rezulta ca admite inmum (cel mai mare minorant) n R,
= inf f (x)[ c < x < b R. Are loc < f (c) < +.
Demonstram ca lim
xc
x<c
f (x) = . Din = supf (x)[ a < x < c rezulta ca f (x) , x (a, c) si
pentru orice > 0 exista u (a, c) cu proprietatea f (u) > . T inand cont si de monotonia funct iei f
pentru ecare x (u, c) avem < f (u) f (x) < +. Prin urmare pentru orice > 0 exista
() > 0 astfel ncat
x (a, c) , [x c[ < () [f (x) [ <
si anume () = c u. Asadar lim
xc
x<c
f (x) = .
Cu un rat ionament asemanator obt inem lim
xc
x>c
f (x) = .

In acest moment avem demonstrate egalitat ile


3.2 Limita unei funct ii f : D R R ntr-un punct 137
f(c 0) = supf (x)[ a < x < c, f (c + 0) = inf f (x)[ c < x < b
si concluziile i), ii) ale teoremei.

In continuare pe baza monotoniei funct iei f rezulta inegalitat ile din iii).
Pentru a demonstra iv) consideram x astfel ncat c < x < d. Rezulta ca f (c + 0) f (x) f (d 0)
si deci f (c + 0) f (d 0) .

In mod asemanator se demonstreaza urmatoarea teorema.


3.2.23 Teorema. Fie a, b R , + , a < b si f : (a, b) R o funct ie descrescatoare.
Pentru ecare c (a, b) are loc:
i) Exista f(c 0), f (c + 0) si f(c 0) R, f (c + 0) R;
ii) f(c 0) = inf f (x)[ a < x < c , f (c + 0) = supf (x)[ c < x < b;
iii) f(c 0) f (c) f (c + 0);
iv) Daca a < c < d < b atunci f(c + 0) f (d 0) .
3.2.24. Observat ie. i) Daca a, b R, a < b si funct ia f : [a, b) R este monotona, atunci exista
f(a + 0).
ii) Daca a, b R, a < b si funct ia f : (a, b] R este monotona, atunci exista f (b 0) .
Urmatoarea teorema cont ine o tehnic a frecvent utilizata la calculul limitelor de funct ii.
3.2.25 Teorema (Criteriul majorarii) Fie f, g : D R R doua funct ii, x
0
R , + un
punct de acumulare pentru D si l R. Daca
i) exista o vecinatate U a lui x
0
astfel ncat [f (x) l[ g (x) , x D U x
0
;
ii) lim
xx
0
g (x) = 0,
atunci lim
xx
0
f (x) = l.
Demonstrat ie. Fie (x
n
)
n1
un sir arbitrar de numere reale din DU x
0
, cu lim
n
x
n
= x
0
. Pe
baza ipotezei i) rezulta [f (x
n
) l[ g (x
n
) , n N

. Din ii) lim


xx
0
g (x) = 0 si Denit ia 3.2.1 (ii) rezulta
ca lim
n
g (x
n
) = 0. Deducem ca sirul de numere reale (f (x
n
))
n1
are limta l; lim
n
f (x
n
) = l. Utilizand
din nou Denit ia 3.2.1 (ii) rezulta ca lim
xx
0
f (x) = l.
3.2.26 Exemplu. Pentru f : R 0 R, f (x) =
1
x
sinx, x
0
= +, l = 0 avem
i) [f (x) l[ =

1
x
sinx 0

1
x

[sinx[
1
x
1 =
1
x
, x (0, +)
ii) lim
x+
1
x
= 0
asadar lim
x+
1
x
sinx = 0.
Urmatoarea teorema se refera la doua funct ii comparate pe o vecinatate a punctului x
0
printr-o
inegalitate.
3.2.27 Teorema (Trecerea la limita ntr-o inegalitate) Fie f, g : D R R doua funct ii si
x
0
R , + un punct de acumulare pentru D. Daca
i) funct iile f si g au limita n punctul x
0
,
ii) exista o vecinatate U a lui x
0
astfel ncat f (x) g (x) , x D U x
0
,
atunci lim
xx
0
f (x) lim
xx
0
g (x) .
Demonstrat ie. Fie (x
n
)
n1
un sir de numere reale din D U x
0
cu lim
n
x
n
= x
0
. Din ipoteza
i) si Denit ia 3.2.1. (ii) a limitei funct iei ntr-un punct rezulta ca lim
n
f (x
n
) = lim
xx
0
f (x), respectiv
lim
n
g (x
n
) = lim
xx
0
g (x) . Conform ipotezei ii) are loc f (x
n
) g (x
n
) , n N

, de unde, cu privire la
sirurile de numere reale (f (x
n
))
n1
, (g (x
n
))
n1
rezulta ca lim
n
f (x
n
) lim
n
g (x
n
) . Asadar
lim
xx
0
f (x) lim
xx
0
g (x) .
3.2.28 Observat ie. Atragem atent ia asupra faptului ca din ipoteza i) si comparat ia f (x) < g (x),
x D U x
0
, nu rezulta inegalitatea stricta lim
xx
0
f (x) < lim
xx
0
g (x) .
3.2.29 Exemplu. Fie f, g : R R, f(x) =
1
1 +x
2
, g(x) =
1
1 +x
4
. Avem f(x) < g(x) pentru orice
x (1, 1) 0, iar lim
x0
f (x) = lim
x0
g (x) = 1.
138 3. Funct ii f : D R R continue
3.3 Funct ii f : D R R continue ntr-un punct

Incepem cu denit ia not iunii de continuitate a unei funct ii ntr-un punct bazata pe not iunea de
vecinatate.
3.3.1 Denit ie. Funct ia f : D R R se numeste continua n punctul x
0
D daca pentru
ecare vecinatate V a punctului f (x
0
) exista o vecinatate U a punctului x
0
cu proprietatea f (D U)
V, ceea ce revine la
x D U f (x) V .
Daca funct ia f nu este continua n punctul x
0
, atunci spunem ca f este discontinua n x
0
.
3.3.2 Observat ie. Subliniem faptul ca not iunea de continuitate (respectiv discontinuitate) a funct iei
f : D R Rntr-un punct x
0
are sens numai pentru puncte x
0
care apart in mult imii de denit ie D a
funct iei f.
Urmatoarea teorema ne ofera caracterizari ale funct iilor continue ntr-un punct.
3.3.3 Teorema. Fie funct ia f : D R R si punctul x
0
D. Urmatoarele armat ii sunt
echivalente:
(i) Funct ia f este continua n punctul x
0
;
(ii) Pentru orice numar real > 0 (oricat de mic ar ) exista un numar real () > 0 astfel ncat
x D, [x x
0
[ < () [f (x) f (x
0
)[ < ;
(iii) Pentru orice sir de numere reale (x
n
)
n1
format cu elemente din D, convergent catre x
0
, sirul
valorilor corespunzatoare (f (x
n
))
n1
este convergent catre f (x
0
) .
Demonstrat ie. Presupunem (i) funct ia f continua n punctul x
0
si demonstram (ii).
Fie > 0 arbitrar. Consideram intervalul deschis V = (f (x
0
) , f (x
0
) +) . Intervalul V ind o
vecinatatea a punctului f (x
0
) si funct ia f ind continua n punctul x
0
, pe baza Denit iei 3.3.1. exista o
vecinatatea U a punctului x
0
astfel ncat
x D U f (x) V.
Deoarece U este o vecinatate a punctului x
0
, exista () > 0 astfel ca (x
0
() , x
0
+ ()) U.
Daca [x x
0
[ < () atunci rezulta () < x x
0
< () x
0
() < x < x
0
+ () x
(x
0
() , x
0
+ ()) x U.
Astfel x D, [x x
0
[ < () x D, x U x DU f (x) V = (f (x
0
) , f (x
0
) +)
f (x
0
) < f (x) < f (x
0
) + < f (x) f (x
0
) < [f (x) f (x
0
)[ < .
Asadar pentru > 0 luat arbitrar, exista () > 0 cu proprietatea ca
x D, [x x
0
[ < () [f (x) f (x
0
)[ <
si deci (ii) are loc.
Presupunem (ii) si demonstram (iii). Fie (x
n
)
n1
un sir arbitrar de numere reale format cu elemente
din D, convergent catre x
0
. Fie > 0 arbitrar. Conform (ii) exista () > 0 astfel ncat
x D, [x x
0
[ < () [f (x) f (x
0
)[ < .
Deoarece sirul (x
n
)
n1
are limita x
0
, exista n

N astfel ca [x
n
x
0
[ < (), n > n

. Pe baza implicat iei


precedente rezulta ca [f (x
n
) f (x
0
)[ < , n > n

, ceea ce nseamna ca sirul (f (x


n
))
n1
este convergent
catre f (x
0
) . Deci (iii) are loc.
Demonstram ca (iii) implica (i). Rat ionam prin reducere la absurd. Consideram (iii) adevarata.
Presupunem ca (i) nu are loc, deci ca funct ia f nu este continua n punctul x
0
. Atunci exista o vecinatate
V a punctului f (x
0
) cu proprietatea ca pentru orice vecinatate U a punctului x
0
exista x D U cu
f (x) / V .

In particular, pentru ecare n N

intervalul U
n
=
_
x
0

1
n
, x
0
+
1
n
_
ind o vecinatate a lui
x
0
, exista x
n
D U
n
cu f (x
n
) / V. Din x
n

_
x
0

1
n
, x
0
+
1
n
_
, n N

rezulta ca sirul de numere


reale (x
n
)
n1
este convergent catre x
0
, iar din f (x
n
) / V, n N

rezulta ca sirul de numere reale


(f (x
n
))
n1
nu converge catre f (x
0
) . Contradict ie cu (iii). Presupunerea noastra duce la contradict ie,
deci este incorecta; rezulta ca (i) are loc, adica funct ia f este continua n punctul x
0
.
Urmatoarele doua teoreme se refera la doua cazuri particulare importante ale Denit iei 3.3.1.
3.3 Funct ii f : D R R continue ntr-un punct 139
3.3.4 Teorema. Fie f : D R R o funct ie si x
0
D. Daca x
0
este punct izolat al lui D, atunci
funct ia f este continua n x
0
.
Demonstrat ie. Fie x
0
D punct izolat al lui D.

Inseamna ca exista o vecinatate W a punctului x


0
astfel ncat DW = x
0
. Deducem ca pentru orice vecinatate V a punctului f (x
0
) exista o vecinatate
U a lui x
0
, anume U = W, astfel ncat f (D U) V, si deci, pe baza Denit iei 3.3.1, f este continua
n punctul x
0
. Am t inut cont de D U = x
0
f (D U) = f (x
0
) si de incluziunea f (x
0
) V
care are loc deoarece V este o vecinatate a lui f (x
0
) si deci f (x
0
) V.
3.3.5 Exemplu. Fie dat (x
n
) un sir de numere reale.

Inseamna ca avem denita funct ia f : N
R, f (n) = x
n
, n N. Consideram un element n N. Deoarece n este punct izolat al mult imii de denit ie
N, funct ia f este continua n punctul n.
3.3.6 Teorema. Fie f : D R R o funct ie si x
0
D. Daca x
0
este punct de acumulare pentru
D, atunci urmatoarele armat ii sunt echivalente
(i) Funct ia f este continua n punctul x
0
.
(ii) Exista limita lim
xx
0
f (x) si lim
xx
0
f (x) = f (x
0
) .
Demonstrat ie. Demonstram ca (i) implica (ii). Daca f este continua n x
0
, atunci conform
Denit iei 3.3.1, pentru orice vecinatate V a lui f (x
0
) exista o vecinatate U a lui x
0
astfel ncat f (D U)
V. Atunci f (D U x
0
) V are loc, ceea ce, pe baza Denit iei 3.2.1 (i), nseamna ca lim
xx
0
f (x) =
f (x
0
) .
Demonstram ca (ii) implica (i). Din lim
xx
0
f (x) = f (x
0
) si Denit ia 3.2.1 (i) rezulta ca pentru orice
vecinatate V a lui f (x
0
) exista o vecinatate U a lui x
0
astfel ncat f (D U x
0
) V. Dar f (x
0
) V
deoarece V este o vecinatate lui f (x
0
) , si deci f (D U) V. Pe baza Denit iei 3.3.1 funct ia f este
continua n x
0
.
3.3.7 Observat ie. Teoremele 3.3.4 si 3.3.6 arata ca funct ia f : D R R este continua n punctul
x
0
D daca si numai daca are loc una din urmatoarele situat ii
(i) Punctul x
0
D este punct izolat al mult imii D.
(ii) Punctul x
0
D este punct de acumulare pentru mult imea D si lim
xx
0
f (x) = f (x
0
) .
Urmatoarele doua teoreme evident iaza proprietat i remarcabile ale funct iilor continue ntr-un punct.
3.3.8 Teorema. Daca funct ia f : D R R este continua n punctul x
0
D si daca f (x
0
) ,= 0,
atunci exista o vecinatate U a punctului x
0
cu proprietatea ca pentru orice x DU valorile f (x) sunt
de acelasi semn cu valoarea f (x
0
) .
Demonstrat ie. Din f (x
0
) ,= 0 rezulta ca [f (x
0
)[ > 0. Consideram intervalul
V =
_
f (x
0
)
[f (x
0
)[
2
, f (x
0
) +
[f (x
0
)[
2
_
.
Acesta ind o vecinatatae a punctului f (x
0
) , si deoarece funct ia f este continua n punctul x
0
, pe baza
Denit iei 3.3.1 deducem existent a unei vecinatat i U a punctului x
0
astfel ncat
x U D f (x)
_
f (x
0
)
]f(x
0
)]
2
, f (x
0
) +
]f(x
0
)]
2
_
.
Daca f (x
0
) < 0, atunci
_
f (x
0
)
[f (x
0
)[
2
, f (x
0
) +
[f (x
0
)[
2
_
(, 0) si deci pentru x DU rezulta
ca f (x) < 0. Daca f (x
0
) > 0, atunci
_
f (x
0
)
[f (x
0
)[
2
, f (x
0
) +
[f (x
0
)[
2
_
(0, +) si deci pentru
x D U rezulta ca f (x) > 0.
3.3.9 Teorema. Daca funct ia f : D R R este continua n punctul x
0
D, atunci exista o
vecinatate U a punctului x
0
cu proprietatea ca f este marginita pe D U.
Demonstrat ie. Consideram intervalul (f (x
0
) 1, f (x
0
) + 1) . Acesta este o vecinatate a punctului
f (x
0
) . Deoarece funct ia f este continua n punctul x
0
, pe baza Denit iei 3.3.1 exista o vecinatate U a
punctului x
0
astfel ncat
f (D U ) (f (x
0
) 1, f (x
0
) + 1),
140 3. Funct ii f : D R R continue
de unde rezulta ca funct ia f este marginit a pe D U.

In continuare consideram not iuni de continuitate laterala.


3.3.10 Denit ie. Fie f : D R R o funct ie.
i) Daca x
0
D este un punct de acumulare pentru mult imea D (, x
0
) si lim
xx
0
x<x
0
f (x) = f (x
0
)
atunci spunem ca funct ia f n punctul x
0
este continua la stanga.
ii) Daca x
0
D este un punct de acumulare pentru mult imea D (x
0
, +) si lim
xx
0
x>x
0
f (x) = f (x
0
)
atunci spunem ca funct ia f n punctul x
0
este continua la dreapta.
3.3.11 Teorema. Fie f : D R R o funct ie. Fie x
0
D punct de acumulare pentru mult imea
D (, x
0
) si pentru mult imea D (x
0
, +) . Funct ia f este continua n punctul x
0
daca si numai
daca lim
xx
0
x<x
0
f (x) = f (x
0
) = lim
xx
0
x>x
0
f (x) .
Demonstrat ie. Aplicam Teorema 3.3.6 de caracterizare a continuitat ii funct iei ntr-un punct de
acumulare cu ajutorul limitei funct iei si Teorema 3.2.20 care arata legatura dintre limita ntr-un punct a
funct iei si limitele laterale n acel punct ale funct iei.
3.3.12 Observat ie. Fie a, b R, a < b si o funct ie f : [a, b] R.

In punctul x
0
= a continuitatea
funct iei f (n sensul Denit iei 3.3.1) revine la continuitatea la dreapta n punctul x
0
= a a funct iei f.

In punctul x
0
= b continuitatea funct iei f (n sensul Denit iei 3.3.1) revine la continuitatea la stanga n
x
0
= b a funct iei f.
Urmatoarele doua teoreme se refera la continuitatea ntr-un punct a funct iei suma, produs, cat, a
modulului funct iei, respectiv a funct iei compuse. Teoremele arata ca proprietatea de continuitate ntr-un
punct se transmite prin operat iile precizate.
3.3.13 Teorema.
i) Daca funct iile f, g : D R R sunt continue n punctul x
0
D, atunci funct ia suma f +g este
continua n punctul x
0
.
ii) Daca funct iile f, g : D R R sunt continue n punctul x
0
D, atunci funct ia produs f g este
continua n punctul x
0
.
iii) Daca c R si daca funct ia f : D R R este continua n punctul x
0
D, atunci funct ia c f
este continua n punctul x
0
.
iv) Dac a funct iile f, g : D R R sunt continue n punctul x
0
D si daca g (x
0
) ,= 0, atunci
exista o vecinatate U a punctului x
0
cu proprietatea ca restrict ia funct iei cat
f
g
la DU este continua
n punctul x
0
.
v) Daca funct ia f : D R R este continua n punctul x
0
D, atunci funct ia [f[ este continua
n punctul x
0
.
Demonstrat ie. Daca x
0
este punct izolat al mult imii D, atunci toate funct iile ment ionate sunt
continue n x
0
pe baza Teoremei 3.3.4.
Daca x
0
D este punct de acumulare pentru mult imea D, atunci utilizam Teorema 3.3.3 (iii) si
proprietat ile sirurilor de numere reale. Fie (x
n
)
n1
un sir arbitrar de numere reale din D convergent
catre x
0
.
Din continuitatea funct iilor f si g n punctul x
0
pe baza Teoremei 3.3.3 deducem ca sirul de numere
reale (f (x
n
))
n1
este convergent catre f (x
0
) , respectiv sirul de numere reale (g (x
n
))
n1
este convergent
catre g (x
0
) .
i) Cum sirul de numere reale ((f +g) (x
n
))
n1
= (f (x
n
) +g (x
n
))
n1
este convergent catre f (x
0
) +
g (x
0
) = (f +g) (x
0
) , pe baza Teoremei 3.3.3 funct ia f +g este continua n punctul x
0
.
ii) Cum sirul de numere reale ((f g) (x
n
))
n1
= (f (x
n
) g (x
n
))
n1
este convergent catre f (x
0
)
g (x
0
) = (f g) (x
0
) , pe baza Teoremei 3.3.3 funct ia f g este continua n punctul x
0
.
iii) Din punctul ii), n cazul cand funct ia g este funct ia constanta g (x) = c, x D, obt inem iii).
iv) Daca funct ia g este continua n punctul x
0
D si g (x
0
) ,= 0, utilizand Teorema 3.2.13 deducem
ca exista o vecinatate U a punctului x
0
cu proprietatea ca x D U g (x) ,= 0 (de fapt g (x) are
semnul valorii g (x
0
)).
Consideram funct ia cat
f
g
restrict ionata la mult imea D U. Fie acum (x
n
)
n1
un sir arbitrar
de numere reale din D U convergent catre x
0
. Deoarece funct iile f si g sunt continue n punctul
3.4 Funct ii f : D R R continue pe o mult ime 141
x
0
D U, pe baza Teoremei 3.3.3 sirul de numere reale (f (x
n
))
n1
este convergent catre f (x
0
) , res-
pectiv sirul de numere reale nenule (g (x
n
))
n1
este convergent catre g (x
0
) . Rezulta ca sirul de numere
reale
_
f
g
(x
n
)
_
n1
=
_
f (x
n
)
g (x
n
)
_
n1
este convergent catre
f (x
0
)
g (x
0
)
=
f
g
(x
0
) , asadar pe baza Teoremei
3.3.3 funct ia
f
g
: D U R este continua n punctul x
0
.
v) Cum sirul de numere reale ([f[ (x
n
))
n1
= ([f (x
n
)[)
n1
este convergent catre [f (x
0
)[ , pe baza
Teoremei 3.3.3 funct ia [f[ este continua n punctul x
0
.
3.3.14 Teorema. Daca funct ia f : D R R este continua n punctul x
0
D si daca funct ia
g : f (D) R este continua n punctul y
0
= f (x
0
) , atunci funct ia compusa g f : D R este continua
n punctul x
0
.
Demonstrat ie. Daca x
0
este punct izolat al mult imii D, atunci pe baza Teoremei 3.3.4 funct ia g f
este continua n x
0
. Daca x
0
D este punct de acumulare pentru D atunci utilizam Teorema 3.3.3. Fie
(x
n
)
n1
un sir arbitrar de numere reale din D convergent catre x
0
. Deoarece funct ia f este continua n
punctul x
0
, pe baza Teoremei 3.3.3 deducem ca sirul (f (x
n
))
n1
este convergent catre f (x
0
) . Deoarece
sirul (y
n
)
n1
, unde y
n
= f (x
n
) , n N

, este format cu numere reale din mult imea de denit ie f (D) a


funct iei g, si dupa cum s-a vazut acest sir este convergent catre f (x
0
) = y
0
f (D) , din continuitatea
funct iei g n punctul y
0
, cu Teorema 3.3.3, deducem ca sirul (g (y
n
))
n1
este convergent catre g (y
0
) . Dar
g (y
n
) = g (f (x
n
)) = (g f) (x
n
) , n N

si g (y
0
) = g (f (x
0
)) = (g f) (x
0
) , deci sirul ((g f) (x
n
))
n1
este convergent catre (g f) (x
0
) . Pe baza Teoremei 3.3.3 rezulta ca funct ia g f este continua n punctul
x
0
.
3.4 Funct ii f : D R R continue pe o mult ime
3.4.1 Denit ie. Spunem ca funct ia f : D R R este continua pe mult imea nevida D
0
D,
daca funct ia f este continua n ecare punct x
0
D
0
.
Daca f este continua pe mult imea de denit ie D, atunci spunem ca funct ia f este continua.
3.4.2 Exemplu. Fie (x
n
) un sir dat de numere reale. Funct ia f : N R, f (n) = x
n
, n N este
continua pe N.

Intr-adevar, ecare n N este punct izolat al mult imii N, deci n ecare punct n N funct ia f este
continua. Am utilizat Teorema 3.3.4.
3.4.3 Teorema. Funct iile c, [x[ , x

, a
x
, log
a
x, sinx, cos x, tgx, ctgx, arcsinx, arccos x, arctgx,
arcctgx sunt continue pe mult imea maxima de denit ie.
Demonstrat ie. Se utilizeaza (iii) din Teorema 3.3.3 pentru ecare funct ie n parte.
3.4.4 Teorema. Fie funct iile f, g : I R unde I R este un interval. Consideram funct iile
h, H : I R, h = minf, g , H = max f, g. Daca funct iile f, g sunt continue pe I, atunci funct iile
h, H sunt continue pe I.
Demonstrat ie. Utilizam exprimarile h =
f +g [f g[
2
, H =
f +g +[f g[
2
.
Funct iile f, g ind continue pe I, ele sunt continue n ecare x
0
I; aplicand Teorema 3.3.13, pentru
ecare x
0
I n parte, rezulta ca funct iile
f +g [f g[
2
= h, respectiv
f +g +[f g[
2
= H sunt
continue n ecare x
0
I, si deci sunt continue pe I.
3.4.5 Observat ie. Pe baza teoremei precedente, prin induct ie matematica, din continuitatea
funct iilor f
1
, f
2
, . . ., f
n
pe I se obt ine continuitatea funct iilor h = minf
1
, f
2
, . . . , f
n
,
H = max f
1
, f
2
, . . . , f
n
pe I.
Urmatoarea teorema stabileste marginirea funct iilor continue pe un interval nchis.
3.4.6 Teorema (Teorema lui Weierstrass) Daca funct ia f : [a, b] R este continua pe [a, b] , unde
a, b R, a < b, atunci
i) funct ia f este marginita pe [a, b];
ii) funct ia f si atinge inmumul si supremumul pe [a, b], adica exista x

[a, b] astfel ncat


f (x

) = inf f (x)[ x [a, b] si exista x

[a, b] astfel ncat f (x

) = supf (x)[ x [a, b] .


Demonstrat ie. i) Vom rat iona prin reducere la absurd. Consideram funct ia f continua pe mult imea
[a, b] . Presupunem ca funct ia f nu este marginita pe [a, b] . Atunci pentru orice interval (n, n) R,
142 3. Funct ii f : D R R continue
unde n N

, exista x
n
[a, b] cu proprietatea ca f (x
n
) / (n, n) . Din [f (x
n
)[ n, n N

rezulta ca
sirul de numere reale ([f (x
n
)[)
n1
are limita +.
Deoarece elementele sirului (x
n
)
n1
sunt numere reale din intervalul [a, b] , sirul (x
n
)
n1
este marginit
si deci are un subsir (x
n
k
)
k1
convergent. Notam cu x
0
limita acestui subsir. Din x
n
k
[a, b] , k N

rezulta ca x
0
[a, b] . Funct ia f ind continua pe [a, b] , ea este continua n ecare punct al interval-
ului, deci si n punctul x
0
. Pe baza Teoremei 3.3.3 rezulta ca sirul de numere reale (f (x
n
k
))
k1
este
convergent catre f (x
0
) . Prin urmare sirul valorilor absolute ([f (x
n
k
)[)
k1
este convergent catre [f (x
0
)[ .
Contradict ie. Rezulta ca funct ia f este m arginita pe [a, b] .
ii) Consideram funct ia f continua pe [a, b] . Cu i) deducem ca f este marginita pe [a, b], deci exista
(c, d) R astfel ncat f ([a, b]) = f (x)[ x [a, b] (c, d) . Evident mult imea f (x)[ x [a, b] este
nevida deoarece a < b. Mult imea f (x)[ x [a, b] ind submult ime nevida si marginita a lui R, are in-
mum (cel mai mare minorant) si supremum (cel mai mic majorant) n R: exista
m = inf f (x)[ x [a, b] R si exista M = supf (x)[ x [a, b] R.
Aratam ca exista x

[a, b] astfel ncat f (x

) = m. Rat ionam prin reducere la absurd. Presupunem


ca nu exista x

[a, b] astfel ncat f (x

) = m. Atunci, t inand cont de proprietatea inmumului, are loc


f (x) > m, x [a, b] . Funct ia g : [a, b] R, g (x) =
1
f (x) m
este pozitiva, pe baza Teoremei 3.3.13
este continua pe [a, b] , si utilizand i) rezulta ca g este marginita pe [a, b] . Prin urmare exista > 0 astfel
ncat 0 <
1
f (x) m
< , x [a, b] . Deducem ca f (x) > m +
1

, x [a, b] , inegalitate ce contrazice


calitatea lui m = inf f (x)[ x [a, b] de a cel mai mare minorant al mult imii f (x)[ x [a, b] .
Rezulta ca exista x

[a, b] astfel ca f (x

) = m = inf f (x)[ x [a, b] .

In mod asemanator se arata ca exista x

[a, b] cu proprietatea ca f (x

) = supf (x)[ x [a, b] .


3.4.7 Observat ie.

In ipotezele teoremei elementul x

[a, b] are proprietatea ca f (x

) f (x),
pentru orice x [a, b], iar x

[a, b] are proprietatea ca f (x

) f (x) , x [a, b] .
Urmatoarea teorema fundamenteaza o importanta metoda de aproximare a solut iilor ecuat iilor,
metoda numita metoda njumatat irii intervalului.
3.4.8 Teorema (Teorema lui Bolzano-Cauchy) Fie a, b R, a < b si f : [a, b] R o funct ie
continua. Daca f (a) f (b) < 0, atunci exista cel put in un punct c (a, b) astfel ncat f (c) = 0.
Demonstrat ie. Numerele f (a) , f (b) au semnele contrare. Construim sirurile de numere reale
(a
n
)
n1
, (b
n
)
n1
, (c
n
)
n1
astfel: a
1
= a, b
1
= b si c
1
=
a
1
+b
1
2
la mijlocul intervalului [a
1
, b
1
] . Deoarece
f (a
1
) si f (b
1
) au semnele contrare, cel put in una din urmatoarele doua situat ii are loc: f (a
1
) f (c
1
) 0,
f (c
1
) f (b
1
) 0; daca are loc prima, atunci denim a
2
= a
1
, b
2
= c
1
, iar n caz contrar denim a
2
= c
1
,
b
2
= b
1
, dupa care consideram c
2
=
a
2
+b
2
2
la mijlocul intervalului [a
2
, b
2
] . Deoarece f (a
2
)f (b
2
) 0,
cel put in una din urmatoarele doua situat ii are loc: f (a
2
)f (c
2
) 0, f (c
2
)f (b
2
) 0; daca are loc
prima, atunci denim a
3
= a
2
, b
3
= c
2
, iar n caz contrar denim a
3
= c
2
, b
3
= b
2
; dupa care consideram
c
3
=
a
3
+b
3
2
la mijlocul intervalului [a
3
, b
3
] . Continuam procedura tot la fel.
Sirurile (a
n
)
n1
, (b
n
)
n1
astfel denite au urmatoarele proprietat i:
i) a a
n
a
n+1
b
n+1
b
n
b, n N

ii) b
n
a
n
=
b a
2
n1
, n N

iii) f (a
n
) f (b
n
) 0, n N

.
Aplicand Teorema lui Cantor rezulta ca cele doua siruri (a
n
)
n1
, (b
n
)
n1
sunt covergente catre o
limita comuna pe care o notam c. T inand cont de i) rezulta ca c [a, b] . Funct ia f ind continua pe
[a, b] , este continua n punctul c, rezulta lim
n
f (a
n
) = lim
n
f (b
n
) = f (c) . Obt inem
lim
n
(f (a
n
) f (b
n
)) = f
2
(c)
si, pe baza iii), f
2
(c) 0. De aici rezulta c a f (c) = 0. Dar c ,= a si c ,= b deoarece f (a) f (b) < 0, asadar
c (a, b) .
3.4.9 Observat ie.

In ipotezele teoremei ecuat ia f (x) = 0, x (a, b) are cel put in o solut ie.
3.4 Funct ii f : D R R continue pe o mult ime 143
Daca intervalul (a, b) cont ine o singura solut ie a ecuat iei, x
0
, aceasta se poate aproxima prin c
1
=
a +b
2
, eroarea comisa ind [x
0
c
1
[
b a
2
; pentru a obt ine o aproximare mai buna pentru x
0
, calculam
c
2
ca mai sus (n demonstrat ia teoremei), eroarea aproximarii x
0
c
2
ind [x
0
c
2
[
b a
2
2
; daca dorim o
aproximare si mai buna pentru x
0
, calculam c
3
ca mai sus (n demonstrat ia teoremei), eroarea aproximarii
x
0
c
3
ind [x
0
c
3
[ b a2
3
; continuand n acelasi mod se poate ajunge la o aproximat ie oricat de
buna, n sensul de eroare oricat de mica, a solut iei x
0
unica n intervalul (a, b) a ecuat iei considerate.
Metoda descrisa aici se numeste metodanjumatat irii intervalului.
Urmatoarea teorema pune n evident a una dintre cele mai importante proprietat i ale funct iilor con-
tinue pe un interval.
3.4.10 Denit ie. Fie f : D R R o funct ie si I D un interval. Spunem ca funct ia f are
proprietatea lui Darboux pe intervalul I daca pentru orice x
1
, x
2
I, x
1
< x
2
si orice numar y
cuprins strict ntre valorile f (x
1
) , f (x
2
) exista cel put in un punct c (x
1
, x
2
) astfel ncat y = f (c) .
3.4.11 Teorema (Teorema lui Bolzano-Darboux) Fie I R un interval si f : I R o funct ie.
Daca funct ia f este continua pe intervalul I, atunci funct ia f are proprietatea lui Darboux pe I.
Demonstrat ie. Fie x
1
, x
2
I, x
1
< x
2
si y
0
cuprins strict ntre valorile f (x
1
) , f (x
2
) . Atunci
(f (x
1
) y
0
) (f (x
2
) y
0
) < 0. Denim funct ia g : I R, g (x) = f (x) y
0
. Inegalitatea precedenta se
scrie g (x
1
) g (x
2
) < 0. Din continuitatea funct iei f si Teorema 3.3.13 rezulta ca funct ia g este continua
pe [a, b] , deci si pe [x
1
, x
2
] . Pe baza teoremei lui Bolzano-Cauchy exista c (x
1
, x
2
) astfel ncat g (c) = 0
adica astfel ncat f (c) y
0
= 0, deci f(c) = y
0
.
3.4.12 Observat ie. Reciproca teoremei nu are loc. Exista funct ii care au proprietatea lui Darboux
pe un interval si nu sunt continue pe acel interval.
3.4.13 Exemplu. Fie [1, 1] . Funct ia f : R [1, 1],
f (x) =
_
sin
1
x
, daca x (, 0) (0, +)
, daca x = 0
are proprietatea lui Darboux pe R si nu este continua pe R.
Urmatoarea teorema ne ofera caracterizari pentru funct ii care au proprietatea lui Darboux pe un
interval.
3.4.14 Teorema. Fie f : D R R o funct ie si I D un interval. Urmatoarele armat ii sunt
echivalente:
(i) Funct ia f are proprietatea lui Darboux pe intervalul I.
(ii) Oricare ar J I un interval, mult imea f (J) este interval.
(iii) Oricare ar a, b I, a < b, mult imea f([a, b]) este interval.
Demonstrat ie. Aratam ca (i) implica (ii). Presupunem ca f are proprietatea lui Darboux pe
I. Fie J I un interval. Pentru a arata ca f (J) este interval, consideram arbitrar y
1
, y
2
f (J) cu
y
1
< y
2
, urmand ca despre ecare y (y
1
, y
2
) sa aratam ca y f (J) . Din y
1
, y
2
f (J) rezulta ca
exista x
1
, x
2
J cu f (x
1
) = y
1
, f (x
2
) = y
2
. Mai departe, pe baza proprietat ii lui Darboux a funct iei f
deducem ca exista c situat strict ntre x
1
, x
2
astfel ncat f (c) = y; prin urmare y f (J) . Asadar f(J)
este interval.
Este evident ca armat ia (ii) implica (iii).
Aratam ca (iii) implica (i). Presupunem ca (iii) este adevarata si demonstram ca f are proprietatea lui
Darboux pe I. Fie x
1
, x
2
I, x
1
< x
2
si un numar y cuprins strict ntre valorile f (x
1
) , f (x
2
) . Mult imea
f ([x
1
, x
2
]) , pe baza armat iei (iii) ind interval, rezulta ca y f ([x
1
, x
2
]) . Acest lucru nseamna ca
exista c [x
1
, x
2
] astfel ncat f (c) = y. Cum y ,= f (x
1
) , y ,= f (x
2
) deducem ca c ,= x
1
, c ,= x
2
si deci
c (x
1
, x
2
) . Prin urmare f are proprietatea lui Darboux pe I.
3.4.15 Teorema. Fie I R un interval si f : I R o funct ie continua. Atunci oricare ar un
interval J I, mult imea f (J) este interval.
Demonstrat ie. Pe baza teoremei lui Bolzano-Darboux funct ia continua f : I R are proprietatea
lui Darboux pe I. Aplicand (ii) din teorema precedenta se obt ine concluzia.
Urmatoarea teorema ne arata ca proprietatea lui Darboux se transmite prin operat ia de compunere
a funct iilor.
144 3. Funct ii f : D R R continue
3.4.16 Teorema. Fie I, J R intervale si f : I R, g : J R doua funct ii. Daca f are
proprietatea lui Darboux pe intervalul I, g are proprietatea lui Darboux pe intervalul J si f (I) J,
atunci funct ia g f : I R are proprietatea lui Darboux pe intervalul I.
Demonstrat ie. Aratam ca pentru orice interval I
1
I mult imea (g f) (I
1
) este interval. Atunci,
pe baza Teoremei 3.4.14, rezulta ca funct ia g f : IR are proprietatea lui Darboux pe intervalul
I. Fie I
1
I un interval arbitrar. Deoarece f are proprietatea lui Darboux pe I, pe baza Teoremei
3.4.14, mult imea f (I
1
) este interval. Evident f (I
1
) f (I) . Avand f (I) J deducem ca f (I
1
) J.
Deoarece funct ia g are proprietatea lui Darboux pe J, rezulta ca mult imea g (f (I
1
)) este interval. Dar
g (f (I
1
)) = (g f) (I
1
) , deci mult imea (g f) (I
1
) este interval.

In mod evident orice funct ie strict monotona este injectiva, nsa nu toate funct iile injective sunt strict
monotone. Urmatoarea teorema arata ca n cazul funct iilor cu proprietatea lui Darboux pe un interval
injectivitatea funct iei implica monotonia stricta a ei.
3.4.17 Teorema. Fie I R un interval nedegenerat si f : I R o funct ie cu proprietatea lui
Darboux pe I. Funct ia f este injectiva daca si numai daca f este strict monotona.
Demonstrat ie. Daca funct ia f este strict monotona atunci n mod evident este injectiva.
Fie funct ia f cu proprietatea lui Darboux si injectiva. Sa aratam ca f este strict monotona. Rat ionam
prin reducere la absurd. Presupunem ca f nu este si strict monotona. Atunci exista x
1
, x
2
, x
3
I,
x
1
< x
2
< x
3
astfel ncat f (x
1
) < f (x
2
) > f (x
3
) sau f (x
1
) > f (x
2
) < f (x
3
) . Ne ocupam de prima
situat ie, a doua se discuta n mod asemanator. Fie deci f (x
1
) < f (x
2
) > f (x
3
) . Comparat ia ntre f (x
1
)
si f (x
3
) induce trei cazuri f (x
1
) < f (x
3
) , f (x
3
) < f (x
1
), respectiv f (x
1
) = f (x
3
) pe care le analizam
pe rand.
i) Daca f (x
1
) < f (x
3
) < f (x
2
) : cum f are proprietatea lui Darboux pe intervalul I, exista
c (x
1
, x
2
) astfel ncat f (x
3
) = f (c) .

Insa c < x
2
< x
3
, deci x
3
,= c, contradict ie cu proprietatea de
injectivitate a funct iei f.
ii) Daca f (x
3
) < f (x
1
) < f (x
2
) : cum f are proprietatea lui Darboux pe I, exista c (x
2
, x
3
)
astfel ncat f (x
1
) = f (c) .

Insa x
1
< x
2
< c, deci x
1
,= c, contradict ie cu proprietatea de injectivitate a
funct iei f.
iii) f (x
1
) = f (x
3
) < f (x
2
) : egalitatea f (x
1
) = f (x
3
) unde x
1
,= x
3
este n contradict ie cu
proprietatea de injectivitate a funct iei f.
Presupunerea noastra duce la contradict ie, deci este incorecta. Asadar f este strict monotona.
3.4.18 Teorema. Fie I R un interval nedegenerat si f : I R o funct ie continua pe I. Funct ia
f este injectiva daca si numai daca f este strict monotona.
Demonstrat ie. Funct ia f este continua pe I. Rezulta ca f are proprietatea lui Darboux pe I,
asadar este aplicabila teorema precedenta.

In continuare aratam ca prin operat ia de inversare a funct iilor se transmite proprietatea de continu-
itate pe interval. Vom avea nevoie de urmatoarea teorema care este chiar evidenta.
3.4.19 Teorema. O funct ie f : D R E R bijectiva este strict crescatoare (strict
descrescatoare) daca si numai daca funct ia inversa f
1
: E D este strict crescatoare (strict de-
screscatoare).
3.4.20 Teorema. Fie I, J R intervale. Fie f : I J o funct ie continua si bijectiva. Atunci
funct ia inversa f
1
: J I este continua.
Demonstrat ie. Funct ia f ind bijectiva, este injectiva. Fiind si continua, pe baza Teoremei 3.4.18
rezulta ca funct ia f este strict monotona. Consideram ca f este strict crescatoare (cazul f descrescatoare
se discuta n mod asemanator). Pe baza Teoremei 3.4.19 si funct ia inversa f
1
: J I este strict
crescatoare.
Fie y J arbitrar. Aratam ca f
1
este continua n y. Cum y J este arbitrar, va rezulta ca f
1
este continua pe J. Fie V o vecinatate arbitrara a lui f
1
(y) ; aratam ca exista o vecinatate U a lui y cu
proprietatea f
1
(J U) V, ceea ce nseamna ca f
1
este continua n y. Consideram cazul cand y nu
este extremitate a intervalului J (cazul contrar se discutan mod asemanator). Atunci, cum f
1
este strict
crescatoare, nici x = f
1
(y) I nu este extremitate a intervalului I. Prin urmare exista > 0 astfel ncat
[x , x +] I V. Din x < x < x+ si f strict crescatoare rezulta f (x ) < f (x) < f (x +) ,
de unde deducem ca intervalul (f (x ) , f (x +)) este o vecinatate a numarului f (x) = y. Aratam
ca f
1
((f (x ) , f (x +)) V, adica pentru u (f (x ) , f (x +)) aratam ca are loc f
1
(u) V.
Deoarece f are proprietatea lui Darboux pe intervalul I, din u (f (x ) , f (x +)) deducem ca exista
3.4 Funct ii f : D R R continue pe o mult ime 145
c (x , x +) cu f (c) = u de unde rezulta ca f
1
(u) = c (x , x +) V. Asadar, pentru
vecinatatea V arbitrar luata a lui f
1
(y) exista o vecinatate U a lui y astfel ncat f
1
(J U) V, si
anume U = (f (x ) , f (x +)) intervalul determinat mai sus.
3.4.21 Exemplu. Funct ia f : [0, 1] [1, 3] , f (x) = x
5
+x + 1 este continua si bijectiva. Aplicand
teorema precedenta rezulta ca inversa lui f, funct ia f
1
: [1, 3] [0, 1] este continua.
Punctele de discontinuitate n cazul funct iilor f : D R R le clasicam n doua categorii.
3.4.22 Denit ie. Fie f : D R R o funct ie si x
0
D un punct de discontinuitate a funct iei f.
i) Spunem ca punctul de discontinuitate x
0
al funct iei f are spet a ntai, daca n punctul x
0
funct ia
f are limite laterale nite.
ii) Spunem ca punctul de discontinuitate x
0
al funct iei f are spet a a doua, daca punctul de
discontinuitate nu are spet a ntai.
Urmatoarele doua teoreme se refera la punctele de discontinuitate ale funct iilor monotone.
3.4.23 Teorema. Fie I R un interval deschis si f : I R o funct ie monotona pe I. Atunci
punctele de discontinuitate ale funct iei f sunt de spet a ntai.
Demonstrat ie. Pe baza Teoremelor 3.2.22, 3.2.23 de existent a a limitelor laterale n cazul funct iilor
monotone, n ecare punct x
0
I funct ia monotona f are limite laterale nite. Rezulta ca orice punct
de discontinuitate a funct iei f are spet a ntai.
3.4.24 Teorema. Fie I R un interval si f : I R o funct ie monotona pe I. Atunci mult imea
punctelor de discontinuitate a funct iei f este o mult ime cel mult numarabila.
Demonstrat ie. Presupunem ca funct ia f este crescatoare pe I. Pentru f descrescatoare pe I se
va rat iona ntr-un mod asemanator. Deoarece f este monotona, n ecare x
0
I funct ia f are limite
laterale nite.
Notam cu A mult imea punctelor de discontinuitate a funct iei f diferite de cele doua extremitat i
ale intervalului I. Evident A I. Consideram doua puncte de discontinuitate x
1
, x
2
A, x
1
< x
2
. Fie
c =
x
1
+x
2
2
. Din
f (x
1
+ 0) = inf f (x)[ x I, x > x
1
f (c) supf (x)[ x I, x < x
2
= f (x
2
0)
rezulta ca (f (x
1
0) , f (x
1
+ 0)) (f (x
2
0) , f (x
2
+ 0)) = .
Asadar funct ia : A , care ataseaza ecarui punct x A un punct rat ional din intervalul
(f (x 0) , f (x + 0)) , este o funct ie injectiva. Mult imea ind numarabila, deducem ca mult imea A
este cel mult numarabila.

In continuare studiem punctele de discontinuitate ale funct iilor cu proprietatea lui Darboux.
3.4.25 Teorema. Fie I R un interval si f : I R o funct ie. Daca f are n intervalul I cel
put in un punct de discontinuitate de spet a ntai, atunci f nu are proprietatea lui Darboux pe I.
Demonstrat ie. Fie x
0
I un punct de discontinuitate de spet a ntai pentru f.

Inseamna ca exista
limitele laterale f (x
0
0) R, f (x
0
+ 0) R. Deoarece f este discontinua n punctul x
0
, cel put in una
dintre limitele laterale f (x
0
0) , f (x
0
+ 0) este diferita de f (x
0
) . Consideram f (x
0
0) ,= f (x
0
) .

In
cazul f (x
0
+ 0) ,= f (x
0
) se poate rat iona n mod asemanator. Notam [f (x
0
) f (x
0
0)[ = d. Are loc
d > 0. Consideram intervalul V =
_
f (x
0
0)
d
3
, f (x
0
0) +
d
3
_
. Cum intervalul V este vecinatate a
numarului f (x
0
0) , pe baza denit iei limitei la stanga n punctul x
0
exista o vecinatate U a lui x
0
cu
proprietatea ca f (I (, x
0
) U) V. Cum U este vecinatate pentru x
0
, exista > 0 astfel ncat
(x
0
, x
0
+) U. Fixam x
1
I (, x
0
) (x
0
, x
0
+) . Atunci [x
1
, x
0
) I (, x
0
) U,
deci f ([x
1
, x
0
)) V. Fie y =
f(x
0
0) +f (x
0
)
2
; evident y / V =
_
f (x
0
0)
d
3
, f (x
0
0) +
d
3
_
si y
este situat strict ntre f (x
1
) si f (x
0
) . Din f ((x
1
, x
0
)) V si y / V rezulta ca nu exista c (x
1
, x
0
) cu
proprietatea f(c) = y si din acest motiv f nu are proprietatea lui Darboux pe I.
3.4.26 Teorema. Punctele de discontinuitate ale unei funct ii cu proprietatea lui Darboux au spet a
a doua.
Demonstrat ie. Concluzia teoremei decurge direct din teorema precedenta.
Urmatoarea teorema da o caracterizare a funct iilor continue pe o mult ime.
3.4.27 Teorema. Funct ia f : D R R este continua pe mult imea nevida D
0
D daca si numai
daca pentru orice x D
0
si > 0 exista (x, ) > 0 astfel ncat pentru orice u D
0
cu [x u[ < (x, )
sa avem [f (x) f (u)[ < .
146 3. Funct ii f : D R R continue
Demonstrat ie. Conform Denit iei 3.4.1 funct ia f este continua pe mult imea D
0
daca si numai
daca ea este continua n ecare punct x D
0
. Aplicand pentru ecare x D
0
armat ia (ii) din Teorema
3.3.3 de caracterizare a funct iilor continue ntr-un punct obt inem ca f este continua n ecare x D
0
,
daca si numai daca pentru ecare x D
0
si ecare > 0 exista (x, ) > 0 astfel ncat u D
0
,
[x u[ < (x, ) [f (x) f (u)[ < .

In cele ce urmeaza consideram o not iune speciala de continuitate, cea de continuitate uniforma a
funct iei pe o mult ime.
3.4.28 Denit ie. Spunem ca funct ia f : D R R este uniform continua pe mult imea nevida
D
0
D daca pentru orice > 0 exista () > 0 astfel ncat
x, u D
0
, [x u[ < () [f (x) f (u)[ < .
Daca f este uniform continua pe mult imea de denit ie D, atunci spunem ca funct ia f este uniform
continua.
3.4.29 Observat ie. Cele doua not iuni, cea de continuitate pe mult imea D
0
, si respectiv uniform
continuitatea pe D
0
, difera la ordinea cuanticatorilor logici (exista,oricare) si implicit prin rolul
numarului care n cazul continuitat ii uniforme nu depinde de x D
0
.
3.4.30 Teorema. Daca funct ia f : D R R este uniform continua pe mult imea nevida D
0
D,
atunci f este continua pe mult imea D
0
.
Demonstrat ie. Pe baza denit iei continuitat ii uniforme a funct iei f pe mult imea D
0
pentru orice
> 0 exista () > 0 astfel ncat pentru ecare x, u D
0
cu [x u[ < () sa avem [f (x) f (u)[ < .
Prin urmare, pentru x D
0
si > 0 arbitrare exista (x, ) > 0 cu proprietatea ca u D
0
, [x u[ <
(x, ) implica [f (x) f (u)[ < , si anume (x, ) = (x) . Pe baza Teoremei 3.4.27 de caracterizare a
funct iilor continue pe o mult ime rezulta ca f este continua pe D
0
.
3.4.31 Observat ie. Reciproca teoremei nu are loc. Exista funct ii continue pe un interval care nu
sunt uniform continue pe acel interval.
3.4.32 Exemplu. Funct ia f : RR, f (x) = x
2
este continua pe R. Aratam ca funct ia f nu
este uniform continua pe R. Presupunem ca f este uniform continua pe R. Atunci, pe baza denit iei
continuitat ii uniforme, pentru = 1 exista > 0 cu proprietatea ca pentru orice x, u R cu [x u[ <
rezulta [f (x) f (u)[ < 1. Luand x =
1

si u =
1

+

2
are loc [x u[ =

2
< , dar
[f (x) f (u)[ = 1 +

2
4
> 1,
deci [f (x) f (u)[ < 1 nu are loc. Asadar f nu este uniform continua pe R.
Urmatoarea teorema arata ca pe un interval nchis [a, b] R continuitatea funct iilor este uniforma.
3.4.33 Teorema. Fie a, b R, a < b si f : [a, b] R o funct ie. Daca f este continua pe [a, b] ,
atunci f este uniform continua pe [a, b] .
Demonstrat ie. Rat ionam prin reducere la absurd. Fie f : [a, b] R o funct ie continua pe [a, b] .
Presupunem ca funct ia f nu este uniform continua pe [a, b] . Atunci exista > 0 cu proprietatea ca pentru
ecare > 0 exista x, u [a, b] astfel ncat
_
[x u[ <
[f (x) f (u)[ .

In particular, pentru ecare n N

,
considerand =
1
n
, exista x
n
, u
n
[a, b] cu proprietatea
_
[x
n
u
n
[ <
1
n
[f (x
n
) f (u
n
)[ .
Sirul (x
n
)
n1
are tot i
termenii n intervalul [a, b] . Asadar sirul de numere reale (x
n
)
n1
este marginit. Rezulta ca are un subsir
(x
n
k
)
k1
convergent. Notam cu x
0
limita acestui subsir. Din x
n
k
[a, b] , k N

rezulta ca x
0
[a, b] .
Funct ia f ind continua pe [a, b] , ea este continua n ecare punct al intervalului, deci si n punctul x
0
.
Din [x
n
k
u
n
k
[ <
1
n
k
, k N

rezulta lim
k
(x
n
k
u
n
k
) = 0 si
lim
k
u
n
k
= lim
k
(x
n
k
(x
n
k
u
n
k
)) = x
0
0 = x
0
.
Pe baza continuitat ii funct iei f n punctul x
0
avem lim
k
f (x
n
k
) = f (x
0
) si lim
k
f (u
n
k
) = f (x
0
) . Rezulta
ca lim
k
(f (x
n
k
) f (u
n
k
)) = 0, n contradict ie cu [f (x
n
) f (u
n
)[ , n N

. Presupunerea noastra
duce la contradict ie, deci este incorecta. Rezulta ca funct ia f este uniform continua pe [a, b] .
3.4 Funct ii f : D R R continue pe o mult ime 147
3.4.34 Exemplu. Am vazut n Exemplul 3.4.32 ca funct ia f : R R, f (x) = x
2
, continua pe R,
nu este uniform continua pe R. Consideram acum restrict ia funct iei f la intervalul [0, 1],
g : [0, 1] R, g (x) = x
2
.
Conform teoremei precedente funct ia g este uniform continua pe [0, 1] .
Capitolul 4
Funct ii f : D R R derivabile
4.1 Derivata funct iei ntr-un punct. Derivabilitatea funct iei ntr-
un punct
Utilitatea practica a raportarii variat iei funct iei f ntr-un punct x
0
, f (x) f (x
0
) , la variat ia lui x,
x x
0
, conduce la urmatoarea denit ie.
4.1.1. Denit ie. Fie funct ia f : D R R si e x
0
D un punct de acumulare pentru D.
Spunem ca funct ia f are derivata n punctul x
0
daca exista limita lim
xx
0
f (x) f (x
0
)
x x
0
pe care o
notam f
t
(x
0
) sau
df
dx
(x
0
) si o numim derivata n punctul x
0
a funct iei f.
Spunem ca funct ia f este derivabila n punctul x
0
daca exista
f
t
(x
0
) = lim
xx
0
f (x) f (x
0
)
x x
0
si f
t
(x
0
) R.
4.1.2. Exemplu. i) Fie f : R R, f (x) =
3

x.
Pentru x
0
R 0 :
f
t
(x
0
) = lim
xx
0
f (x) f (x
0
)
x x
0
= lim
xx
0
3

x
3

x
0
x x
0
= lim
xx
0
_
3

x
3

x
0
_
_
3

x
2
+
3

xx
0
+
3
_
(x
0
)
2
_
(x x
0
)
_
3

x
2
+
3

xx
0
+
3
_
(x
0
)
2
_ =
lim
xx
0
x x
0
(x x
0
)
_
3

x
2
+
3

xx
0
+
3
_
(x
0
)
2
_ = lim
xx
0
1
3

x
2
+
3

xx
0
+
3
_
(x
0
)
2
=
1
3
3
_
(x
0
)
2
R,
deci funct ia f este derivabila n punctul x
0
R 0 .
Pentru x
0
= 0 avem f
t
(0) = lim
x0
f (x) f (0)
x 0
= lim
x0
3

x
x
= lim
x0
1
3

x
2
= + / R, deci funct ia f nu
este derivabila n punctul x
0
= 0.
ii) Fie f : R R, f (x) = sinx.
Pentru ecare x
0
R avem
f
t
(x
0
) = lim
xx
0
f (x) f (x
0
)
x x
0
= lim
xx
0
sinx sinx
0
x x
0
= lim
xx
0
2 sin
x x
0
2
cos
x +x
0
2
x x
0
= lim
xx
0
sin
x x
0
2
x x
0
2
cos
x +x
0
2
= 1 cos
x
0
+x
0
2
= cos x
0
.
Din f
t
(x
0
) = cos x
0
R rezulta ca f este derivabila n x
0
.
148
4.1 Derivata funct iei ntr-un punct. Derivabilitatea funct iei ntr-un punct 149
iii) Fie f : R R, f (x) = cos x.
Pentru ecare x
0
R avem
f
t
(x
0
) = lim
xx
0
f (x) f (x
0
)
x x
0
= lim
xx
0
cos x cos x
0
x x
0
= lim
xx
0
2 sin
x +x
0
2
sin
x x
0
2
x x
0
= lim
xx
0
(1)
sin
x x
0
2
x x
0
2
sin
x +x
0
2
= 1 sin
x
0
+x
0
2
= sinx
0
.
Din f
t
(x
0
) = sinx
0
R rezulta ca f este derivabila n x
0
.
iv) Fie f : (0, +) R, f (x) = lnx.
Pentru ecare x
0
(0, +) avem
f
t
(x
0
) = lim
xx
0
f (x) f (x
0
)
x x
0
= lim
xx
0
lnx lnx
0
x x
0
= lim
xx
0
1
x x
0
_
ln
x
x
0
_
= lim
xx
0
ln
_
x
x
0
_
1
x x
0
= lne
1
x
0
=
1
x
0
,
deoarece
lim
xx
0
_
x
x
0
_
1
x x
0
= lim
xx
0
_
1 +
_
x
x
0
1
__
1
x x
0
= lim
xx
0
_

_
_
1 +
x x
0
x
0
_
x
0
x x
0
_

_
1
x
0
= e
1
x
0
.
Din f
t
(x
0
) =
1
x
0
R rezulta ca f este derivabila n x
0
(0, +) .
v) Fie f : R R, f (x) = x
n
, unde n N

.
Pentru ecare x
0
R avem
f
t
(x
0
) = lim
xx
0
f (x) f (x
0
)
x x
0
= lim
xx
0
x
n
(x
0
)
n
x x
0
= lim
xx
0
(x x
0
)
_
x
n1
+x
n2
x
0
+x
n3
(x
0
)
2
+. . . +x(x
0
)
n2
+ (x
0
)
n1
_
x x
0
= lim
xx
0
_
x
n1
+x
n2
x
0
+x
n3
(x
0
)
2
+. . . +x(x
0
)
n2
+ (x
0
)
n1
_
= (x
0
)
n1
+ (x
0
)
n1
+. . . + (x
0
)
n1
= n(x
0
)
n1
.
Din f
t
(x
0
) = n(x
0
)
n1
R rezulta ca f este derivabila n x
0
.
Not iunea de derivata ntr-un punct a unei funct ii f : D R R se considera numai n acele puncte
x
0
D care sunt puncte de acumulare pentru D.

Intr-un punct izolat al mult imii D nu se pune problema


existent ei derivatei a unei funct ii f : D R R.
Not iunile de derivata ntr-un punct x
0
a funct iei f, cea de limita n x
0
a funct iei f, cat si cea de
continuitate/discontinuitate n x
0
a funct iei f au un caracter local, n sensul ca pentru studiul acestor
not iuni punctele unde trebuie sa cunoastem valorile funct iei f se ncadreaza ntr-o vecinatate a lui x
0
.
Urmatoarea teorema evident iaza legatura dintre derivabilitatea funct iei ntr-un punct si continuitatea
ei n acelasi punct.
4.1.3. Teorema. Fie D R si x
0
D un punct de acumulare pentru D. Daca funct ia f : D R
este derivabila n punctul x
0
, atunci funct ia f este continua n punctul x
0
.
Demonstrat ie. Pentru ecare x D x
0
are loc
150 4. Funct ii f : D R R derivabile
f (x) =
f (x) f (x
0
)
x x
0
(x x
0
) +f (x
0
).
Pe baza acestei identitat i calculam lim
xx
0
f (x) . Deoarece funct ia f este derivabila n punctul x
0
avem
lim
xx
0
f (x) f (x
0
)
x x
0
= f
t
(x
0
) R si deoarece lim
xx
0
(x x
0
) = 0 obt inem lim
xx
0
f (x) = f
t
(x
0
) 0 +f (x
0
) ,
deci lim
xx
0
f (x) = f (x
0
) . Ultima egalitate arata ca funct ia f este continua n punctul x
0
.
4.1.4. Observat ie. Din existent a derivatei funct iei f n punctul x
0
nu rezulta continuitatea funct iei
n x
0
.
4.1.5. Exemplu. Pentru funct ia
f : R R , f (x) =
_
3

x, daca x 0
3

x + 1, daca x > 0
avem f
t
(0) = lim
x0
f (x) f (0)
x 0
= lim
x0
f (x)
x
= +, deoarece ambele limite laterale
lim
x0
x<0
f (x)
x
= lim
x0
x<0
3

x
x
= lim
x0
x<0
1
3

x
2
= +,
lim
x0
x>0
f (x)
x
= lim
x0
x>0
3

x + 1
x
= lim
x0
x>0
_
1
3

x
2
+
1
x
_
= +
sunt egale cu +. Funct ia f are derivata n punctul x
0
= 0 si nu este derivabila n acest punct deoarece
f
t
(0) / R. Din lim
x0
x<0
f (x) = 0 ,= lim
x0
x>0
f (x) = 1 rezulta ca funct ia f nu este continua n x
0
= 0.
4.1.6. Observat ie. Reciproca Teoremei 4.1.3 nu are loc. Este posibil ca o funct ie f : D R R
sa e continua ntr-un punct x
0
si sa nu e derivabila n acest punct.
4.1.7. Exemplu. Funct ia f : R R, f(x) = [x[ este continua n x
0
= 0 si nu este derivabila n
x
0
= 0.
Daca o funct ie f : D R R nu este continua n punctul x
0
D care este punct de acumulare
pentru D, atunci, pe baza teoremei de nainte, funct ia f nu este derivabila n acest punct. Condit ia de
continuitate n x
0
a funct iei este o condit ie necesara derivabilitat ii funct iei n punctul x
0
.
Not iunile de limita la stanga respectiv la dreapta ntr-un punct a unei funct ii ne conduc la not iuni
de derivata laterala.
4.1.8. Denit ie. Fie funct ia f : D R R.
i) Fie x
0
D, x
0
punct de acumulare pentru mult imea D (, x
0
).
Spunem ca funct ia f are n punctul x
0
derivata la stanga, daca exista limita
lim
xx
0
x<x
0
f (x) f (x
0
)
x x
0
,
pe care o notam f
t
s
(x
0
) si o numim derivata la stanga n punctul x
0
a funct iei f.
Spunem ca funct ia f este derivabila la stanga n punctul x
0
daca exista
f
t
s
(x
0
) = lim
xx
0
x<x
0
f (x) f (x
0
)
x x
0
si f
t
s
(x
0
) R.
ii) Fie x
0
D, x
0
punct de acumulare pentru mult imea D (x
0
, +).
Spunem ca funct ia f are n punctul x
0
derivata la dreapta, daca exista limita
lim
xx
0
x>x
0
f (x) f (x
0
)
x x
0
,
pe care o notam f
t
d
(x
0
) si o numim derivata la dreapta n punctul x
0
a funct iei f.
Spunem ca funct ia f este derivabila la dreapta n punctul x
0
daca exista
4.1 Derivata funct iei ntr-un punct. Derivabilitatea funct iei ntr-un punct 151
f
t
d
(x
0
) = lim
xx
0
x>x
0
f (x) f (x
0
)
x x
0
si f
t
d
(x
0
) R.
Derivatele f
t
s
(x
0
) si f
t
d
(x
0
) se numesc derivate laterale ale funct iei f n punctul x
0
.
4.1.9. Observat ie. Fie a, b R, a < b si f : [a, b] R o funct ie. Derivabilitatea n punctul
a a funct iei f n sensul Denit iei 4.1.1 revine la derivabilitatea la dreapta n punctul a a funct iei f.
Derivabilitatea n punctul b a funct iei f n sensul Denit iei 4.1.1 revine la derivabilitatea la stanga n
punctul b a funct iei f.
Urmatoarea teorema arata legatura dintre derivata funct iei ntr-un punct si derivatele laterale ale ei
n acelasi punct.
4.1.10. Teorema. Fie D R o mult ime nevida si f : D R o funct ie. Fie x
0
D un punct de
acumulare atat pentru mult imea D (, x
0
) cat si pentru mult imea D (x
0
, +).
i) Daca funct ia f are derivata n punctul x
0
, atunci ea are derivata la stanga n x
0
, are derivata la
dreapta n x
0
si f
t
s
(x
0
) = f
t
d
(x
0
) = f
t
(x
0
) .
ii) Daca funct ia f are derivata la stanga n x
0
, are derivata la dreapta n x
0
si f
t
s
(x
0
) = f
t
d
(x
0
) ,
atunci funct ia f are derivata n punctul x
0
si f
t
(x
0
) = f
t
s
(x
0
) = f
t
d
(x
0
) .
Demonstrat ie. Se utilizeaza Teorema 3.2.20 care arata legatura dintre limita funct iei ntr-un punct
si limitele laterale ale funct iei n acelasi punct.
Urmatoarea teorema arata legaturea dintre derivabilitatea laterala ntr-un punct a unei funct ii si
continuitatea laterala n acelasi punct a funct iei.
4.1.11. Teorema. Fie funct ia f : D R R si e x
0
D.
i) Daca x
0
este un punct de acumulare pentru mult imea D (, x
0
) si funct ia f este derivabila
la stanga n punctul x
0
, atunci funct ia f este continua la stanga n punctul x
0
.
ii) Daca x
0
este un punct de acumulare pentru mult imea D (x
0
, +) si funct ia f este derivabila
la dreapta n punctul x
0
, atunci funct ia f este continua la dreapta n punctul x
0
.
Demonstrat ie. i) Pentru ecare x D x
0
are loc
f (x) =
f (x) f (x
0
)
x x
0
(x x
0
) +f (x
0
)
Pe baza acestei identitat i calculam lim
xx
0
x<x
0
f (x) . Deoarece funct ia f este derivabila la stanga n punctul
x
0
avem lim
xx
0
x<x
0
f (x) f (x
0
)
x x
0
= f
t
s
(x
0
) R si deoarece lim
xx
0
x<x
0
(x x
0
) = 0 obt inem
lim
xx
0
x<x
0
f (x) = f
t
s
(x
0
) 0 +f (x
0
) ,
asadar lim
xx
0
x<x
0
f (x) = f (x
0
) , egalitate care ne spune ca funct ia f este continua la stanga n punctul x
0
.
ii) Pe baza aceleiasi identitat i calculam lim
xx
0
x>x
0
f (x) . Deoarece funct ia f este derivabila la dreapta n
punctul x
0
avem lim
xx
0
x>x
0
f (x) f (x
0
)
x x
0
= f
t
d
(x
0
) R si deoarece lim
xx
0
x>x
0
(x x
0
) = 0 obt inem
lim
xx
0
x>x
0
f (x) = f
t
d
(x
0
) 0 +f (x
0
) ,
asadar lim
xx
0
x>x
0
f (x) = f (x
0
) , egalitate care ne spune ca funct ia f este continua la dreapta n punctul x
0
.
4.1.12. Observat ie. Este posibil ca o funct ie f : D R R sa e continua la stanga ntr-un
punct si sa nu e derivabila la stanga n acel punct. Este posibil ca o funct ie sa e continua la dreapta
ntr-un punct si sa nu e derivabila la dreapta n acel punct.
4.1.13. Exemplu. Funct ia
f : R R, f (x) =
_
xsin
1
x
, daca x (, 0) (0, +)
0, daca x = 0
152 4. Funct ii f : D R R derivabile
este continua la stanga n punctul x
0
= 0 si nu este derivabila la stanga n x
0
= 0.
Interpretarea geometrica a derivatei f
t
(x
0
)
Fie I R un interval nedegenerat, x
0
I un punct interior intervalului si f : I R o funct ie care
are derivata n x
0
.
Consideram mult imea G = (x, f (x))[ x I numita gracul funct iei f. Mult imea G poate
reprezentata ntr-un sistem cartezian de coordonate xOy prin mult imea punctelor (x, y) de abscisa x si
ordonata y = f (x) , unde x parcurge mult imea I. Consideram punctele M
0
(x
0
, f (x
0
)) si M(x, f (x)) ale
gracului G. Unghiul dintre dreapta M
0
M si axa Ox are panta (coecientul unghiular)
tg =
f (x) f (x
0
)
x x
0
.
Deoarece f are derivata n punctul x
0
, exista
lim
xx
0
f (x) f (x
0
)
x x
0
= f
t
(x
0
) .
Rezulta ca pozit ia dreptei M
0
M tinde catre o pozit ie limita M
0
T atunci cand x tinde la x
0
. Dreapta M
0
T,
adica dreapta care trece prin punctul M
0
(x
0
, f (x
0
)) si care are panta (coecientul unghiular) f
t
(x
0
) , se
numeste dreapta tangenta n punctul M
0
(x
0
, f (x
0
)) la gracul funct iei f.
Daca funct ia f este derivabila n punctul x
0
, atunci f
t
(x
0
) R si prin urmare ecuat ia dreptei
tangente n punctul M
0
(x
0
, f (x
0
)) la gracul funct iei f se scrie cunoscand un punct al dreptei (x
0
, f (x
0
))
si panta dreptei egala cu f
t
(x
0
):
y f(x
0
) = f
t
(x
0
) (x x
0
) .
Daca f
t
(x
0
) = sau f
t
(x
0
) = + atunci din tg = , respectiv tg = + obt inem =

2
sau
=

2
si rezulta ca dreapta tangenta n punctul M
0
(x
0
, f (x
0
)) la gracul funct iei f este paralela cu axa
Oy; asadar ecuat ia ei este x = x
0
.
Daca n punctul x
0
o funct ie f are derivata laterala la stanga atunci semidreapta care trece prin
punctul M
0
(x
0
, f (x
0
)), are panta (coecientul unghiular) f
t
s
(x
0
) , si este formata din puncte de abscisa
x x
0
, se numeste semidreapta tangenta la stanga n punctul M
0
(x
0
, f (x
0
)) gracului funct iei f. Prin
analogie se deneste not iunea de semidreapta tangenta la dreapta n punctul M
0
(x
0
, f (x
0
)) gracului
funct iei f n cazul cand o funct ie f are derivata laterala la dreapta n punctul x
0
.
Daca o funct ie f : I R are derivata n punctul x
0
I care este un punct interior intervalului,
atunci cele doua semidrepte tangente n punctul M
0
(x
0
, f (x
0
)) la gracul funct iei f sunt n prelungire.
Distingem doua tipuri de puncte n care funct ia f este continua si are derivate laterale, fara sa e
derivabila.
4.1.14. Denit ie. Fie I R un interval nedegenerat, x
0
I un punct interior intervalului si
f : I R o funct ie care este continua n x
0
, are derivate laterale n x
0
, dar derivatele laterale sunt
diferite adica f
t
s
(x
0
) ,= f
t
d
(x
0
) .
i) Daca f
t
s
(x
0
) R sau f
t
d
(x
0
) R atunci punctul M
0
(x
0
, f (x
0
)) se numeste punct unghiular al
gracului funct iei f.
ii) Daca f
t
s
(x
0
) = si f
t
d
(x
0
) = + sau invers, atunci punctul M
0
(x
0
, f (x
0
)) se numeste punct
de ntoarcere al gracului funct iei f.
4.1.15. Exemplu. i) Funct ia f : R R, f (x) = [sinx[ este continua n x
0
= 0,
f
t
s
(0) = lim
x0
x<0
f (x) f (0)
x 0
= lim
x0
x<0
[sinx[ 0
x
= lim
x0
x<0
sinx
x
= 1 R,
f
t
d
(0) = lim
x0
x>0
f (x) f (0)
x 0
= lim
x0
x>0
[sinx[ 0
x
= lim
x0
x>0
sinx
x
= 1 R.
4.1 Derivata funct iei ntr-un punct. Derivabilitatea funct iei ntr-un punct 153
Obt inem f
t
s
(0) ,= f
t
d
(0), asadar (0, f (0)) = (0, 0) este punct unghiular al gracului funct iei f.
ii) Funct ia f : R R, f (x) =
3

x
2
este continua n x
0
= 0,
f
t
s
(0) = lim
x0
x<0
f (x) f (0)
x 0
= lim
x0
x<0
3

x
2
0
x
= lim
x0
x<0
1
3

x
= ,
f
t
d
(0) = lim
x0
x>0
f (x) f (0)
x 0
= lim
x0
x>0
3

x
2
0
x
= lim
x0
x>0
1
3

x
= +,
asadar (0, f (0)) = (0, 0) este punct de ntoarcere al gracului funct iei f.
iii) Funct ia f : R R, f (x) =
_
x, daca x (, 0)

x, daca x [0, +)
este continua n x
0
= 0,
f
t
s
(0) = lim
x0
x<0
f (x) f (0)
x 0
= lim
x0
x<0
x 0
x
= lim
x0
x<0
1 = 1 R,
f
t
d
(0) = lim
x0
x>0
f (x) f (0)
x 0
= lim
x0
x>0

x 0
x
= lim
x0
x>0
1

x
= +,
asadar (0, f (0)) = (0, 0) este punct unghiular al gracului funct iei f.
Operat ii cu funct ii derivabile ntr-un punct
Urmatoarele doua teoreme se refera la derivabilitatea ntr-un punct a funct iei suma, produs, cat,
respectiv a funct iei compuse.
4.1.16. Teorema. Fie D R si x
0
D un punct de acumulare pentru mult imea D.
i) Daca funct iile f, g : D R sunt derivabile n punctul x
0
, atunci funct ia suma f +g este derivabila
n punctul x
0
si are loc (f +g)
t
(x
0
) = f
t
(x
0
) +g
t
(x
0
) .
ii) Daca funct iile f, g : D R sunt derivabile n punctul x
0
, atunci funct ia produs fg este derivabila
n punctul x
0
si are loc (f g)
t
(x
0
) = f
t
(x
0
) g (x
0
) +f (x
0
) g
t
(x
0
) .
iii) Daca funct ia f : D R este derivabila n punctul x
0
, atunci funct ia c f, unde c R, este
derivabila n punctul x
0
si are loc (c f)
t
(x
0
) = c f
t
(x
0
) .
iv) Daca funct iile f, g : D R sunt derivabile n punctul x
0
si g
t
(x
0
) ,= 0, atunci funct ia cat
f
g
este derivabila n punctul x
0
si are loc
_
f
g
_
t
(x
0
) =
f
t
(x
0
) g (x
0
) f (x
0
) g
t
(x
0
)
(g
t
(x
0
))
2
.
Demonstrat ie. Cum cele doua funct ii f si g sunt derivabile n punctul x
0
, avem
lim
xx
0
f (x) f (x
0
)
x x
0
= f
t
(x
0
) R, respectiv lim
xx
0
g (x) g (x
0
)
x x
0
= g
t
(x
0
) R.
i) Pentru orice x D x
0
are loc
(f +g) (x) (f +g) (x
0
)
x x
0
=
f (x) f (x
0
)
x x
0
+
g (x) g (x
0
)
x x
0
si
deducem
(f +g)
t
(x
0
) = lim
xx
0
(f +g) (x) (f +g) (x
0
)
x x
0
= lim
xx
0
_
f (x) f (x
0
)
x x
0
+
g (x) g (x
0
)
x x
0
_
= f
t
(x
0
) +g
t
(x
0
) R.
ii) Pentru orice x D x
0
are loc
(f g) (x) (f g) (x
0
)
x x
0
=
f (x) f (x
0
)
x x
0
g (x) +f (x
0
)
g (x) g (x
0
)
x x
0
;
154 4. Funct ii f : D R R derivabile
t inem cont ca derivabilitatea funct iei g n punctul x
0
implica proprietatea de continuitate n x
0
a funct iei
g, deci are loc lim
xx
0
g (x) = g (x
0
) R si deducem
(f g)
t
(x
0
) = lim
xx
0
(f g) (x) (f g) (x
0
)
x x
0
= lim
xx
0
_
f (x) f (x
0
)
x x
0
g (x) +f (x
0
)
g (x) g (x
0
)
x x
0
_
= f
t
(x
0
) g (x
0
) +f (x
0
) g
t
(x
0
) R.
iii) Din punctul ii), n cazul cand funct ia g este constanta g (x) = c, pentru orice x D, rezulta
g (x
0
) = c, g
t
(x
0
) = 0, si obt inem (f c)
t
(x
0
) = f
t
(x
0
) c +f (x
0
) 0 = c f
t
(x
0
) R.
iv) Am vazut la demonstrarea part ii ii) ca funct ia g este continua n x
0
; cum g (x
0
) ,= 0, utilizand
Teorema 3.3.8 rezulta existent a unei vecinatate U a punctului x
0
astfel ca x DU g (x) ,= 0. Pentru
orice x D U x
0
are loc
f
g
(x)
f
g
(x
0
)
x x
0
=
1
g (x) g (x
0
)
_
f (x) f (x
0
)
x x
0
g (x
0
) f (x
0
)
g (x) g (x
0
)
x x
0
_
si deducem
_
f
g
_
t
(x
0
) = lim
xx
0
f
g
(x)
f
g
(x
0
)
x x
0
= lim
xx
0
1
g (x) g (x
0
)
_
f (x) f (x
0
)
x x
0
g (x
0
) f (x
0
)
g (x) g (x
0
)
x x
0
_
=
1
(g

(x
0
))
2
[f
t
(x
0
) g (x
0
) f (x
0
) g
t
(x
0
)] R.
4.1.17. Exemplu. Utilizam iv) pentru a calcula (tg)
t
(x
0
) si (ctg)
t
(x
0
) .
i) Pentru x
0
R
_
(2k + 1)

2

k Z
_
avem
(tg)
t
(x
0
) =
_
sin
cos
_
t
(x
0
) =
(sin)
t
(x
0
) cos x
0
sinx
0
(cos)
t
(x
0
)
cos
2
x
0
=
cos (x
0
) cos x
0
sinx
0
(sinx
0
)
cos
2
x
0
=
cos
2
x
0
+ sin
2
x
0
cos
2
x
0
=
1
cos
2
x
0
.
ii) Pentru x
0
R k[ k Z avem
(ctg)
t
(x
0
) =
_
cos
sin
_
t
(x
0
) =
(cos)
t
(x
0
) sinx
0
cos x
0
(sin)
t
(x
0
)
sin
2
x
0
=
sin(x
0
) sinx
0
cos x
0
cos x
0
sin
2
x
0
=

_
sin
2
x
0
+ cos
2
x
0
_
sin
2
x
0
=
1
sin
2
x
0
.
4.1.18. Observat ie. Pornind de la teorema precedenta, din derivabilitatea funct iilor f
1
, f
2
, . . . , f
n
n punctul x
0
, prin induct ie matematica, se obt ine derivabilitatea n punctul x
0
a funct iei suma f
1
+f
2
+
. . . +f
n
, a funct iei produs f
1
f
2
. . . f
n
, cat si formulele de derivare
(f
1
+f
2
+. . . +f
n
)
t
(x
0
) = f
t
1
(x
0
) +f
t
2
(x
0
) +. . . +f
t
n
(x
0
),
(f
1
f
2
. . . f
n
)
t
(x
0
) =
n

k=1
(f
1
(x
0
) f
2
(x
0
) . . . f
k1
(x
0
) f
t
k
(x
0
) f
k+1
(x
0
) . . . f
n
(x
0
)) .
4.1.19. Teorema. Fie I, J R doua intervale si funct iile f : I J, g : J R. Daca funct ia
f este derivabila n punctul x
0
I si funct ia g este derivabila n punctul y
0
= f (x
0
) , atunci funct ia
compusa g f : I R este derivabila n punctul x
0
si are loc (g f)
t
(x
0
) = g
t
(f (x
0
)) f
t
(x
0
) .
Demonstrat ie.

In ipotezele teoremei, cum funct ia f este derivabila n punctul x
0
si funct ia g este
derivabila n punctul y
0
= f (x
0
) , avem
lim
xx
0
f (x) f (x
0
)
x x
0
= f
t
(x
0
) R, lim
yy
0
g (y) g (y
0
)
y y
0
= g
t
(y
0
) = g
t
(f (x
0
)) R.
4.1 Derivata funct iei ntr-un punct. Derivabilitatea funct iei ntr-un punct 155
Consideram funct ia p : J R, p (y) =
_
_
_
g (y) g (y
0
)
y y
0
, daca y J y
0

g
t
(y
0
) , daca y = y
0
.
Calculand
lim
yy
0
p (y) = lim
yy
0
g (y) g (y
0
)
y y
0
= g
t
(y
0
)
constatam egalitatea lim
yy
0
p (y) = p (y
0
), deci funct ia p este continua n punctul y
0
.
Din denit ia funct iei p explicitam g (y) g (y
0
) = p (y) (y y
0
) , y J y
0
si deducem
g (f (x)) g (f (x
0
)) = p (f (x)) (f (x) f (x
0
)) , x I.
Asadar pentru x I x
0
are loc
g (f (x)) g (f (x
0
))
x x
0
= p (f (x))
f (x) f (x
0
)
x x
0
. Cum funct ia f este
derivabila n punctul x
0
, rezulta ca ea este continua n x
0
, iar funct ia p ind continua n y
0
= f (x
0
) ,
deducem continuitatea funct iei p f n punctul x
0
. Prin urmare
lim
xx
0
p (f (x)) = p (f (x
0
)) = p (y
0
) = g
t
(y
0
) = g
t
(f (x
0
)) .
Utilizand si lim
xx
0
f (x) f (x
0
)
x x
0
= f
t
(x
0
) obt inem
(g f)
t
(x
0
) = lim
xx
0
(g f) (x) (g f) (x
0
)
x x
0
= lim
xx
0
g (f (x)) g (f (x
0
))
x x
0
= lim
xx
0
p (f (x))
f (x) f (x
0
)
x x
0
= g
t
(f (x
0
)) f
t
(x
0
) R.
Ment ionam ca existent a expresiei
g (f (x)) g (f (x
0
))
f (x) f (x
0
)

f (x) f (x
0
)
x x
0
nu este garantata pe o veci-
natate a punctului x
0
, deoarece egalitatea f (x) = f (x
0
) poate adevarata n unele puncte x ale
vecinatat ii. Din acest motiv n demonstrat ia teoremei am evitat utilizarea (de altfel tentanta) a ex-
presiei ment ionate.
4.1.20. Exemplu. i) Fie f : R R, f (x) = cos (x
n
) , g : R R, g (x) = cos
n
x cu n N

si e
x
0
R. Pe baza teoremei precedente obt inem f
t
(x
0
) = sin((x
0
)
n
) n(x
0
)
n1
= n(x
0
)
n1
sin((x
0
)
n
),
respectiv g
t
(x
0
) = n(cos x
0
)
n1
(sinx
0
) = n(cos x
0
)
n1
sinx
0
.
ii) Fie f : (0, +) R, f (x) = x
r
cu r R si e x
0
(0, +) . Exprimam
f (x) = x
r
= e
ln x
r
= e
r ln x
si aplicam teorema precedenta f
t
(x
0
) = e
r ln x
0
.
_
r
1
x
0
_
, deci obt inem
f
t
(x
0
) = (x
0
)
r
_
r
1
x
0
_
= r (x
0
)
r1
.
Urmatoarea teorema formuleaza condit ii suciente pentru derivabilitatea ntr-un punct a funct iei
inverse.
4.1.21. Teorema. Fie I, J R intervale. Fie funct ia f : I J bijectiva. Daca funct ia f
este derivabila n punctul x
0
I, f
t
(x
0
) ,= 0 si funct ia inversa f
1
: J I este continua n punctul
y
0
= f(x
0
), atunci funct ia f
1
este derivabila n punctul y
0
= f (x
0
) si are loc
_
f
1
_
t
(y
0
) =
1
f
t
(x
0
)
.
Demonstrat ie. Fie y J, y ,= y
0
= f(x
0
). Pe baza injectivitat ii funct iei f, cu privire la x = f
1
(y)
deducem x ,= x
0
. Pentru orice y J, y ,= y
0
are loc
f
1
(y) f
1
(y
0
)
y y
0
=
f
1
(f (x)) f
1
(f (x
0
))
f (x) f (x
0
)
=
x x
0
f (x) f (x
0
)
=
1
f (x) f (x
0
)
x x
0
,
156 4. Funct ii f : D R R derivabile
unde x = f
1
(y) . Cand y tinde la y
0
, din continuitatea funct iei f
1
n punctul y
0
obt inem ca f
1
(y) = x
tinde catre f
1
(y
0
) = x
0
. Utilizand lim
xx
0
f (x) f (x
0
)
x x
0
= f
t
(x
0
) ,= 0 obt inem
_
f
1
_
t
(y
0
) = lim
yy
0
f
1
(y) f
1
(y
0
)
y y
0
= lim
xx
0
1
f (x) f (x
0
)
x x
0
=
1
f
t
(x
0
)
.
4.1.22. Exemplu. i) Funct ia sin :
_

2
,

2
_
[1, 1] este continua si bijectiva. Inversa ei,
funct ia arcsin : [1, 1]
_

2
,

2
_
este continua. Pentru x
0

_

2
,

2
_
avem (sin)
t
(x
0
) = cos x
0
,= 0,
funct ia arcsin este continua n y
0
= sinx
0
deci, pe baza teoremei, funct ia arcsin este derivabila n punctul
y
0
= sinx
0
(1, 1) si (arcsin)
t
(y
0
) =
1
(sin)
t
(x
0
)
=
1
cos x
0
. T inand cont de cos x
0
=
_
1 sin
2
x
0
=
_
1 (y
0
)
2
, obt inem
(arcsin)
t
(y
0
) =
1
_
1 (y
0
)
2
, unde y
0
(1, 1) .
ii) Funct ia cos : [0, ] [1, 1] este continua si bijectiva. Inversa ei, funct ia arccos : [1, 1] [0, ]
este continua. Pentru x
0
(0, ) avem (cos)
t
(x
0
) = sinx
0
,= 0, funct ia arccos este continua n
y
0
= cos x
0
, deci, pe baza teoremei, funct ia arccos este derivabila n punctul y
0
= cos x
0
(1, 1) si
(arccos)
t
(y
0
) =
1
(cos)
t
(x
0
)
=
1
sinx
0
. T inand cont de sinx
0
=

1 cos
2
x
0
=
_
1 (y
0
)
2
obt inem
(arccos)
t
(y
0
) =
1
_
1 (y
0
)
2
, unde y
0
(1, 1) .
iii) Funct ia tg :
_

2
,

2
_
R este continua si bijectiva. Inversa ei, funct ia arctg : R
_

2
,

2
_
este continua. Pentru x
0

_

2
,

2
_
avem (tg)
t
(x
0
) =
1
cos
2
x
0
,= 0, funct ia arctg este continua n
y
0
= tgx
0
, deci, pe baza teoremei, funct ia arctg este derivabila n punctul y
0
= tgx
0
si (arctg)
t
(y
0
) =
1
(tg)
t
(x
0
)
= cos
2
x
0
. T inand cont de cos
2
x
0
=
1
1 +tg
2
x
0
=
1
1 + (y
0
)
2
obt inem
(arctg)
t
(y
0
) =
1
1 + (y
0
)
2
.
iv) Funct ia ctg : (0, ) R este continua si bijectiva. Inversa ei, funct ia arcctg : R (0, ) este
continua. Pentru x
0
(0, ) avem (ctg)
t
(x
0
) =
1
sin
2
x
0
,= 0, funct ia arcctg este continua n y
0
= ctgx
0
,
deci, pe baza teoremei, funct ia arcctg este derivabilan punctul y
0
= ctgx
0
si (arcctg)
t
(y
0
) =
1
(ctg)
t
(x
0
)
=
sin
2
x
0
. T inand cont de sin
2
x
0
=
1
1 + ctg
2
x
0
=
1
1 + (y
0
)
2
obt inem
(arcctg)
t
(y
0
) =
1
1 + (y
0
)
2
.
v) Funct ia ln : (0, +) R este continua si bijectiva. Inversa ei, funct ia exp : R (0, +) ,
exp(x) = e
x
, este continua. Pentru x
0
(0, +) avem (ln)
t
(x
0
) =
1
x
0
,= 0, funct ia exp este continua
n y
0
= lnx
0
, deci, pe baza teoremei, funct ia exp este derivabila n punctul y
0
= lnx
0
si (exp)
t
(y
0
) =
1
(ln)
t
(x
0
)
= x
0
. T inand cont de x
0
= e
ln x
0
= e
y
0
obt inem
(exp)
t
(y
0
) = e
y
o
.
4.2 Funct ii derivabile pe o mult ime 157
4.1.23. Teorema. Daca funct ia strict pozitiva f : D R R si funct ia g : D R sunt derivabile
n punctul x
0
D, atunci funct ia h : D R, h(x) = (f (x))
g(x)
este derivabila n x
0
si
h
t
(x
0
) = g (x
0
) (f (x
0
))
g(x
0
)1
f
t
(x
0
) + (f (x
0
))
g(x
0
)
(lnf (x
0
)) g
t
(x
0
) .
Demonstrat ie. Avem h(x) = e
ln(f(x))
g(x)
= e
g(x) ln f(x)
. Utilizand Teorema 4.1.19 obt inem
h
t
(x
0
) = e
g(x
0
) ln f(x
0
)

_
(g (x) lnf (x))
t

x=x
0
= (f (x
0
))
g(x
0
)

_
g
t
(x
0
) lnf (x
0
) +g (x
0
)
1
f (x
0
)
f
t
(x
0
)
_
si dupa efectuarea nmult irii obt inem formula din enunt ul teoremei.
4.2 Funct ii derivabile pe o mult ime
4.2.1. Denit ie. i) Spunem ca funct ia f : D R R este derivabila pe mult imea D
0
D,
D
0
nevida, daca ea este derivabila n ecare punct x
0
D
0
.
ii) Daca funct ia f este derivabila pe mult imea D
0
, atunci funct ia f
t
: D
0
R care ecarui x D
0
ataseaza numarul real f
t
(x) se numeste derivata funct iei f pe mult imea D
0
.
Daca f este derivabila pe mult imea de denit ie D, atunci spunem ca funct ia f este derivabila.
4.2.2. Exemplu. i) Fie n 3, 5, 7, . . . . Consideram funct ia f : R R, f (x) =
n

x. Pentru
x
0
R 0 avem
f
t
(x
0
) = lim
xx
0
f (x) f (x
0
)
x x
0
= lim
xx
0
n

x
n

x
0
x x
0
= lim
xx
0
_
n

x
n

x
0
_
_
n

x
n1
+
n

x
n2
x
0
+
n
_
x
n3
(x
0
)
2
+. . . +
n
_
x(x
0
)
n2
+
n
_
(x
0
)
n1
_
(x x
0
)
_
n

x
n1
+
n

x
n2
x
0
+
n
_
x
n3
(x
0
)
2
+. . . +
n
_
x(x
0
)
n2
+
n
_
(x
0
)
n1
_
= lim
xx
0
1
n

x
n1
+
n

x
n2
x
0
+
n
_
x
n3
(x
0
)
2
+. . . +
n
_
x(x
0
)
n2
+
n
_
(x
0
)
n1
=
1
n
n
_
(x
0
)
n1
R.
Asadar funct ia f este derivabila n ecare punct x
0
R 0 .
Pentru x
0
= 0 avem
f
t
(0) = lim
x0
f (x) f (x
0
)
x x
0
= lim
x0
n

x
n

0
x 0
= lim
x0
n

x
x
= lim
x0
1
n

x
n1
=
1
0
+
= + / R,
deci funct ia f nu este derivabila n punctul x
0
= 0.
Funct ia f
t
: R 0 R, f
t
(x) =
1
n
n
_
(x)
n1
este derivata funct iei f pe mult imea D
0
= R 0 .
ii) Reluand unele exemple din Paragraful 4.1 avem
(c)
t
= 0, cu c R, reprezentand derivata funct iei constante
(x
n
)
t
= nx
n1
, x R, unde n N

(x
r
)
t
= rx
r1
, x (0, +) , unde r R
(
n

x)
t
=
_
x
1
n
_
t
=
1
n
x
1
n
1
=
1
n

1
n

x
n1
, pentru
_
x (0, +) , daca n 2, 4, . . .
x R 0 , daca n 3, 5, . . .
(lnx)
t
=
1
x
, x (0, +)
(e
x
)
t
= e
x
, x R
(log
a
x)
t
=
_
lnx
lna
_
t
=
1
lna
(lnx)
t
=
1
lna

1
x
, x (0, +) , unde a (0, 1) (0, +)
158 4. Funct ii f : D R R derivabile
(a
x
)
t
= a
x
lna, x R, unde a (0, 1) (0, +)
(sinx)
t
= cos x, x R
(cos x)
t
= sinx, x R
(tgx)
t
=
1
cos
2
x
, x R
_
(2k + 1)

2

k Z
_
(ctgx)
t
=
1
sin
2
x
, x R k[ k Z
(arcsinx)
t
=
1

1 x
2
, x (1, 1)
(arccos x)
t
=
1

1 x
2
, x (1, 1)
(arctgx)
t
=
1
x
2
+ 1
, x R
(arcctgx)
t
=
1
x
2
+ 1
, x R
De multe ori relativ la o funct ie f : D R R ne intereseaza valori optime ale ei.
4.2.3. Denit ie. Consideram funct ia f : D R R. Fie D
0
D o mult ime nevida.
i) Punctul x
0
D se numeste punct de minim al funct iei f relativ la mult imea D
0
, daca
pentru orice x D
0
are loc f (x
0
) f (x) .
Punctul x
0
D se numeste punct de maxim al funct iei f relativ la mult imea D
0
, daca pentru
orice x D
0
are loc f (x
0
) f (x) .
ii) Punctul x
0
D se numeste punct de minim local al funct iei f relativ la mult imea D
0
, daca
exista o vecinatate U a punctului x
0
astfel ncat f (x
0
) f (x) , x D
0
U.
Punctul x
0
D se numeste punct de maxim local al funct iei f relativ la mult imea D
0
, daca
exista o vecinatate U a punctului x
0
astfel ncat f (x
0
) f (x) , x D
0
U.
Relativ la o mult ime data, punctele de minim, respectiv de maxim se numesc puncte de optim, iar
punctele de minim local, respectiv de maxim local se numesc puncte de optim local. Daca un punct
de optim (local) este relativ la D
0
= D adica la mult imea de denit ie a funct iei f, atunci se poate numi
simplu punct de optim (local), fara specicarea lui D
0
.
Este evident ca, relativ la o mult ime data, un punct de optim este totodata punct de optim local,
dar un punct de optim local nu neaparat este punct de optim. Urmatoarea teorema arata o condit ie
necesara pentru ca un punct din interiorul mult imii de denit ie a funct iei f : D R R, punct n care
f are derivata, sa e punct de optim local al funct iei relativ la D.
4.2.4. Teorema (Teorema lui Fermat) Consideram funct ia f : D R R si un punct x
0
D.
Daca
i) exista > 0 astfel ncat (x
0
, x
0
+) D ( x
0
este punct interior al mult imii D),
ii) funct ia f are derivata n punctul x
0
,
iii) punctul x
0
este punct de optim local al funct iei f relativ la D,
atunci f
t
(x
0
) = 0.
Demonstrat ie. Consideram ca punctul x
0
este punct de minim local al funct iei f relativ la D. Cazul
cand x
0
este punct de maxim local al funct iei f relativ la D se studiaza n mod asemanator. Din faptul
ca x
0
este punct de minim local al funct iei f relativ la D, t inand cont de i), deducem ca exista
1
> 0
astfel ncat f (x
0
) f (x) , x (x
0

1
, x
0
+
1
) . Asadar f (x) f (x
0
) 0, x (x
0

1
, x
0
+
1
) .
Deoarece funct ia f are derivata n punctul x
0
, au loc egalitat ile f
t
s
(x
0
) = f
t
d
(x
0
) = f
t
(x
0
) .
Pentru ecare x din intervalul (x
0

1
, x
0
) are loc x x
0
0,
f (x) f (x
0
)
x x
0
0, prin urmare
f
t
s
(x
0
) = lim
xx
0
x<x
0
f (x) f (x
0
)
x x
0
0.
Pentru ecare x din intervalul (x
0
, x
0
+
1
) are loc x x
0
0,
f (x) f (x
0
)
x x
0
0, prin urmare
f
t
d
(x
0
) = lim
xx
0
x>x
0
f (x) f (x
0
)
x x
0
0.
4.2 Funct ii derivabile pe o mult ime 159
Pe baza relat iilor f
t
s
(x
0
) 0, f
t
d
(x
0
) 0, f
t
s
(x
0
) = f
t
d
(x
0
) rezulta f
t
s
(x
0
) = f
t
d
(x
0
) = 0 si prin
urmare f
t
(x
0
) = 0.
Interpretare geometrica.

In ipotezele i) si ii) ale teoremei, daca punctul x
0
este punct de optim local
al funct iei f relativ la D, atunci tangenta n punctul (x
0
, f (x
0
)) la gracul funct iei f este paralela cu
axa Ox.
Punctele x
0
D
0
n care derivata unei funct ii f : D R R se anuleaza se numesc puncte
stat ionare (sau puncte critice) ale funct iei f relativ la D
0
. Daca un punct stat ionar este relativ la
D
0
= D, adica la mult imea de denit ie a funct iei f, atunci se poate numi simplu punct stat ionar, fara
specicarea lui D
0
.
Cu privire la o funct ie derivabila f : I R, unde I este interval, teorema lui Fermat ne spune ca
punctele de optim local ale funct iei f, interioare lui I, se gasesc printre punctele stat ionare ale funct iei.
Condit ia i), ca punctul x
0
sa e interior mult imii D, este o condit ie esent iala pentru valabilitatea
teoremei. Analizand funct ia f : [1, 1] R, f (x) = x, vedem ca punctul x
0
= 1 este punct de minim
al funct iei relativ la [1, 1] si f
t
(1) = 1 ,= 0. Punctul x
0
= 1 nu este n interiorul mult imii [1, 1] .
4.2.5. Observat ie. Condit ia f
t
(x
0
) = 0, necesara, n cadrul teoremei, pentru ca x
0
sa e punct de
optim local al funct iei f, nu este si sucienta.
4.2.6. Exemplu.

In cazul f : R R, f (x) = x
3
, x
0
= 0: derivata f
t
(x) = 3x
2
se anuleaza n
x
0
= 0, punct situat n interiorul mult imii de denit ie; punctul x
0
= 0 nu este punct de optim local
relativ R al funct iei f, deoarece n orice vecinatate U a lui x
0
= 0 exista x < 0 cu f (x) < 0 = f (0)
(asadar x
0
= 0 nu este punct de minim local pentru f), si exista x > 0 cu f (x) > 0 = f (0) (asadar
x
0
= 0 nu este punct de maxim local pentru f).
Urmatoarea teorema o demonstram utilizand teorema lui Weierstrass si teorema lui Fermat.
4.2.7. Teorema (Teorema lui Rolle) Fie a, b R, a < b si f : [a, b] R o funct ie. Daca
i) funct ia f este continua pe [a, b],
ii) funct ia f este derivabila pe (a, b),
iii) f (a) = f (b),
atunci exista cel put in un punct c (a, b) astfel ncat f
t
(c) = 0.
Demonstrat ie. Cazul 1. Funct ia f este o funct ie constanta.

In acest caz f
t
(x) = 0, x (a, b) si
deci c poate orice punct din (a, b).
Cazul 2. Funct ia f nu este constanta pe [a, b] . Deoarece f este continua pe un interval nchis,
pe [a, b] , conform teoremei lui Weierstrass, exista x

[a, b] unde f si realizeaza minimul relativ la


[a, b] si exista x

[a, b] unde f si realizeaza maximul relativ la [a, b] . Deoarece f nu este constanta


pe [a, b] , minimul lui f si maximul lui f relativ la [a, b] sunt doua valori diferite f (x

) ,= f (x

) . Din
f (x

) ,= f (x

) si f (a) = f (b) deducem ca cel put in unul dintre punctele x

, x

nu este situat n capatul


a sau b al intervalului [a, b] . Asadar cel put in un punct de optim relativ la [a, b] al funct iei f este situat
n (a, b) , adica n interiorul mult imii [a, b] ; notam un asemenea punct cu c. Pe baza teoremei lui Fermat
rezulta ca f
t
(c) = 0.
Interpretare geometrica. Consideram extremitat ile gracului funct iei f, adica punctele A(a, f(a)) si
B(b, f(b)).
Condit ia ii) geometric nsemna ca gracul funct iei f admite tangenta n ecare punct al gracului
cu except ia, eventual, al extremitat ilor A(a, f(a)) si B(b, f(b)).
Condit ia iii) f (a) = f (b) geometric nseamna ca dreapta AB este paralela cu axa Ox.
Egalitatea f
t
(c) = 0 geometric nsemna ca dreapta tangenta n punctul (c, f(c)) la gracul funct iei
f este paralela cu axa Ox.
Asadar teorema lui Rolle arma ca daca gracul funct iei continue f : [a, b] R admite tangenta
n ecare punct al gracului, exceptand, eventual, extremitat ile A(a, f(a)), B(b, f(b)), si daca dreapta
AB (care uneste cele doua extremitat i) este paralela cu axa Ox, atunci gracul are cel put in un punct
(c, f (c)) diferit de extremitat ile sale A(a, f(a)) si B(b, f(b)), n care tangenta dusa la grac este paralela
cu axa Ox (si implicit este paralela cu dreapta AB).
Urmatoarea teorema este o consecint a directa a teoremei lui Rolle.
4.2.8. Teorema.

Intre oricare doua zerouri consecutive ale unei funct ii derivabile pe un interval
n mod necesar se gaseste cel put in un zero al derivatei funct iei.
Urmatoarele doua teoreme se numesc teoreme de medie.
160 4. Funct ii f : D R R derivabile
4.2.9. Teorema (Teorema lui Cauchy)
Fie a, b R, a < b si f, g : [a, b] R doua funct ii. Daca
i) funct iile f si g sunt continue pe [a, b],
ii) funct iile f si g sunt derivabile pe (a, b),
iii) g
t
(x) ,= 0, x (a, b),
atunci exista cel put in un punct c (a, b) astfel ncat
f
t
(c)
g
t
(c)
=
f (b) f (a)
g (b) g (a)
.
Demonstrat ie. Aratam ca n ipotezele i), ii), iii) ale teoremei are loc g (b) g (a) ,= 0. Rat ionam
prin reducere la absurd. Presupunem ca g (b) g (a) = 0; atunci, avand funct ia g continua pe [a, b] ,
derivabila pe (a, b) si cu g (a) = g (b) , pe baza teoremei lui Rolle ar rezulta existent a unui c (a, b) cu
g
t
(c) = 0, n contradict ie cu ipoteza iii).
Consideram funct ia h : [a, b] R , h(x) = f(x) +g (x) unde =
f (b) f (a)
g (b) g (a)
. Numarul a fost
ales astfel ca sa avem h(a) = h(b) . Din proprietat ile de continuitate pe [a, b], respectiv de derivabilitate
pe (a, b) ale funct iilor f si g rezulta ca funct ia h este continua pe [a, b] si este derivabila pe (a, b) . Aplicand
teorema lui Rolle funct iei h rezulta ca exista cel put in un punct c (a, b) astfel ncat h
t
(c) = 0. Ultima
egalitate se scrie
f
t
(c) +g
t
(c) = 0
f
t
(c)
g
t
(c)
=
f
t
(c)
g
t
(c)
=
f (b) f (a)
g (b) g (a)
.

In cazul particular al funct iei g : [a, b] R, g (x) = x, din teorema lui Cauchy se obt ine urmatorul
rezultat important.
4.2.10. Teorema (Teorema lui Lagrange) Fie a, b R, a < b si funct ia f : [a, b] R. Daca
i) funct ia f este continua pe [a, b],
ii) funct ia f este derivabila pe (a, b),
atunci exista cel put in un punct c (a, b) astfel ncat
f
t
(c) =
f (b) f (a)
b a
.
Demonstrat ie. Consideram funct ia g : [a, b] R, g (x) = x. Funct ia g este continua pe [a, b] si
este derivabila pe (a, b) . Evident g
t
(x) = 1, x (a, b), deci g
t
(x) ,= 0, x (a, b) . Pe baza teoremei lui
Cauchy exista cel put in un punct c (a, b) astfel ncat
f
t
(c)
g
t
(c)
=
f (b) f (a)
g (b) g (a)
. Ultima egalitate se scrie
f
t
(c)
1
=
f (b) f (a)
b a
deoarece g
t
(c) = 1 si g (b) g (a) = b a.
Interpretare geometrica. Consideram punctele A(a, f(a)) si B(b, f(b)) ale gracului funct iei f.
Dreapta AB are panta (coecientul unghiular)
f (b) f (a)
b a
. Tangenta n punctul (c, f(c)) la gracul
funct iei f are panta (coecientul unghiular) f
t
(c) .
Teorema lui Lagrange arma ca daca gracul funct iei continue f : [a, b] R admite tangenta n
ecare punct, exceptand, eventual, extremitat ile A(a, f(a)), B(b, f(b)), atunci cel put in ntr-un punct
(c, f (c)) al gracului, diferit de extremitat ile A(a, f(a)) si B(b, f(b)), tangenta la gracul funct iei este
paralela cu dreapta AB.

In continuare discutam patru teoreme care sunt consecint e importante ale teoremei lui Lagrange.
Teorema care urmeaza se utilizeaza la studiul existent ei derivatei ntr-un punct si la studiul deriv-
abilitat ii ntr-un punct a unei funct ii.
4.2.11. Teorema. Fie I R un interval, x
0
I, funct ia f : I R derivabila pe I x
0
si
continua n x
0
.
i) Daca exista lim
xx
0
f
t
(x) , atunci funct ia f are derivata n punctul x
0
si f
t
(x
0
) = lim
xx
0
f
t
(x) .
ii) Daca exista lim
xx
0
f
t
(x) si lim
xx
0
f
t
(x) R, atunci funct ia f este derivabila n punctul x
0
,
f
t
(x
0
) = lim
xx
0
f
t
(x) si derivata f
t
: I R este continua n punctul x
0
.
4.2 Funct ii derivabile pe o mult ime 161
Demonstrat ie. i) Notam lim
xx
0
f
t
(x) = l. Aratam ca lim
xx
0
f (x) f (x
0
)
x x
0
= l.
Fie (x
n
)
n1
un sir arbitrar de numere reale din I x
0
convergent catre x
0
. Pentru ecare
n 1, 2, . . . consideram intervalul [a
n
, b
n
] =
_
[x
n
, x
0
] , daca x
n
< x
0
[x
0
, x
n
] , daca x
0
< x
n
.
Deoarece f este continua
pe I si derivabila pe I x
0
, rezulta ca f este continua pe [a
n
, b
n
] si este derivabila pe (a
n
, b
n
) .
Aplicand teorema lui Lagrange funct iei f pe intervalul [a
n
, b
n
] xam un c
n
(a
n
, b
n
) pentru care
f
t
(c
n
) =
f (b
n
) f (a
n
)
b
n
a
n
. Acest c
n
este situat strict ntre x
n
si x
0
si are loc f
t
(c
n
) =
f (x
n
) f (x
0
)
x
n
x
0
.
Deoarece [c
n
x
0
[ < [x
n
x
0
[ , n N

si lim
n
x
n
= x
0
, obt inem lim
n
c
n
= x
0
.
Din faptul ca sirul (c
n
)
n1
, format cu numere reale din I x
0
, este convergent catre x
0
si
lim
xx
0
f
t
(x) = l rezulta ca lim
n
f
t
(c
n
) = l.

Inlocuind aici f
t
(c
n
) =
f (x
n
) f (x
0
)
x
n
x
0
, obt inem
lim
n
f (x
n
) f (x
0
)
x
n
x
0
= l.
Cum sirul (x
n
)
n1
de numere reale din I x
0
, convergent catre x
0
, a fost arbitrar, deducem ca
lim
xx
0
f (x) f (x
0
)
x x
0
= l. Ultima egalitate nsemna ca f
t
(x
0
) = l, asadar are loc f
t
(x
0
) = lim
xx
0
f
t
(x) .
ii) Daca are loc i) si n plus lim
xx
0
f
t
(x) R, atunci pe baza egalitat ii f
t
(x
0
) = lim
xx
0
f
t
(x) obt inem
f
t
(x
0
) R si deci funct ia f este derivabila n x
0
. Astfel f are derivata pe I, f
t
: I R. Pe baza egalitat ii
lim
xx
0
f
t
(x) = f
t
(x
0
) funct ia f
t
este continua n x
0
.
4.2.12. Observat ie. Continuitatea funct iei f n punctul x
0
este esent iala pentru valabilitatea
teoremei precedente.
4.2.13. Exemplu. Pentru funct ia f : R R, f (x) =
_
x 1, daca x (, 0)
x, daca x [0, +)
avem
f
t
(x) = 1, x (, 0) (0, +) . Exista lim
x0
f
t
(x) = lim
x0
1 = 1. Funct ia f nu este continua n x
0
= 0.
Funct ia f nu are derivata n x
0
= 0 deoarece limitele
lim
x0
x<0
f (x) f (0)
x 0
= lim
x0
x<0
x 1
x
= +, lim
x0
x>0
f (x) f (0)
x 0
= lim
x0
x>0
x
x
= 1,
ind diferite rezulta ca limita f
t
(0) = lim
x0
f (x) f (0)
x 0
nu exista.
4.2.14. Observat ie. Existent a limitei lim
xx
0
f
t
(x) nu este necesara pentru derivabilitatea n punctul
x
0
a funct iei f .
4.2.15. Exemplu. Funct ia f : R R, f (x) =
_
x
2
sin
1
x
, daca x (, 0) (0, +)
0, daca x = 0
este
derivabila n x
0
= 0, f
t
(0) = 0, dar derivata
f
t
: R R, f
t
(x) =
_
2xsin
1
x
cos
1
x
, daca x (, 0) (0, +)
0, daca x = 0
nu are limita n x
0
= 0.
4.2.16. Observat ie. Pe baza Teoremei 4.2.11 din existent a limitei la stanga n punctul x
0
a
derivatei f
t
(x) si din continuitatea la stanga n punctul x
0
a funct iei f rezulta ca n punctul x
0
funct ia
f are derivata la stanga si f
t
s
(x
0
) = lim
xx
0
x<x
0
f
t
(x) . Un rezultat similar este valabil cu privire la f
t
d
(x
0
) .
Teorema urmatoare se utilizeaza la studiul monotoniei funct iilor derivabile pe un interval.
4.2.17. Teorema. Fie I R un interval si f : I R o funct ie derivabila.
i) Funct ia f este crescatoare pe I daca si numai daca f
t
(x) 0, x I.
ii) Funct ia f este descrescatoare pe I daca si numai daca f
t
(x) 0, x I.
iii) Daca f
t
(x) > 0, x I, atunci funct ia f este strict crescatoare pe I.
162 4. Funct ii f : D R R derivabile
iv) Daca f
t
(x) < 0, x I atunci funct ia f este strict descrescatoare pe I.
Demonstrat ie. i) Aratam ca daca funct ia f este crescatoare pe I atunci f
t
(x) 0, x I.
Fie funct ia f crescatoare pe I. Consideram x
0
I arbitrar. Deoarece funct ia f este crescatoare,
pentru x I, x < x
0
rezulta f (x) f (x
0
) si
f (x) f (x
0
)
x x
0
0, iar pentru x I, x > x
0
rezulta
f (x) f (x
0
) si
f (x) f (x
0
)
x x
0
0. Asadar pentru orice x I x
0
are loc
f (x) f (x
0
)
x x
0
0 si
deducem ca lim
xx
0
f (x) f (x
0
)
x x
0
0. Pe baza denit iei derivatei lim
xx
0
f (x) f (x
0
)
x x
0
= f
t
(x
0
) obt inem
f
t
(x
0
) 0. Cum x
0
I a fost arbitrar, deducem f
t
(x) 0, x I.
Aratam ca daca f
t
(x) 0, x I, atunci funct ia f este crescatoare pe I. Consideram x
1
, x
2
I,
x
1
< x
2
, arbitrare. Pe baza teoremei lui Lagrange exista c (x
1
, x
2
) astfel ncat f
t
(c) =
f (x
2
) f (x
1
)
x
2
x
1
.
Din egalitatea f (x
2
) f (x
1
) = f
t
(c) (x
2
x
1
) , stiind ca f
t
(c) 0 si x
2
x
1
> 0, obt inem
f (x
2
) f (x
1
) 0, adica f (x
1
) f (x
2
) . Prin urmare avem implicat ia
x
1
, x
2
I, x
1
< x
2
f (x
1
) f (x
2
) ,
deci funct ia f este crescatoare pe I.
ii) Rat ionam n mod asemanator ca si la partea i).
iii) Fie x
1
, x
2
I, x
1
< x
2
, arbitrare. Pe baza teoremei lui Lagrange exista c (x
1
, x
2
) astfel
ncat f
t
(c) =
f (x
2
) f (x
1
)
x
2
x
1
. Din egalitatea f (x
2
) f (x
1
) = f
t
(c) (x
2
x
1
) , stiind ca f
t
(c) > 0 si
x
2
x
1
> 0, obt inem f (x
2
) f (x
1
) > 0, adica f (x
1
) < f (x
2
) . Avem implicat ia
x
1
, x
2
I, x
1
< x
2
f (x
1
) < f (x
2
) ,
deci funct ia f este strict crescatoare pe I.
iv) Rat ionam n mod asemanator ca si la partea iii).
4.2.18. Exemplu. Fie n N

. Funct ia f : R R, f (x) = x
2n
este derivabila pe R si
f
t
(x) = 2nx
2n1
, x R. Pentru x (, 0] avem f
t
(x) 0, deci f este descrescatoare pe inter-
valul (, 0]. Pentru x [0, ) avem f
t
(x) 0, deci f este crescatoare pe intervalul [0, ).
4.2.19. Observat ie. Reciproca armat iei iii) nu are loc. Reciproca armat iei iv) nu are loc.
Exista funct ii derivabile pe un interval, strict monotone, ale caror derivata se anuleaza n unele puncte
ale intervalului.
4.2.20. Exemplu. Funct ia f : R R , f (x) = x
3
este strict crescatoare si derivata ei
f
t
: R R, f (x) = 2x
2
se anuleaza n punctul x
0
= 0. Pentru a arata ca f este strict crescatoare
consideram x
1
, x
2
R, x
1
< x
2
si aratam ca f (x
1
) < f (x
2
) . Avem
f (x
1
) f (x
2
) = (x
1
)
3
(x
2
)
3
= (x
1
x
2
)
_
(x
1
)
2
+x
1
x
2
+ (x
2
)
2
_
< 0,
deoarece x
1
x
2
< 0 si (x
1
)
2
+x
1
x
2
+ (x
2
)
2
=
_
x
1
+
x
2
2
_
2
+
3
4
(x
2
)
2
> 0.
Urmatoarea teorema o completeaza pe cea precedenta.
4.2.21. Teorema. Fie I R un interval si f : I R o funct ie derivabila pe I. Funct ia f este
constanta pe I daca si numai daca f
t
(x) = 0, oricare ar x I.
Demonstrat ie. Daca funct ia f este constanta pe I, f(x) = k, x I unde k R, atunci
f
t
(x) = (k)
t
= 0, x I.
Consideram acum ca are loc f
t
(x) = 0, x I si aratam ca f este constanta. Fie x
0
I xat. Consideram
x I, x ,= x
0
arbitrar. Aplicam teorema lui Lagrange funct iei f pe intervalul
[a, b] =
_
[x, x
0
] , daca x < x
0
[x
0
, x] , daca x
0
< x.
Obt inem ca exista c situat strict ntre x si x
0
astfel ncat
f
t
(c) =
f (x) f (x
0
)
x x
0
. De aici, doarece f
t
(c) = 0, obt inem ca f (x) f (x
0
) = 0 adica f (x) = f (x
0
) .
4.2 Funct ii derivabile pe o mult ime 163
Cum x I, x ,= x
0
a fost arbitrar, egalitatea f (x) = f (x
0
) are loc pentru ecare x I, deci funct ia f
este constanta pe I.
Urmatoarea teorema se refera la funct ii derivabile pe un interval avand derivatele egale pe acel
interval.
4.2.22. Teorema. Fie I R un interval si f, g : I R doua funct ii derivabile pe I. Daca f
t
= g
t
pe I, atunci funct ia f g este constanta pe I.
Demonstrat ie. Consideram funct ia h = f g, adica h : IR, h(x) = f (x) g (x) . Deoarece f si
g sunt derivabile pe I, deducem ca h este derivabila pe I si h
t
(x) = f
t
(x) g
t
(x) , x I. Cum f
t
si g
t
sunt egale pe I, f
t
(x) = g
t
(x) , x I, rezulta ca h
t
(x) = 0, x I. Pe baza teoremei precedente funct ia
h este constanta pe I. Dar h = f g, asadar funct ia f g este constanta pe I.
Exista funct ii f derivabile pe un interval avand derivata f
t
fara proprietatea de continuitate pe
intervalul respectiv.
4.2.23. Exemplu. Funct ia f : R R, f (x) =
_
x
2
sin
1
x
, daca x (, 0) (0, +)
0, daca x = 0
este
derivabila pe R, f
t
: R R, f
t
(x) =
_
2xsin
1
x
cos
1
x
, daca x (, 0) (0, +)
0, daca x = 0.
Funct ia f
t
nu
are limita n x
0
= 0, prin urmare f
t
nu este continua n x
0
= 0 si f
t
nu este continua pe R.
Urmatoarea teorema ne arata ca derivata unei funct ii derivabile pe un interval are proprietatea
lui Darboux pe acel interval.

In demonstrat ia teoremei utilizam teorema lui Weierstrass si teorema lui
Fermat.
4.2.24. Teorema (Teorema lui Darboux) Fie I R un interval si f : I R o funct ie derivabila
pe I. Atunci derivata funct iei f, funct ia f
t
: I R, are proprietatea lui Darboux pe I.
Demonstrat ie. Daca f
t
: IR este funct ie constanta, atunci evident f
t
are proprietatea lui
Darboux pe I. Daca f
t
nu este funct ie constanta pe I, atunci e x
1
, x
2
I, x
1
< x
2
cu f
t
(x
1
) ,= f
t
(x
2
) .
Considerand arbitrar y
0
cuprins strict ntre f
t
(x
1
) si f
t
(x
2
) , avem de aratat existent a unui c (x
1
, x
2
)
cu proprietatea f
t
(c) = y
0
.
Denim funct ia g : IR, g(x) = f (x) y
0
x. Din faptul ca f este derivabila pe I, rezulta ca g este
derivabila pe I si g
t
: IR, g
t
(x) = f
t
(x) y
0
. Deci g este derivabila pe [x
1
, x
2
] . Prin urmare g este
continua pe intervalul nchis [x
1
, x
2
] . Aplicand teorema lui Weierstrass obt inem ca exista x

[x
1
, x
2
]
n care g si realizeaza minimul relativ la [x
1
, x
2
] . Aratam ca x

(x
1
, x
2
) . Pe baza teoremei lui Fermat
rezulta ca g
t
(x

) = 0. Aceasta egalitate se scrie f


t
(x

)y
0
= 0, asadar exista c (x
1
, x
2
) cu proprietatea
f
t
(c) = y
0
, de exemplu c = x

.
Sa aratam ca x

,= x
1
si x

,= x
2
. Avand f
t
(x
1
) ,= f
t
(x
2
) , nseamna ca f
t
(x
1
) < f
t
(x
2
) sau
f
t
(x
1
) > f
t
(x
2
) . Consideram primul caz.

In al doilea caz se rat ioneaza n mod asemanator. Deoarece
f
t
(x
1
) < y
0
< f
t
(x
2
) si g
t
(x) = f
t
(x) y
0
avem g
t
(x
1
) < 0, g
t
(x
2
) > 0.
Din g
t
(x
1
) < 0 si g
t
(x
1
) = lim
xx
1
g (x) g (x
1
)
x x
1
deducem ca exista
1
> 0 astfel ncat
g (x) g (x
1
)
x x
1
<
0, x (x
1
, x
1
+
1
) ; rezulta ca g (x) < g (x
1
) , x (x
1
, x
1
+
1
) , inegalitate ce ne spune ca funct ia g si
realizeaza minimul relativ la [x
1
, x
2
] ntr-un punct diferit de x
1
, asadar x

,= x
1
.
Din g
t
(x
2
) > 0 si g
t
(x
2
) = lim
xx
2
g (x) g (x
2
)
x x
2
deducem ca exista
2
> 0 astfel ncat
g (x) g (x
2
)
x x
2
>
0, x (x
2

2
, x
2
) ; rezulta ca g (x) < g (x
2
) , x (x
2

2
, x
2
) , inegalitate ce ne spune ca g si
realizeaza minimul relativ la [x
1
, x
2
] ntr-un punct diferit de x
2
, asadar x

,= x
2
.
Urmatoarele doua teoreme de utilitate practica sunt consecint e imediate ale teoremei lui Darboux
n combinat ie cu Teorema 4.2.17, si respectiv Teorema 3.4.26.
4.2.25. Teorema. Fie I R un interval si f : I R o funct ie derivabila pe I.
i) Daca x
1
, x
2
I, x
1
< x
2
si f
t
(x
1
) f
t
(x
2
) < 0, atunci exista c (x
1
, x
2
) cu f
t
(c) = 0.
ii) Daca f
t
nu se anuleaza n nici un punct al intervalului I, atunci f
t
pastreaza acelasi semn pe I,
iar f este strict monotona pe I.
4.2.26. Teorema. Fie I R un interval si f : I R o funct ie derivabila pe I. Atunci derivata
funct iei f, funct ia f
t
: I R poate sa aiba puncte de discontinuitate numai de spet a a doua.
Urmatoarele doua teoreme se utilizeaza la calculul limitelor unor fract ii.
164 4. Funct ii f : D R R derivabile
4.2.27. Teorema (Cauchy) Fie I R un interval si f, g : I R doua funct ii derivabile n punctul
x
0
I. Daca
i) f(x
0
) = g (x
0
) = 0,
ii) g
t
(x
0
) ,= 0,
atunci exista o vecinatate U a punctului x
0
astfel ncat g
t
(x) ,= 0, x U x
0
si exista limita
lim
xx
0
f (x)
g (x)
=
f
t
(x
0
)
g
t
(x
0
)
.
Demonstrat ie. Din g
t
(x
0
) = lim
xx
0
g (x) g (x
0
)
x x
0
,= 0 rezulta ca exista o vecinatate U a punctului
x
0
astfel ncat
g (x) g (x
0
)
x x
0
,= 0, x U x
0
. Rezulta ca g (x) ,= g (x
0
) = 0, x U x
0
.
Pentru x U x
0
, t inand cont ca f(x
0
) = g (x
0
) = 0 si g (x) g (x
0
) ,= 0, exprimam fract ia
f (x)
g (x)
astfel
f (x)
g (x)
=
f (x) f (x
0
)
g (x) g (x
0
)
=
f (x) f (x
0
)
x x
0

x x
0
g (x) g (x
0
)
. Din
f (x)
g (x)
=
f (x) f (x
0
)
x x
0

x x
0
g (x) g (x
0
)
, x U x
0
,
t inand cont ca lim
xx
0
f (x) f (x
0
)
x x
0
= f
t
(x
0
) si lim
xx
0
g (x) g (x
0
)
x x
0
= g
t
(x
0
) ,= 0, rezulta ca
lim
xx
0
f (x)
g (x)
= f
t
(x
0
)
1
g
t
(x
0
)
.
4.2.28. Teorema (Teorema lui lHospital) Fie I R un interval, x
0
R , + un punct
de acumulare pentru I si f, g : I x
0
R doua funct ii. Daca
i) funct iile f, g sunt derivabile pe I x
0
,
ii) g
t
(x) ,= 0, x I x
0
,
iii) exista lim
xx
0
f
t
(x)
g
t
(x)
= L R , +,
iv) lim
xx
0
f (x) = lim
xx
0
g (x) = 0,
atunci exista lim
xx
0
f (x)
g (x)
si lim
xx
0
f (x)
g (x)
= L.
Demonstrat ie. Consideram x
0
R. Denim funct iile F, G : I R,
F (x) =
_
f (x) , daca x I x
0

0, daca x = x
0
si G(x) =
_
g (x) , daca x I x
0

0, daca x = x
0
.
Avem F
t
(x) = f
t
(x) , G
t
(x) = g
t
(x) ,= 0, x I x
0
. Funct iile F, G sunt derivabile pe I x
0
. Din
lim
xx
0
F (x) = lim
xx
0
f (x) = 0 = F (x
0
), respectiv lim
xx
0
G(x) = lim
xx
0
g (x) = 0 = G(x
0
), rezulta ca funct iile
F si G sunt continue n punctul x
0
.
Fie x I x
0
arbitrar. Pe intervalul [x, x
0
] sau [x
0
, x] , dupa cum x < x
0
, respectiv x > x
0
, sunt
ndeplinite condit iile teoremei lui Cauchy cu privire la funct iile F si G, deci exista c
x
cuprins strict ntre
x si x
0
astfel ncat
F (x) F (x
0
)
G(x) G(x
0
)
=
F
t
(c
x
)
G
t
(c
x
)
. Aceasta egalitate, stiind ca
F (x) F (x
0
)
G(x) G(x
0
)
=
f (x) 0
g (x) 0
=
f (x)
g (x)
si
F
t
(c
x
)
G
t
(c
x
)
=
f
t
(c
x
)
g
t
(c
x
)
,
se scrie
f (x)
g (x)
=
f
t
(c
x
)
g
t
(c
x
)
. Daca x x
0
, atunci c
x
x
0
(deoarece c
x
este situat strict ntre x si x
0
),
astfel din iii) si egalitatea
f (x)
g (x)
=
f
t
(c
x
)
g
t
(c
x
)
, deducem lim
xx
0
f (x)
g (x)
= lim
c
x
x
0
f
t
(c
x
)
g
t
(c
x
)
= lim
xx
0
f
t
(x)
g
t
(x)
= L. Daca
4.3 Derivate de ordin superior 165
x
0
= sau x
0
= + pentru a demonstra concluzia teoremei se poate proceda la schimbarea de
variabila x =
1
t
si atunci x x
0
revine la t 0.
4.2.29. Exemplu. Ca aplicat ie, utilizand teorema lui lHospital, calculam limita sirului
a
n
= n(
n

n 1). Avem
lim
n
n
_
n

n 1
_
= lim
n
n
_
_
n
1
n
1
_
_
= lim
x+
x
_
_
x
1
x
1
_
_
= lim
x+
x
1
x
1
1
x
(
0
0
)
= lim
x+
_
_
x
1
x
1
_
_
t
_
1
x
_
t
= lim
x+
1
x
x
1
x
1
+x
1
x
lnx
1
x
2

1
x
2
= lim
x+
x
1
x
(lnx 1) = +(+1) = +.
Am t inut cont ca lim
x+
x
1
x
= lim
x+
e
ln

x
1
x

= lim
x+
e
1
x
ln x
= e
0
= 1, deoarece utilizand teorema lui
lHospital lim
x+
1
x
lnx = lim
x+
lnx
x
(

)
= lim
x+
(lnx)
t
(x)
t
= lim
x+
1
x

1
x
2
= lim
x+
(x) = 0.
4.2.30. Observat ie. Poate exista lim
xx
0
f (x)
g (x)
fara ca limita lim
xx
0
f
t
(x)
g
t
(x)
sa existe.
4.2.31. Exemplu.

In cazul funct iilor f : R R, f (x) =
_
x
2
sin
1
x
, daca x (, 0) (0, +)
0, daca x = 0
si g : R R, g (x) = x, avem lim
x0
f (x)
g (x)
= lim
x0
xsin
1
x
= 0 si lim
x0
f
t
(x)
g
t
(x)
= lim
x0
2xsin
1
x
cos
1
x
1
nu exista.
4.3 Derivate de ordin superior
Denit ia 4.3.1. Fie funct ia f : D R R si x
0
D, x
0
punct de acumulare pentru D.
i) Spunem ca funct ia f este de doua ori derivabila n punctul x
0
daca exista o vecinatate U
a punctului x
0
astfel ncat funct ia f este derivabila pe mult imea D U si daca funct ia f
t
: D U R
este derivabila n punctul x
0
. Derivata n punctul x
0
a funct iei f
t
: D U R se numeste derivata de
ordinul doi n punctul x
0
a funct iei f si se noteaza f
tt
(x
0
) sau f
(2)
(x
0
) sau
d
2
f
dx
2
(x
0
) .
ii) Daca funct ia f este de doua ori derivabila n ecare punct x D
0
, unde D
0
D, D
0
mult ime
nevida, atunci spunem ca funct ia f este de doua ori derivabila pe mult imea D
0
.

In acest caz funct ia care
ecarui x D
0
ataseaza f
tt
(x) se numeste derivata de ordinul doi pe mult imea D
0
a funct iei f si
se noteaza f
tt
: D
0
R, pe scurt f
tt
sau f
(2)
sau
d
2
f
dx
2
.
iii) Fie n 2, 3, . . . . Spunem ca funct ia f este de n ori derivabila n punctul x
0
daca exista o
vecinatate U a punctului x
0
astfel ncat funct ia f este de n 1 ori derivabila pe mult imea DU si daca
derivata de ordinul n 1 pe mult imea D U a funct iei f, notata f
(n1)
: D U R, este derivabila n
punctul x
0
. Derivata n punctul x
0
a funct iei f
(n1)
: DU R se numeste derivata de ordinul n n
punctul x
0
a funct iei f si se noteaza f
(n)
(x
0
) sau
d
n
f
dx
n
(x
0
) .
iv) Fie n 2, 3, . . . . Daca funct ia f este de n ori derivabilan ecare punct x D
0
unde D
0
D, D
0
mult ime nevida, atunci spunem ca funct ia f este de n ori derivabila pe mult imea D
0
.

In acest caz funct ia


166 4. Funct ii f : D R R derivabile
care ecarui x D
0
ataseaza f
(n)
(x) se numeste derivata de ordinul n pe mult imea D
0
a funct iei
f si se noteaza f
(n)
: D
0
R, pe scurt f
(n)
sau
d
n
f
dx
n
.
v) Prin convent ie consideram f
(0)
= f.
vi) Daca funct ia f pentru orice n N este de n ori derivabila pe mult imea D
0
D, atunci spunem ca
funct ia f este ori de cate ori derivabila pe mult imea D
0
sau ca funct ia f este indenit derivabila
pe mult imea D
0
.
vii) Spunem ca funct ia f : D R R este de n ori derivabila daca funct ia f este de n ori derivabila
n ecare punct x
0
D.
Urmatoarea teorema referitoare la derivabilitatea de ordinul n ntr-un punct a funct iei suma si a
funct iei produs se obt ine prin induct ie matematica pornind de la i) respectiv ii) din Teorema 4.1.16.
4.3.2. Teorema. Fie D R si x
0
D un punct de acumulare pentru mult imea D. Fie n N

xat.
i) Daca funct iile f, g : D R sunt de n ori derivabile n punctul x
0
, atunci funct ia suma f +g este
de n ori derivabila n punctul x
0
si are loc
(f +g)
(n)
(x
0
) = f
(n)
(x
0
) +g
(n)
(x
0
) .
ii) (Leibniz) Daca funct iile f, g : D R sunt de n ori derivabile n punctul x
0
, atunci funct ia produs
f g este de n ori derivabila n punctul x
0
si are loc egalitatea
(f g)
(n
(x
0
) = f
(n)
(x
0
) g (x
0
) +C
1
n
f
(n1)
(x
0
) g
t
(x
0
) +C
2
n
f
(n2)
(x
0
) g
(2)
(x
0
)
+ . . . +C
k
n
f
(nk)
(x
0
) g
(k)
(x
0
) +. . . +C
n
n
f (x
0
) g
(n)
(x
0
) .
iii) Daca funct ia f : D R este de n ori derivabila n punctul x
0
atunci funct ia c f, unde c R,
este de n ori derivabila n punctul x
0
si are loc
(c f)
(n)
(x
0
) = c f
(n)
(x
0
) .
4.3.3. Exemplu. i) Pentru a R 0 , b R, prin derivari succesive si induct ie matematica rezulta
_
1
ax +b
_
(n)
= (1)
n
a
n
n!
1
(ax +b)
n+1
, x R
_

b
a
_
, n N
(ln(ax +b))
(n)
= (1)
n1
a
n
(n 1)!
1
(ax +b)
n
, x R
_

b
a
_
, n N

_
e
ax+b
_
(n)
= a
n
e
ax+b
, x R, n N
(sin(ax +b))
(n)
= a
n
sin
_
ax +b +n

2
_
, x R, n N
(cos (ax +b))
(n)
= a
n
cos
_
ax +b +n

2
_
, x R, n N.
ii) Pentru funct ia : R R, (x) = e
2x
sinx calculam
(2006)
() utilizand formula lui Leibniz.
Notam f (x) = e
2x
, g (x) = sinx. Avem f
(k)
() = (2)
k
e
2
, g
(k)
() = sin
_
+k

2
_
. Obt inem
f
(2006)
() =
2006

i=0
C
i
2006
(2)
2006i
e
2
sin
_
+i

2
_
= e
2
2006

i=0
(1)
i+1
C
i
2006
2
2006i
sin
_
i

2
_
= e
2
1002

j=0
(1)
j
C
2j+1
2006
2
2006(2j+1)
,
deoarece sin(i

2
) =
_
0, daca i = 2j
(1)
j
, daca i = 2j + 1.
Urmatoarea teorema referitoare la limita unei fract ii se obt ine prin induct ie matematica pornind de
la Teorema 4.2.27.
4.4 Funct ii convexe 167
4.3.4. Teorema (Cauchy) Fie I R un interval, n N

xat si f, g : I R doua funct ii derivabile


de n ori n punctul x
0
I. Daca
i) f
(k)
(x
0
) = g
(k)
(x
0
) = 0, k 0, 1, . . . , n 1,
ii) g
(n)
(x
0
) ,= 0.
atunci exista limita lim
xx
0
f (x)
g (x)
=
f
(n)
(x
0
)
g
(n)
(x
0
)
.
Urmatoarea teorema referitoare la limita unei fract ii se obt ine prin induct ie matematica pornind de
la teorema lui lHospital prezentata n paragraful anterior.
4.3.5. Teorema (Teorema lui lHospital cu derivate de ordin superior) Fie I R un interval,
x
0
R , + un punct de acumulare pentru I, f, g : I x
0
R doua funct ii si n N

xat.
Daca
i) funct iile f, g sunt de n ori derivabile pe I x
0
,
ii) g
(n)
(x) ,= 0, x I x
0
,
iii) exista lim
xx
0
f
(n)
(x)
g
(n)
(x)
= L R , +,
iv) lim
xx
0
f
(k)
(x) = lim
xx
0
g
(k)
(x) = 0, k 0, 1, 2, . . . , n 1,
atunci exista lim
xx
0
f (x)
g (x)
si lim
xx
0
f (x)
g (x)
= L.
4.3.6. Exemplu.
lim
x+
1 +
x
1!
+
x
2
2!
+
x
3
3!
+. . . +
x
n
n!
e
x
(

)
= lim
x+
_
1 +
x
1!
+
x
2
2!
+
x
3
3!
+. . . +
x
n
n!
_
t
(e
x
)
t
= lim
x+
1 +
x
1!
+
x
2
2!
+
x
3
3!
+. . . +
x
n1
(n 1)!
e
x
(

)
= lim
x+
_
1 +
x
1!
+
x
2
2!
+
x
3
3!
+. . . +
x
n1
(n 1)!
_
t
(e
x
)
t
= lim
x+
1 +
x
1!
+
x
2
2!
+
x
3
3!
+. . . +
x
n2
(n 2)!
e
x
(

)
= . . .
= lim
x+
1 +x
e
x
(

)
= lim
x+
(1 +x)
t
(e
x
)
t
= lim
x+
1
e
x
=
1
+
= 0.
4.4 Funct ii convexe
O clasa importanta de funct ii este cea a funct iilor convexe.
4.4.1. Denit ie. Fie funct ia f : D R R si I D un interval.
i) Spunem ca funct ia f este convexa pe I daca pentru orice x
1
, x
2
I si pentru orice t (0, 1)
are loc
f((1 t)x
1
+tx
2
) (1 t) f (x
1
) +tf (x
2
) .
ii) Spunem ca funct ia f este strict convexa pe I daca pentru orice x
1
, x
2
I si pentru orice
t (0, 1) are loc
f((1 t)x
1
+tx
2
) < (1 t) f (x
1
) +tf (x
2
) .
iii) Spunem ca funct ia f este concava pe I daca pentru orice x
1
, x
2
I si pentru orice t (0, 1)
are loc
f((1 t)x
1
+tx
2
) (1 t) f (x
1
) +tf (x
2
) .
168 4. Funct ii f : D R R derivabile
iv) Spunem ca funct ia f este strict concava pe I daca pentru orice x
1
, x
2
I si pentru orice
t (0, 1) are loc
f((1 t)x
1
+tx
2
) > (1 t) f (x
1
) +tf (x
2
) .
Daca D este interval si funct ia f : D R este (strict) convexa sau (strict) concava pe D, atunci
spunem ca funct ia f este (strict) convexa, respectiv (strict) concava. Din Denit ia 4.4.1 rezulta ca funct ia
f este (strict) concava pe I daca si numai daca funct ia f este (strict) convexa pe I. Din Denit ia 4.4.1
rezulta ca o funct ie f strict convexa (strict concava) pe I este convexa (concava) pe I. Armat ia reciproca
nu are loc.
4.4.2. Exemplu. Funct ia f : R R, f (x) = x este convexa pe R si nu este strict convexa pe R; n
acelasi timp funct ia f este concava pe R si nu este strict concava pe R.
Funct ia f : R R, f (x) = x
2
este strict convexa pe R.
Interpretare geometrica. Fie funct ia f : I R unde I R este un interval. Consideram gracul
funct iei f si punctele A(x
1
, f(x
1
)), B(x
2
, f(x
2
)) ale gracului unde x
1
, x
2
I, x
1
< x
2
. Fie t (0, 1) si
x
t
= (1 t)x
1
+tx
2
. Studiem punctele (x
t
, f (x
t
)) respectiv (x
t
, (1 t)f (x
1
) +tf (x
2
)) .
Din t (0, 1) si x
1
< x
2
rezulta ca x
1
< (1 t)x
1
+ tx
2
< x
2
, deci x
t
(x
1
, x
2
) I. Cum x
t
I,
punctul (x
t
, f (x
t
)) apart ine gracului funct iei f, si deoarece x
1
< x
t
< x
2
, el este situat pe arcul de
grac cuprins strict ntre punctele A si B.
Deoarece punctul ((1 t)x
1
+tx
2
, (1 t)f (x
1
) +tf (x
2
))) verica ecuat ia dreptei AB, el apart ine
dreptei AB, si deoarece x
1
< (1 t)x
1
+tx
2
< x
2
, el este situat pe segmentul delimitat de punctele A, B,
strict ntre A si B.
Pe baza celor ment ionate mai sus interpretam inegalitat ile din Denit ia 4.4.1.

In cazul unei funct ii convexe (concave) oricare ar doua puncte distincte ale gracului, arcul de
grac delimitat de aceste doua puncte este situat sub (deasupra) segmentul de dreapta delimitat de cele
doua puncte.

In cazul unei funct ii strict convexe (strict concave) oricare ar doua puncte distincte ale
gracului, arcul de grac cuprins strict ntre aceste doua puncte este situat strict sub (strict deasupra)
segmentul de dreapta delimitat de cele doua puncte.
Urmatoarea teorema evident iaza o proprietate importanta a funct iilor convexe care sunt derivabile
ntr-un punct.
4.4.3. Teorema. Fie I R un interval si funct ia f : I R convexa pe I. Daca funct ia f este
derivabila n punctul x
0
I, atunci
f
t
(x
0
) (x x
0
) f (x) f (x
0
) , x I.
Demonstrat ie. Consideram x I, x ,= x
0
. Deoarece funct ia f este convexa pe I, pentru ecare
t (0, 1) are loc inegalitatea f((1 t)x
0
+ tx)) (1 t) f (x
0
) + tf (x) . Scriind aceasta inegalitatea n
forma f (x
0
+t(x x
0
)) f (x
0
) t (f (x) f (x
0
)) si apoi
f (x
0
+t(x x
0
)) f (x
0
)
(x
0
+t (x x
0
)) x
0
(x x
0
) f (x) f (x
0
) ,
prin trecere la limita n ultima inegalitate rezulta
lim
t0
t>0
f (x
0
+t(x x
0
)) f (x
0
)
(x
0
+t (x x
0
)) x
0
(x x
0
) f (x) f (x
0
) .
T inand cont ca funct ia f este derivabila n punctul x
0
si deci lim
t0
t>0
f (x
0
+t(x x
0
)) f (x
0
)
(x
0
+t (x x
0
)) x
0
= f
t
(x
0
) ,
obt inem f
t
(x
0
) (x x
0
) f (x) f (x
0
) .
Interpretare geometrica. Daca funct ia f este derivabila n x
0
, atunci gracul funct iei admite dreapta
tangenta n punctul (x
0
, f (x
0
)), si anume dreapta de ecuat ie y f(x
0
) = f
t
(x
0
) (x x
0
) . Ordonata
punctului de abscisa x I al acestei drepte este y = f(x
0
) + f
t
(x
0
) (x x
0
) . Ordonata punctului de
aceeasi abscisa pe grac este f(x). Asadar teorema arma ca punctele de abscisa x I ale unei drepte
tangente la gracul funct iei convexe f : I R sunt situate sub gracul funct iei.
Teoremele urmatoare se utilizeaza pentru studiul convexitat ii funct iilor derivabile pe un interval.
4.4 Funct ii convexe 169
4.4.4. Teorema. Fie I R un interval si f : I R o funct ie derivabila pe I. Urmatoarele
armat ii sunt echivalente:
(i) Funct ia f este convexa pe I,
(ii) Pentru orice x
1
, x
2
I are loc f
t
(x
1
) (x
2
x
1
) f (x
2
) f (x
1
) ,
(iii) Derivata f
t
este funct ie crescatoare pe I.
Demonstrat ie. Implicat ia (i)(ii) este adevarata pe baza teoremei precedente.
Aratam ca (ii)(i). Fie x
1
, x
2
I, x
1
< x
2
. Pentru t (0, 1) rezulta (1 t)x
1
+ tx
2
I. Pe baza
ipotezei (ii) avem
f
t
((1 t)x
1
+tx
2
) [x
1
((1 t)x
1
+tx
2
)] f (x
1
) f ((1 t)x
1
+tx
2
)
f
t
((1 t)x
1
+tx
2
) [x
2
((1 t)x
1
+tx
2
)] f (x
2
) f ((1 t)x
1
+tx
2
)

Inmult im prima relat ie cu (1 t), a doua relat ie cu t, si apoi le adunam. Obt inem
f((1 t)x
1
+tx
2
) (1 t) f (x
1
) +tf (x
2
) ,
adica inegalitatea din denit ia funct iei convexe.
Aratam ca (ii)(iii). Fie x
1
, x
2
I, x
1
< x
2
. Pe baza ipotezei (ii) avem inegalitat ile
f
t
(x
1
) (x
2
x
1
) f (x
2
) f (x
1
) ,
respectiv
f
t
(x
2
) (x
1
x
2
) f (x
1
) f (x
2
) ,
care prin adunare conduc la (f
t
(x
1
) f
t
(x
2
)) (x
2
x
1
) 0 si de aici, dupa mpart ire cu (x
2
x
1
)
obt inem f
t
(x
1
) f
t
(x
2
) . Asadar funct ia f
t
este crescatoare.
Aratam ca (iii)(ii). Fie x
1
, x
2
I, x
1
< x
2
. Pe baza teoremei lui Lagrange exista c (x
1
, x
2
) astfel
ca f
t
(c) =
f (x
2
) f (x
1
)
x
2
x
1
. Pe baza ipotezei (iii) funct ia f
t
este crescatoare, deducem f
t
(x
1
) f
t
(c) , si
deci f
t
(x
1
)
f (x
2
) f (x
1
)
x
2
x
1
. Prin nmult ire cu (x
2
x
1
) obt inem
f
t
(x
1
) (x
2
x
1
) f (x
2
) f (x
1
) .
Interpretare geometrica. Teorema arma ca o funct ie derivabila pe un interval I este convexa pe I
daca si numai daca punctele de abscisa x I ale dreptelor tangente la gracul funct iei sunt situate sub
gracul funct iei.
Urmatoarea teorema se demonstreaza n mod asemanator.
4.4.5. Teorema. Fie I R un interval si f : I R o funct ie derivabila pe I. Urmatoarele
armat ii sunt echivalente:
(i) Funct ia f este strict convexa pe I,
(ii) Pentru orice x
1
, x
2
I, x
1
< x
2
are loc f
t
(x
1
) (x
2
x
1
) < f (x
2
) f (x
1
) ,
(iii) Derivata f
t
este funct ie strict crescatoare pe I.
4.4.6. Teorema. Fie I R un interval si funct ia f : I R convexa pe I. Daca funct ia f este
derivabila de doua ori n punctul x
0
I, atunci f
tt
(x
0
) 0.
Demonstrat ie. Pe baza denit iei derivabilitat ii de ordinul doi a funct iei f n punctul x
0
, exista
> 0 astfel ncat funct ia f este derivabila pe I (x
0
, x
0
+) si funct ia f
t
: I (x
0
, x
0
+) R
este derivabila n punctul x
0
, adica exista
f
tt
(x
0
) = lim
xx
0
f
t
(x) f
t
(x
0
)
x x
0
.
Deoarece mult imea I (x
0
, x
0
+) este interval si f este convexa pe acest interval, pe baza teoremei
precedente derivata f
t
: I (x
0
, x
0
+) R este funct ie crescatoare, asadar pentru orice x
I (x
0
, x
0
+) are loc
f
t
(x) f
t
(x
0
)
x x
0
0 si prin urmare f
tt
(x
0
) = lim
xx
0
f
t
(x) f
t
(x
0
)
x x
0
0.
170 4. Funct ii f : D R R derivabile
4.4.7. Teorema. Fie I R un interval si e funct ia f : I R derivabila de doua ori pe I.
Funct ia f este convexa pe I daca si numai daca f
tt
(x) 0, x I.
Demonstrat ie. Daca funct ia f de doua ori derivabila pe I este convexa pe I, atunci pe baza
teoremei precedente, pentru ecare x I are loc f
tt
(x) 0. Daca f
tt
(x) 0, x I, atunci funct ia
f
t
: I R este crescatoare si aplicand Teorema 4.4.4 rezulta ca funct ia f este convexa pe I.
4.4.8. Teorema. Fie I R un interval si e funct ia f : I R derivabila de doua ori pe I. Daca
f
tt
(x) > 0, x I, atunci funct ia f este strict convexa pe I.
Demonstrat ie. Din f
tt
(x) > 0, x I rezulta ca derivata f
t
este strict crescatoare, si atunci, pe
baza Teoremei 4.4.5, funct ia f este strict convexa pe I.
4.4.9. Observat ie. Reciproca Teoremei 4.4.8 nu are loc. Exista funct ii f derivabile de doua ori pe
un interval I si strict convexe pe I fara ca funct ia f
tt
sa e srtict pozitiva pe I.
4.4.10. Exemplu. Funct ia f : R R, f (x) = x
4
este strict convexa pe R. Derivata de ordinul doi
f
tt
: R R, f (x) = 12x
2
nu este strict pozitiva pe R, se anuleaza n x
0
= 0.
Teoremele 4.4.3-4.4.8 au analoage cu privire la o funct ie f concava, inegalitat ile ind inversate pentru
acest caz. Enunt am aceste rezultate pentru ultimele doua teoreme ment ionate.
4.4.11. Teorema. Fie I R un interval si e funct ia f : I R derivabila de doua ori pe I.
Funct ia f este concava pe I daca si numai daca f
tt
(x) 0, x I.
4.4.12. Teorema. Fie I R un interval si e funct ia f : I R derivabila de doua ori pe I. Daca
f
tt
(x) < 0, x I, atunci funct ia f este strict concava pe I.
4.4.13. Denit ie. Fie I R un interval si f : IR o funct ie. Spunem ca x
0
I este punct de
inexiune pentru f, daca exista > 0 astfel ncat
i) (x
0
, x
0
+) I (adica x
0
este punct interior al intervalului I),
ii) funct ia f este continua n x
0
,
iii) funct ia f are derivata n x
0
(nita sau nu),
iv) funct ia f este convexa pe (x
0
, x
0
] si este concava pe [x
0
, x
0
+), sau invers.
4.4.14. Exemplu. i) Funct ia f : R R, f (x) = x
3
este derivabila de doua ori pe R,
f
tt
(x) = 6x, pentru orice x R. Din f
tt
(x) 0, x (, 0] rezulta ca funct ia f este concava pe
intervalul (, 0], iar din f
tt
(x) 0, x [0, +) rezulta ca funct ia f este convexa pe intervalul
[0, +). Funct ia f este continua n x
0
= 0, are derivata n x
0
= 0 care este punct interior al lui R, asadar
x
0
= 0 este punct de inexiune pentru f.
ii) Funct ia f : [0, 2] R, f (x) = sinx este derivabila de doua ori pe [0, 2] si f
tt
(x) = sinx,
pentru orice x [0, 2] . Din f
tt
(x) 0, x [0, ] rezulta ca funct ia f este concava pe intervalul [0, ] ,
iar din f
tt
(x) 0, x [, 2] rezulta ca funct ia f este convexa pe intervalul [, 2] . Evident f este
continua n x
0
= 0 si are derivata n x
0
= 0 care este punct interior al intervalului [0, 2] , asadar x
0
= 0
este punct de inexiune pentru f.
4.4.15. Observat ie. Conform denit iei punctului de inexiune continuitatea funct iei f ntr-un
punct de inexiune x
0
cat si existent a derivatei f
t
(x
0
) sunt esent iale.
4.4.16. Exemplu. Funct ia f : R R, f (x) =
_
x
2
+ 2, x < 1
x
2
, x 1
este concava pe (, 1] si este
convexa pe [1, +), este continua n x
0
= 1, dar nu are derivata n x
0
= 1. Punctul x
0
= 1 nu este punct
de inexiune pentru f (de fapt (1, f(1)) este punct unghiular al gracului funct iei).
Inegalitatea lui Jensen. Pornind de la Denit ia 4.4.1 i), notand 1 t = t
1
, t = t
2
si observand ca
t
1
, t
2
(0, 1) , t
1
+t
2
= 1, prin induct ie matematica deducem ca daca f este convexa pe I, atunci pentru
ecare n 2, 3, . . . are loc inegalitatea
f(t
1
x
1
+t
2
x
2
+. . . +t
n
x
n
) t
1
f (x
1
) +t
2
f (x
2
) +. . . +t
n
f (x
n
) ,
pentru orice x
1
, x
2
, . . . , x
n
I si orice t
1
, t
2
, . . . , t
n
(0, 1) cu t
1
+t
2
+. . . +t
n
= 1.
Acest rezultat este numit Inegalitatea lui Jensen. Pentru funct ii strict convexe inegalitatea este
stricta. Pentru funct ii concave (strict concave) inegalitatea este inversata.
4.4.17. Exemplu.

In continuare consideram unghiurile masurate n radiani si aratam ca:
i) pentru orice triunghi ABC este valabila inegalitatea sinA+ sinB + sinC
3

3
2
ii) pentru orice patrulater ABCD este valabila inegalitatea sin
A
2
+ sin
B
2
+ sin
C
2
+ sin
D
2
2

2.
4.4 Funct ii convexe 171
Fie f : [0, ] [1, 1] , f(x) = sinx. Calculam derivatele funct iei f
t
(x) = cos x, f
tt
(x) = sinx
0, x [0, ] , deci f este concava pe [0, ] . Cu inegalitatea lui Jensen corespunzatoare lui n = 3, aplicata
funct iei concave f pentru x
1
= A, x
2
= B, x
3
= C si t
1
= t
2
= t
3
=
1
3
, adica f(
1
3
A +
1
3
B +
1
3
C)
1
3
f (A) +
1
3
f (B) +
1
3
f (C) , stiind ca A + B + C = , obt inem sin

3

1
3
sinA +
1
3
sinB +
1
3
sinC; cu
sin

3
=

3
2
, dupa nmult ire cu 3, rezulta inegalitatea i).
Fie g : [0, 2] [1, 1] , g(x) = sin
x
2
. Calculam derivatele funct iei g si avem g
t
(x) =
1
2
cos
x
2
,
g
tt
(x) =
1
4
sin
x
2
0, x [0, 2] , deci g este concava pe [0, 2] . Cu inegalitatea lui Jensen core-
spunzatoare lui n = 4, aplicata funct iei concave g pentru x
1
= A, x
2
= B, x
3
= C, x
4
= D si t
1
=
t
2
= t
3
= t
4
=
1
4
, adica f(
1
4
A +
1
4
B +
1
4
C +
1
4
D)
1
4
f (A) +
1
4
f (B) +
1
4
f (C) +
1
4
f (D) , stiind ca
A + B + C + D = 2, obt inem sin

4

1
4
sin
A
2
+
1
4
sin
B
2
+
1
4
sin
C
2
+
1
4
sin
D
2
; cu sin

4
=

2
2
, dupa
nmult ire cu 4, rezulta inegalitatea ii).
Urmatoarele teoreme arata cateva proprietat i importante ale funct iilor convexe legate de continuitate
si derivabilitatea laterala.
4.4.18. Denit ie. Fie I R un interval si f : I R o funct ie. Pentru x
0
I xat consideram
funct ia p
x
0
: I x
0
R, p
x
0
(x) =
f (x) f (x
0
)
x x
0
numita funct ia panta.
4.4.19. Teorema. Fie I R un interval si f : I R o funct ie.
i) Daca funct ia f este convexa pe I, atunci pentru orice x
0
I funct ia panta p
x
0
este crescatoare.
ii) Daca funct ia f este strict convexa pe I, atunci pentru orice x
0
I funct ia panta p
x
0
este strict
crescatoare.
Demonstrat ie. i) Funct ia f ind convexa pe I, pentru orice x
1
, x
2
I si pentru orice t (0, 1)
are loc f ((1 t) x
1
+tx
2
) (1 t) f (x
1
) + tf (x
2
) . Fie x
0
I xat si e x
1
< x
2
I x
0
. Aratam
ca p
x
0
(x
1
) p
x
0
(x
2
), adica
f (x
1
) f (x
0
)
x
1
x
0

f (x
2
) f (x
0
)
x
2
x
0
.
Privind pozit ia numerelor x
1
, x
2
fat a de x
0
deosebim trei cazuri.
a) Daca x
0
< x
1
< x
2
atunci are loc x
1
=
x
2
x
1
x
2
x
0
x
0
+
x
1
x
0
x
2
x
0
x
2
si pe baza inegalitat ii din
denit ia funct iei convexe obt inem inegalitatea f (x
1
)
x
2
x
1
x
2
x
0
. .
1t
f (x
0
) +
x
1
x
0
x
2
x
0
. .
t(0,1)
f (x
2
) echivalenta cu
f (x
1
) f (x
0
)
x
1
x
0
x
2
x
0
(f (
2
) f (x
0
)), adica
f (x
1
) f (x
0
)
x
1
x
0

f (x
2
) f (x
0
)
x
2
x
0
.
b) Daca x
1
< x
2
< x
0
atunci are loc x
2
=
x
0
x
2
x
0
x
1
x
1
+
x
2
x
1
x
0
x
1
x
0
si pe baza inegalitat ii din
denit ia funct iei convexe obt inem inegalitatea f (x
2
)
x
0
x
2
x
0
x
1
. .
t(0,1)
f (x
1
) +
x
2
x
1
x
0
x
1
. .
1t
f (x
0
) echivalenta cu
f (x
2
) f (x
0
)
x
0
x
2
x
0
x
1
(f (x
1
) f (x
0
)), adica
f (x
1
) f (x
0
)
x
1
x
0

f (x
2
) f (x
0
)
x
2
x
0
.
c) Daca x
1
< x
0
< x
2
atunci are loc x
0
=
x
2
x
0
x
2
x
1
x
1
+
x
0
x
1
x
2
x
1
x
2
si pe baza inegalitat ii din denit ia
funct iei convexe, obt inem inegalitatea f (x
0
)
x
2
x
0
x
2
x
1
. .
1t
f (x
1
) +
x
0
x
1
x
2
x
1
. .
t
f (x
2
), echivalenta cu
_
x
2
x
0
x
2
x
1
+
x
0
x
1
x
2
x
1
_
f (x
0
)
x
2
x
0
x
2
x
1
f (x
1
) +
x
0
x
1
x
2
x
1
f (x
2
) ,
172 4. Funct ii f : D R R derivabile
ceea ce se scrie (x
2
x
0
) (f (x
0
) f (x
1
)) (x
0
x
1
) (f (x
2
) f (x
0
)), adica
f (x
1
) f (x
0
)
x
1
x
0

f (x
2
) f (x
0
)
x
2
x
0
.
4.4.20. Observat ie.

In mod asemanator se demonstreaza ca daca funct ia f este (strict) concava
pe intervalul I, atunci pentru orice x
0
I funct ia panta p
x
0
este (strict) descrescatoare.
4.4.21. Teorema. Fie I R un interval si f : I R o funct ie. Daca funct ia f este convexa pe I,
atunci f este continua n ecare punct interior intervalului.
Demonstrat ie. Fie x
0
I, situat n interiorul intervalului I. Aratam ca funct ia f, convexa pe I,
este continua n x
0
. Cum x
0
este punct interior lui I, exista > 0 astfel ncat (x
0
, x
0
+) I. Notand
x
0


2
= a, x
0
+

2
= b, avem a < x
0
< b si [a, b] I. Daca funct ia f este convexa pe I, atunci funct ia
panta p
x
0
: I x
0
R, p
x
0
(x) =
f (x) f (x
0
)
x x
0
este crescatoare, asadar pentru orice x [a, b] x
0

are loc
f (a) f (x
0
)
a x
0

f (x) f (x
0
)
x x
0

f (b) f (x
0
)
b x
0
.
Daca notam M = max
_

f (a) f (x
0
)
a x
0

f (b) f (x
0
)
b x
0

_
, deducem
[f (x) f (x
0
)[ M[x x
0
[ , x [a, b] x
0
,
de unde rezulta continuitatea funct iei f n punctul x
0
.
4.4.22. Teorema. Fie a, b R, a < b si f : (a, b) R o funct ie convexa pe (a, b). Atunci are loc
i) f
t
s
(x) R, x (a, b),
ii) f
t
d
(x) R, x (a, b),
iii) funct iile f
t
s
: (a, b) R, f
t
d
: (a, b) R sunt crescatoare,
iv) f
t
s
(x) f
t
d
(x) , x (a, b).
Demonstrat ie. i), ii) Fie x
0
(a, b) xat. Cum funct ia f este convexa pe (a, b), funct ia panta
p
x
0
: (a, b) x
0
R, p
x
0
(x) =
f (x) f (x
0
)
x x
0
este monotona (crescatoare) pe mult imea (a, x
0
) (x
0
, b) ,
asadar funct ia p
x
0
are limite laterale nite n punctul x
0
: lim
xx
0
x<x
0
p
x
0
(x) R, lim
xx
0
x>x
0
p
x
0
(x) R. Utilizam
denit ia derivatelor laterale si obt inem
f
t
s
(x
0
) = lim
xx
0
x<x
0
f (x) f (x
0
)
x x
0
= lim
xx
0
x<x
0
p
x
0
(x) R si f
t
d
(x
0
) = lim
xx
0
x>x
0
f (x) f (x
0
)
x x
0
= lim
xx
0
x>x
0
p
x
0
(x) R.
iii), iv) Fie x
1
, x
2
R, x
1
< x
2
. Aratam ca f
t
s
(x
1
) f
t
s
(x
2
) . Utilizam funct iile panta
p
x
1
: (a, b) x
1
R, p
x
1
(x) =
f (x) f (x
1
)
x x
1
, p
x
2
: (a, b) x
2
R, p
x
2
(x) =
f (x) f (x
2
)
x x
2
.
Deoarece f este convexa pe (a, b) , funct iile p
x
1
, p
x
2
sunt crescatoare, asadar considerand x
t
, x
tt
astfel ca
a < x
t
< x
1
< x
tt
< x
2
avem
f (x
t
) f (x
1
)
x
t
x
1

f (x
tt
) f (x
1
)
x
tt
x
1

f (x
tt
) f (x
2
)
x
tt
x
2
. Prin trecere la limita
pentru x
t
x
1
si x
tt
x
2
obt inem lim
x

x
1
x

<x
1
f (x
t
) f (x
1
)
x
t
x
1
lim
x

x
2
x

<x
2
f (x
tt
) f (x
2
)
x
tt
x
2
, adica f
t
s
(x
1
) f
t
s
(x
2
) .
Pentru a arata ca f
t
d
(x
1
) f
t
d
(x
2
) se procedeaza n mod asemanator.
Prin trecere la limta pentru x
t
x
1
si pentru x
tt
x
1
obt inem
lim
x

x
1
x

<x
1
f (x
t
) f (x
1
)
x
t
x
1
lim
x

x
1
x

>x
1
f (x
tt
) f (x
1
)
x
tt
x
1
,
adica f
t
s
(x
1
) f
t
d
(x
1
) .
4.5 Formula lui Taylor 173
4.5 Formula lui Taylor
4.5.1. Denit ie. Fie I R un interval, n N

xat si f : IR o funct ie derivabila de n ori n


punctul x
0
I. Funct ia T
n
: R R,
T
n
(x) = f (x
0
)+
f
t
(x
0
)
1!
(x x
0
)+
f
tt
(x
0
)
2!
(x x
0
)
2
+. . . +
f
(k)
(x
0
)
k!
(x x
0
)
k
+. . . +
f
(n)
(x
0
)
n!
(x x
0
)
n
se numeste polinomul lui Taylor de ordinul n atasat funct iei f si punctului x
0
. Pentru x I
notand f(x) T
n
(x) prin R
n
(x) are loc f(x) = T
n
(x) +R
n
(x)x I, adica
f (x) = f (x
0
) +
f
t
(x
0
)
1!
(x x
0
) +
f
tt
(x
0
)
2!
(x x
0
)
2
+. . . +
f
(k)
(x
0
)
k!
(x x
0
)
k
+. . .
+
f
(n)
(x
0
)
n!
(x x
0
)
n
+R
n
(x), x I
formula numita formula lui Taylor corespunzatoare funct iei f si punctului x
0
.
Funct ia R
n
: IR, R
n
(x) = f(x)T
n
(x) se numeste restul de ordinul n al formulei lui Taylor.
Deoarece polinoamele sunt funct ii convenabile din punct de vedere computat ional, avem interesul
sa aproximam numarul f (x) prin T
n
(x), respectiv funct ia f prin polinomul T
n
. Comportarea restului
R
n
(x) pentru x I ne arata calitatea acestor aproximari.
Urmatoarele doua teoreme ne ofera reprezentari ale restului R
n
(x).
4.5.2. Teorema (Formula lui Taylor cu restul lui Peano) Fie I R un interval deschis, n N

xat si f : I R o funct ie derivabila de n ori n punctul x


0
I. Atunci exista o funct ie : I R
continua n x
0
, cu (x
0
) = 0, astfel ncat
f (x) = f (x
0
) +
f
t
(x
0
)
1!
(x x
0
) +
f
tt
(x
0
)
2!
(x x
0
)
2
+. . . +
f
(k)
(x
0
)
k!
(x x
0
)
k
+. . .
+
f
(n)
(x
0
)
n!
(x x
0
)
n
+(x)
(x x
0
)
n
n!
, x I
Demonstrat ie. Denim funct ia : I R astfel:
(x) =
_
_
_
n!
f (x) T
n
(x)
(x x
0
)
n
, daca x I x
0

0, daca x = x
0
,
unde
T
n
(x) = f (x
0
)+
f
t
(x
0
)
1!
(x x
0
)+
f
tt
(x
0
)
2!
(x x
0
)
2
+. . .+
f
(k)
(x
0
)
k!
(x x
0
)
k
+. . .+
f
(n)
(x
0
)
n!
(x x
0
)
n
.
Aratam ca funct ia astfel denita are proprietat ile ment ionate n teorema. Calculam
T
n
(x) +
1
n!
(x) (x x
0
)
n
= T
n
(x) +
1
n!
n!
f (x) T
n
(x)
(x x
0
)
n
(x x
0
)
n
= f (x) , x I x
0
.
Calculam
lim
xx
0
(x) = lim
xx
0
n!
f (x) T
n
(x)
(x x
0
)
n
= n! lim
xx
0
f (x) T
n
(x)
(x x
0
)
n
utilizand teorema lui Cauchy privind calculul limitei unei fract ii cu ajutorul derivatelor de ordin superior,
Teorema 4.3.4. Avem
lim
xx
0
(f (x) T
n
(x)) = f (x
0
) T
n
(x
0
) = f(x
0
) f (x
0
) = 0, lim
xx
0
(x x
0
)
n
= 0.
174 4. Funct ii f : D R R derivabile
Funct iile R(x) = f (x) T
n
(x) , Q(x) = (x x
0
)
n
sunt derivabile de n ori n punctul x
0
. Calculand
T
(k)
n
(x
0
) = f
(k)
(x
0
) rezulta R
(k)
(x
0
) = 0, k 0, 1, 2, . . . , n . Evident Q
(k)
(x
0
) = 0, pentru orice
k 0, 1, 2, . . . , n 1 si Q
(n)
(x
0
) = n! ,= 0. Pe baza teoremei ment ionate obt inem
lim
xx
0
(x) = n! lim
xx
0
R(x)
Q(x)
= n!
R
(n)
(x
0
)
Q
(n)
(x
0
)
= n!
0
n!
= 0.
Comparam cu (x
0
) = 0 si egalitatea lim
xx
0
(x) = (x
0
) ne spune ca funct ia f este continua n punctul
x
0
.
4.5.3. Observat ie.

In ipotezele teoremei exista o funct ie : IR continua n x
0
, cu (x
0
) = 0,
astfel ncat restul de ordinul n al formulei lui Taylor se reprezinta R
n
(x) = (x)
(x x
0
)
n
n!
.
4.5.4. Teorema (Formula lui Taylor cu restul lui Lagrange) Fie I R un interval deschis. Fie
n N xat. Daca funct ia f : I R este derivabila de n + 1 ori pe I, atunci pentru orice doua puncte
distincte din I, x, x
0
I, x ,= x
0
, exista cel put in un punct situat strict ntre x si x
0
astfel ncat
f (x) = f (x
0
) +
f
t
(x
0
)
1!
(x x
0
) +
f
tt
(x
0
)
2!
(x x
0
)
2
+. . . +
f
(k)
(x
0
)
k!
(x x
0
)
k
+. . .
+
f
(n)
(x
0
)
n!
(x x
0
)
n
+
f
(n+1)
()
(n + 1)!
(x x
0
)
n+1
.
Demonstrat ie. Fixam x, x
0
I, x ,= x
0
. Determinam A R din condit ia R
n
(x) = A(x x
0
)
n+1
,
unde R
n
(x) = f (x) T
n
(x) si apoi consideram funct ia : IR,
(t) = f (t)+
f
t
(t)
1!
(x t)+
f
tt
(t)
2!
(x t)
2
+. . . +
f
(k)
(t)
k!
(x t)
k
+. . . +
f
(n)
(t)
n!
(x t)
n
+A(x t)
n+1
.
Prin calcule simple gasim (x) = f (x),
(x
0
) = T
n
(x) +A(x x
0
)
n+1
= T
n
(x) +R
n
(x) = f (x) ,
asadar (x) = (x
0
) .
Din faptul ca funct ia f este derivabil a de n + 1 ori pe I rezulta ca funct ia este derivabila pe I :

t
(t) = f
t
(t)
f
t
(t)
1!
+
f
tt
(t)
1!
(x t)
f
tt
(t)
2!
2 (x t) +
f
ttt
(t)
2!
(x t)
2
+ . . .
f
(k)
(t)
k!
k (x t)
k1
+
f
(k+1)
(t)
k!
(x t)
k
+. . .
f
(n)
(t)
n!
n(x t)
n1
+
f
(n+1)
(t)
n!
(x t)
n
A(n + 1) (x t)
n
, t I

t
(t) =
f
(n+1)
(t)
n!
(x t)
n
A(n + 1) (x t)
n
, t I.
Cu privire la funct ia sunt ndeplinite condit iile teoremei lui Rolle pe intervalul [x, x
0
] daca x < x
0
,
respectiv pe intervalul [x
0
, x] daca x
0
< x, deci exista cel put in un punct situat strict ntre x si x
0
astfel
ncat
t
() = 0. Ultima relat ie se scrie
f
(n+1)
()
n!
(x )
n
A(n + 1) (x )
n
= 0 si de aici obt inem
A =
f
(n+1)
()
(n + 1)!
. Prin urmare R
n
(x) =
f
(n+1)
()
(n + 1)!
(x x
0
)
n+1
.
4.5.5. Observat ie.

In ipotezele teoremei exista situat strict ntre x si x
0
astfel ncat restul de
ordinul n al formulei lui Taylor se reprezinta R
n
(x) =
f
(n+1)
()
(n + 1)!
(x x
0
)
n+1
.
4.5.6. Observat ie. Pentru f si n date, punctul evident depinde de x si x
0
. Punctul se mai
poate reprezenta n forma = x
0
+(x x
0
) cu (0, 1) convenabil ales.
4.5.7. Observat ie.Teorema precedenta pentru n = 0 coincide cu teorema lui Lagrange.
Formula lui Taylor cu restul lui Lagrange n cazul particular x
0
= 0 ne da urmatorul rezultat.
4.5.8. Teorema (Formula lui MacLaurin) Fie I R un interval deschis si 0 I. Fie n N xat.
Daca funct ia f : I R este derivabila de n + 1 ori pe I, atunci pentru orice punct x I, x ,= 0 exista
cel put in un punct situat strict ntre x si 0 astfel ncat
f (x) = f (0) +
f
t
(0)
1!
x +
f
tt
(0)
2!
x
2
+. . . +
f
(k)
(0)
k!
x
k
+. . . +
f
(n)
(0)
n!
x
n
+
f
(n+1)
()
(n + 1)!
x
n+1
.
4.5 Formula lui Taylor 175
4.5.9. Exemplu. i) Scriem formula lui MacLaurin de ordinul n pentru funct ia f : R R, f(x) = e
x
.
Calculam f
(k)
(x) = e
x
, k N, f
(k)
(0) = e
0
= 1, k N. Obt inem formula
e
x
= 1 +
1
1!
x +
1
2!
x
2
+. . . +
1
k!
x
k
+. . . +
1
n!
x
n
+
e

(n + 1)!
x
n+1
,
unde este situat strict ntre x si 0.
ii) Scriem formula lui MacLaurin de ordinul 2n pentru funct ia f : R R, f(x) = sinx.
Avem f
(k)
(x) = sin
_
x +k

2
_
, k N, si f
(k)
(0) = sin
_
k

2
_
=
_
0, daca k = 2j
(1)
j
, daca k = 2j + 1
.
Obt inem formula
sinx =
1
1!
x
1
3!
x
3
+. . . + (1)
j
1
(2j + 1)!
x
(2j+1)
+. . . + (1)
n1
1
(2n 1)!
x
2n1
+ (1)
n
sin
_
+ (2n + 1)

2
_
(2n + 1)!
x
2n+1
,
unde este situat strict ntre x si 0.
iii) Scriem formula lui MacLaurin de ordinul 2n 1 pentru funct ia f : R R, f(x) = cos x.
Calculam f
(k)
(x) = cos
_
x +k

2
_
, k N, deci f
(k)
(0) = cos
_
k

2
_
=
_
0, daca k = 2j + 1
(1)
j
, daca k = 2j
.
Obt inem formula
cos x = 1
1
2!
x
2
+
1
4!
x
4
+. . . + (1)
j
1
(2j)!
x
(2j)
+. . . + (1)
n1
1
(2n 2)!
x
2n2
+ (1)
n
cos ( +n)
(2n)!
x
2n
,
unde este situat strict ntre x si 0.
iv) Aproximam numarul
10

e utilizand formula i) cu n = 3:
10

e = e
1
10
= 1 +
1
1!
1
10
+
1
2!
_
1
10
_
2
+
1
3!
_
1
10
_
3
+
e

4!
_
1
10
_
4
, 0 < <
1
10
.
Prin omiterea termenului rest rezulta aproximarea
10

e 1 +
1
1!
1
10
+
1
2!
1
10
2
+
1
3!
1
10
3
, deci
10

e
6631
6000
.
Eroarea comisa la aceasta aproximare se delimiteaza prin inegalitatea

4!
1
10
4

<
e
1
10
4!
1
10
4
<
e
4!
1
10
4
<
3
4!
1
10
4
=
1
80000
.
v) Organizam polinomul P(x) = 2x
5
4x
3
+ 7x
2
+ 2x 3 ca suma de puteri ale binomului (x + 2).

In acest scop utilizam formula lui Taylor de ordinul 5 cu x


0
= 2.
P(x) = P (2) +
P
t
(2)
1!
(x + 2) +
P
tt
(2)
2!
(x + 2)
2
+
P
(3)
(2)
3!
(x + 2)
3
+
P
(4)
(2)
4!
(x + 2)
4
+
P
(5)
(2)
5!
(x + 2)
5
+
P
(6)
()
6!
(x + 2)
6
,
unde este situat strict ntre x si 2.
176 4. Funct ii f : D R R derivabile
Calculam P (2) = 11 si derivatele P
t
(x) = 10x
4
12x
2
+ 14x + 2, P
tt
(x) = 40x
3
24x + 14,
P
(3)
(x) = 120x
2
24, P
(4)
(x) = 240x, P
(5)
(x) = 240, P
(6)
(x) = 0; Obt inem P
t
(2) = 86, P
tt
(2) =
258, P
(3)
(2) = 456, P
(4)
(2) = 480, P
(5)
(2) = 240, P
(6)
() = 0. Scriem formula lui Taylor
P (x) = 11 +
86
1!
(x + 2) +
258
2!
(x + 2)
2
+
456
3!
(x + 2)
3
+
480
4!
(x + 2)
4
+
240
5!
(x + 2)
5
+
0
6!
x
6
.
Deci
2x
5
4x
3
+ 7x
2
+ 2x 3 = 2 (x + 2)
5
20 (x + 2)
4
+ 76 (x + 2)
3
129 (x + 2)
2
+ 86 (x + 2) 11.
4.6 Caracterizarea punctelor de optim ale unei funct ii cu aju-
torul derivatelor
Consideram o funct ie f : D R R derivabila ntr-un punct x
0
din interiorul mult imii D. Teorema
lui Fermat arma ca daca funct ia f are optim local n x
0
, atunci f
t
(x
0
) = 0. Din f
t
(x
0
) = 0 nsa, dupa
cum ne arata si Exemplul 4.2.6, nu rezult a ca x
0
este punct de optim local pentru f.
Urmatoarea teorema ofera condit ii suciente pentru puncte de optim local apeland la derivatele de
ordin superior ale funct iei f n punctul x
0
.
4.6.1. Teorema. Fie I R un interval, x
0
un punct interior intervalului I, n 2, 3, xat si
f : I R o funct ie de n ori derivabila n x
0
cu
f
t
(x
0
) = f
tt
(x
0
) = ... = f
(n1)
(x
0
) = 0, f
(n)
(x
0
) ,= 0.
Au loc urmatoarele armat ii:
i) Daca n este numar par atunci x
0
este punct de optim local relativ la I al funct iei f, si anume
daca f
(n)
(x
0
) < 0, atunci x
0
este punct de maxim local,
daca f
(n)
(x
0
) > 0, atunci x
0
este punct de minim local.
ii) Daca n este numar impar atunci x
0
nu este punct de optim local relativ la I al funct iei f.
Demonstrat ie. Funct ia f este derivabila de n ori n punctul x
0
. Scriem formula lui Taylor atasata
funct iei f si punctului x
0
cu restul lui Peano
f (x) = f (x
0
) +
f
t
(x
0
)
1!
(x x
0
) +
f
tt
(x
0
)
2!
(x x
0
)
2
+. . . +
f
(k)
(x
0
)
k!
(x x
0
)
k
+. . .
+
f
(n)
(x
0
)
n!
(x x
0
)
n
+(x)
(x x
0
)
n
n!
, x I
cu funct ia : I R continua n x
0
si (x
0
) = 0.

Inlocuim aici f
t
(x
0
) = f
tt
(x
0
) = ... = f
(n1)
(x
0
) = 0
si obt inem
f (x) = f (x
0
) +
f
(n)
(x
0
)
n!
(x x
0
)
n
+(x)
(x x
0
)
n
n!
, x I,
adica
f (x) f (x
0
) =
_
f
(n)
(x
0
) +(x)
_
(x x
0
)
n
n!
, x I.
Aratam ca exista o vecinatate a punctului x
0
pe care funct ia f
(n)
(x
0
) + (x) de variabila x are
semnul constant.
Funct ia continua n punctul x
0
cu (x
0
) = 0 are limita lim
xx
0
(x) = 0 si rezulta ca
lim
xx
0
_
f
(n)
(x
0
) +(x)
_
= f
(n)
(x
0
) .
Conform ipotezei f
(n)
(x
0
) ,= 0.
Daca f
(n)
(x
0
) < 0 atunci avem lim
xx
0
_
f
(n)
(x
0
) +(x)

< 0, inegalitate care implica existent a unei


vecinatat i U a lui x
0
astfel ca f
(n)
(x
0
) +(x) < 0, x I U x
0
.
4.6 Caracterizarea punctelor de optim ale unei funct ii cu ajutorul derivatelor 177
Daca f
(n)
(x
0
) > 0, atunci avem lim
xx
0
_
f
(n)
(x
0
) +(x)

> 0, inegalitate care implica existent a unei


vecinatat i U a lui x
0
astfel ca f
(n)
(x
0
) +(x) > 0, x I U x
0
.
i) Daca numarul n este par, atunci
(x x
0
)
n
n!
0, x I.

In cazul f
(n)
(x
0
) < 0 pentru orice x I U are loc
f (x) f (x
0
) =
_
f
(n)
(x
0
) +(x)
_
(x x
0
)
n
n!
0.
Asadar f (x) f (x
0
) , x I U, deci x
0
este punct de maxim local relativ la I al funct iei f.

In cazul f
(n)
(x
0
) > 0 pentru orice x I U are loc
f (x) f (x
0
) =
_
f
(n)
(x
0
) +(x)
_
(x x
0
)
n
n!
0.
Asadar f (x) f (x
0
) , x I U, deci x
0
este punct de minim local relativ la I al funct iei f.
ii) Daca numarul n este impar, atunci funct ia
(x x
0
)
n
n!
cu variabila x n orice vecinatate a lui x
0
realizeaza atat valori negative cat si valori pozitive. Diferent a
f (x) f (x
0
) =
_
f
(n)
(x
0
) +(x)
_
(x x
0
)
n
n!
cu variabila x, pe nici o vecinatate a lui x
0
nu pastreaza un acelasi semn, prin urmare x
0
nu este punct
de minim local si nici punct de maxim local al funct iei f relativ la I.
4.6.2. Exemplu. i) Determinam punctele de optim local ale funct iei f : R R, f (x) = x
3
e
x
.
Calculam derivatele
f
t
(x) =
_
3x
2
+x
3
_
e
x
, f
tt
(x) =
_
6x + 6x
2
+x
3
_
e
x
, f
ttt
(x) =
_
6 + 18x + 9x
2
+x
3
_
e
x
Punctele de optim local ale funct iei f, conform teoremei lui Fermat, se gasesc printre punctele stat ionare
ale funct iei, adica printre solut iile ecuat iei
f
t
(x) = 0
_
3x
2
+x
3
_
e
x
= 0 x
1
= 0, x
2
= 3.
Din f
t
(0) = 0, f
tt
(0) = 0, f
ttt
(0) ,= 0, deoarece numarul de derivari n = 3 este numar impar, rezulta
ca x
1
= 0 nu este punct de optim local al funct iei f.
Din f
t
(3) = 0, f
tt
(3) ,= 0, deoarece numarul de derivari n = 2 este numar par si f
tt
(3) > 0
rezulta ca x
2
= 3 este punct de minim local al funct iei f.
ii) Determinam punctele de optim local ale funct iei g : R R, f (x) = x
4
e
x
. Calculam derivatele
g
t
(x) =
_
4x
3
+x
4
_
e
x
, g
tt
(x) =
_
12x
2
+ 8x
3
+x
4
_
e
x
, g
ttt
(x) =
_
24x + 36x
2
+ 12x
3
+x
4
_
e
x
,
g
(4)
(x) =
_
24 + 96x + 72x
2
+ 16x
3
+x
4
_
e
x
.
Punctele de optim local ale funct iei g, conform teoremei lui Fermat, se gasesc printre punctele stat ionare
ale funct iei, adica printre solut iile ecuat iei
g
t
(x) = 0
_
4x
3
+x
4
_
e
x
= 0 x
1
= 0, x
2
= 4.
Din g
t
(0) = 0, g
tt
(0) = 0, g
ttt
(0) = 0, g
(4)
(0) ,= 0, deoarece numarul de derivari n = 4 este numar
par si g
(4)
(0) > 0, rezulta ca x
1
= 0 este punct de minim local al funct iei g.
Din g
t
(4) = 0, f
tt
(4) ,= 0, deoarece numarul de derivari n = 2 este numar par si f
tt
(4) < 0,
rezulta ca x
2
= 4 este punct de maxim local al funct iei g.
Capitolul 5
Funct ii integrabile Riemann si
primitive
5.1 Funct ii integrabile. Integrala Riemann.
Not iunea de integrala a aparut din necesitatea practica de a calcula aria unor guri plane si de-a
rezolva anumite probleme de mecanica. Pana la denit ia actuala a integralei data de Riemann (1854)
si-au adus contribut ia la aceasta denit ie mai mult i matematicieni: Newton, Leibniz, Cauchy. Ulterior,
conceptul de integrala a fost extins de Stieltjes si Lebesgue, iar n secolul al XX-lea au aparut noi
generalizari ale integralei datorate lui Denjoy, Peron, Kurzweil-Henstock, integrale care nu mai constituie
obiectul studiului din aceasta carte.
5.1.1 Denit ie. (Diviziune a unui interval compact). Fie a, b R, a < b. Se numeste
diviziune a intervalului compact [a, b], mult imea ordonata de n +1 puncte x
0
, x
1
, x
2
, . . . , x
n
[a, b],
notata
:= (x
0
, x
1
, . . . , x
n1
, x
n
),
cu proprietatea
a = x
0
< x
1
< x
2
< . . . < x
n1
< x
n
= b.
Vom nota cu Div[a, b] mult imea tuturor diviziunilor intervalului [a, b].
5.1.2 Denit ie. (Norma unei diviziuni). Daca = (x
0
, x
1
, . . . , x
n
) este o diviziune a interva-
lului [a, b], se numeste norma a diviziunii , numarul notat () si reprezentand lungimea celui mai
mare interval part ial [x
i
, x
i+1
] determinat de diviziunea , adica
() := maxx
1
x
0
, x
2
x
1
, . . . , x
n
x
n1
= max
1in
x
i
x
i1
.
5.1.3 Exemplu. Fie intervalul [0, 1] si consideram doua diviziuni ale acestui interval

1
=
_
0,
1
3
,
1
2
, 1
_
;
2
=
_
0,
1
4
,
1
2
,
3
4
, 1
_
.
Normele acestor diviziuni sunt
(
1
) =
1
2
, (
2
) =
1
4
.
5.1.4 Observat ie. Pentru orice > 0, putem gasi o diviziune a intervalului [a, b] asa ca () < .

Intr-adevar, daca > 0 si n este un numar natural astfel ncat


b a
n
< , atunci diviziunea prin
puncte echidistante
=
_
a, a +
b a
n
, a +
2(b a)
n
, . . . , a +
(n 1)(b a)
n
, b
_
,
178
5.1 Funct ii integrabile. Integrala Riemann. 179
are proprietatea ca () =
b a
n
< .
5.1.5 Denit ie. Fie ,
t
Div [a, b], doua diviziuni ale intervalului [a,b]. Spunem ca diviziunea ,
este mai na decat divizuinea
t
si scriem
t
sau
t
, daca cont ine toate punctele diviziunii

t
si eventual si puncte n plus.
5.1.6 Teorema. Daca este o diviziune a intervalului [a, b] mai na decat
t
, adica
t
,
atunci () (
t
).
5.1.7 Observat ie. Reciproca teoremei 5.1.6 nu este, n general, adevarata, dupa cum se vede n
exemplul urmator:
Div[0, 1], =
_
0,
1
3
,
1
2
, 1
_
,
() =
1
2
,
t
Div[0, 1],

t
=
_
0,
1
4
, 1
_
, (
t
) =
3
4
.
Avem () < (
t
), dar nu este mai na decat
t
.
5.1.8 Denit ie. (Sistem de puncte intermediare). Fie := (x
0
, x
1
, . . . , x
n
) o diviziune a
intervalului [a, b], a < b. Mult imea de puncte =
1
,
2
, . . . ,
n
cu proprietatea ca

i
[x
i1
, x
i
] , oricare ar i = 1, n,
se numeste sistem de puncte intermediare asociat diviziunii . Evident, pentru o diviziune data,
exista oricate sisteme de puncte intermediare asociate lui . Vom nota cu P(), mult imea tuturor
sistemelor de puncte intermediare asociate diviziunii .
5.1.9 Denit ie. (Suma Riemann). Fie f : [a, b] R o funct ie, := (x
0
, x
1
, . . . , x
n
) o diviziune
a intervalului [a, b] si =
1
,
2
, . . . ,
n
, un sistem de puncte intermediare asociat lui . Se numeste
suma Riemann (sau suma integrala Riemann) asociata funct iei f, diviziunii si sistemului de puncte
intermediare , numarul
(f, , ) :=
n

i=1
f(
i
)(x
i
x
i1
). (5.1.1.)
Este evident ca, pentru funct ia f : [a, b] R, se pot construi o innitate de sume Riemann, avand n
vedere ca exista oricate diviziuni ale intervalului [a, b] si pentru ecare diviziune aleasa, exista oricate
sisteme de puncte intermediare asociate.
5.1.10 Observat ie. Daca presupunem ca intervalul [a, b] este degenerat, adica a = b, atunci orice
diviziune Div[a, b] cont ine puncte identice. Asadar, x
i
= a, i = 1, n, x
i
x
i1
= 0 si (f, , ) = 0.
5.1.11 Denit ie. (Funct ie integrabila, integrala Riemann). Spunem ca funct ia f : [a, b] R
este integrabila Riemann pe [a, b] (sau simplu, integrabila) daca exista un numar real I cu propri-
etatea ca pentru orice > 0 exista un () > 0 astfel ncat pentru orice diviziune a intervalului [a, b],
cu () < si orice sistem de puncte intermediare =
1
,
2
, . . . ,
n
P (), sa avem
[ (f, , ) I [ < . (5.1.2)
Numarul I se numeste integrala Riemann a funct iei f pe intervalul [a, b] si se noteaza
I :=
b
_
a
f(x) dx. (5.1.3)

In aceasta notat ie a, b se numesc limite de integrare, a se numeste limita inferioara, b se numeste


limita superioara, f este funct ia de sub semnul integrala (sau funct ia de integrat), x se numeste
variabila de integrare, iar dx arata deocamdata care este variabila de integrare
5.1.12 Observat ie. Este usor de aratat ca exista cel mult un numar I care sa satisfaca relat ia
(5.1.2) si daca pentru orice I R, exista > 0 astfel ncat pentru orice > 0 sa existe Div[a, b] cu
() < si un P() ncat
[(f, , )I [ ,
180 5. Funct ii integrabile Riemann si primitive
atunci funct ia f nu este integrabila Riemann.

Intr-adevar, daca ar exista doua numere I


1
si I
2
care sa satisfaca denit ia 5.1.11 atunci pentru orice
> 0 avem
[ I
1
I
2
[ = [ I
1
(f, , ) + (f, , ) I
2
[
[ (f, , )I
1
[ +[ (f, , ) I
2
[ < + = 2 ,
deci I
1
= I
2
.
5.1.13 Exemple. 1
0
. Funct ia f : [a, b] R, f(x) = k = constant, pentru orice x [a, b], este
integrabila pe [a, b] si avem
b
_
a
f(x) dx =
b
_
a
k dx = k(b a). (5.1.4)

Intr-adevar, daca
= (x
0
, x
1
, . . . , x
n
) Div[a, b]
si
=
1
,
2
, . . . ,
n
P(),
suma Riemann asociata lui f, , , este
(f, , ) =
n

i=1
f(
i
)(x
i
x
i1
) =
=
n

i=1
k(x
i
x
i1
) = k(x
n
x
0
) = k(b a),
adica este constanta. Deci este clar ca luand I = k (b a), pentru > 0, avem
[ (f, , ) I [ = [ k(b a) k(b a) [ = 0 < ,
ceea ce arata ca funct ia f este integrabila pe [a, b] si are loc (5.1.4).
2
0
. Funct ia f : [a, b] R, data prin
f(x) =
_
0, daca x [a, b] Q,
1, daca x [a, b] Q,
numita funct ia lui Dirichlet, nu este integrabila Riemann pe [a, b].
Sa presupunem contrarul, anume ca f este integrabila Riemann pe [a, b]. Atunci, dupa denit ia
5.1.11 exista I R cu proprietatea ca pentru orice > 0, exista () > 0 astfel ncat oricare ar o
diviziune Div[a, b] cu () < si pentru orice sistem de puncte intermediare P() sa avem
[ (f, , )I [ < .
Sa presupunem ca sistemul de puncte intermediare =
1
,
2
, . . . ,
n
este format din puncte
rat ionale
t
1
,
t
2
, . . . ,
t
n
,
t
i
[x
i1
, x
i
] , i = 1, n. Atunci
(f, , ) =
n

i=1
f(
t
i
) (x
i
x
i1
) = 0,
astfel ca pentru orice > 0.
[ (f, , )I[ = [ I [ < . (5.1.5)
Sa facem acum alta alegere pentru sistemul de puncte intermediare, anume sa luam
tt
i
[x
i1
, x
i
] i = 1, n,
numere irat ionale. Atunci,
(f, , ) =
n

i=1
f(
tt
i
) (x
i
x
i1
) =
n

i=1
(x
i
x
i1
) = x
n
x
0
= b a,
5.1 Funct ii integrabile. Integrala Riemann. 181
si atunci
[ (f, , )I[ = [ b a I [ < , (5.1.6)
pentru orice > 0. Daca =
1
4
(b a), cu (5.1.5) si (5.1.6) gasim
b a = [b a I +I [ [ b a I [ +[ I [ < + =
1
2
(b a),
adica am ajuns la o contradict ie. Aceasta arata ca funct ia f nu este integrabila.
5.1.14 Teorema. Daca funct ia f : [a, b] R este integrabila Riemann pe [a, b], atunci funct ia f
este marginita pe [a, b], adica mult imea f(x) [x [a, b] R este marginita.
Demonstrat ie. Sa presupunem ca f este integrabila pe [a, b]. Atunci, exista I R astfel ncat
pentru orice > 0, exista > 0 asa ncat pentru := (x
0
, x
1
, . . . , x
n
) Div[a, b] cu () < si orice
=
1
,
2
, . . . ,
n
P() sa avem
[ (f, , )I [ < .
Daca luam = 1 si xam diviziunea , ultima inegalitate se poate scrie
1 I < (f, , ) < 1 +I. (5.1.7)
Sa presupunem prin absurd ca f nu este marginita pe [a, b]. Atunci exista un interval part ial
[x
k1
, x
k
] al diviziunii pe care f nu este marginita, sa zicem ca nu este majorata si astfel
supf(x) [x [x
k1
, x
k
] = +. (5.1.8)
Sa notam cu
:=
n

i=1
i,=k
f (
i
) f (
i
) (x
i
x
i1
) . (5.1.9)
Deoarece pe [x
k1
, x
k
], f nu este marginita, putem alege punctul
k
[x
k1
, x
k
] asa ca
f(
k
) >
I + 2
x
k
x
k1
.
De aici
f(
k
)(x
k
x
k1
) > I + 2, (5.1.10)
si putem scrie
(f, , ) =
n

i=1
f(
i
)(x
i
x
i1
) = +f(
k
)(x
k
x
k1
) >
> +I + 2 = I + 2 > 1 +I,
adica
(f, , ) > 1 +I. (5.1.11)
Dar inegalitatea aceasta, contrazice inegalitatea (5.1.7) valabila pentru diviziunea . Aceasta
contradict ie arata ca presupunerea facuta, anume ca f este nemarginita pe [a, b], este falsa si deducem
concluzia teoremei, ca funct ia f este marginita pe [a, b].
5.1.15 Observat ie. Denit ia integrabilitat ii Riemann 5.1.11 nu este comoda n practica, pentru
a stabili daca o funct ie este sau nu este integrabila, deoarece n aceasta denit ie apare n mod explicit
integrala funct iei, adica numarul I. De aceea, vom da cateva teoreme echivalente cu denit ia 5.1.11,
teoreme care folosesc siruri de diviziuni.
5.1.16 Teorema. Fie f : [a, b] R, a,b R, a < b. Condit ia necesara si sucienta ca funct ia
f sa e integrabila Riemann pe [a, b] si
b
_
a
f(x) d x = I este, ca pentru orice sir de diviziuni (
n
)
n1
ale intervalului [a, b] cu lim
n
(
n
) = 0 si pentru orice sir (
n
)
n1
P(
n
) de sisteme de puncte
intermediare, sirul sumelor Riemann ((f,
n
,
n
)) converge catre numarul I.
182 5. Funct ii integrabile Riemann si primitive
Demonstrat ie. Necesitatea: Presupunem ca f este integrabila pe [a, b] si I =
b
_
a
f(x)d x. Atunci
pentru orice > 0 exista () > 0 asa ca s a avem
[(f, , )I [ < , (5.1.12)
pentru orice Div[a, b] cu () < si orice P (). Fie acum (
n
) un sir de diviziuni ale lui
[a, b] cu proprietatea ca (
n
) 0, cand n si e (
n
) un sir de sisteme de puncte intermediare

n
P(
n
), n N

. Daca > 0, din faptul ca (


n
) 0, deducem ca exista un N() N asa ca
(
n
) < () pentru n N().

Insa acum din (5.1.12), deducem ca
[(f,
n
,
n
)I [ < ,
pentru orice n N, n N(). Ultima relat ie arata ca sirul ( (f,
n
,
n
) ) converge catre I, adica
lim
n
(f,
n
,
n
) = I.
Sucient a: Sa presupunem ca pentru orice sir de diviziuni (
n
) cu (
n
) 0, cand n
avem (f,
n
,
n
) I, pentru orice
n
P(
n
) dar ca f nu este integrabila pe [a, b]. Deci exista > 0
asa ca pentru orice > 0 exista Div[a, b] cu () < si P() asa ncat [ (f, , )I [ . Sa
luam =
1
n
, n N

. Atunci pentru n N

exista diviziunea
n
Div[a, b] cu (
n
) <
1
n
si exista

n
P(
n
) asa ca sa avem
[ (f,
n
,
n
) I [ . (5.1.13)

Insa, deoarece (
n
) 0, cand n , din ipoteza, deducem ca (f,
n
,
n
) I, ceea ce
contravine inegalitat ii (5.1.13).
Din aceasta teorema se deduce imediat urmatorul rezultat:
5.1.17 Corolar. Funct ia f : [a, b] R este integrabila pe [a, b] daca si numai daca pentru orice sir
de diviziuni (
n
) ale intervalului [a, b] cu lim
n
(
n
) = 0 si pentru orice sir
n
P(
n
) de sisteme de
puncte intermediare asociate lui (
n
, sirul ((f,
n
,
n
) ) este convergent.
5.1.18 Teorema. (Criteriu de tip Cauchy de integrabilitate Riemann). Funct iaf:[a,b] R
este integrabila Riemann pe intervalul [a, b] daca si numai daca pentru orice > 0 exista () > 0 astfel
ca oricare ar diviziunile
t
,
tt
Div [a, b] cu (
t
) < si (
t
t
) < si oricare ar sistemele de
puncte intermediare
t
P(
t
) ,
tt
P(
tt
) avem
[ (f,
t
,
t
) (f,
tt
,
tt
) [ < . (5.1.14)
Demonstrat ie. Necesitatea Presupunem ca f este integrabila pe [a, b] si e > 0. Atunci exista
I R si exista () > 0 astfel ncat pentru orice
t
,
tt
Div [a, b] cu (
t
) < , (
t
t
) < si orice

t
P(
t
) ,
tt
P(
tt
) sa avem
[ (f,
t
,
t
)I [ <

2
, [ (f,
tt
,
tt
)I [ <

2
.
Putem scrie
[ (f,
t
,
t
) (f,
tt
,
tt
)[ = [( (f,
t
,
t
)I) + (I (f,
tt
,
tt
)[
[ (f,
t
,
t
)I [ +[ (f,
tt
,
tt
)I [ <

2
+

2
= ,
adica inegalitatea (5.1.14).
Sucient a: Presupunem satisfacuta condit ia (5.1.14) si demonstram ca f este integrabila pe [a, b].
Fie (
n
) un sir de diviziuni ale intervalului [a,b] cu (
n
) 0 cand n si
n
P(
n
) un sir
de sisteme de puncte intermediare. Daca > 0, atunci exista > 0 astfel ncat
t
,
tt
Div [a, b] cu
(
t
) < , (
t
t
) < si pentru
t
P(
t
) ,
tt
P(
tt
) avem
[ (f,
t
,
t
) (f,
tt
,
tt
) [ < .
5.2 Criteriul de integrabilitate al lui Darboux. Clase de funct ii integrabile 183
Din faptul ca (
n
) 0, cand n , exista N () N astfel ca pentru n N () sa avem (
n
) < .
Fie n, m N asa ca n, m N (). Atunci (
n
) < si (
m
) < si avem
[ (f,
n
,
n
) (f,
m
,
m
) [ < ,
ceea ce dovedeste ca sirul ( (f,
n
,
n
)) este un sir fundamental de numere reale si deci el este convergent.
Cu corolarul precedent se deduce ca funct ia f este integrabila Riemann pe [a, b].
O condit ie oarecum simplicata, de integrabilitate Riemann, este data de
5.1.19 Teorema. Funct ia f : [a, b] R este integrabila Riemann pe intervalul [a, b] daca si numai
daca pentru orice > 0 exista ()> 0 asa ncat pentru orice diviziune Div[a, b] cu () < si
oricare ar sistemele de puncte intermediare
t
,
tt
P(), sa avem
[ (f, ,
t
) (f, ,
tt
) [ < .
Renunt am la demonstrat ie.
5.2 Criteriul de integrabilitate al lui Darboux. Clase de funct ii
integrabile
Vom da n cele ce urmeaza un criteriu de integrabilitate care foloseste asa numitele sume ale lui
Darboux.
Fie f : [a, b] R o funct ie marginita pe [a, b] si avand marginile
m := inf f(x) [ x [a, b] ,
M := supf(x) [ x [a, b] .
Daca = (x
0
, x
1
, x
2
, . . . , x
n
) Div [a, b], deducem ca f este marginita pe ecare interval part ial
[x
i1
, x
i
]. Vom nota atunci
m
i
:= inff(x) [ x [x
i1
,x
i
] , M
i
:= supf(x) [ x [x
i1
,x
i
] ,
marginile funct iei f pe [x
i1
, x
i
], i = 1, n.
5.2.1 Denit ie. (Sumele lui Darboux). Numerele reale
s

(f) = s(f, ) :=
n

i=1
m
i
(x
i
x
i1
) ,
S

(f) = S(f, ) :=
n

i=1
M
i
(x
i
x
i1
) ,
se numesc suma inferioara Darboux asociata funct iei f si diviziunii , respectiv suma superioara
Darboux asociata funct iei f si diviziunii .
5.2.2 Teorema Daca f : [a, b] R este marginita pe [a, b] si are marginile m, M atunci pentru orice
diviziune Div[a, b] si orice sistem de puncte intermediare P(), avem
m(b a) s (f, ) (f, , ) S(f, ) M(b a). (5.2.1)
Demonstrat ia este imediata, daca se t ine seama de denit ia sumelor Darboux s (f,) si S(f, ), a sumei
Riemann si de inegalitat ile evidente
m m
i
f(
i
) M
i
M , i = 1, n. (5.2.2)
5.2.3 Teorema. Daca diviziunii := (x
0
, x
1
, . . . , x
n
) Div [a, b] i se adauga cel put in un nou punct
de diviziune atunci noua diviziune
t
Div[a, b] este mai na ca si avem
s(f, ) s(f,
t
) , S(f, ) S(f,
t
). (5.2.3)
184 5. Funct ii integrabile Riemann si primitive
Demonstrat ie. Sa presupunem ca
t
= c , x
i1
< c < x
i
, adica,

t
:= (x
0
, x
1
, . . . , x
i1
, c, x
i
, . . . , x
n
).
Atunci n suma s(f, ) =
n

i=1
m
i
(x
i
x
i1
) numarul m
i
(x
i
x
i1
) se nlocuieste cu m
t
i
(c x
i1
) +
m
tt
i
(x
i
c), unde
m
t
i
:= inff(x) [ x [x
i1
, c] , m
t
t
i
:= inff(x) [ x [c, x
i
] .
si evident m
i
m
t
i
, m
i
m
tt
i
. Deci
m
i
(x
i
x
i1
) = m
i
(x
i
c +c x
i1
) =
m
i
(x
i
c) +m
i
(c x
i1
) m
t
i
(c x
i1
) +m
tt
i
(x
i
c).
Deducem ca
s(f, ) s(f,
t
).
Analog se deduce cea de-a doua inegalitate din (5.2.2).
5.2.4 Teorema Oricare ar diviziunile
t
,
tt
Div [a, b] avem
s(f,
t
) S(f,
tt
) (5.2.4)
Demonstrat ie. Fie
t
,
tt
Div [a, b] si notam =
t

tt
. Atunci este clar ca
t
si
tt
si dupa teorema 5.2.3 avem
s(f,
t
) s(f, ) si S(f, ) S(f,
tt
).
De aici, deoarece s(f, ) S(f, ) conform (5.2.1) avem s(f,
t
) S(f,
tt
), adica inegalitatea ceruta.
5.2.5 Observat ie. Daca f : [a, b] R este marginita, atunci mult imile s(f, ) [ Div [a, b]
si S (f, ) [ Div [a, b] sunt marginite.
Armat ia rezulta din inegalitatea (5.2.1).
5.2.6 Denit ie. Daca f : [a, b] R este marginita, notam
I(f) := sups(f, ) [ Div[a, b] ,
I(f) := infS(f, ) [ Div[a, b] .
Numerele reale I(f) , I (f) se numesc integralele lui Darboux inferioara, respectiv superioara. Din
aceasta denit ie rezulta ca
s (f, ) I(f) I (f) S (f, ). (5.2.5)
Mai mult, are loc
5.2.7 Teorema. Fie f :[a, b] R marginita si Div[a, b] o diviziune xata a intervalului [a, b].
Atunci
s (f, ) = inf (f, , ) [ P() , (5.2.6)
S (f, ) = sup (f, , ) [ P() . (5.2.7)
Demonstrat ie. Fie = (x
0
, x
1
, x
2
, . . . , x
n
) si = (
1
,
2
, . . . ,
n
) P(). Din (5.2.1) deducem ca
s (f, ) (f, , ), pentru orice P().
Sa aratam ca s (f, ) este cel mai mare minorant al mult imii (f, , ) [ P(). Fie > 0.
Daca m
i
:= inff(x) [ x [x
i1
, x
i
] , i = 1, n, putem spune ca pentru ecare i 1, 2, . . . , n exista

i
[x
i1
, x
i
], deci exista = (
1
,
2
, . . . ,
i
, . . . ,
n
) astfel ca f(
i
) < m
i
+

b a
, caci m
i
este margine
inferioara. Atunci pentru acest P() avem
(f, , ) =
n

i=1
f(
i
)(x
i
x
i1
)
n

i=1
_
m
i
+

b a
_
(x
i
x
i1
) =
5.2 Criteriul de integrabilitate al lui Darboux. Clase de funct ii integrabile 185
=
n

i=1
m
i
(x
i
x
i1
) +

b a
n

i=1
(x
i
x
i1
) = s (f, ) +.
Aceasta arata ca are loc (5.2.6).

In mod analog se demonstreaza si (5.2.7).
5.2.8 Teorema. (Criteriul lui Darboux de integrabilitate Riemann). Fie f : [a, b] R
marginita. Funct ia f este integrabila Riemann pe [a, b] atunci si numai atunci cand pentru orice > 0
exista un > 0 asa ncat oricare ar Div [a, b] cu () < , sa avem
S (f, ) s (f, ) < . (5.2.8)
Demonstrat ie. Necesitatea: Presupunem ca funct ia f este integrabila si e > 0. Atunci exista
() > 0 astfel ncat pentru orice Div [a, b] cu () < si pentru orice
t
,
tt
P() avem (conform
teoremei 5.1.19)
[ (f, ,
t
) (f, ,
tt
) [ <

2
. (5.2.9)
Din teorema precedenta, cum s(f, ) si S(f, ) sunt marginile inferioara, respectiv superioara ale
mult imii (f, , ) [ P() putem spune ca exista doua siruri de sisteme de puncte intermediare
(
n
) si (

n
) astfel ncat
s (f, ) = lim
n
(f, ,
n
) , S (f, ) = lim
n
(f, ,

n
).

In plus, cum () <, cu (5.2.9) deducem ca

(f, ,

n
) (f, ,
n
)

<

2
, n N.
De aici, pentru n obt inem (5.2.8).
Sucient a. Daca Div [a, b] sa presupunem ca are loc (5.2.8) pentru orice > 0 si () <. Din
(5.2.5), avem
0 I(f) I(f) S (f, ) s (f, ) < ,
pentru orice > 0. Aceasta implica egalitatea
I(f) I(f) = 0.
Fie I = I(f) = I(f), valoarea comuna. Atunci avem, cu (5.2.5)
s (f, ) I S (f, ), (5.2.10)
pentru orice Div [a, b].

Insa din (5.2.1) mai avem
s (f, ) (f, , ) S (f, ), (5.2.11)
pentru orice Div [a, b] si P() Cu inegalitat ile (5.2.10) si (5.2.11) putem scrie [ (f, , )I [
S (f, ) s (f, ) < , pentru orice Div [a, b] cu () < si orice P(). Aceasta dovedeste ca f
este integrabila Riemann pe [a, b] si
b
_
a
f(x) d x = I = I(f) = I(f).
5.2.9 Corolar. Daca f : [a, b] R este marginita si daca f este integrabila Riemann pe [a, b] avem
I(f) = I(f) =
b
_
a
f(x) d x. (5.2.12)
Concluzia rezulta pe baza unicitat ii integralei I =
b
_
a
f(x) d x, pentru o funct ie integrabila.
186 5. Funct ii integrabile Riemann si primitive
5.2.10 Observat ie. Daca notam
i
= M
i
m
i
, i = 1, n, oscilat ia funct iei f pe intervalul
[x
i1
, x
i
] atunci
S (f, ) s (f, ) =
n

i=1
(M
i
m
i
)(x
i
x
i1
) =
n

i=1

i
(x
i
x
i1
)
si criteriul lui Darboux se enunt a astfel:
Funct ia f : [a, b] R, marginita, este integrabila pe [a, b] daca si numai daca ()> 0, () () > 0,
astfel ncat pentru () Div[a, b] cu () <, avem
n

i=1

i
(x
i
x
i1
) < . (5.2.13)
5.2.11 Observat ie. Din teorema 5.2.8. rezulta ca f : [a, b] R este integrabila Riemann daca si
numai daca pentru orice sir de diviziuni (
n
) a intervalului [a, b] cu proprietatea ca lim
n
(
n
) = 0 si
orice sir (
n
) de sisteme de puncte intermediare avem
lim
n
S (f,
n
) = lim
n
s (f,
n
) = lim
n
(f,
n
,
n
) = I.
5.2.12 Observat ie. (Interpretarea geometrica a integralei Riemann). Fie f : [a, b] R o
funct ie continua (desi nu este obligatoriu) si pozitiva, si e gracul sau
G
f
:= (x, y) R
2
[y =f(x) , x [a, b] .
Gracul G
f
este arcul de curba AB situat deasupra axei Ox. Mult imea de puncte T cuprinsa ntre axa
Ox, gracul G
f
si dreptele x = a, x = b se numeste trapez curbiliniu. Se pune problema evaluarii
ariei trapezului curbiliniu T, deocamdata acceptand pentru arie denit ia data la liceu.
Fig 5.2.1.
Deoarece funct ia f este marginita pe [a, b], ind continua, trapezul curbiliniu T = ABba, este o mult ime
de puncte din plan, marginita. Fie = (x
0
, x
1
, x
2
, . . . , x
n
) Div [a, b] si n punctele diviziunii vom ridica
perpendiculare pe Ox. Daca pentrui = 1, 2, . . . , n, notam
m
i
:= inf f(x) [ x [x
i1
, x
i
] , M
i
= sup f(x) [ x [x
i1
, x
i
] ,
vom considera dreptunghiurile avand ca baze pe x
i
x
i1
si ca nalt imi, odata pe m
i
si apoi pe M
i
.
Ariile acestor dreptunghiuri vor , pentru dreptunghiurile mici incluse n trapezul curbiliniu,
m
1
(x
1
x
0
) , m
2
(x
2
x
1
) , . . . , m
n
(x
n
x
n1
) ,
iar pentru dreptunghiurile mai mari, care au si part i exterioare trapezului curbiliniu,
M
1
(x
1
x
0
) , M
2
(x
2
x
1
) , . . . , M
n
(x
n
x
n1
) .
Reuniunea tuturor dreptunghiurilor cu nalt imile m
i
, i = 1, n este un poligon P cont inut n trapezul
curbiliniu, avand aria
Aria P =
n

i=1
m
i
(x
i
x
i1
) = s (f, ), (5.2.14)
iar reuniunea tuturor dreptunghiurilor cu nalt imile M
i
, i = 1, n este un poligon Q care cont ine n
ntregime trapezul curbiliniu si care are aria
Aria Q =
n

i=1
M
i
(x
i
x
i1
) = S (f, ). (5.2.15)
5.2 Criteriul de integrabilitate al lui Darboux. Clase de funct ii integrabile 187
Evident, aceste arii sunt chiar sumele Darboux s(f, ), respectiv S (f, ). Asadar, ariile poligoanelor
P si Q aproximeaza, prin lipsa si respectiv prin adaos, aria trapezului curbiliniu. Aproximarea este din
ce n ce mai buna cand n creste si lungimile intervalelor [x
i
x
i1
] scad, adica () 0 cand n .
Considerand acum un sir de diviziuni (
n
) ale intervalului [a, b], vom obt ine doua siruri de poligoane
(P
n
), incluse n trapezul curbiliniu si (Q
n
), care cont in trapezul curbiliniu, ariile lor ind
Aria P
n
= s (f,
n
) ; Aria Q
n
= S (f,
n
). (5.2.16)
Daca f este integrabila Riemann pe [a, b] atunci conform criteriului lui Darboux si observat iei 5.1.11
se deduce ca daca pentru sirul de diviziuni (
n
), avem (
n
) 0 cand n , atunci
lim
n
(S (f,
n
) s (f,
n
)) = 0 , lim
n
S (f,
n
) = lim
n
s (f,
n
) = I
si deci
lim
n
Aria P
n
= lim
n
Aria Q
n
=
b
_
a
f(x) d x. (5.2.17)
Asadar, n acest caz, deoarece
Aria P
n
Aria T Aria Q
n
,
se deduce ca trapezul curbiliniu T are arie (se spune ca este o mult ime masurabila) si aria sa este
Aria T =
b
_
a
f(x) d x. (5.2.18)
Sa observam ca putem aproxima Aria T si cu o suma Riemann
(f,
n
,
n
) :=
n

i=1
f(
n
)(x
i
x
i1
),
unde
n
=
n
1
,
n
2
, . . . ,
n
n
P(
n
), adica cu suma ariilor dreptunghiurilor cu bazele x
i
x
i1
, i = 1, n
si nalt imile f(
n
i
), datorita lui (5.2.1).
Daca funct ia f : [a, b] R nu este neaparat pozitiva, atunci
Aria T =
b
_
a
[f(x)[ dx. (5.2.19)
Vom prezenta n cele ce urmeaza cateva clase de funct ii integrabile Riemann.
5.2.13 Teorema. Orice funct ie f : [a, b] R continua, este integrabila Riemann.
Demonstrat ie. Fie f : [a, b] R continua. Atunci ea este uniform continua (teorema lui Cantor)
si deci oricare ar > 0, exista () > 0 asa ncat pentru orice x
t
, x
tt
[a, b] cu

x
t
x
t
t

< , avem
[ f(x
t
)f(x
tt
) [ <

b a
. (5.2.20)
Daca luam o diviziune = (x
0
, x
1
, . . . , x
n
) Div [a, b] cu () <, se deduce ca f este continua
pe ecare interval part ial [x
i1
, x
i
] si pe baza teoremei lui Weierstrass, ea este marginita si si atinge
marginile pe ecare [x
i1
, x
i
]. Atunci exista punctele
t
i
,
tt
i
[x
i1
, x
i
] asa ncat
m
i
:= inff(x) [ x [x
i1
, x
i
]=f(
t
i
) ,
M
i
:= supf(x)

x [x
i1
, x
i
]=f(
t
t
i
) .
Cu (5.2.20) vom avea
M
i
m
i
= f(
t
t
i
) f(
t
i
) <

b a
, i = 1, n.
188 5. Funct ii integrabile Riemann si primitive
De aici rezulta
S (f, ) s (f, ) =
n

i=1
(M
i
m
i
)(x
i
x
i1
) <

b a
n

i=1
(x
i
x
i1
) < ,
si pe baza criteriului lui Darboux, se deduce ca funct ia f este integrabila Riemann pe [a, b].
5.2.14 Teorema. Orice funct ie f : [a, b] R monotona, este integrabila Riemann.
Demonstrat ie. Fie deci f o funct ie monotona, de exemplu crescatoare pe [a, b]. Daca
= (x
0
, x
1
, . . . , x
n
) Div [a, b] atunci f este crescatoare si pe [x
i1
, x
i
], i = 1, n si deci m
i
= f(x
i1
) si
M
i
= f(x
i
). Atunci
S (f, ) s (f, ) =
n

i=1
(M
i
m
i
)(x
i
x
i1
) =
n

i=1
[f(x
i
)f(x
i1
)] (x
i
x
i1
).

Insa x
i
x
i1
() pentru orice i = 1, n si atunci
S(f, ) s(f, ) ()
n

i=1
[f(x
i
) f(x
i1
)] = ()[f(b) f(a)].
Fie acum > 0 si () =

f(b) f(a)
. Daca () < ( ) atunci obt inem
S (f, ) s (f, ) < [f(b) f(a)] = ,
pentru orice diviziune cu () < . Pe baza criterului lui Darboux se deduce ca f este integrabila
Riemann pe [a, b].
5.2.15 Observat ie.

In clasa funct iilor monotone, deci integrabile pe[a, b] intra si unele funct ii
discontinue. Daca notam D(f) mult imea punctelor de discontinuitate ale funct iei f : [a, b] R, este
importanta structura mult imii D(f) n studiul integrabilitat ii funct iei f.
5.2.16 Teorema. O funct ie f : [a, b] R marginita si avand un numar nit de puncte de
discontinuitate, este integrabila Riemann.
Demonstrat ie. Este sucient sa presupunem ca f are un singur punct de discontinuitate c ]a, b[,
caci n caz contrar vom descompune [a, b] ntr-un numar nit de intervale, ecare avand cate un punct
de discontinuitate.
Fie > 0 asa ca ] c , c + [ [a, b] si atunci putem scrie
[a, b] = [a, c ]] c , c + [[c +, b].
Pe intervalele [a, c ] si [c +, b], funct ia f este continua si deci uniform continua Astfel ca pentru
> 0, exista numerele
1
() > 0,
2
( ) > 0 asa ncat pentru orice
x
t
, x
tt
[a, c ] cu [ x
t
x
tt
[ <
1
, avem [ f(x
t
) f(x
tt
)[ < .
La fel daca
x
t
, x
tt
[c +, b ] cu [ x
t
x
tt
[ <
2
, avem [ f(x
t
) f(x
tt
) [ < .
De aici, daca = min
1
,
2
, rezulta ca pentru orice
x
t
, x
tt
[a, c ] [c +, b] cu [ x
t
x
tt
[ < ,
avem
[ f(x
t
) f(x
tt
) [ < . (5.2.20)
Fie := (x
0
, x
1
, . . . , x
n
) Div[a, b] cu () < . Este clar ca o parte a punctelor de diviziune, sa
zicem x
k
, x
k+1
, . . . , x
j
[c , c + ]. Atunci vom avea
S(f, ) s(f, ) =
k1

i=1
(M
i
m
i
)(x
i
x
i1
)+
5.2 Criteriul de integrabilitate al lui Darboux. Clase de funct ii integrabile 189
+
j

i=k
(M
i
m
i
)(x
i
x
i1
)+
n

i=j+1
(M
i
m
i
)(x
i
x
i1
). (5.2.21)

In prima si n a treia suma, utilizam (5.2.20), iar n a doua suma putem scrie
M
i
m
i
M m si
j

i=k
(x
i
x
i1
) < 2,
unde
M := supf(x) [ x [a, b] ,
m := inff(x) [ x [a, b] .
Atunci (5.2.21) ne da
S(f, ) s(f, ) =
k1

i=1
(x
i
x
i1
) + (M m)
j

i=k
(x
i
x
i1
)+ +
n

i=j+1
(x
i
x
i1
)
<
n

i=1
(x
i
x
i1
) + (M m) 2 =
= (b a) + 2(M m) = [b a + 2(M m)].
Cum > 0 este arbitrar, cu criteriul lui Darboux rezulta ca funct ia f este integrabila pe [a, b].
5.2.17 Teorema. Fie f, g : [a, b] R cu proprietat ile
1
0
. f este integrabila Riemann pe [a, b];
2
0
. g(x) = f(x) pentru orice x [a, b]A, unde A [a, b] este nita.
Atunci funct ia g este integrabila pe [a, b] si avem
b
_
a
g(x) d x =
b
_
a
f(x) d x. (5.2.22)
Demonstrat ie. Vom presupune, pentru simplicare, ca mult imea A are un singur punct c, deci A
= c. Evident f este marginita, [ f(x) [ M
t
, M
t
> 0 si notand M = max M
t
, [ g(c)[ , atunci este
clar ca vom avea
[ f(x) [ M , [ g(x) [ M , x [a, b]. (5.2.23)
Faptul ca f este integrabila, nseamna ca pentru orice > 0, exista un () > 0 astfel ncat pentru
orice Div[a, b] cu () < si orice P() sa avem

(f, , )
b
_
a
f(x) d x

<

2
. (5.2.24)
Alegem o diviziune := (x
0
, x
1
, . . . , x
n
) cu () < min
_

8M
,
_
si un sistem de puncte intermediare
=
1
,
2
, . . . ,
n
P().
Daca punctul c este un punct al diviziunii , e c = x
j
, atunci este posibil ca c =
j
sau c =
j+1
.
Vom putea scrie atunci, cu ipoteza 2
0
[ (f, , ) (g, , ) [ =

i=1
[f(
i
)g(
i
)](x
i
x
i1
)

=
= [ [f(
j
)g(
j
)] (x
j
x
j1
) + [f(
j+1
)g(
j+1
)] (x
j+1
x
j
)[
[ f(
j
)g(
j
) [ (x
j
x
j1
) +[ f(x
j+1
)g(
j+1
) [ (x
j+1
x
j
)
190 5. Funct ii integrabile Riemann si primitive
( [ f(
j
) [ +[ g(
j
)[) () + (f [(
j+1
) [ +[g (
j+1
)[) ()
4 M () < 4 M

8 M
=

2
.
Daca punctul c nu este punct de diviziune n , e atunci x
j1
< c < x
j
si n acest caz c ar putea
coincide cu
j
. Cu ipoteza 2
0
, putem scrie
[ (f, , ) (g, , ) [ = [ f(
j
)g(
j
) [ (x
j
x
j1
)
(f[(
j
) [ +[g(
j
) [ ) () 2M () <

4
<

2
.
Asadar, oriunde s-ar aa punctul c n [a, b], putem scrie
[ (f, , ) (g, , ) [ <

2
. (5.2.25)
Folosind acum (5.2.24) si (5.2.25) avem

(g, , )
b
_
a
f(x)dx

(g, , ) (f, , ) +(f, , )


b
_
a
f(x)dx

[(g, , ) (f, , ) [ +

(f, , )
b
_
a
f(x) d x

<

2
+

2
< ,
ceea ce arata ca g este integrabila pe [a, b] si are loc (5.2.22).
5.2.18 Exemplu. Funct ia g : [1, 2] R, data prin
g(x) =
_
2 , daca x [1, 2[ ,
5 , daca x = 2 ,
este integrabila Riemann pe [1, 2] si
2
_
1
g(x) d x = 2.

Intr-adevar, funct ia f : [1, 2] R, f(x) = 2 oricare ar x [1, 2], conform exemplului 5.1.13,1
0
este
integrabila si
2
_
1
f(x) d x = 2 (2 1) = 2.
Pe de alta parte, avem f(x) = g(x), pentru orice x [1, 2]2. Aplicand teorema precedenta,
deducem ca g este integrabila pe [1, 2] si avem
2
_
1
g(x) d x =
2
_
1
f(x) d x = 2.
5.2.19 Observat ie. Teorema 5.2.17 ne permite sa modicam valorile unei funct ii f : [a, b] R
integrabile, ntr-un numar nit de puncte din [a, b], si funct ia obt inuta sa ramana tot integrabila si sa
aiba aceeasi integrala ca si f.
Pentru a demonstra un nou criteriu de integrabilitate Riemann, criteriul lui Lebesgue, care
foloseste doar structura mult imii punctelor de continuitate (sau discontinuitate) ale funct iei, avem nevoie
de not iunile de mult ime de masura Jordan nula si de mult ime de masura Lebesgue nula.
5.2.20 Denit ie. (Mult ime de masura Lebesgue nula). Spunem ca mult imea A R este
de masura Lebesgue nula (sau este neglijabila) daca pentru orice > 0, exista o familie nita sau
5.2 Criteriul de integrabilitate al lui Darboux. Clase de funct ii integrabile 191
numarabila de intervale deschise I
n
=] a
n
, b
n
[, care acopera mult imea A si suma lungimilor lor este mai
mica decat , adica
A

_
nI
0
] a
n
, b
n
[;

nI
0
(b
n
a
n
) < ,
unde I
0
N, este o submult ime nita sau numarabila a lui N.
5.2.21 Exemplu. a) Orice mult ime numarabila din R este de masura Lebesgue nula.

Intr-adevar,
e sirul A = (x
n
) de numere reale, deci o mult ime numarabila si sa consideram intervalele
I
n
:=
_
x
n


2 3
n
, x
n
+

2 3
n
_
, > 0.
Este clar ca
A

_
n=1
_
x
n


2 3
n
, x
n
+

2 3
n
_
si

n=1
(b
n
a
n
) =

n=1
1
3
n
=

2
< ,
deci sirul A = (x
n
) este o mult ime de masura Lebesgue nula.
b)

In particular, mult imea numerelor rat ionale este neglijabila, de asemenea si mult imile N si Z.
Cu acest exemplu se deduce imediat ca si o mult ime nita
1
,
2
, . . . ,
n
R este de masura
Lebesgue nula.
Pentru mult imile din R de masura Lebesgue nula avem:
5.2.22 Teorema. 1
0
. Orice submul t ime a unei mul t imi de masura Lebesgue nula, este de masura
Lebesgue nula;
2
0
. O reuniune cel mult numarabila de mult imi de masura Lebesgue nula este o mult ime de masura
Lebesgue nula.
Demonstrat ie. Sa demonstram doar propozit ia 2
0
. Fie (A
n
), n N
t
, o familie nita sau
numarabila de mult imi de masura Lebesgue nula si > 0. Atunci pentru orice n N
t
(mult ime cel
mult numarabila de indici), exista o familie cel mult numarabila de intervale I
nk
=] a
n
k
, b
n
k
[ , k N
tt
N,
astfel ncat
A
n

_
kN

] a
n
k
, b
n
k
[ si

kN

(b
n
k
a
n
k
) <

2
n
.
De aici se deduce ca
_
nN

A
n

_
nN

_
_
kN

] a
n
k
, b
n
k
[
_
,

nN

_

kN

(b
n
k
a
n
k
)
_
<

nN

2
n
<

n=1

2
n
= ,
ceea ce arata ca familia de mult imi (A
n
)
nN
este de masura nula.
5.2.23 Denit ie. (Mult ime de masura Jordan nula). O mult ime A R se spune ca este de
masura Jordan nula, daca pentru orice > 0, exista o familie nita de intervale I
k
=] a
k
, b
k
[ , k = 1, n
cu proprietat ile
A
n
_
k=1
] a
k
, b
k
[ si
n

k=1
(b
k
a
k
) < .
5.2.24 Exemplu. O mult ime nita A =
1
,
2
, . . . ,
n
R este de masura Jordan nula.

Intr-adevar pentru > 0 sa luam intervalele:


I
k
:=
_
a
k


4n
,a
k
+

4n
_
, k = 1, n.
192 5. Funct ii integrabile Riemann si primitive
Vom avea
A
n
_
k=1
_


4n
,
k
+

4n
_
si
n

k=1
_

k
+

4n

k
+

4n
_
=
n

k=1

2n
=

2
< ,
adica A este de masura Jordan nula.
5.2.25 Observat ie. Este evident ca orice mult ime de masura Jordan nula este de masura Lebesgue
nula, dar reciproca nu este n general adevarata.
5.2.26 Observat ie. Se spune despre proprietatea P referitoare la punctele unei mult imi A R ca
are loc aproape peste tot (a.p.t) pe mult imea A, daca mult imea punctelor din A n care proprietatea
P nu are loc este de masura Lebesgue nul a.
De exemplu, spunem ca funct ia f : [a, b]R este continua aproape peste tot pe [a, b], daca mult imea
punctelor de discontinuitate ale lui f are masura Lebesgue nula (este neglijabila).
5.2.27 Exemplu. Funct ia f : R R, data prin f(x) = [x] (partea ntreaga a lui x) pentru orice
x R, este continua a.p.t. pe R deoarece mult imea punctelor sale de discontinuitate este egala cu Z si
deci o mult ime de masura Lebesgue nula (exemplul 5.2.21.b).
5.2.28 Exemplu. Funct ia Dirichlet f : [0, 1] R, data prin
f(x) =
_
0, daca x [0, 1]
1, daca x [0, 1],
nu este continua a.p.t. pe [0, 1] deoarece mult imea punctelor sale de discontinuitate este ntreg intervalul
[0, 1], care nu este de masura Lebesgue nula.
5.2.30 Denit ie. Daca f : [a, b] R este o funct ie marginita pe [a, b] si D [a, b]. Numarul

f
(D) := sup f(D) inf f(D), (5.2.26)
se numeste oscilat ia funct iei f pe mult imea D, iar daca x [a, b] numarul

f
(x) := inf
f
([a, b]] x r, x +r [) ; r > 0, (5.2.27)
se numeste oscilat ia funct iei f n punctul x.
5.2.31 Teorema. Funct ia f : [a, b] R, marginita este continua n punctul x
0
[a, b] daca si
numai daca
f
(x
0
) = 0.
Demonstrat ie. Necesitatea. Funct ia f este continua n x
0
implica faptul ca pentru orice > 0,
exista () > 0 astfel ncat pentru orice x [a, b]] x
0
, x
0
+ [ sa avem
[ f(x) f(x
0
) [ <

3
. (5.2.28)
De aici, daca u, v [a, b] ] x
0
, x
0
+ [ deducem
[ f(u) f(v) [ [ f(u) f(x
0
) [ +[ f(x
0
) f(v) [ <
2
3
,
si atunci

f
([a, b] ] x
0
, x
0
+ [) = supf(u) f(v)

u, v [a, b] ] x
0
, x
0
+ [
2
3
.
Rezulta acum
0
f
(x
0
) = inf
f
([a, b]] x
0
, x
0
+ [) [ > 0
2
3
< ,
pentru orice > 0, ceea ce nseamna ca
f
(x
0
) = 0.
5.2 Criteriul de integrabilitate al lui Darboux. Clase de funct ii integrabile 193
Sucient a. Presupunem ca

f
(x
0
) = inf
f
( [a, b] ] x
0
, x
0
+ [) [ > 0 = 0 .
Daca > 0, atunci exista r > 0 astfel ncat

f
([a, b] ] x
0
r, x
0
+r [ ) < .
Avand
[ f(u) f(v) [
f
([a, b] ] x
0
r, x
0
+r [ )
oricare ar u, v [a, b] ] x
0
r, x
0
+r [, deducem ca pentru x [a, b] ] x
0
r, x
0
+r [ avem
[ f(x) f(x
0
) [ < .
Aceasta arata continuitatea funct iei f n punctul x
0
.
5.2.32 Observat ie. Daca f : [a, b] R este discontinua n punctul x
0
, atunci
f
(x
0
) > 0. De
exemplu, pentru funct ia f : R R,
f(x) =
_
0 , dac a x ,
1 , dac a x R,
avem
f
(x
0
) = 1, oricare ar x R. Se deduce ca f este discontinua n orice x R.
5.2.33 Teorema. Daca f : [a, b] R este marginita, atunci pentru orice > 0, mult imea
D = x [a, b] [
f
(x) este compacta.
Demonstrat ie. Este sucient sa aratam ca D este nchisa, deci ca si cont ine toate punctele sale
de acumulare. Sa presupunem contrarul, ca exista x
0
[a, b], punct de acumulare al lui D dar x
0
/ D.
Atunci exista r > 0 astfel ncat

f
(D] x
0
r, x
0
+r [) < .

Insa x
0
ind punct de acumulare al lui D si ] x
0
r, x
0
+ r [ o vecinatate a sa, nseamna ca exista
cel put in un x D]x
0
r, x
0
+r [ si deci
f
(x) .

Insa

f
(D] x
0
r, x
0
+r [)
f
(x) .
Am ajuns la o contradict ie. Deducem ca x
0
D, deci D este nchisa, ceea ce implica D compacta.
5.2.34 Teorema. Daca funct ia f : [a, b] R este integrabila Riemann pe [a, b], atunci pentru orice
> 0, mult imea
D := x [a, b] [
f
(x) ,
este de masura Jordan nula.
Demonstrat ie. Fie > 0 si = (x
0
, x
1
, . . . , x
n
) Div[a, b]. Cum f este integrabila, cu criteriul lui
Darboux se deduce ca
S (f, ) s (f, ) < . (5.2.29)
Evident, o parte dintre intervalele part iale [x
i1
, x
i
] au puncte comune cu D. Atunci, sa notam cu
N
t
1, 2, . . . , n mult imea acelor indici i 1, 2, . . . , n pentru care [x
i1
, x
i
] D ,= .

Inseamna ca
[a, b]
_
iN

[x
i1
, x
i
] D (5.2.30)
si pentru orice i N
t
si orice x [x
i1
, x
i
] D, avem

f
(x)
f
([x
i1
, x
i
]) = M
i
m
i
.
De aici, deducem (x
i
x
i1
) (M
i
m
i
)(x
i
x
i1
) , i N
t
, inegalitat i pe care le nsumam, si
gasim

iN

(x
i
x
i1
)

iN

(M
i
m
i
)(x
i
x
i1
)
194 5. Funct ii integrabile Riemann si primitive

i=1
(M
i
m
i
)(x
i
x
i1
) = S (f, ) s (f, ) < .
Simplicand cu , rezulta ca

iN

(x
i
x
i1
) < .
Am demonstrat astfel ca intervalele [x
i1
, x
i
], i N
t
acopera mult imea D si suma lungimilor lor
este mai mica decat > 0, deci D este o mult ime de masura Jordan nula.
5.2.35 Teorema. (Criteriul lui Paul de Bois Reymond de integrabilitate Riemann).
Funct ia f : [a, b] R este integrabila Riemann pe [a, b] daca si numai daca f este marginita si pentru
orice > 0 mul t imea
D

:= x [a, b] [
f
(x) ,
este de masura Jordan nula.
Demonstrat ie. Necesitatea rezulta din teoremele 5.1.14 si 5.2.34, iar la demonstrarea sucient ei
renunt am, datorita ntinderii ei.
5.2.36 Teorema. (Criteriul lui Lebesgue de integrabilitate Riemann). Funct ia f : [a, b]R
este integrabila Riemann pe [a, b] daca si numai daca f este marginita pe [a, b] si continua aproape peste
tot pe [a, b].
Demonstrat ie. Necesitatea. Presupunem ca f este integrabila Riemann pe [a, b]. Atunci ea este
marginita, pe baza teoremei 5.1.14. Pe baza teoremei 5.2.35, pentru orice n N

, mult imea
D
n
:=
_
x [a, b]

f
(x)
1
n
_
,
este de masura Jordan nula si deci si de masura Lebesgue nula. Pe baza teoremei 5.2.22, deducem ca si
mult imea
D(f) :=

_
n=1
D
n
,
este de masura Lebesgue nula.

Insa D(f) = x [a, b] [
f
(x) > 0 este chiar mult imea punctelor de
discontinuitate ale lui f si deci mult imea punctelor de discontinuitate ale lui f este de masura Lebesgue
nula, ceea ce este echivalent cu faptul ca f este continua aproape peste tot pe [a, b].
Sucient a. Sa presupunem acum ca f este marginita si ca ea este continua aproape peste tot pe
[a, b]. Fie > 0 si sa consideram mult imea
D

= x [a, b] [
f
(x) .
Trebuie sa aratam ca D

este de masura Jordan nula. Fie pentru aceasta un alt numar


t
> 0. Din
faptul ca f este continua a.p.t pe [a, b], rezulta ca mult imea punctelor sale de discontinuitate
D(f) = x [a, b] [
f
(x) > 0 ,
este de masura Lebesgue nula. Deci exista o familie cel mult numarabila de intervale dechise ( ] a
n
, b
n
[ ),
n N
t
astfel ncat
D(f)
_
nN

] a
n
, b
n
[ si

nN

(b
n
a
n
) <
t
.

Insa D

D(f), adica familia de intervale ( ] a


n
, b
n
[ ) , n N
t
acopera si pe D

, si cum D

este
compacta (teorema 5.2.33) rezulta ca exista o subacoperire nita (] a
i
, b
i
[), i I
0
, I
0
N, I
0
nita,
astfel ncat
D


_
iI
0
] a
i
, b
i
[ si

iI
0
(b
i
a
i
)

nN

(b
n
a
n
) <
t
.
Rezulta astfel ca mult imea D

are masura Jordan nula si cu teorema 5.2.35 se deduce ca f este


integrabila Riemann pe [a, b].
5.3 Operat ii cu funct ii integrabile Riemann. Propriet at ile funct iilor integrabile si ale integralei Riemann 195
5.3 Operat ii cu funct ii integrabile Riemann. Proprietat ile funct iilor
integrabile si ale integralei Riemann
5.3.1 Teorema. (Linearitatea integralei Riemann). Daca funct iile f, g : [a, b] R sunt
integrabile Riemann atunci funct ia f + g este integrabila Riemann pentru orice , R si
b
_
a
[ f(x)+ g(x)] d x =
b
_
a
f(x) d x +
b
_
a
g(x) d x. (5.3.1)
Demonstrat ie. Utilizand denit ia integrabilitat ii si a integralei (denit ia 1.1.11), pentru > 0 va
exista numarul
t
( ) > 0 astfel ncat

(f, , )
b
_
a
f(x) d x

<

1 +[ [ +[ [
, (5.3.2)
pentru orice Div[a, b] cu () <
t
. Analog pentru funct ia g exista
tt
( ) > 0 astfel ncat

(g, , )
b
_
a
g(x) d x

<

1 +[ [ +[ [
, (5.3.3)
pentru Div[a, b] cu () <
tt
si oricare ar P() un sistem de puncte intermediare asociat
diviziunii .
Fie acum Div[a, b] cu () < min
t
,
tt
= si P(). Atunci, utilizand (5.3.2) si (5.3.3),
obt inem

(f + g, , )
_
_

b
_
a
f(x) d x+
b
_
a
g(x) d x
_
_

=
=

_
_
(f, , )
b
_
a
f(x) d x
_
_
+
_
_
(g, , )
b
_
a
g(x) dx
_
_

[[

(f, , )
b
_
a
f(x) d x

+[[

(g, , )
b
_
a
g(x) d x

[[

1 +[ [ +[ [
+[[

1 +[ [ +[ [
=
[[ +[[
1 +[[ +[[
< .
De aici se deduce ca f + g este integrabila si are loc (5.3.1).
Din aceasta teorema se deduce ca f+g, fg si cf sunt integrabile Riemann, daca f, g sunt integrabile
si c este o constanta si n plus
_
b
a
[f(x) g(x)] dx =
_
b
a
f(x)dx g(x)dx,
_
b
a
cf(x)dx = c
_
b
a
f(x)dx.
5.3.2 Teorema. Produsul a doua funct ii integrabile Riemann este o funct ie integrabila Riemann.
Demonstrat ie. Fie f, g : [a, b] R, doua funct ii integrabile Riemann. Atunci ele sunt marginite
(teorema 5.1.14) si produsul lor f g va o funct ie marginita. Fie
D(f) : = x [a, b] [
f
(x) > 0 ,
D(g) : = x [a, b] [
g
(x) > 0 ,
mult imile punctelor de discontinuitate ale celor doua funct ii. Mult imile D(f) si D(g) sunt de masura
Lebesgue nula pe baza criteriului lui Lebesgue. Mult imea punctelor de discontinuitate ale funct iei produs
196 5. Funct ii integrabile Riemann si primitive
f g va D(f g) D(f)D(g) si atunci aceasta mult ime D(f g) are masura Lebesgue nula. Prin urmare,
funct ia f g este continua aproape peste tot pe [a, b] si pe baza criteriului lui Lebesgue ea va integrabila
Riemann.
5.3.3 Observat ie. Cu toate ca produsul f g a doua funct ii integrabile este o funct ie integrabila,
integrala produsului nu este egala cu produsul integralelor.
5.3.4 Teorema. Daca funct iile f, g : [a, b] R sunt integrabile Riemann pe [a, b], g(x) ,= 0 oricare
ar x [a, b] si
1
g
este marginita pe [a, b] atunci catul
f
g
este o funct ie integrabila pe [a, b].
Demonstrat ie. Cum g(x) ,= 0 pentru orice x [a, b], nseamna ca funct ia
1
g
este continua n
aceleasi puncte ca si g si deci
1
g
este continua a.p.t pe [a, b]. Funct ia f de asemenea ind integrabila, este
continua a.p.t pe [a, b]. Atunci, cu teorema 5.3.2 se deduce ca f
1
g
=
f
g
este integrabila Riemann.
5.3.5 Observat ie. Daca a = b, deci [a, b] = a si f : a R, atunci f este integrabila pe [a, a] si
avem
b
_
a
f(x) d x = 0.

Intr-adevar, n acest caz orice diviziune a intervalului [a, a] este = a si orice sistem de puncte
intermediare asociat lui este = a si atunci (f, , ) = f(a)(a a) = 0, deci I = 0. De asemenea
daca f : [a, b] R este integrabila pe [a, b] atunci se considera, prin convent ie, ca este integrabila si pe
[b, a] si avem
b
_
a
f(x) dx :=
a
_
b
f(x) dx. (5.3.4)
5.3.6 Teorema. Daca f : [a, b] R este integrabila Riemann pe [a, b] si daca f(x) 0 pentru orice
x [a, b], atunci avem
b
_
a
f(x) d x 0.
Demonstrat ie. Fie
n
:= (x
n
0
, x
n
1
, . . . , x
n
k
n
) Div[a, b] , n N

, un sir de diviziuni ale lui [a, b] cu


lim
n
(
n
) = 0 si e
n
= (
n
1
,
n
2
, . . . ,
n
k
n
) P(
n
) un sir de sisteme de puncte intermediare. Atunci
sirul sumelor Riemann asociate este
(f,
n
,
n
) =
n

i=1
f(
n
k
i
)(x
n
k
i
x
n
k
i1
) 0,
si atunci avem
lim
n
(f,
n
,
n
) =
b
_
a
f(x) d x 0.
5.3.7 Teorema. Daca funct iile f, g : [a , b] R sunt integrabile Riemann pe [a, b] si daca
f(x) g(x) pentru orice x [a, b], atunci
b
_
a
f(x) d x
b
_
a
g(x) d x. (5.3.5)
Demonstrat ie. Fie funct ia h : [a, b] R, h(x) = g(x) f(x) , x [a, b]. Prin ipoteza avem
h(x) 0 pentru orice x [a, b] si atunci aplicand teorema 5.3.6. si teorema 5.3.1, obt inem
b
_
a
h(x) dx =
b
_
a
g(x) dx
b
_
a
f(x) dx 0,
5.3 Operat ii cu funct ii integrabile Riemann. Propriet at ile funct iilor integrabile si ale integralei Riemann 197
ceea ce este echivalent cu (5.3.5).
5.3.8 Corolar. Daca f, g : [a, b] R sunt integrabile pe [a, b], g(x) 0 pentru orice x [a, b],
atunci avem
m
b
_
a
g(x) d x
b
_
a
f(x) g(x) d x M
b
_
a
g(x) d x, (5.3.6)
unde m := inf
x[a,b]
f(x) si M := sup
x[a,b]
f(x).
Demonstrat ie. Avem, evident, pentru orice x [a, b]
m f(x) M
si cum g(x) 0, se obt ine
m g(x) f(x) g(x) M g(x).
Deoarece f g este integrabila, aplicand teorema 5.3.7 avem
m
b
_
a
g(x) d x
b
_
a
f(x) g(x) d x M
b
_
a
g(x) d x.
5.3.9 Corolar. Daca f:[a, b]R este integrabila Riemann pe [a, b], atunci
m(b a)
b
_
a
f(x) d x M (b a), (5.3.7)
unde m si M sunt marginile funct iei f pe [a, b].
Demonstrat ie. Demonstrat ia rezult a imediat, daca n corolarul 5.3.8 consideram funct ia g(x) = 1,
pentru orice x [a, b].
5.3.10 Teorema. Daca f:[a, b] R este integrabila Riemann pe [a, b], atunci funct ia [ f [ este
integrabila pe [a, b] si avem

b
_
a
f(x) d x

b
_
a
[f(x)[ d x. (5.3.8)
Demonstrat ie. Cum f este integrabila Riemann pe [a, b], ea este marginita pe [a, b]. Atunci, si
[ f [ este marginita. Daca D(f) este mult imea punctelor de discontinuitate a lui f, atunci D(f) este o
mult ime neglijabila. Atunci, mult imea punctelor de discontinuitate ale lui [ f [ este D( [ f [ ) D(f), deci
ea are masura Lebesgue nula. Deducem ca funct ia [ f [ este integrabila Riemann pe [a, b]. Evident, mai
avem, pentru orice x [a, b]
[ f(x) [ f(x) [ f(x) [ ,
si atunci cu (5.3.5) obt inem

b
_
a
[f(x)[ dx
b
_
a
f(x) dx
b
_
a
[ f(x) [ dx,
adica

b
_
a
f(x) d x

b
_
a
[f(x) [ d x. (5.3.9)
5.3.11 Teorema. (Prima formula generalizata de medie). Daca f, g : [a, b] R sunt
integrabile Riemann pe [a, b] si daca g(x) 0 pentru orice x [a, b], atunci exista un numar [m, M],
unde m si M sunt marginile funct iei f, astfel ncat
b
_
a
f(x) g(x) d x =
b
_
a
g(x) d x. (5.3.10)
198 5. Funct ii integrabile Riemann si primitive
Demonstrat ie. Daca m := inf
x[a,b]
f(x) si M := sup sup
x[a,b]
f(x), atunci dupa (5.3.6) avem
m
b
_
a
g(x) d x
b
_
a
f(x) g(x) d x M
b
_
a
g(x) d x. (5.3.11)
Cum g(x) 0 avem
b
_
a
g(x) d x 0. Daca
b
_
a
g(x) d x > 0, mpart im inegalitat ile (5.3.11) cu
b
_
a
g(x) d x
si gasim
m
b
_
a
f(x) g(x) d x
b
_
a
g(x) d x
M.
Daca notam cu
:=
b
_
a
f(x) g(x) dx
b
_
a
g(x) dx
,
atunci m M si
b
_
a
f(x) g(x) d x =
b
_
a
g(x) d x.
Daca, nsa
b
_
a
g(x) d x = 0, atunci din (5.3.11) obt inem
b
_
a
f(x) g(x) d x = 0. Atunci putem scrie
pentru orice [m, M]
b
_
a
f(x) g(x) d x =
b
_
a
g(x) d x.
5.3.12 Corolar. Daca f : [a, b] R este continua, iar g : [a, b] R este integrabila Riemann pe
[a, b] si g(x) 0, x [a, b], atunci exista cel put in un c [a, b] astfel ncat
b
_
a
f(x) g(x) d x = f(c)
b
_
a
g(x) d x. (5.3.12)
Demonstrat ie. Daca f este continua pe [a, b], atunci f are proprietatea lui Darboux pe [a, b] si
deci pentru orice m M exista c [a, b] astfel ncat f(c) = . Acum din teorema 5.3.11 rezulta
concluzia (5.3.12).
Pentru g(x) 1, din (5.3.10), se obt ine prima formula de medie
b
_
a
f(x) d x = (b a) , [m, M ].
5.3.13 Corolar. Daca f : [a, b]R este continua atunci exista c [a, b] astfel ncat
b
_
a
f(x) d x = f(c) (b a). (5.3.15)
Formula (5.3.13) se numeste prima formula de medie pentru funct ii continue. Formula rezulta
direct din (5.3.12) daca se ia g(x) = 1 pentru orice x [a, b].
Are loc si teorema urmatoare:
5.3 Operat ii cu funct ii integrabile Riemann. Propriet at ile funct iilor integrabile si ale integralei Riemann 199
5.3.14 Teorema. (Teorema a doua de medie). Daca f, g : [a,b] R satisfac condit iile:
(i) f este integrabila Riemann pe [a, b];
(ii) g este monotona pe [a, b], atunci are loc relat ia
b
_
a
f(x) g(x) d x = g(a)
c
_
a
f(x) d x +g(b)
b
_
c
f(x) d x, c [a, b]. (5.3.14)
Renunt am la demonstrarea teoremei, ntrucat demonstrat ia necesita anumite proprietat i ale integralei cu
limita superioara variabila, care se vor studia n paragraful 5.4 al acestui capitol.
5.3.15 Teorema. (Aditivitatea integralei ca funct ie de interval). Daca f : [a, b] R este
integrabila Riemann pe [a, b] atunci pentru orice c ] a, b [ funct ia f va integrabila Riemann atat pe [a, c]
cat si pe [c, b] si are loc relat ia
b
_
a
f(x) dx =
c
_
a
f(x) d x +
b
_
c
f(x) d x. (5.3.15)
Demonstrat ie. Fie f integrabila Riemann pe [a, b] si c ]a, b[. Atunci f este marginita si continua
a.p.t. pe [a, b].

Inseamna ca oricare ar intervalul [, ] [a, b] funct ia f va marginita pe intervalul
[, ] si de asemenea continua a.p.t. pe [, ], deci integrabila pe [, ]. Deducem ca f este integrabila
Riemann pe [a, c] si pe [c, b]. Mai trebuie sa demonstram egalitatea (5.3.15). Fie
t
Div [a, c] si

tt
Div [c, b] diviziuni oarecare si =
t

tt
Div [a, b]. Pentru > 0 exista () > 0 astfel ncat daca
(
t
) < si (
tt
) < si oricare ar
t
P(
t
) si
tt
P(
tt
) vom avea

(f,
t
,
t
)
c
_
a
f(x) d x

<

2
, (5.3.16)

(f,
tt
,
tt
)
b
_
c
f(x) d x

<

2
. (5.3.17)
Este clar ca () (
t
) < si atunci pentru =
t

tt
P (), avem
(f, , ) = (f,
t
,
t
) + (f,
tt
,
tt
).
Mai avem

(f, , )
_
_
c
_
a
f(x) d x +
b
_
c
f(x) dx
_
_

(f,
t
,
t
)
c
_
a
f(x) dx+
+ (f,
tt
,
tt
)
b
_
c
f(x) d x

(f,
t
,
t
)
c
_
a
f(x) d x

+
+

(f,
tt
,
tt
)
b
_
c
f(x) d x

<

2
+

2
= .
Aceasta arata ca
c
_
a
f(x) d x +
b
_
c
f(x) d x =
b
_
a
f(x) dx.
Se mai poate demonstra o varianta a teoremei precedente
200 5. Funct ii integrabile Riemann si primitive
5.3.16 Teorema. Fie f : [a, b] R si c ] a, b [. Daca funct ia f este integrabila Riemann pe [a, c]
si pe [c, b], atunci f este integrabila Riemann pe [a, b] si are loc (5.3.15).
Demonstrat ie. Sa presupunem ca f este integrabila Riemann atat pe [a, c] cat si pe [c, b]. Atunci
f este marginita atat pe [a, c] cat si pe [c, b], deci f va marginita si pe [a, b]. De asemenea daca A este
mult imea punctelor de discontinuitate ale lui f pe [a, c] si B este mult imea punctelor de discontinuitate
ale lui f pe [c, b], atunci A si B au masura Lebesgue nula si mult imea punctelor de discontinuitate ale lui
f pe [a, b] va A B si va tot de masura Lebesgue nula (Teorema 5.2.22). Asadar f va integrabila
Riemann pe [a, b]. Egalitatea (5.3.15) se demonstreaza ca si la Teorema 5.3.15.
5.3.17 Denit ie. Fie f: I R R, unde I este un interval (posibil chiar R). Spunem ca f este
local integrabila Riemann pe I daca f este integrabila Riemann pe orice interval compact [a, b] I,
cu a < b.
5.3.18 Exemplu. Fie I R un interval (nedegenerat) si f : IR o funct ie continua. Atunci f
este local integrabila pe I.

Intr-adevar, pentru orice [a, b] I, f va continua pe [a, b], deci integrabila
Riemann pe [a, b] si astfel va local integrabila pe I.
5.3.19 Teorema. Daca I R este un interval nedegenerat si f: I R o funct ie local integrabila
pe I, atunci oricare va a,b,c I avem
b
_
a
f(x) d x =
c
_
a
f(x) d x +
b
_
c
f(x) d x. (5.3.18)
Demonstrat ie. Pentru demonstrat ia relat iei (5.3.18) deosebim mai multe cazuri:
a) a = b = c, atunci este clar ca avem
b
_
a
f(x) d x = 0 ,
c
_
a
f(x) d x = 0 ,
b
_
c
f(x) d x = 0 ,
si evident, are loc (5.3.18).
b) a = b ,= c, caz n care
b
_
a
f(x) d x = 0 ,
c
_
a
f(x) d x =
b
_
c
f(x) d x =
b
_
c
f(x) d x,
si egalitatea rezulta imediat.
Analog se trateaza cazurile a ,= b = c si a = c ,= b.
c) a < c < b, atunci pe baza Teoremei 5.3.15, are loc relat ia (5.3.18);
d) a < b < c, tot pe baza Teoremei 5.3.15, avem
c
_
a
f(x) dx =
b
_
a
f(x) dx +
c
_
b
f(x) dx,
iar de aici obt inem
b
_
a
f(x) d x =
c
_
a
f(x) d x
c
_
b
f(x) d x =
c
_
a
f(x) d x +
b
_
c
f(x) d x.
Celelalte posibilitat i de ordonare b < a < c; b < c < a; c < a < b; c < b < a, se discuta n mod
analog.
Putem acum da o teorema mai tare decat Teorema 5.3.6.
5.3.20 Teorema. Daca funct ia f : [a, b] R, a < b este continua, pozitiva si neidentic nula pe
[a, b], atunci avem
b
_
a
f(x) d x > 0. (5.3.19)
5.4 Primitive. Integrala nedenit a. Primitivabilitatea funct iilor continue 201
Demonstrat ie. Daca f este pozitiva si neidentic nula pe [a , b] atunci exista cel put in un punct
x
0
[a , b] astfel ncat f(x
0
) > 0. Fiind continua pe [a , b], deci si n x
0
, exista o vecinatate,
V =]x
0
, x
0
+ [ , > 0 astfel ncat f(x) > 0 oricare ar x ]x
0
, x
0
+ [ [a, b].
Evident, exista [c, d] ] x
0
, x
0
+ [ [a, b] si deci f(x) > 0 pentru orice x [c, d]. Daca notam
cu m = inf
x[c,d]
f(x) , pe baza teoremei Weierstrass, aceasta margine inferioara este atinsa, adica exista
x
t
[c, d] astfel ncat m = f(x
t
) > 0 si nca f(x) m > 0, oricare ar x [c, d].
Aplicand Corolarul 5.3.9 pentru [c, d] [a, b], avem
0 < m(d c)
d
_
c
f(x) d x
b
_
a
f(x) d x, (5.3.20)
adica chiar (5.3.19).
5.4 Primitive. Integrala nedenita. Primitivabilitatea funct iilor
continue
5.4.1 Denit ie. Fie D R, nevida, f:D R si I D, nevida. Spunem ca funct ia f admite
primitive (este primitivabila, este o derivata) pe I, daca exista o funct ie F: I R, derivabila pe
I si cu F
t
(x) = f(x) oricare ar x I. Funct ia F se numeste o primitiva a funct iei f pe mult imea I.
Daca f admite primitive pe ntreg domeniul D atunci spunem simplu ca f admite primitive (fara a
mai preciza mult imea D).
5.4.2 Exemplu. Funct ia f:R R, f(x) = x
2
pentru orice x R, admite primitive, deoarece
funct ia F:R R, F(x) =
x
3
3
+ 1, este derivabila pe R si F
t
(x) = x
2
= f(x), oricare ar x R.
5.4.3 Teorema. Daca f: I RR, I este un interval , admite primitive si daca F
1
, F
2
: I R
sunt doua primitive ale sale, atunci
F
1
(x) F
2
(x) = c = const , oricare ar x I.
Demonstrat ie.

Intr-adevar, avem F
t
1
= F
t
2
= f, adica (F
1
F
2
)
t
= 0 si funct ia F
1
F
2
are derivata
nula pe intervalul I R. Rezulta ca ea este o constanta pe acest interval, deci exista c R astfel ncat
F
1
(x) F
2
(x) = c = const , pentru orice x I.
5.4.4 Observat ie. Daca f: I R, dar I nu este un interval, atunci teorema precedenta nu este
adevarata dupa cum se vede din urmatorul exemplu. Fie f :] , 0 []0, +[R, data prin f(x) = 0,
pentru orice x R0. Funct iile F
1
, F
2
: R0 R, date prin F
1
(x) = 1, pentru orice x R0 si
F
2
(x) =
_
3 , x < 0 ,
0 , x > 0 ,
sunt primitive ale lui f pe R0 deoarece F
t
1
(x) = 0 = f(x) , x R0 iar F
t
2
(x) = 0 = f(x) pentru
orice x R0. Cu toate acestea diferent a lor este
F
2
(x) F
1
(x) =
_
2 , x ] , 0 [ ,
1 , x ] 0, + [ ,
adica nu exista un c R astfel ncat F
2
(x) F
1
(x) = c pentru orice x R0.
5.4.5 Teorema. Fie D R nevida, f:D R si I D un interval nedegenerat. Daca f admite
primitive pe I atunci f are proprietatea lui Darboux pe I.
Demonstrat ie. Daca f admite primitive pe I, atunci e F:IR una dintre primitive, deci o
funct ie derivabila pe I si cu F
t
(x) = f(x) pentru orice x I.

Insa derivata oricarei funct ii derivabile
pe un interval are proprietatea lui Darboux (Teorema lui Darboux). Astfel ca f are proprietatea lui
Darboux pe I.
202 5. Funct ii integrabile Riemann si primitive

In practica, pentru a arata ca o funct ie nu admite primitive pe un interval este sucient sa aratam
ca ea nu are proprietata lui Darboux pe acel interval. De exemplu, funct ia f:R R, data prin f(x) = [x]
(partea ntreaga a lui x), pentru orice x R, nu are proprietatea lui Darboux pe R, asa ca nu admite
primitive pe R.
5.4.6 Teorema. Fie f, g : I R, I R un interval
a) Daca f si g admit primitive pe I atunci funct ia f+g, admite primitive pe I pentru orice , R.
b) Daca f admite primitive pe I si g este derivabila cu derivata g
t
continua atunci funct ia f g admite
primitive pe I.
Demonstrat ia.
a) Fie F, G : I R primitive ale lui f, respectiv g si , R. Atunci
(F +G)
t
= F
t
+G
t
= f +g,
adica f +g admite primitive.
b) Fie F : I R o primitiva a lui f. Avem
(F g)
t
= F
t
g +F g
t
= f g +F g
t
,
adica
f g = (F g)
t
F g
t
.
Cum (F g)
t
admite primitive (pe F g), iar F g
t
e continua deci admite primitive, rezulta (cu punctul
a)) ca produsul lor, adica f g admite primitive pe I.
5.4.7 Denit ie. Fie f: I R R, I un interval nedegenerat. Daca f admite primitive pe I,
mult imea tuturor primitivelor lui f pe I se numeste integrala nedenita a funct iei f si ea se noteaza
cu simbolul
_
f(x) d x, xI.
5.4.8 Observat ie. Daca I R este un interval nedegenerat, vom nota cu C mult imea tuturor
funct iilor constante denite pe I cu valori n R, adica
C = f : I R[ exista c R astfel ncat f(x) = c, pentru orice x I
Cu aceasta notat ie se constata ca
a) C +C = h : I R[exista f C si g C astfel ncat h = f +g = C,
b) C = C, pentru orice R, ,= 0.
Acum putem spune, folosind teorema 5.4.3, ca
_
f(x) dx = F : I R[ F este o primitiva a lui f pe I
= F
0
+c [F
0
este o primitiva a lui f pe I, c C = F
0
+C.
2.4.9 Teorema. Fie I R un interval si f :I R o funct ie local integrabila Riemann pe I. Daca
a I, atunci funct ia F:IR data prin
F(x) =
x
_
a
f(t) d t, (5.4.1)
este continua pe I.
Demonstrat ie. Fie x
0
un punct oarecare din intervalul I. Daca x
0
este interior lui I atunci exista
r > 0 astfel ncat [x
0
r, x
0
+ r] I. Cum f este local integrabila pe I, f este integrabila Riemann
5.4 Primitive. Integrala nedenit a. Primitivabilitatea funct iilor continue 203
pe [x
0
r, x
0
+ r] si deci marginita pe acest interval. Fie M > 0 astfel ncat [ f(x) [ M oricare ar
x [x
0
r, x
0
+r]. Atunci,
[ F(x) F(x
0
) [ =

x
_
a
f(t) d t
x
0
_
a
f(t) d t

=
=

x
_
x
0
f(t) d t

x
_
x
0
[ f(t) [ d t

M [ x x
0
[ , x ] x
0
r, x
0
+r [.
Daca > 0 si luam ( ) = min
_
r,

M
_
, atunci pentru orice x I pentru care [x x
0
[ < , vom avea
[ F(x) F(x
0
)[ < M , adica f este continua n x
0
si deci continua pe I.
Daca x
0
I si este extremitatea stanga a intervalului I, atunci exista r > 0 astfel ncat [x
0
, x
0
+r] I
si f este evident, integrabila pe [x
0
, x
0
+r] si marginita, adica [ f(x) [ M, pentru orice x [x
0
, x
0
+r].
Daca x ] x
0
, x
0
+r [ putem scrie, ca si mai nainte
[ F(x) F(x
0
) [ =

x
_
x
0
f(t) d t

x
_
x
0
[ f(t) [ d t M(x x
0
) < ,
pentru orice > 0, ( ) = min
_
r,

M
_
si x I cu [ x x
0
[ < , si astfel, din nou, funct ia f este
continua n x
0
. Analog se trateaza cazul n care x
0
este extremitatea dreapta a intervalului I.
5.4.10 Teorema. Fie f :I R R, I interval, o funct ie local integrabila Riemann pe I. Daca f
este continua pe punctul x
0
I, atunci pentru orice a I, funct ia F:I R data prin
F(x) :=
x
_
a
f(t) dt,
oricare ar x I, este derivabila n x
0
si F
t
(x
0
) = f(x
0
).
Demonstrat ie. Deoarece f este presupusa continua n x
0
, oricare ar > 0, exista ( ) > 0 astfel
ncat pentru orice t I cu [ t x
0
[ < sa avem [ f(t) f(x
0
) [ < , adica
f(x
0
) < f(t) < f(x
0
) +. (5.4.2)
Fie acum x I, x > x
0
astfel ncat [ x x
0
[ < . Atunci daca t [x
0
, x] avem[ t x
0
[ [ x x
0
[ <
si deci are loc (5.4.2). Integrand (5.4.2) ntre limitele x
0
si x, obt inem
x
_
x
0
(f(x
0
) ) d t
x
_
x
0
f(t) d t
x
_
x
0
(f(x
0
) + ) d t,
sau
(f(x
0
) ) (x x
0
) F(x) F(x
0
) (f(x
0
) + ) (x x
0
).
De aici rezulta
f(x
0
)
F(x) F(x
0
)
x x
0
f(x
0
) +,
adica

F(x) F(x
0
)
x x
0
f(x
0
)

< . (5.4.3)
La aceasta inegalitate se ajunge si daca x < x
0
si t [x, x
0
]. Asadar, pentru orice x I, x ,= x
0
cu
[ x x
0
[ < are loc (5.4.3), ceea ce nseamna ca
lim
x x
0
F(x) F(x
0
)
x x
0
= f(x
0
),
204 5. Funct ii integrabile Riemann si primitive
adica F este derivabila n x
0
si F
t
(x
0
) = f(x
0
).
5.4.11 Teorema. (Teorema de existent a a primitivelor unei funct ii continue). Daca f :
I R R, I-interval nedegenerat, este continua pe I, atunci ea admite primitive pe I pentru orice
a I, funct ia F : I R, denita prin
F(x) :=
x
_
a
f(t) dt,
pentru orice x I, este o primitiva a funct iei f (adica F
t
(x) = f(x), x I) cu proprietatea F(a) = 0.
Concluzia teoremei rezulta imediat din teorema 5.4.10, daca f este continua n orice x I. Teorema
arma ca orice funct ie continua pe un interval admite primitive pe acest interval.
5.4.12 Teorema. (Teorema de reprezentare a primitivelor unei funct ii continue).
Fie f: I R R, I-interval, o funct ie continua pe I. Daca F : I R este o primitiva a lui f pe I cu
proprietatea F(a) = 0 pentru a I, atunci ea se reprezinta sub forma
F(x) :=
x
_
a
f(t) dt (5.4.4)
oricare ar x I.
Demonstrat ie. Am vazut la Teorema 5.4.11 ca funct ia : I R, data prin
(x) :=
x
_
a
f(t) dt,
este o primitiva a lui f pe I. Atunci exista o constanta c astfel ncat F(x) = (x) + c , x I.

Insa
F(a) = (a) = 0 si deci c = 0, astfel ca F(x) = (x), pentru orice x I.
5.4.13 Observat ie. Teorema precedenta arma ca orice funct ie continua pe un interval admite
primitive, nsa aceasta nu nseamna ca nu exista si funct ii discontinue care admit primitive, cum se vede
din exemplul urmator.
Fie funct ia f : R R
f(x) =
_
sin
1
x
, x ,= 0,
0, x = 0,
discontinua n x = 0 (nu are limita n x = 0), x = 0 ind un punct de discontinuitate de spet a a 2-a.
Sa aratam ca f admite totusi primitive pe R. Fie funct ia h : R R, data prin
h(x) =
_
x
2
cos
1
x
, x ,= 0,
0, x = 0.
Se constata ca h e derivabila pe R caci
h
t
(0) = lim
x
h(x) h(0)
x 0
= lim
x
xcos
1
x
= 0.
Deci avem
h
t
(x) =
_
2xcos
1
x
+ sin
1
x
, x ,= 0,
0, x = 0,
=
_
2xcos
1
x
, x ,= 0,
0, x = 0,
+
_
sin
1
x
, x ,= 0,
0, x = 0,
adica
h
t
(x) = H(x) +f(x), f(x) = h
t
(x) H(x).
5.4 Primitive. Integrala nedenit a. Primitivabilitatea funct iilor continue 205
Funct ia h
t
(x) admite primitive (caci exista funct ia h = R R derivabila avand derivata h
t
(x) ) iar
funct ia H : R R
H(x) =
_
2xcos
1
x
, x ,= 0,
0, x = 0,
admite primitive pe R, ind continua. Deducem ca funct ia f = h
t
H admite primitive pe R (cu Teorema
5.4.6)
5.4.13
t
. Observat ie. Daca f: I R, Iinterval are un punct de discontinuitate de spet a 1-a,
x
0
I, atunci ea nu poate admite primitive.

Intr-adevar, daca am presupune contrariul, e F : I R o
primitiva a sa, deci
F
t
(x) = f(x), x I.

Insa F
t
s
(x
0
) ,= F
t
d
(x
0
), caci f(x
0
0) ,= f(x
0
+ 0), adica F nu e derivabila n x
0
, contrar presupunerii.
Not iunea de primitiva a unei funct ii se poate generaliza n sensul urmator
5.4.14 Denit ie. Fie I R un interval si f : I R o funct ie. Se numeste primitiva generalizata
a lui f, orice funct ie F : I R, derivabila pe I cu except ia unei mult imi nite de puncte din I si astfel
ca F
t
(x) = f(x) n toate punctele de derivabilitate si F continua n punctele de except ie.
5.4.15 Teorema (Formula lui NewtonLeibniz). Fie f:[a, b] R integrabila Riemann pe [a, b]
si primitivabila pe [a, b]. Atunci oricare ar F : [a, b] R, o primitiva a lui f are loc egalitatea
b
_
a
f(x) d x = F(b) F(a) = F(x) [
b
a
. (5.4.5)
Demonstrat ie. Sa consideram un sir de diviziuni
n
:= (x
0
, x
(n)
1
, x
(n)
2
, . . . , x
(n)
k
n
) ale intervalului
[a,b] cu (
n
) 0 cand n si
n
=
1
,
2
, . . . ,
n
P(
n
) un sistem de puncte intermediare
asociate diviziunii
n
, obt inut prin aplicarea formulei cresterilor nite primitivei F pe [x
(n)
i1
, x
(n)
i
],
F(x
(n)
i
) F(x
(n)
i1
) = F
t
(
i
) (x
(n)
i
x
(n)
i1
) = f(
i
)(x
(n)
i
x
(n)
i1
),
valabila pentru orice i = 1, n. Atunci suma Riemann corespunzatoare va
(f,
n
,
n
) =
n

i=1
f(
i
)(x
(n)
i
x
(n)
i1
) =
n

i=1
( F(x
(n)
i
) F(x
(n)
i1
)) = F(b) F(a).
Deducem ca
b
_
a
f(x) d x = lim
n
(f,
n
,
n
) = F(b) F(a).
5.4.15 Exemplu. Sa consideram funct ia f : [2, 5] R data prin
f(x) =
1
x(x 1)
, x [2, 5].
Funct ia f este continua pe [2, 5], deci integrabila Riemann (Teorema 5.4.11). Funct ia f admite si
primitive pe [2, 5] deoarece funct ia F : [2, 5] R, data prin F(x) = ln (x 1) ln x, este o primitiva a
sa, avand
F
t
(x) =
1
x 1

1
x
=
1
x(x 1)
.
Aplicand formula lui NewtonLeibniz (5.4.5) obt inem
5
_
2
1
x(x 1)
dx = [ln(x 1) lnx][
5
2
= ln 4 ln5 ln1 + ln2 =
8
5
.
206 5. Funct ii integrabile Riemann si primitive
5.4.16 Observat ie. Aplicarea formulei Newton Leibniz pretinde ca f sa e n acelasi timp si
integrabila si primitivabila pe intervalul [a, b]. Acest lucru nu este realizat de multe ori deoarece exista
funct ii care admit primitive pe [a, b] dar nu sunt integrabile pe [a, b] si de asemenea exista funct ii
integrabile pe [a, b] dar care nu admit primitive pe [a, b].
5.4.17 Exemplu (Funct ie integrabila Riemann care nu admite primitive).
Fie f : [1, 1] R, data prin
f(x) =
_
1 , daca x [1, 0] ,
2 , daca x ]0, 1] .
Funct ia f este marginita pe [1, 1] si are un singur punct de discontinuitate x
0
= 0, deci este
integrabila Riemann pe [1, 1 ] (Teorema 5.2.16), nsa funct ia f nu are proprietatea Darboux pe [1, 1 ]
(deoarece nu ia nici o valoare ] 1, 2 [), deci nu are primitive.
5.4.18. Exemplu. (Funct ie care admite primitive dar care nu este integrabila). Funct ia
f : [0, 1 ] R, data prin
f(x) =
_
2x cos
1
x
2
+
2
x
sin
1
x
2
, daca x ] 0, 1],
0, daca x = 0 .
Funct ia f nu este marginita pe [0, 1] n vecinatatea lui 0, deci f nu este integrabila Riemann pe [0, 1].

Insa, funct ia f admite primitive pe [0, 1] deoarece funct ia F : [0, 1 ] R,


F(x) =
_
x
2
cos
1
x
2
, daca x ] 0, 1],
0, daca x = 0 ,
este o primitiva a lui f avand F
t
(x) = f(x) pentru orice x [0, 1].
5.4.19. Exemplu. Sa se arate ca funct ia f : R R data prin
f(x) =
_
e
x
, x 0,
x 1, x < 0,
admite primitive si sa se determine primitivele sale.
Se constata ca f e continua pe ] , 0[]0, +[ ind exprimata pe aceste intervale prin funct ii
elementare, deci continue.

In punctul x = 0 avem
f(0 0) = lim
x,0
f(x) = lim
x,0
(x 1) = 1,
f(0 + 0) = lim
x0
f(x) = lim
x0
e
x
= e
0
= 1,
f(0) = e
0
= 1
Deci f e continua n x = 0, deci continua pe R. Cu Teorema 5.4.11 se deduce ca f admite primitive.
Primitivele ei vor avea forma
F(x) =
_
_
_
_
e
x
dx, x 0,
_
(x 1)dx, x < 0
=
_
_
_
e
x
+C
1
, x 0,
x
2
2
x +C
2
, x < 0,
unde C
1
, C
2
sunt constante arbitrare. Leg atura dintre constantele C
1
si C
2
se obt ine din condit ia ca F,
ca primitiva a lui f sa e derivabila, deci continua pe R, deci si n x = 0. Atunci
F(0 0) = lim
x,0
F(x) = lim
x,0
_
x
2
2
x +C
2
_
= C
2
,
F(0 + 0) = lim
x0
F(x) = lim
x0
(e
x
+C
1
) = C
1
1 = F(0).
Deducem ca C
2
= C
1
1. Daca punem C
1
= C, atunci C
2
= C 1 si primitivele lui f vor
F(x) =
_
_
_
e
x
+C, x 0
x
2
2
x 1 +C, x < 0, C R.
5.5 Calculul unor primitive si metode de integrare 207
Pentru C = 0, o primitiva a lui f va
F
0
(x) =
_
_
_
e
x
, x 0
x
2
2
x 1, x < 0
si atunci F(x) = F
0
(x) +C, reprezinta mult imea primitivelor, adica
_
f(x)dx = F
0
(x) +C.
Formula lui Newton Leibniz (5.4.5) ramane valabila pentru o funct ie f : [a, b] R integrabila
Riemann pe [a, b] care admite pe [a, b] primitive generalizate ( conform denit iei 5.4.13) . Nu facem
demonstrat ia.
5.4.20 Observat ie. Vom enumera primitivele unor funct ii elementare pe un interval I inclus n
domeniul maxim de denit ie.
1
0
.
_
0 d x= C;
2
0
.
_
x

d x =
x
+1
+ 1
+C , R, ,= 1;
3
0
.
_
1
x
d x = ln [ x[ +C;
4
0
.
_
a
x
d x =
a
x
ln a
+C , a > 0, a ,= 1;
5
0
.
_
e
x
d x = e
x
+C;
6
0
.
_
sinxd x = cos x +C;
7
0
.
_
cos xd x = sin x +C;
8
0
.
_
1
cos
2
x
d x=tg x+C;
9
0
.
_
1
sin
2
x
d x = ctg x +C;
10
0
.
_
dx
x
2
+a
2
=
1
a
arctg
x
a
+C , a ,= 0;
11
0
.
_
d x
x
2
a
2
=
1
2a
ln

x a
x +a

+C , a ,= 0;
12
0
.
_
d x

a
2
+x
2
= arcsin
x
a
+C ;
13
0
.
_
d x

x
2
+a
2
= ln( x+

x
2
+a
2
) +C ;
14
0
.
_
d x

x
2
a
2
= ln

x+

x
2
a
2

+C .
5.5 Calculul unor primitive si metode de integrare

In acest paragraf vom prezenta proprietat ile integralei nedenite, formula integrarii prin part i si
teoremele de schimbare de variabila atat pentru integrala nedenita cat si pentru integrala Riemann.
5.5.1 Teorema. Daca f, g:I R R, Iinterval, admit primitive pe I atunci si funct ia f + g
admite primitive pe I si are loc pentru orice x I relat ia
_
(f +g) (x) d x =
_
f(x) d x +
_
g(x) d x.
Proprietatea se extinde usor si la o suma nita de funct ii care admit primitive.
Demonstrat ie. Consideram primitivele F:I R a lui f si G:I R a lui g. Funct ia F +G : I R
va atunci derivabila si
(F +G)
t
(x) = F
t
(x) +G
t
(x) = f(x) +g(x) = (f +g)(x),
208 5. Funct ii integrabile Riemann si primitive
pentru orice x I, deci va primitiva a lui f +g, adica f +g admite primitive pe I si avem pentru orice
x I
_
f(x) d x = F(x) +C ,
_
g(x) d x = G(x) +C ,
si deci
_
f(x) d x+
_
g(x) d x=F(x) +C +G(x) +C=(F +G)(x)+C=
_
(f +g)(x) d x.
5.5.2 Teorema. Daca f : I R admite primitive pe intervalul I R si a R, atunci funct ia a f
cu a ,= 0 admite primitive si avem
_
a f(x) d x = a
_
f(x) d x , x I.

Intr-adevar, daca F este o primitiva a lui f avem pentru orice x I


a
_
f(x) d x = a(F(x) +C) = a F(x) +a C = (a F)(x) +C =
_
(af)(x) d x +C.
5.5.3 Observat ie. Daca F : I R R, I interval, este o funct ie derivabila pe I, primitiva
funct iei f : I R, atunci avem
1
0
.
_
F
t
(x) d x = F(x) +C , x I;
2
0
.
_ _
f(x) d x
_
t
= f(x) , x I.
Relat iile acestea sunt evidente, avand n vedere denit ia primitivei si a integralei nedenite.
5.5.4 Teorema. (formula integrarii prin part i). Daca funct iile f,g:I R R, I interval,
sunt derivabile pe I si derivatele lor f
t
si g
t
sunt continue pe I, atunci are loc relat ia
_
f
t
(x) g(x) d x=f(x) g(x)
_
f(x) g
t
(x) d x. (5.5.1)
Demonstrat ie. Deoarece f
t
si g
t
sunt presupuse continue pe I, rezulta ca si funct iile f si g sunt
continue si atunci funct iile f
t
g si fg
t
sunt continue pe I, deci admit primitive pe I.
Din relat ia (f g)
t
= f
t
g +f g
t
, se deduce ca f g este o primitiva a funct iei f
t
g +f g
t
, astfel ca
_
(f
t
g +f g
t
)(x) d x= (f g)(x) +C , x I.
De aici se deduce ca
_
(f
t
g)(x) d x+
_
(f g
t
)(x) d x=f(x) g(x) +C , x I.

Insa, deoarece
_
(f g
t
)(x) d x C=
_
(f g
t
)(x) d x se deduce ca
_
(f
t
g)(x) dx=f(x) g(x)
_
(f g
t
)(x) dx,
adica relat ia ceruta.
5.5.5 Observat ie. Formula (5.5.1) este o egalitate ntre doua mult imi de funct ii si daca F
1
, este de
exemplu o primitiva particulara a funct iei f
t
g si F
2
, o primitiva particulara a lui f g
t
este posibil ca sa
avem F
1
,= f g F
2
.
5.5.6 Exemplu. Utilizand formula integrarii prin part i, avem
_
lnxd x =
_
x
t
lnxd x = x ln x
_
x(ln x)
t
d x
= x ln x
_
d x = xlnx x +C , x > 0.
5.5 Calculul unor primitive si metode de integrare 209
5.5.7 Exemplu.
_
x cos x dx =
_
x(sin x)
t
= x sin x
_
sinx dx
= x sin x + cos x +C, xI R.
5.5.8 Exemplu.
_
arctg xd x =
_
x
t
arctg xd x = xarctg x
_
x
1
1 +x
2
d x =
= xarctg x
1
2
ln(1 +x
2
) +C , x R sau x I R.
5.5.9 Exemplu. Sa calculam integrala
_
_
4 x
2
d x =
_
4 x
2

4 x
2
d x = 4
_
d x

4 x
2
+
_
x (
_
4 x
2
)
t
dx =
= 4 arcsin
x
2
+x
_
4 x
2

_
_
4 x
2
d x , x ] 2 , 2[.
De aici se expliciteaza
_
_
4 x
2
d x = 2 arcsin
x
2
+
1
2
x
_
4 x
2
+C , x ] 2, 2 [.
5.5.10 Teorema. (Formula integrarii prin part i generalizata). Daca f, g : I R R,
Iinterval, au derivate pana la ordinul n > 1 continue pe I, atunci are loc formula
_
f
(n)
(x) g(x) d x = f
(n1)
(x) g(x) f
(n2)
(x) g
t
(x) +f
(n3)
(x)g
tt
(x) . . .
. . . + (1)
n1
f(x) g
(n1)
(x) + (1)
n
_
f(x) g
(n)
(x) d x, x I. (5.5.2)
Demonstrat ie. Cu formula integrarii prin part i (5.5.1), putem scrie
_
f
(nk)
(x) g
(k)
(x) d x = f
(nk1)
(x) g
(k)
(x)
_
f
(nk1)
(x) g
(k+1)
(x) d x,
pentru k = 0, 1, 2, . . . , n 1 si x I. Aici f
(0)
(x) nseamna f(x).
De aici, daca nmult im n ambii membri egalitatea cu (1)
k
, obt inem
(1)
k
_
f
(nk)
(x) g
(k)
(x) d x = (1)
k
f
(nk1)
(x) g
(k)
(x) + (1)
k+1
_
f
(nk1)
(x) g
(k+1)
(x) d x,.
Daca adunam membru cu membru egalitat ile obt inute de aici pentru k = 0, 1, 2, . . . , n1, se obt ine
tocmai formula ceruta (5.5.2).
5.5.11 Exemplu. Sa se calculeze
_
x
n
e
x
d x, n R, n > 1.
Daca consideram funct iile f(x) = e
x
, g(x) = x
n
, x R, avem
f
t
(x) = e
x
, f
tt
(x) = e
x
, . . . , f
(n)
(x) = e
x
g
t
(x) = nx
n1
, g
tt
(x) = n(n 1) x
n2
, . . . , g
(k)
(x) =
= n(n 1) . . . (n k + 1) x
nk
, . . . g
(n1)
(x) = n! x, g
(n)
(x) = n!
Aplicand formula (5.5.2), avem
_
x
n
e
x
d x=e
x
x
n
nx
n1
e
x
+n(n 1) x
n2
e
x
. . . +
210 5. Funct ii integrabile Riemann si primitive
+(1)
k
n(n 1) . . . (n k + 1) x
nk
e
x
+. . . + (1)
n
_
e
x
n ! dx=
= e
x
[x
n
nx
n1
+n(n 1)x
n2
. . . + (1)
k
n(n 1) . . . (n k + 1)x
nk
+. . .
+(1)
n1
n! x + (1)
n
n!] = e
x
n

k=0
(1)
k
n(n 1) . . . (n k + 1) x
nk
, x R.
5.5.12 Teorema (prima metoda de schimbare de variabila). Fie I, J intervale din R si
funct iile u : I J, f:J R cu proprietat ile:
(i) u este derivabila pe I, cu u
t
,= 0,
(ii) f admite primitive pe J si F : J R este o primitiva a sa.
Atunci funct ia (f u) u
t
admite primitive pe I si F u este o primitiva a sa, adica are loc egalitatea
_
f(u(x)) u
t
(x) d x = F(u(x)) +C , x I (5.5.3)
Demonstrat ie. Deoarece F
t
(t) = f(t) oricare ar t J si funct ia compusa F u este derivabila
pe I, avem pentru orice x I
((f u) (x) +C)
t
= ( F(u(x)) +C)
t
= F
t
(u(x)) u
t
(x) = f(u(x)) u
t
(x),
ceea ce dovedeste ca F u este o primitiva a lui (f u) u
t
.
5.5.13 Observat ie. Practic, pentru a calcula
_
g(x) d x, unde g:I R se procedeaza astfel:
a)

Incercam sa punem n evident a o funct ie u : I R derivabila si o funct ie f care admite primitive,
f : u(I) R astfel ca g(x) = f(u(x)) u
t
(x).
b) Determinam o primitiva F a lui f pe intervalul J = u(I ), adica
_
f(t) d t = F(t) +C;
c) Atunci, o primitiva a funct iei g = (f u) u
t
este F u, adica
_
g(x) d x =
_
f(u(x)) u
t
(x) d x = F(u(x)) +C , x I.
5.5.14 Exemplu. Sa calculam
_
tg xd x , x
_
0,

2
_
= I.
Putem scrie g(x) =
1
cos x
(cos x)
t
pentru x
_
0 ,

2
_
si atunci luam u(x) = cos x, u:
_
0,

2
_
R si
f(t) =
1
t
, t ] 0 , 1 ]. Atunci g(x) = f(u(x)) u
t
(x), x
_
0,

2
_
si o primitiva a funct iei f pe ]0, 1] este
F(t) = ln t si deci
_
f(t) d t =
_

1
t
d t = ln t +C , t ] 0 , 1 ].
Rezulta ca o primitiva a funct iei g pe
_
0,

2
_
este F u, adica
_
tg xd x= ln cos x +C , x
_
0,

2
_
.
5.5.15 Observat ie. Formal, daca interpretam pe dx ca diferent iala, atunci u
t
(x) d x = d u (diferent iala
funct iei u) si formula schimbarii de variabila (5.5.3) se scrie formal
_
f(u) d u=F(u) +C.
5.5 Calculul unor primitive si metode de integrare 211
Se poate atunci, formal, proceda astfel: pentru a calcula
_
f(u(x)) u
t
(x) d x, se face nlocuirea u(x) = t
si se diferent iaza ca o egalitate de funct ii obt inandu-se u
t
(x) d x = d t si atunci
_
f(u(x)) u
t
(x) d x=
_
f(t) d t = F(t) +C=F(u(x))+C.
Aceasta egalitate este formala, nu este egalitate obisnuita de funct ii deoarece
_
f(u(x)) u
t
(x) d x sunt
funct ii denite pe I pe cand
_
f(t) d t sunt primitivele lui f, deci sunt denite pe J.
5.5.16 Exemplu. Sa se calculeze
_
2x
x
4
+ 5
d x , x R.
Deoarece
2x
x
4
+ 5
=
(x
2
)
t
(x
2
)
2
+ 5
,
luam u : R R, u(x) = x
2
si f : R R, f(t) =
1
5 +t
2
, t R. Avem
_
f(t) d t=
_
1
5 +t
2
d t=
1

5
arctg
t

5
+C , t R,
si apoi
_
2x
x
4
+ 5
d x =
_
(x
2
)
t
(x
2
)
2
+ 5
d x =
1

5
arctg
x
2

5
+C , x R.
Formal, putem proceda asa: nlocuim x
2
= t , 2 xd x = d t si atunci
_
2 x
x
4
+ 5
d x=
_
d t
5 +t
2
=
1

5
arctg
t

5
+C =
1

5
arctg
x
2

5
+C.
Aplicand observat iile 5.5.13 si 5.5.15 putem da cateva primitive ale funct iilor compuse (n ipoteza ca
212 5. Funct ii integrabile Riemann si primitive
u : I R, Iinterval, este derivabila cu derivata continua):
1
0
.
_
u
n
(x) u
t
(x) d x=
u
n+1
(x)
n + 1
+C , nZ, n ,= 1;
2
0
.
_
u

(x) u
t
(x) d x =
u
+1
(x)
+ 1
+C , R, ,= 1;
3
0
.
_
u
t
(x)
u(x)
d x= ln [ u(x) [ +C;
4
0
.
_
a
u(x)
u
t
(x) d x=
a
u(x)
ln a
+C , a > 0 , a ,= 1;
5
0
.
_
sin u(x) u
t
(x) d x = cos u(x) +C;
6
0
.
_
cos u(x) u
t
(x) d x= sinu(x) +C;
7
0
.
_
u
t
(x)
cos
2
u(x)
d x = tg u(x) +C;
8
0
.
_
u
t
(x)
sin
2
u(x)
d x = ctg u(x) +C;
9
0
.
_
u
t
(x)
_
a
2
u
2
(x)
d x=arcsin
u(x)
a
+C, a R, a ,= 0;
10
0
.
_
u
t
(x)
a
2
+u
2
(x)
d x =
1
a
arctg
u(x)
a
+C , a ,= 0;
11
0
.
_
u
t
(x)
_
u
2
(x) +a
2
d x = ln
_
u(x)+
_
a
2
+u
2
(x)
_
+C, a ,= 0;
12
0
.
_
u
t
(x)
_
u
2
(x) a
2
d x = ln

u(x)+
_
u
2
(x) a
2

+C , a ,= 0.
5.5.17 Teorema. (a doua metoda de schimbare de variabila). Fie I, J R intervale si
f : I R, u : J I, funct ii cu proprietat ile:
1
0
. u este bijectiva, derivabila pe J si cu u
t
(x) ,= 0, oricare ar x J,
2
0
. funct ia compusa h = (f u) u
t
admite primitive pe J, H ind o primitiva a sa. Atunci funct ia f
admite primitive pe I si o primitiva a sa este H u
1
, adica avem egalitatea
_
f(x) d x=H (u
1
(x))+C. (5.5.4)
Demonstrat ie. Daca H : J R este o primitiva a lui h = (f u) u
t
pe J, atunci avem
H
t
(t) = h(t) = f(u(t)) u
t
(t) oricare ar t J. Apoi se deduce ca u
1
: I J este derivabila pe
I si
(u
1
)
t
(x) =
1
u
t
(t)
=
1
u
t
(u
1
(x))
.
Atunci
(H u
1
)
t
(x) = [H(u
1
(x))]
t
= H
t
(u
1
(x)) (u
1
(x))
t
= h(u
1
(x))(u
1
(x))
t
= f(u(u
1
(x)) u
t
(u
1
(x))
1
u
t
(u
1
(x))
= f(x), oricare ar x I.
5.5 Calculul unor primitive si metode de integrare 213
De aici se deduce ca H u
1
este o primitiva a lui f si are loc (5.1.4).
5.5.18 Observat ie. Aplicarea concreta a teoremei 5.5.17 se face n felul urmator: vrem sa calculam
_
f(x) d x, unde f: I R R este o funct ie care admite primitive pe I, atunci procedam astfel
1
0
. Punem n evident a o funct ie u:J I, Jinterval, bijectiva si derivabila pe J cu derivata nenula
pe J si e u
1
: I J, inversa ei (se spune ca u
1
schimba variabila x n variabila t).
2
0
. Determinam o primitiva H:J R a funct iei h = (f u) u
t
, deci
_
f(u(t)) u
t
(t) d t = H(t)+C.
3
0
. Atunci H u
1
va o primitiva a lui f pe I, adica
_
f(x) d x = H(u
1
(x)) +C , x I.
Formal se face nlocuirea x = u(t) , d x = u
t
(t) d t , si atunci t = u
1
(x). Integrala devine
_
f(x) d x =
_
f(u(t)) u
t
(t) d t = H(t) +C = H(u
1
(x))+C.
5.5.19 Exemplu. Sa calculam
_
_
1 x
2
d x , x ] 1, 1 [ = I.
Consideram funct ia u :
_

2
,

2
_
] 1, 1[, u(t) = sin t, derivabila si bijectiva pe
_

2
,

2
_
si
u
t
(t) = cos t ,= 0 pentru orice t
_

2
,

2
_
. Atunci funct ia h = (f u) u
t
este
h(t) =
_
1 u
2
(t) u
t
(t) =
_
1 sin
2
t cos t = cos
2
t,
pentru orice t
_

2
,

2
_
sau h(t) =
1
2
(1 + cos 2 t) si are primitive pe
_

2
,

2
_
; o primitiva a sa este
H :
_

2
,

2
_
R,
H(t) =
1
2
_
t +
1
2
sin 2 t
_
=
1
2
(t + sin t cos t), t
_

2
,

2
_
.
Atunci
_
f(x) d x =
_
_
1 x
2
d x = H(u
1
(x)) +C = H(arcsinx) +C =
1
2
(arcsin x +x
_
1 x
2
) +C.
5.5.20 Exemplu. Sa calculam
_
d x
sin x
, x] 0, [=I.
Formal, nlocuim tg
x
2
= t, x = 2arctg t , dx =
2
1 +t
2
dt.
Mai avem
sin x =
2tg
x
2
1 +tg
2
x
2
=
2 t
1 +t
2
, t ] 0 , +[,
si integrala se scrie succesiv
_
d x
sin x
=
_
1 +t
2
2 t

2
1 +t
2
d t=
=
_
1
t
d t = ln [ t [ +C = ln

tg
x
2

+C.
214 5. Funct ii integrabile Riemann si primitive
5.5.21 Observat ie. Fie I, J R intervale si u:I J, f:J R funct ii cu proprietat ile
(i) u este bijectiva, derivabila pe I cu derivata continua si diferita de zero pe I;
(ii) f este continua pe J.
Evident ca n ipotezele noastre f admite primitive si e F:J R o primitiva a sa. Atunci, pe baza
Teoremei 5.5.12, deducem ca funct ia F u este o primitiva a funct iei (f u) u
t
pe I.
Invers, sa presupunem ca H = F u este o primitiva a funct iei (f u) u
t
pe I. Pe baza Teoremei
5.5.17 funct ia H u
1
= F u u
1
= F este o primitiva a lui f. Asadar, n ipotezele (i) si (ii), funct ia
F:J R este o primitiva a lui f pe J daca si numai daca F u este o primitiva a funct iei (f u) u
t
.
Altfel spus, n ipotezele (i) si (ii) cele doua metode de schimbare de variabile sunt echivalente, exista doar
mai multe variante de aplicare a unei metode unice de schimbare de variabila.
Varianta I. Pentru a calcula
_
f(x) d x , x I;
1
0
. Scriem pe f(x) = g(u(x)) u
t
(x), unde u : I J, derivabila iar g : J R o funct ie care are
primitive, G ind o primitiva;
2
0
. Se nlocuieste formal u(x) : = t si u
t
(x) d x := d t si se obt ine
_
g(t) d t = G(t) +C , t J;
3
0
. Revenim la variabila x, nlocuind t : = u(x) n G(t) si obt inem
_
f(x) d x = G(u(x))+C.
Varianta a II-a. Pentru a calcula
_
f(x) d x , x I;
1
0
. Punem n evident a un interval J R si o funct ie u :J I, derivabila si bijectiva si nlocuim
x := u(t) , t J, d x := u
t
(t) d t
si obt inem integrala
_
f(u(t)) u
t
(t) d t , t J;
2
0
. Gasim o primitiva H si atunci
_
f(u(t)) u
t
(t) d t = H(t) +C , t J;
3
0
. Revenim la variabila x, nlocuind n H pe t := u
1
(x) si obt inem
_
f(x) d x = H(u
1
(x)) +C.
Varianta a III-a. Pentru a calcula
_
f(x) d x , x I;
1
0
. Punem n evident a n expresia lui f(x), o funct ie u : I R injectiva si derivabila cu u
1
:
u(I ) I si o funct ie g : u(I ) R astfel ncat f(x) = g(u(x)) , x I;
2
0
.

Inlocuim formal
u(x) : = t, x : = u
1
(t) ,
d x : = (u
1
(t))
t
d t
5.5 Calculul unor primitive si metode de integrare 215
si se obt ine integrala
_
g(t) (u
1
(t))
t
d t = F(t) +C , t u(I );
3
0
. Revenim la variabila x, punand t : = u(x) n expresia lui F si astfel avem
_
f(x) d x = F(u(x)) +C , x I.
5.5.22 Exemplu. Sa se calculeze (cu varianta a III-a)
_
sin x
3 sin
2
x + 1
d x, x ] 0, [.
Punem cos x = t , x : = arccos t, t ] 1 , 1 [ , d x =
1

1 t
2
d t si se obt ine
_

1 t
2
3 (1 t
2
) + 1
d t

1 t
2
=
_
d t
3t
2
4
=
1
3
_
d t
t
2

4
3
=
1
3
1
2
2

3
ln

t
2

3
t +
2

+C =
1
4

3
ln

3 2
t

3 + 2

+C , t ] 1, 1[.
Atunci
_
sin x
3 sin
2
x + 1
d x =
1
4

3
ln

3 cos x 2

3 cos x + 2

+C.
5.5.23 Observat ie. Problema determinarii primitivelor este mult mai dicila decat problema de-
rivarii. Determinarea primitivelor se poate face, cu metodele pe care le-am prezentat, doar pentru clase
restranse de funct ii elementare. Exista multe funct ii despre care desi stim ca admit primitive, acestea
nu se pot calcula, nu se pot exprima ntr-un numar nit de operat ii asupra unor funct ii elementare,
deci nu sunt funct ii elementare. Primitivele acestor funct ii ne denesc noi funct ii, numite funct ii tran-
scendente, funct ii care sunt tabelate, valorile lor sunt date n tabele. Ment ionam cateva funct ii dintre
acestea:
er (x) =
_
e
x
2
d x, x R; li (x) =
_
1
ln x
d x, x ] 1 , [ (5.5.5)
si (x) =
_
sin x
x
d x , x ] 0, +[; ci (x) =
_
cos x
x
d x, x ]0, +[ (5.5.6)
Tot n aceasta categorie intra si integralele eliptice, avand forma
_
R (x,
_
P(x) ) d x, (5.5.7)
unde P (x) este un polinom de gradul 3 sau 4, iar R(x, y) este o funct ie rat ionala n x si y.
Se arata ca integralele (5.5.7) se pot transforma n una din urmatoarele trei integrale
I
1
(x) =
_
d x
_
(1 x
2
)(1 a
2
x
2
)
, (5.5.8)
I
2
(x) =
_
x
2
d x
_
(1 x
2
)(1 a
2
x
2
)
, (5.5.9)
I
3
(x) =
_
dx
(1 +kx
2
)
_
(1 x
2
)(1 a
2
x
2
)
, (5.5.10)
216 5. Funct ii integrabile Riemann si primitive
unde a,k R, a ]0, 1[.
Formula integrarii prin part i, simpla si generalizata, si metodele de schimbare de variabila se pot
extinde, cu ajutorul formulei Newton Leibniz si la integrale Riemann (integrale denite).
5.5.24 Teorema. (formula integrarii prin part i pentru integrale Riemann). Daca
f, g : [a, b] R sunt derivabile pe [a, b] si au derivatele f
t
, g
t
continue pe [a, b] atunci are loc egalitatea
b
_
a
f
t
(x) g(x) d x = f(x)g(x)

b
a

b
_
a
f(x) g
t
(x) d x. (5.5.11)
Demonstrat ie. Din teorema de derivare a unui produs, avem
(f(x) g(x))
t
= f
t
(x) g(x) +f(x) g
t
(x) , x [a, b].
Pe baza formulei Newton Leibniz (5.4.5.), obt inem
f(x) g(x)[
b
a
=
b
_
a
f
t
(x) g(x) d x +
b
_
a
f(x) g
t
(x) d x,
iar de aici rezulta (5.5.11).
5.5.25 Teorema. (formula generalizata de integrare prin part i). Daca f,g : [a, b] R au
derivate continue pana la ordinul n, n > 1 pe [a, b] atunci are loc egalitatea
b
_
a
f
(n)
(x) g(x) d x =
_
f
(n1)
(x) g(x) f
(n2)
(x) g
t
(x) +f
(n3)
(x) g
tt
(x) . . .
+ (1)
n1
f(x) g
(n1)
(x)
_

b
a
+ (1)
n
b
_
a
f(x) g
(n)
(x) d x (5.5.12)
Demonstrat ie. Demonstrat ia se bazeaza pe teorema 5.5.10 si formula Newton Leibniz, obt inandu-
se fara probleme relat ia (5.5.12).
5.5.26 Exemplu. Sa se calculeze integralele
A
n
=

2
_
0
sin
n
xd x si B
n
=

2
_
0
cos
n
xd x.
Pentru A
n
aplicam teorema 5.5.24 si avem
A
n
=

2
_
0
sin
n1
x (cos x)
t
d x = sin
n1
xcos x

2
0
+ (n 1)

2
_
0
sin
n2
x cos
2
xdx
= (n 1)

2
_
0
sin
n2
(x) ( 1 sin
2
x) d x = (n 1) A
n2
(n 1) A
n
De aici se obt ine relat ia de recurent a
A
n
=
n 1
n
A
n2
, n 2.
Pentru n = 2k, obt inem, prin aplicare repetata pentru k = 1, 2, ...,
A
2k
=
(2k 1)(2k 3) . . . 5 3 1
2k(2k 2) . . . 6 4 2
A
0
,
5.5 Calculul unor primitive si metode de integrare 217
cu A
0
=

2
_
0
d x =

2
. Deci
A
2k
=
1 3 5 . . . (2k 3)(2k 1)
2 4 6 . . . 2k


2
=
(2k 1) !!
(2k) !!


2
. (5.5.13)
Daca n = 2k + 1 se obt ine fara dicultate
A
2k+1
=
2 4 6 . . . (2k 2)(2k)
1 3 5 . . . (2k 1)(2k + 1)
=
(2k) !!
(2k + 1) !!
. (5.5.14)
Pentru integrala B
n
se observa ca daca se face schimbarea de variabila x =

2
t, se obt ine B
n
= A
n
si
astfel
A
n
= B
n
=
_

_
(n 1) !!
n!!


2
, daca n este par ,
(n 1) !!
n!!
, daca n este impar .
(5.5.16)
Aici, notat ia n !! (dublu factorial de n) nseamna ca factorii produsului n ! se iau din doi n doi, adica
n!! =
_
2 4 6 . . . n , daca n este par ,
1 3 5 . . . n , daca n este impar .
5.5.27 Teorema. (Formula lui Taylor cu restul sub forma de integrala). Daca f : I R,
I interval, admite derivate pana la ordinul n + 1, n N, continue pe I si daca x
0
, x I, atunci avem
egalitatea
f(x) = f(x
0
) +
x x
0
1 !
f
t
(x
0
) +
(x x
0
)
2
2 !
f
tt
(x
0
) +. . . +
+... +
(x x
0
)
n
n!
f
(n)
(x
0
) +
1
n!
x
_
x
0
(x t)
n
f
(n+1)
(t) d t.
(5.5.17)
Demonstrat ie. Aplicam formula integrarii prin part i generalizata (5.5.12) pentru funct iile
F(t) = f
t
(t) , G(t) =
(x t)
n
n!
, t [x
0
, x] I si obt inem
x
_
x
0
F
(n)
(t)G(t) dt = F
(n1)
(t) G(t)

x
x
0
F
(n2)
(t) G
t
(t)

x
x
0
+. . . +
+(1)
n1
F(t) G
(n1)
(t)

x
x
0
+ (1)
n
x
_
x
0
F(t) G
(n)
(t) d t,
adica
x
_
x
0
(x t)
n
n!
f
(n+1)
dt =
(x t)
n
n!
f
(n)
(t)

x
x
0
+
(x t)
n1
(n 1) !
f
(n1)
(t)

x
x
0
+
+
(x t)
n2
(n 2) !
f
(n2)
(t)

x
x
0
+. . . +
x t
1 !
f
t
(t)

x
x
0
+
x
_
x
0
f
t
(t) d t =
= f(x) f(x
0
)
(x x
0
)
1 !
f
t
(x
0
)
(x x
0
)
2
2 !
f
tt
(x
0
) . . .
(x x
0
)
n
n!
f
(n)
(x
0
).
De aici rezulta imediat relat ia ceruta (5.5.17).
218 5. Funct ii integrabile Riemann si primitive
5.5.28 Teorema. (prima formula de schimbare de variabila pentru integrala Riemann).
Daca I R este un interval, a,b R si funct iile u : [a, b] I, f : I R ndeplinesc condit iile
1
0
. u este continua pe [a, b] si are derivata u
t
continua pe [a, b];
2
0
. f este continua pe I, atunci are loc egalitatea
b
_
a
f(u(x)) u
t
(x) d x =
u(b)
_
u(a)
f(t) d t. (5.5.18)
Demonstrat ie. Funct ia f, continua, admite primitive pe I. Daca F : I R este o primitiva a sa
atunci avem, cu formula lui Newton Leibniz
u(b)
_
u(a)
f(t) d t = F(u(b)) F(u(a)).
Funct ia compusa g:[a, b] R, g(x) = F(u(x)), este derivabila, ind compusa a doua funct ii deriva-
bile, si atunci
g
t
(x) = F
t
(u(x)) u
t
(x) = f(u(x)) u
t
(x) , x [a, b].
Aplicand din nou formula lui Newton Leibniz
b
_
a
f(u(x)) u
t
(x) = g(x)

b
a
= g(b) g(a) = F(u(b)) F(u(a)),
rezulta relat ia (5.5.18).
5.5.29 Exemplu. Sa se calculeze
1
_
0
x
5
1 +x
6
d x.
Fie u(x) = 1 +x
6
, u : [0, 1] R, evident derivabila si u
t
(x) = 6x
5
este continua. Atunci avem
1
_
0
x
5
1 +x
6
d x =
1
6
1
_
0
u
t
(x)
u(x)
= ln (x) = t =
1
6
u(1)
_
u(0)
d t
t
=
1
6
ln [ t [

2
1
=
1
6
ln 2.
5.5.30 Teorema. (a doua formula de schimbare de variabila). Fie funct iile u : [, ]
[a, b] , f : [a, b] R cu proprietat ile
1
0
. funct ia u este bijectiva si derivabila mpreuna cu inversa sa u
1
: [a, b] [, ],
2
0
. u
t
si (u
1
)
t
sunt continue pe [, ] respectiv [a, b];
3
0
. funct ia f este continua pe [a, b].

In aceste condit ii are loc egalitatea

f(u(x)) d x =
u( )
_
u()
f(t) (u
1
)
t
(t) d t. (5.5.19)
Demonstrat ie.

In ipotezele noastre, funct ia compusa f u este continua si deci admite primitive
pe [, ]. Daca F:[, ] R este o primitiva, cu formula lui Newton Leibniz, obt inem

f(u(x)) d x = F() F().

Insa funct ia F u
1
este o primitiva a funct iei f (u
1
)
t
pe [a, b], deoarece
(F u
1
)
t
(t) = F
t
(u
1
(t)) (u
1
)
t
(t) = (f u)(u
1
(t)) (u
1
)
t
(t) =
= f(u(u
1
(t))) (u
1
)
t
(t) = f(t) (u
1
)
t
(t).
5.5 Calculul unor primitive si metode de integrare 219
Aplicam din nou formula lui Newton Leibniz
u( )
_
u( )
f(t) (u
1
)
t
(t) = (F u
1
) (u( )) (F u
1
)(u( )) =
= F(u
1
(u( ))) F(u
1
(u( ))) = F( ) F( ) =
_

f (u(x)) dx.
5.5.31 Exemplu. Sa se calculeze

_
0
x sin x
1 + cos
2
x
d x.
Pentru calculul acestei integrale, o vom descompune n doua integrale astfel

_
0
x sin x
1 + cos
2
x
d x =

2
_
0
x sin x
1 + cos
2
x
d x +

2
x sin x
1 + cos
2
x
d x (5.5.20)
La integrala a doua vom pune x = t si atunci pentru x
_

2
,
_
vom avea t
_
0 ,

2
_
si

2
xsinx
1 + cos
2
x
dx =
0
_

2
( t) sin( t)
1 + cos
2
( t)
(dt) =

2
_
0
( t) sint
1 + cos
2
t
dt =

2
_
0
sint
1 + cos
2
t
dt

2
_
0
t sint
1 + cos
2
t
dt.
Acum din (5.5.20) obt inem

_
0
xsinx
1 + cos
2
x
dx =

2
_
0
sint
1 + cos
2
t
dt,
iar de aici, cu schimbarea de variabila cos t = u, t = arccos u, u [0, 1] cand t
_
0,

2
_
, sintdt = du, se
gaseste

_
0
xsinx
1 + cos
2
x
dx =
0
_
1
du
1 +u
2
=
1
_
0
du
1 +u
2
=
= arctgu[
1
0
=

2
4
.
5.5.32. Observat ie. Teoremele 5.5.28 si 5.5.30 se pot unica ntr-o singura formula de schimbare
de variabila cu cele trei variante de aplicare prezentate la Observat ia 5.5.21, doar ca n cazul acesta
odata cu schimbarile formale de variabila se schimba si limitele de integrare prin nlocuirea u(x) := t sau
x := u(t).
5.5.33 Exemplu. Sa se calculeze
1
_
0
e
3x
1 +e
3x
dx.
Cu substitut ia e
3x
= t adica x :==
1
3
lnt, dx :=
1
3t
dt si daca x = 0, atunci t = 1, iar daca x = 1,
atunci t = e
3
. Integrala devine
1
_
0
e
3x
1 +e
3x
dx =
e
3
_
1
t
1 +t

1
3t
dt =
1
3
e
3
_
1
dt
1 +t
=
1
3
ln[1 +t[

e
3
1
=
1
3
ln
1 +e
3
2
.
5.5.34 Exemplu. Sa se calculeze

4
_
0
tgx
cos
2
x(1 + tgx)
dx
.
220 5. Funct ii integrabile Riemann si primitive
Vom face substitut ia tgx := t sau x := arctgt, dx :=
1
1 +t
2
dt. Noile limite de integrare vor 0 si 1, astfel
ca integrala va

4
_
0
tgx
cos
2
x(1 + tgx)
dx
.
=
1
_
0
t
1 +t
dt =
1
_
0
dt
1
_
0
1
1 +t
dt = t

1
0
ln(1 +t)

1
0
= 1 ln2.
5.6 Integrarea funct iilor rat ionale si a altor clase de funct ii
Vom vedea n acest paragraf ca primitivele funct iilor rat ionale nu sunt ntotdeauna funct ii rat ionale
dar sunt funct ii elementare care se exprima ca o suma de funct ii rat ionale si de funct ii logaritmice sau
funct ii arctangente.
5.6.1 Denit ie. (Funct ie rat ionala simpla). O funct ie rat ionala f se numeste funct ie rat ionala
simpla sau fract ie simpla) daca are una din urmatoarele forme:
I. f(x) := a
0
x
n
+a
1
x
n1
+. . . +a
n1
x +a
n
, n N, a
i
R, i = 0, n;
II. f(x) :=
A
(x a)
n
unde n N, a, A R;
III.f(x) :=
Ax +B
(ax
2
+bx +c)
n
, unde n N, A, B, a, b, c R si b
2
4ac < 0.
Se arata, n teorema urmatoare, ca orice alta funct ie rat ionala f(x) :=
P(x)
Q(x)
, este o combinat ie
liniara de funct ii rat ionale simple.
5.6.2 Teorema. (Teorema de descompunere a funct iilor rat ionale n funct ii rat ionale
simple). Fie funct ia rat ionala f : D R, f(x) =
P(x)
Q(x)
, unde P si Q sunt polinoame cu coecient i reali,
prime ntre ele si D = x R [ Q(x) ,= 0. Daca Q are descompunerea n factori ireductibili
Q(x) = (x x
1
)
n
1
(x x
2
)
n
2
. . . (x xk)
n
k
(a
1
x
2
+b
1
x +c
1
)
m
1
. . . (a
s
x
2
+b
s
x +c
s
)
m/s
(5.6.1)
unde x
1
, x
2
, . . . , x
k
sunt radacinile reale ale lui Q, iar
i
= b
2
i
4a
i
c
i
< 0, pentru i = 1, 2, . . . , s si
n
1
, n
2
, . . . , n
k
, m
1
, m
2
, . . . , m
s
N

asa ca
n
1
+n
2
+. . . +n
k
+ 2(m
1
+m
2
+. . . +m
s
) = gradQ,
atunci exista un polinom cu coecient i reali p(x), si constantele reale A
j
i
, i = 1, k; j = 1, n
i
; B
l
p
, C
l
p
,
p = 1, s ; l = 1, m astfel ncat f sa se scrie n mod unic sub forma
f(x) = p(x) +
k

i=1
_
A
1
i
x x
i
+
A
2
i
(x x
i
)
2
+. . . +
A
n
i
i
(x x
i
)
n
i
_
+
+
s

p=1
_
B
1
p
x +C
1
p
a
p
x
2
+b
p
x +c
p
+
B
2
p
x +C
2
p
(a
p
x
2
+b
p
x +c
p
)
2
+. . . +
B
m
p
p
x +c
m
p
p
(a
p
x
2
+b
p
x +c
p
)
m
p
_
, (5.6.2)
pentru orice x D.
Renunt am la demonstrat ia teoremei, datorita lungimii ei.
Din (5.6.2) se deduce ca pentru a integra funct ia rat ionala f trebuie sa integram un polinom si o
suma de funct ii rat ionale de tipul II si III. Daca polinomul p este
p(x) = a
0
x
n
+a
1
x
n1
+. . . +a
n1
x +a
n
, a
i
R; i = 0, n,
atunci
_
p(x) dx =
a
0
n + 1
x
n+1
+
a
1
n
x
n
+. . . +
a
n1
2
x
2
+a
n
x +C,
daca x I interval inclus n D.
5.6 Integrarea funct iilor rat ionale si a altor clase de funct ii 221
5.6.3 Observat ie. Daca n f(x) =
P(x)
Q(x)
, gradul polinomului P este mai mic decat gradul lui Q,
atunci polinomul p din (5.6.2) este identic nul. Daca funct ia rat ionala simpla care intra n descompunerea
(5.6.2) are forma II, adica f(x) =
A
(x a)
n
, x I D, atunci pentru n = 1
_
A
x a
dx = A ln [ x a [ +C,
iar daca n > 1, atunci
_
A
(x a)
n
d x = A
_
(x a)
n
d x = A
(x a)
n+1
n + 1
+C =
A
1 n
1
(x a)
n1
+C.
Daca funct ia rat ionala simpla este de tipul III, atunci
f(x) =
Ax +B
(ax
2
+bx +c)
n
, x I D, = b
2
4ac < 0,
si pentru a gasi primitiva sa vom scrie
ax
2
+bx +c = a
_
_
x +
b
2a
_
2


4a
2
_
= a
_
_
x +
b
2a
_
2
+
2
_
, unde =

2a
R.
Atunci
_
f(x) d x =
_
Ax +B
(ax
2
+bx +c)
n
d x =
_
Ax +B
a
n
_
_
x +
b
2a
_
2
+
2
_
n
d x.
Daca facem substitut ia x +
b
2a
:= t, obt inem integrala
_ A
_
t
b
2a
_
+B
a
n
(t
2
+
2
)
n
dt =
A
a
n
_
t
(t
2
+
2
)
n
dt +
1
a
n
_
B
Ab
2a
_ _
1
(t
2
+
2
)
n
dt.
Asadar, a ramas sa calculam integralele de forma
_
t
(t
2
+
2
)
n
d t ;
_
1
(t
2
+
2
)
n
d t , t R (5.6.3)
Pentru aceste doua integrale avem doua cazuri, n = 1 si n > 1. Pentru n = 1 se obt ine imediat
_
t
(t
2
+
2
)
n
dt =
1
2
_
(t
2
+
2
)
t
t
2
+
2
dt =
1
2
ln(t
2
+
2
) +C , t R,
si de asemenea
_
1
t
2
+
2
d t =
1

arctg
t

+C , tR.
Daca n > 1, prima integrala (5.6.3) devine, pentru t
2
+
2
= u
_
t
(t
2
+
2
)
n
dt =
1
2
_
(t
2
+
2
)
t
(t
2
+
2
)
n
dt =
1
2
_
du
u
n
=
1
2
u
n+1
n + 1
+C=
=
1
2(1 n)
1
(t
2
+
2
)
n1
+C , t R.
222 5. Funct ii integrabile Riemann si primitive
Cea de-a doua integrala (5.6.3) pentru n > 1, se calculeaza cu ajutorul unei formule de recurent a.
Daca notam I
n
:=
_
dt
(t
2
+
2
)
n
, atunci integrand prin part i pe
I
n1
:=
_
1
(t
2
+
2
)
n1
dt =
_
t
t
1
(t
2
+
2
)
n1
dt,
obt inem
I
n1
=
t
(t
2
+
2
)
n1

_
t
(n 1)(t
2
+
2
)
n2
2t
(t
2
+
2
)
2n2
dt =
t
(t
2
+
2
)
n1
+
+2(n 1)
_
t
2
(t
2
+
2
)
n
dt =
t
(t
2
+
2
)
n1
+ 2(n 1)
_
t
2
+
2

2
(t
2
+
2
)
n
dt=
=
t
(t
2
+
2
)
n1
+ 2(n 1) I
n1
2(n 1)
2
I
n
.
Explicitam n raport cu I
n
si gasim relat ia de recurent a
I
n
=
1

2
_
t
2(n 1)(t
2
+
2
)
n1
+
2n 3
2n 2
I
n1
_
, n = 2, 3, . . . , (5.6.4)
iar
I
1
=
_
1
t
2
+
2
d t =
1

arctg
t

+C , t R.
5.6.4 Exemplu. Sa se calculeze
_
3
2x
2
+ 4x + 7
d x.
Avem = 16 56 = 40 < 0, asa ca avem o funct ie rat ionala simpla de tipul III cu n =1. Putem
scrie
_
3
2x
2
+ 4x + 7
dx = 3
_
1
2
_
x
2
+ 2x + 1 +
7
2
1
_dx =
3
2
_
dx
(x + 1)
2
+
5
2
=
3
2
_
dz
z
2
+
5
2
= J
Acum putem face substitut ia z := x + 1, dz := dx si obt inem
J =
3
2
_
2
5
arctg
z
_
5
2
+C =
3

10
arctg

2(x + 1)

5
+C, x R.
5.6.5 Exemplu. Sa se calculeze
_
x + 1
x
3
+x
dx, x > 0.
Descompunem funct ia rat ionala n funct ii rat ionale simple cu ajutorul relat iei (5.6.2)
x + 1
x
3
+x
=
x + 1
x(x
2
+ 1)
=
A
x
+
Bx +C
x
2
+ 1
.
Vom determina constantele A, B, C prin identicarea celor doi membri ai egalitat ii
x + 1
x(x
2
+ 1)
=
(A+B)x
2
+Cx +A
x(x
2
+ 1)
.
Deducem ca A+B = 0, C = 1, A = 1 si astfel A = 1, B = 1, C = 1. Vom avea
_
x + 1
x
3
+x
dx =
_ _
1
x
+
x + 1
x
2
+ 1
_
dx =
_
dx
x

_
x
x
2
+ 1
dx +
_
1
x
2
+ 1
dx=
5.6 Integrarea funct iilor rat ionale si a altor clase de funct ii 223
= lnx
1
2
ln(x
2
+ 1) + arctgx +C, x > 0.
5.6.6 Exemplu. Sa se calculeze
_
2x
2
+ 2x + 12
(x 1)(x
2
2x + 5)
2
dx, x I, (5.6.5)
I ind un interval din R care nu cont ine punctul x = 1.
Vom descompune, conform Teoremei 5.6.2 funct ia de sub integrala n funct ii rat ionale simple
2x
2
+ 2x + 12
(x 1)(x
2
2x + 5)
2
=
A
x 1
+
Bx +C
x
2
2x + 5
+
Dx +E
(x
2
2x + 5)
2
.
Vom determina constantele A, B, C, D, E prin identicarea numaratorilor din cei doi membri, dupa
aducerea la acelasi numitor
2x
2
+ 2x + 12
(x 1)(x
2
2x + 5)
2
=
=
A(x
2
2x + 5)
2
+ (Bx +C)(x 1)(x
2
2x + 5) + (Dx +E)(x 1)
(x 1)(x
2
2x + 5)
2
.
Se obt ine sistemul liniar
_

_
A+B = 0,
4A3B +C = 0,
14A+ 7B 3C +D = 2,
20A5B + 7C D +E = 2,
25A5C +E = 12.
Solut ia acestui sistem este A = 1, B = 1, C = 1, D = 2, E = 8.
Funct ia rat ionala de sub semnul integrala se va descompune astfel:
2x
2
+ 2x + 12
(x 1)(x
2
2x + 5)
2
=
1
x 1

x 1
x
2
2x + 5

2x 8
(x
2
2x + 5)
2
,
si atunci
_
2x
2
+ 2x + 12
(x 1)(x
2
2x + 5)
2
dx =
_
1
x 1
dx
_
x 1
x
2
2x + 5
dx
_
2x 8
(x
2
2x + 5)
2
dx =
= ln[x 1[ J
1
J
2
(5.6.6)
Daca facem n J
1
si J
2
substitut ia x 1 := t, x := t + 1, dx := dt, x
2
2x + 5 = (x 1)
2
+ 4 = t
2
+ 4,
atunci
J
1
:=
_
t
t
2
+ 4
dt =
1
2
_
(t
2
+ 4)
t
t
2
+ 4
dt =
1
2
ln(t
2
+ 4) =
1
2
ln
_
x
2
2x + 5
_
,
J
2
:=
_
2(t + 1) 8
(t
2
+ 4)
2
dt =
_
2t
(t
2
+ 4)
2
dt 6
_
1
(t
2
+ 4)
2
dt =
_
(t
2
+ 4)
t
(t
2
+ 4)
2
dt 6I
2
=
=
1
t
2
+ 4
6I
2
. (5.6.7)
Integrala
I
2
:=
_
1
(t
2
+ 4)
2
dt, (5.6.8)
se calculeaza cu relat ia de recurent a (5.6.4) pentru n = 2, sau se calculeaza prin part i integrala
I
1
:=
_
1
(t
2
+ 4)
dt =
1
2
arctg
t
2
.
224 5. Funct ii integrabile Riemann si primitive
Se obt ine
I
2
=
1
4
_
t
2(t
2
+ 4)
+
1
4
arctg
t
2
_
.
Acum integrala J
2
devine
J
2
=
1
t
2
+ 4

3
4
t
t
2
+ 4

3
8
arctg
t
2
.

Inlocuind t := x 1 obt inem expresia nala pentru integrala (5.6.6)


_
2x
2
+ 2x + 13
(x 1)(x
2
2x + 5)
2
dx = ln[x 1[
1
2
ln(x
2
2x + 5)
1
x
2
2x + 5

3
4
x 1
x
2
2x + 5

3
8
arctg
x 1
2
+C.
5.6.7 Observat ie. (Un procedeu pentru determinarea coecient ilor din descompunerea
unei funct ii rat ionale). Sa presupunem ca
Q(x) = (x a)
k
Q
1
(x)
si avem descompunerea
f(x) =
P(x)
Q(x)
=
A
1
x a
+
A
2
(x a)
2
+. . . +
A
k
(x a)
k
+
P
1
(x)
Q
1
(x)
, x I (5.6.9)
unde x = a este radacina multipla de ordinul k al numitorului Q(x) si deci Q
1
(a) ,= 0. Pentru deter-
minarea lui A
k
, nmult im n ambii membri egalitatea (5.6.9) cu (x a)
k
si obt inem
(x a)
k
f(x) =
P(x)
Q
1
(x)
= A
k
+A
k1
(x a) +. . . +A
1
(x a)
k1
+ (x a)
k
P
1
(x)
Q
1
(x)
.
De aici obt inem
A
k
= lim
xa
P(x)
Q
1
(x)
=
P(a)
Q
1
(a)
. (5.6.10)
Pentru a determina pe A
k1
, termenul
A
k
(x a)
k
din (5.6.9), cu A
k
determinat cu relat ia (5.6.10), se
trece n membrul stang si efectuandu-se operat iile, gasim
P(x)
(x a)
k
Q
1
(x)

A
k
(x a)
k
=
A
1
x a
+. . . +
A
k2
(x a)
k2
+
A
k1
(x a)
k1
+
P
1
(x)
Q
1
(x)
.
Membrul stang se scrie, evident
(x a)P
2
(x)
(x a)
k
Q
1
(x)
=
P
2
(x)
(x a)
k1
Q
1
(x)
=
A
1
x a
+. . . +
A
k1
(x a)
k1
+
P
1
(x)
Q
1
(x)
, (5.6.11)
unde P
2
este un alt polinom. Acum, pentru determinarea luiA
k1
, repetam acelasi procedeu, nmult im
n ambii membri relat ia (5.6.11) cu (x a)
k1
si apoi facem x = a (sau x a). Se continua procedeul
pana cand se obt ine si coecientul A
1
.

Intr-un mod asemanator se determina coecient ii B
j
i
si C
j
i
din
descompunerea (5.6.2).
5.6.8 Exemplu. Sa se calculeze
_
x
2
+ 3x + 1
(x 2)
2
(x
2
+ 4x + 8)
dx, x [3, +). (5.6.12)
Avem descompunerea n fract ii simple
x
2
+ 5x + 1
(x 2)
2
(x
2
+ 4x + 8)
=
A
x 2
+
B
(x 2)
2
+
Cx +D
x
2
+ 4x + 8
. (5.6.13)
5.6 Integrarea funct iilor rat ionale si a altor clase de funct ii 225
Vom determina coecient ii A, B, C, D R utilizand observat ia 5.6.7.

Inti pentru determinarea lui
B, nmult im (5.6.13) n ambii membri cu (x 2)
2
si apoi facem x = 2.
B =
x
2
+ 5x + 1
x
2
+ 4x + 8

x=2
=
3
4
.

In continuare, scriem relat ia 5.6.13 n felul urmator


x
2
+ 5x + 1
(x 2)
2
(x
2
+ 4x + 8)

3
4(x 2)
2
=
A
x 2
+
Cx +D
x
2
+ 4x + 8
.
Efectuand calculele n membrul stang, obt inem
x
2
+ 8x 20
4(x 2)
2
(x
2
+ 4x + 8)
=
(x 2)(x + 10)
4(x 2)
2
(x
2
+ 4x + 8)
=
x + 10
4(x 2)
2
(x
2
+ 4x + 8)
x + 10
4(x 2)
2
(x
2
+ 4x + 8)
=
A
x 2
+
Cx +D
x
2
+ 4x + 8
(5.6.14)
Acum nmult im ambii membri ai acestei egalitat ii cu x 2 si apoi vom face x = 2. Gasim
A =
x + 10
4(x
2
+ 4x + 8)

x=2
=
3
20
.
Cu A determinat, relat ia (5.6.14) se scrie
x + 10
4(x 2)(x
2
+ 4x + 8)

3
20(x 2)
=
Cx +D
x
2
+ 4x + 8
sau efectuand calculele

3x + 39
20(x
2
+ 4x + 8)
=
Cx +D
x
2
+ 4x + 8
.
De aici se obt ine imediat
C =
3
20
, D =
13
20
. (5.6.15)
Revenind la relat ia (5.6.13), n care nlocuim valorile gasite, avem
_
x
2
+ 5x + 1
(x 2)
2
(x
2
+ 4x + 8)
dx =
3
20
_
dx
x 2
+
3
4
_
dx
(x 2)
2

1
2
_
3x + 13
(x + 2)
2
+ 4
dx =
=
3
20
ln(x 2)
3
4
1
(x 2)

1
2
_
3(x + 2)
(x + 2)
2
+ 4
dx +
7
20
_
dx
(x + 2)
2
+ 4

In nal, am obt inut pentru orice x [3, +[


_
x
2
+ 5x + 1
(x 2)
2
(x
2
+ 4x + 8)
dx =
3
20
ln(x 2)
3
4(x 2)

3
40
ln(x
2
+ 4x + 8)
7
40
arctg
x + 2
2
+C.

In cele ce urmeaza vom prezenta cateva clase de funct ii a caror integrare se reduce printr-o schimbare
de variabila convenabila la integrarea unei funct ii rat ionale (la o integrala rat ionala). Asemenea integrale
le vom numi integrale reductibile la integrale rat ionale.
5.6.9 Observat ie. Integralele de forma
b
_
a
R(u(x))u
t
(x)dx, (5.6.16)
226 5. Funct ii integrabile Riemann si primitive
unde R(t) este o funct ie rat ionala iar u(x) este o funct ie oarecare u : [a, b]R cu derivata continua pe
[a, b], sunt integrale evident reductibile la integrale rat ionale prin schimbarea de variabila u(x) = t.
Integralele de forma
b
_
a
R(u(x))dx, (5.6.17)
nu sunt nsa, n general, reductibile la integrale rat ionale chiar daca R(t) este funct ie rat ionala (except ie
cazul cand u(x) este o funct ie rat ionala). Exista nsa cateva alegeri pentru u(x) asa ca integrala (5.6.17)
sa e reductibila la una rat ionala.
5.6.10 Teorema. Integralele de forma
b
_
a
R(e
x
)dx (5.6.18)
unde R(t) =
P(t)
Q(t)
este o funct ie rat ionala, R, asa ca pentru orice x [a, b] numitorul sau Q(e
x
) ,= 0,
se reduc la integrale rat ionale.

Intr-adevar, cu substitut ia e
x
:= t sau x :=
1

lnt, dx :=
1

1
t
dt, este clar ca t [e
a
, e
b
] pentru
x [a, b] si integrala (5.6.18) devine
e
b
_
e
a
R(t)
1
t
dt =
t
2
_
t
1
R
1
(t)dt,
unde R
1
(t) este o alta funct ie rat ionala.
5.6.11 Teorema. Integralele trigonometrice de forma
b
_
a
R(sinx, cos x)dx, x [a, b] ] , [= I (5.6.19)
unde R(u, v) este o funct ie rat ionala de doua variabile, sunt reductibile la integrale rat ionale cu substitut ia
tg
x
2
:= t, adica x := u(t) = 2arctgt.
Demonstrat ie. Este clar ca daca x I, atunci
x
2

_

2
,

2
_
si funct ia tg
x
2
este strict crescatoare
si inversabila. Utilizand formulele
sinx =
2tg
x
2
1 + tg
2
x
2
=
2t
1 +t
2
, cos x =
1 tg
2
x
2
1 + tg
2
x
2
=
1 t
2
1 +t
2
(5.6.20)
dx = u
t
(t)dt =
2
1 +t
2
dt,
se obt ine
b
_
a
R(sinx, cos x)dx =
t
2
_
t
1
R
_
2t
1 +t
2
,
1 t
2
1 +t
2
_
2
1 +t
2
dt =
t
2
_
t
1
R
1
(t)dt,
unde R
1
(t) este o alta funct ie rat ionala de variabilat pe intervalul [t
1
, t
2
], cu t
1
= tg
a
2
, t
2
= tg
b
2
.
5.6.12 Exemplu. Sa se calculeze

2
_
0
1
sinx + cos x + 1
dx.
5.6 Integrarea funct iilor rat ionale si a altor clase de funct ii 227
Cu substitut ia tg
x
2
= t, x = 2arctgt, dx =
2
1 +t
2
dt, utilizand formulele (5.6.20) obt inem

2
_
0
1
sinx + cos x + 1
dx =
1
_
0
2dt
1 +t
2
2t
1 +t
2
+
1 t
2
1 +t
2
+ 1
= 2
1
_
0
dt
2t + 2
=
1
_
0
dt
t + 1
= ln(t + 1)

1
0
= ln 2.
5.6.13 Observat ie. Uneori pentru calculul integralelor (5.6.19) schimbarea tg
x
2
= t duce la calcule
complicate si atunci exista nca trei cazuri posibile de reducere a integralei la una rat ionala.
1
0
. Daca funct ia rat ionala R(sinx, cos x) satisface relat ia
R(sinx, cos x) = R(sinx, cos x), x [a, b]
_

2
,

2
_
, (5.6.21)
atunci nlocuirea tgx := t sau x := arctgt, ne duce la o integrala rat ionala.

Intr-adevar, daca R(u,v) este
para simultan n raport cu ambele variabile, putem scrie R(u, v) = R
_
u
v
v, v
_
= R
1
_
u
v
, v
_
, unde R
1
este o alta funct ie rat ionala n argumentele sale. Apoi
R(u, v) = R
1
_
u
v
, v
_
= R(u, v) = R
1
_
u
v
, v
_
,
deci R
1
este para n raport cu v si deci putem scrie R(u, v) = R
1
_
u
v
, v
_
= R
2
_
u
v
,v
2
_
, unde R
2
este o
alta funct ie rat ionala. Integrala va atunci
b
_
a
R(sinx, cos x)dx =
b
_
a
R
2
(tgx, cos
2
x)dx =
=
tgb
_
tga
R
2
_
t,
1
1 +t
2
_
1
1 +t
2
dt =
tgb
_
tga
R
3
(t)dt,
unde R
3
este o noua funct ie rat ionala.
5.6.14 Exemplu. Sa se calculeze

3
_

3
dx
sin
4
xcos
2
x
.
Funct ia de sub semnul integral este simultan para n raport cu sin x si cos x, adica satisface (5.6.21).
Deci cu nlocuirea tgx := t,x := arctgt, dx =
1
1 +t
2
dt obt inem

3
_

3
dx
sin
4
xcos
2
x
=

3
_

3
dt
1 +t
2
t
4
(1 +t
2
)
2

1
1 +t
2
=

3
_

3
(1 +t
2
)
2
t
4
dt =

3
_

3
_
1
t
4
+
2
t
2
+ 1
_
dt =
=
_

1
3
1
t
3

2
t
+t
_

3
= 2

3
4

3

2
9

3
=
16

3
27
.
2
0
. Daca funct ia R(u,v) = R(sinx, cos x) este impara n raport cu u = sinx, adica
R(u, v) = R(u, v), (5.6.22)
228 5. Funct ii integrabile Riemann si primitive
atunci nlocuirea cos x := t sau x := arccos t, reduce integrala (5.6.19) la o integrala rat ionala. Justicarea
este imediata deoarece din relat ia (5.6.22) rezulta ca funct ia
R(u,v)
u
este para n raport cu u deci ea se
scrie sub forma
R(u, v)
u
= R
1
(u
2
, v), adica R(u, v) = R
1
(u
2
, v) u, unde R
1
este o alta funct ie rat ionala.
Deci
b
_
a
R(sinx, cos x)dx =
b
_
a
R
1
(sin
2
x, cos x) sinxdx =
=
t
2
_
t
1
R
1
(1 t
2
, t)(dt) =
t
2
_
t
1
R
2
(t)dt
unde R
2
este o funct ie iar t
1
= cos a, t
2
= cos b.
5.6.15 Exemplu. Sa se calculeze

_
0
sin
5
xcos
2
xdx.
Funct ia de sub semnul integrala este impara n raport cu sin x, astfel ca nlocuim
cos x := t, sinxdx = dt, t
1
= cos 0 = 1, t
2
= cos = 1.
Obt inem

_
0
sin
4
xcos
2
xsinxdx =
1
_
1
(1 t
2
)
2
t
2
dt =
1
_
1
(t
6
2t
4
+t
2
)dt =
=
_
t
7
7

2t
5
5
+
t
3
3
_

1
1
= 2
_
1
7

2
5
+
1
3
_
=
16
105
.
3
0
. Daca n integrala (5.6.19) funct ia de sub integrala R(u, v) = R(sinx, cos x) este impara n raport
cu v = cos x, adica
R(u, v) = R(u, v), (5.6.23)
atunci cu substitut ia sinx := t, integrala se reduce la una rat ionala.

Intr-adevar, din (5.6.23) rezulta
ca
R(u, v)
v
este para n raport cu v, deci
R(u, v)
v
= R
1
(u, v
2
), unde R
1
este o funct ie rat ionala. Atunci
R(u, v) = R
1
(u, v
2
) v si avem
b
_
a
R(sinx, cos x)dx =
b
_
a
R
1
(sinx, cos
2
x) cos xdx =
=
t
2
_
t
1
R
1
(t, 1 t
2
) (dt) =
t
2
_
t
1
R
2
(t)dt,
unde R
2
(t) este o funct ie rat ionala.
5.6.16 Exemplu. Sa se calculeze

4
_
0
dx
cos x
.
Funct ia este impara n raport cu cos x si atunci nlocuirea sinx := t, cos xdx = dt, t
1
= sin0 = 0,
t
2
= sin

4
=

2
2
, conduce la
5.6 Integrarea funct iilor rat ionale si a altor clase de funct ii 229

4
_
0
dx
cos x
=

4
_
0
cos xdx
cos
2
x
=

2
2
_
0
dt
1 t
2
=
1
2
ln

t 1
t + 1

2
2
0
=
=
1
2
ln

2 2

2 + 2

=
1
2
ln
_
2 +

2
2

2
_
.
5.6.17 Teorema. Integralele de forma

R
_
x,
n
_
ax +b
cx +d
_
dx, (5.6.24)
unde R(u, v) =
P(u, v)
Q(u, v)
este o funct ie rat ionala de doua variabile, sunt reductibile la integrale rat ionale,
daca se face schimbarea de variabile
ax +b
cx +d
:= t
n
sau x :=
dt
n
b
ct
n
a
= u(t). (5.6.25)
Desigur, funct ia de sub integrala trebuie sa satisfaca cateva condit ii minime de existent a:
a) Q
_
x,
n
_
ax +b
cx +d
_
,= 0 pentru orice x [, ];
b)
ax +b
cx +d
0 n [, ] daca n este numar par;
c) ad ,= bc, deoarece n caz contrar funct ia
ax +b
cx +d
se reduce la o constanta si deci funct ia de sub integrala
nu mai este o funct ie irat ionala.
Cu acestea, integrala (5.6.24) devine

R
_
x,
n
_
ax +b
cx +d
_
dx =
t
2
_
t
1
R(u(t), t) u
t
(t)dt =
t
2
_
t
1
R
1
(t)dt,
unde R
1
(t) este clar o funct ie rat ionala, iar t
1
=
n
_
a +b
c +d
; t
2
=
n
_
a +b
c +d
.
5.6.18 Sa se calculeze
I :=
7/9
_
7/9
1
x + 1
3
_
x 1
x + 1
dx. (5.6.26)
Avem o integrala de forma (5.6.24). Pentru a o reduce la o integrala rat ionala vom face schimbarea
de variabila (5.6.25), care n acest caz este
x 1
x + 1
= t
3
. De aici obt inem
x :=
1 +t
3
1 t
3
, dx :=
6t
2
(1 t
3
)
2
dt.
230 5. Funct ii integrabile Riemann si primitive
Integrala (5.6.26) devine, cu noile limite de integrare,
I := e
1/2
_
2
t
3
1 t
3
dt = 3
1/2
_
2
t
3
1 + 1
t
3
1
dt =
= 3
1/2
_
2
dt 3
1/2
_
2
1
(t 1)(t
2
+t + 1)
dt.
Daca descompunem n funct ii rat ionale simple
1
(t 1)(t
2
+t + 1)
=
1
3

1
t 1

1
3

t + 2
t
2
+t + 1
,
I = 3t 3t[

1
2
2

1/2
_
2
dt
t 1
+
1/2
_
2
t + 2
t
2
+t + 1
dt =
9
2
+ ln2
1
2
ln
7
4
+
0
_
3/2
udu
u
2
+ 3/4
+
+
3
2
0
_
3/2
du
u
2
+ 3/4
=
9
2
+ ln2
1
4
ln
7
4
+
1
2
ln
_
u
2
+
3
4
_

0
3/2
+

3arctg
2u

0
3/2
=

3
3

9
2

1
2
ln7.
5.6.19 Obsevat ie. O integrala de forma
J =
b
_
a
R
_
x,
n
1

_
ax +b
cx +d
_
m
1
, . . . ,
n
k

_
ax +b
cx +d
_
m
k
_
dx, (5.6.27)
unde R(u
1
, u
2
, . . . , u
k+1
) este o funct ie rat ionala de argumentele sale, iar m
i
, n
i
2, i = 1, 2, . . . , k sunt
numere naturale, se reduce la o integrala rat ionala prin schimbarea de variabila
ax +b
cx +d
:= t
n
, unde n este
cel mai mic multiplu comun al numerelor n
1
, n
2
, . . . , n
k
. Justicarea este imediata, n ipoteza ca funct ia
de sub integrala satisface condit ii de existent a similare condit iilor a), b), c) de la teorema 2.6.17.
5.6.20 Exemplu. Sa se calculeze
J :=
0
_
1

x + 1
1 +
3

x + 1
dx.
Cu schimbarea de variabila x + 1 := t
6
, dx := 6t
5
dt, t
1
= 0, t
2
= 1, obt inem
J =
1
_
0
t
3
1 +t
2
6t
5
dt = 6
_
t
8
t
2
+ 1
dt =
= 6
1
_
0
t
8
+t
6
t
6
t
4
+t
4
t
2
+t
2
1 + 1
t
2
+ 1
dt =
= 6
1
_
0
_
t
6
t
4
+t
2
1 +
1
t
2
+ 1
_
dt =
= 6
_
t
7
7

t
5
5
+
t
3
3
t + arctgt
_

1
0
=
5.6 Integrarea funct iilor rat ionale si a altor clase de funct ii 231
= 6
_
1
7

1
5
+
1
3
1 +

4
_
=
3
2

152
35
.
5.6.21 Teorema. Integralele de forma

R
_
x,
_
ax
2
+bx +c
_
dx, (5.6.28)
unde R(x, y) =
P(x, y)
Q(x, y)
este o funct ie rat ionala n doua variabile, se reduce ntotdeauna la o integrala
rat ionala daca urmatoarele condit ii sunt satisfacute:
a) Q
_
x,

ax
2
+bx +c
_
,= 0, oricare ar x [, ];
b) ax
2
+bx +c 0, pentru orice x [, ];
c) a ,= 0 si = b
2
4ac ,= 0.
Demonstrat ie. Vom prezenta schimbarile de variabile ale lui Euler, nsa acestea nu sunt
singurele care reduc integrala (5.6.28) la integrale rat ionale.
1
0
. Daca a > 0 se face nlocuirea
_
ax
2
+bx +c := t

ax, (5.6.29)
care ne conduce la x := u(t) =
t
2
c
2

at +b
. Deoarece t este o funct ie continua de x, nseamna ca daca x
parcurge intervalul [, ] atunci t parcurge intervalul [t
1
, t
2
] unde t
1
si t
2
se obt in din (5.6.29) pentru x
=, respectiv x = . Prin metoda reducerii la absurd se deduce ca b+2

a t ,= 0 pentru orice t [t
1
, t
2
].
Atunci integrala (5.6.28) devine
t
2
_
t
1
R(u(t), t

a u(t)) u
t
(t)dt =
t
2
_
t
1
R
1
(t)dt,
unde R
1
(t) este o funct ie rat ionala de t, deoarece si u(t), u
t
(t) sunt funct ii rat ionale de t.
2
0
. Daca c > 0 se face schimbarea de variabila
_
ax
2
+bx +c := tx +

c. (5.6.30)
De aici se scoate x si avem x = u(t) =
2

ct b
a t
2
, t [t
1
, t
2
]. Integrala (5.6.28) devine
t
2
_
t
1
R(u(t), tu(t) +

c)u
t
(t)dt =
t
2
_
t
1
R
2
(t)dt,
unde R
2
(t) este o funct ie rat ionala de t, deoarece R, u si u
t
sunt funct ii rat ionale.
3
0
. Daca = b
2
4ac > 0, adica trinomul de gradul doi ax
2
+bx +c are radacini reale si distincte
x
1
, x
2
atunci se face schimbarea de variabile

ax
2
+bx +c
x x
1
:= t sau a(x x
2
) = t
2
(x x
1
). (5.6.31)
De aici se obt ine
x = u(t) :=
x
1
t
2
ax
2
t
2
a
, t [t
1
, t
2
],
_
ax
2
+bx +c =
a(x
1
x
2
)t
t
2
a
,
si integrala (5.6.28) devine
t
2
_
t
1
R
_
u(t),
a(x
1
x
2
)t
t
2
a
_
u
t
(t)dt =
t
2
_
t
1
R
3
(t)dt,
232 5. Funct ii integrabile Riemann si primitive
unde R
3
(t) este o funct ie rat ionala deoarece R, u si u
t
sunt funct ii rat ionale.
Asadar n ecare caz 1
0
, 2
0
, 3
0
integralele (5.6.28) se reduc la integrale rat ionale.
5.6.22 Observat ie. Schimbarile de variabile denite de relat iile (5.6.29), (5.6.30) si (5.6.31) se
numesc substitut iile lui Euler.
5.6.23 Observat ie. Este posibil sa avem ndeplinite simultan doua sau trei din condit iile a > 0,
c > 0 si b
2
4ac > 0. Atunci se poate efectua oricare din substitut iile (5.6.29), (5.6.30) si (5.6.31).
5.6.24 Exemplu. Sa se calculeze
1
_
0
dx
x +

x
2
x + 1
.
Avem o integrala de forma (5.6.28) cu a = 1 si c = 1. Deci putem face sau substitut ia (5.6.29) sau
substitut ia (5.6.30). Sa o facem pe (5.6.29)
_
x
2
x + 1 := t x, x
2
x + 1 = t
2
2tx +x
2
, x := u(t) =
t
2
1
2t 1
,
dx = u
t
(t)dt =
2(t
2
t + 1)
(2t 1)
2
dt, t
1
= 1, t
2
= 2.
1
_
0
dx
x +

x
2
x + 1
= 2
2
_
1
t
2
t + 1
t(2t 1)
2
dt =
2
_
1
_
2
t

3
2t 1
+
3
(2t 1)
2
_
dt =
=
_
2 lnt
3
2
ln(2t 1)
3
2

1
2t 1
_

2
1
= 1 + 2 ln2
3
2
ln3.
5.6.25 Denit ie. Integralele de forma

x
m
(ax
n
+b)
p
dx, (5.6.32)
unde a, b R iar m, n, p sunt numere rat ionale, se numesc integrale binoame.
Pentru ca integrala (5.6.32) sa aiba sens sunt necesare satisfacerea urmatoarelor condit ii:
a) x 0 pentru orice x [, ], daca unul dintre numitorii lui m sau n este numar natural par;
b) x ,= 0 daca cel put in unul dintre numerele m si n este negativ;
c) ax
n
+ b 0, oricare ar x [, ] daca p are numitorul un numar par si ax
n
+ b ,= 0 pentru orice
x [, ], daca p 0.
Matematicianul rus P. L. Cebsev a demonstrat ca exista numai trei cazuri n care integralele
binoame (5.6.32) sunt reductibile la integrale rat ionale. Aceste cazuri sunt prezentate n teorema care
urmeaza
5.6.26 Teorema. O integrala binoama ( 5.6.32) se reduce la o integrala rat ionala n urmatoarele
cazuri:
1
0
. Daca p este numar ntreg, atunci substitut ia
x := u(t) = t
r
, (5.6.33)
unde r N

, este cel mai mic multiplu comun al numitorilor lui m si n, reduce integrala binoama la o
integrala rat ionala;
2
0
. Daca
m+ 1
n
este ntreg, atunci substitut ia
ax
n
+b := t
r
, sau x := u(t) =
_
t
r
b
a
_1
n
, (5.6.34)
5.6 Integrarea funct iilor rat ionale si a altor clase de funct ii 233
unde r este numitorul lui p, reduce integrala binoama la o integrala rat ionala.
3
0
. Daca
m+ 1
n
+p este numar ntreg, atunci substitut ia
a +bx
n
= t
r
, sau x := u(t) =
_
b
t
r
a
_1
n
, (5.6.35)
unde r este numitorul lui p, reduce integrala binoama la o integrala rat ionala.
Demonstrat ie.

In cazul 1
0
, daca facem substitut ia (5.6.33), integrala binoama (5.6.32) devine
t
2
_
t
1
t
mr
(at
nr
+b)
p
rt
r1
dt =
t
2
_
t
1
R(t)dt,
unde t
1
=
r

, t
2
=
r

, iar R(t) este evident o funct ie rat ionala de t.


Daca p / Z, dar
m+ 1
n
Z, cu substitut ia (2.6.34), integrala binoama (2.6.32) devine
t
2
_
t
1
_
t
r
b
a
_m
n
t
pr

1
n

r
a
t
r1

_
t
r
b
a
_1
n
1
dt =
=
r
n

1
a
m+1
n
t
2
_
t
1
t
pr+r1
(t
r
b)
m+1
n
1
dt = A
t
2
_
t
1
R
1
(t)dt,
unde A este o constanta iar R
1
(t) este o funct ie rat ionala, deoarece pr +r 1 si
m+ 1
n
1 sunt numere
ntregi.

In cazul n care nici p, nici
m+ 1
n
, nu sunt numerentregi, dar
m+ 1
n
+p estentreg, cu substitut ia
(2.6.35), integrala binoama devine
t
2
_
t
1
_
b
t
r
a
_m
n

b
p
t
pr
(t
r
a)
p
_

r
n
_
b
1
n
t
r1
(t
r
a)

n+1
n
dt =
= B
t
2
_
t
1
t
pr+r1
(t
r
a)
(
m+1
n
+p+1)
dr = B
t
2
_
t
1
R
2
(t)dt,
unde B este o constanta si R
2
(t) este o funct ie rat ionala deoarece pr +r 1 si
_
m+ 1
n
+p + 1
_
sunt
ntregi.
5.6.27 Exemplu. Sa se calculeze
64
_
1
(
6

x 1)
2
6

x
7
dx =
_
64
1
x
7/6
(x
1/6
1)
2
dx.

In acest caz avem m =


7
6
, n =
1
6
, p = 2. Cum p = 2 este ntreg facem schimbarea de variabila (2.6.33),
adica x := t
6
, t :=
4

x, dx := 6t
5
dt, t
1
= 1, t
2
= 2. Obt inem
_
2
1
t
7
(t 1)
2
6t
5
dt = 6
_
t 2 lnt
1
t
_

2
1
= 9 12 ln 2.
5.6.28 Exemplu. Sa se calculeze
I
2
:=
4
_
1
3
_
1 +
4

x
dx =
4
_
1
x
1/2
(x
1/4
+ 1)
1/3
dx.
234 5. Funct ii integrabile Riemann si primitive
Avem m =
1
2
, n =
1
4
, p =
1
3
, deci p / Z dar
m+ 1
n
= 2 Z astfel folosim (2.6.34)
x
1/4
+ 1 := t
3
, x := (t
3
1)
4
, t
1
=
3

2, t
2
=
3

3, dx := 12t
2
(t
3
1)
3
dt.
I
2
:= 12
_ 3

3
3

2
(t
6
t
3
)dt =
3
7
(15
3

3
3

2).
5.6.29 Exemplu. Sa se calculeze
I
3
:=
1
_
1
2
dx
4

1 +x
4
=
1
_
1
2
x
0
(x
4
+ 1)

1
4
dx.
Avem m = 0, n = 4, p =
1
4
si p / Z,
m+ 1
n
/ Z, dar
m+ 1
n
+p = 0 R si cu (5.6.35)
1 +x
4
:= t
4
, t :=
4

1 +x
4
x
, x := (t
4
1)
1/4
, dx := t
3
(t
4
1)
5/4
dt, t
1
=
4

17, t
2
=
4

2.
Integrala devine
I
3
=
4

2
_
4

17
_
t
4
t
4
1
_

1
4
t
3
(t
4
1)

5
4
dt =
4

17
_
4

2
t
2
t
4
1
dt =
=
4

17
_
4

2
_
1
4

1
t + 1

1
4

1
t 1

1
2

1
t
2
+ 1
_
dt =
=
1
4
ln

t + 1
t 1

17
4

1
2
arctgt

17
4

2
=
1
4
ln
(
4

17 + 1)(
4

2 1)
(
4

17 1)(
4

2 + 1)
+
+
1
2
_
arctg
4

2 arctg
4

17
_
.
5.7 Aplicat ii geometrice ale integralei Riemann
Denirea riguroasa a ariilor mult imilor plane, a lungimii arcelor de curba, a volumelor unor domenii
din spat iul R
3
si a ariilor unor suprafet e din spat iul R
3
, este o problema complexa ce t ine de teoria
masurii si nu ne permitem o dezvoltare a acesteia n acest cadru. Vom face aici doar unele considerat ii
sumare privind masura mult imilor din R
2
si R
3
, considerat ii necesare pentru calculul ariilor unor mult imi
particulare, a ariilor suprafet elor de rotat ie si volumele unor corpuri particulare, cum sunt cele de rotat ie.
5.7.1 Aria unei mult imi plane
5.7.1 Denit ie. (masura sau aria unei mult imi elementare). Daca I
1
= [a
1
, b
1
], I
2
= [a
2
, b
2
]
sunt intervale compacte din R atunci mult imea D = I
1
I
2
se numeste dreptunghi din R
2
cu aria
(masura) Aria(D) = (b
1
a
1
)(b
2
a
2
) iar mult imea A R
2
care se reprezinta sub forma
A =
n
_
k=1
D
k
unde D
k
sunt dreptunghiuri cu interioarele disjuncte doua cate doua, se numeste mult ime elementara.
Masura(aria) mult imii elementare A se deneste ca
m(A) = [A[ =
n

k=1
m(D
k
),
5.7 Aplicat ii geometrice ale integralei Riemann 235
unde m(D
k
) este aria (masura) dreptunghiului D
k
.
5.7.2 Denit ie.(aria sau masura Jordan a unei mult imi din R
2
) Fie A R
2
o mult ime
arbitrara si notam cu c - familia tuturor mult imilor elementare din R
2
. Denim masura interioara a
mult imii A prin
m
i
(A) = supm(E); E c, E A
si masura exterioara a mult imii A, prin
m
e
(A) = inf m(F); F c, A F
si vom spune ca mult imea A este masurabila Jordan (sau ca ea are arie) daca m
i
(A) = m
e
(A) =
m(A) =AriaA iar numarul m(A) se numeste aria (sau masura Jordan) a lui A.

In particular, daca A c, atunci m(A) coincide cu aria sau masura mult imii elementare A.

In plus
se arata ca m(A) este o funct ie aditiva de mult ime, deci daca A, B R
2
au arie, atunci AB are arie si
m(A B) = m(A) +m(B), A B = .
5.7.3 Teorema. Fie A R
2
o mult ime marginita (deci exista un dreptunghi care o cont ine).
Mult imea A are arie (este masurabila Jordan) daca si numai daca exista doua siruri de mult imi ele-
mentare (A
n
) si (B
n
) primul crescator iar al doilea descrescator, asa ca sa avem
a) A
n
A B
n
, n 1;
b) lim
n
m(A
n
) = lim
n
m(B
n
) = L

In acest caz avem Aria(A) = m(A) = L Demonstrat ie. () Presupunem ca A R


2
are arie si e
a :=Aria(A), deci
a = m
i
(A) = supm(E); E c, E A .
Din proprietat ile supremului, deducem ca exista E
n
c, E
n
A asa ca m(E
n
) a
1
n
, n N

. Analog
a = m
e
(A) = inf m(F); F c, A F
si din proprietat ile inmului rezulta ca pentru orice n N

exista F
n
c, F
n
A asa ca m(F
n
) a+
1
n
.
Daca punem A
n
=
n

k=1
E
k
, B
n
=
n

k=1
F
k
, vom avea A
n
A B
n
,
m(A
n
) m(E
n
) a
1
n
,
m(B
n
) m(F
n
) a +
1
n
.
Deci
a
1
n
m(A
n
) m(B
n
) a +
1
n
.
De aici, pentru n obt inem
lim
n
m(A
n
) = lim
n
m(B
n
) = a = Aria(A).
Armat ia reciproca, rezulta imediat.
5.7.4 Observat ie.

In mod analog se denesc mult imile masurabile Jordan din R
3
(adica mult imile
care au volum)
5.7.5 Teorema. (Aria unui trapez curbiliniu). Fie f : [a, b] R o funct ie continua si nenegativa
si trapezul curbiliniu
T =
_
(x, y) R
2
[ a x b, 0 y f(x)
_
.
Atunci T are arie (este masurabil Jordan) si
Aria(T) =
_
b
a
f(x)dx. (5.7.1)
236 5. Funct ii integrabile Riemann si primitive
Demonstrat ie. Concluzia teoremei rezulta din Observat ia 5.2.12 (interpretarea geometrica a inte-
gralei Riemann) si din Teorema 5.7.3.
5.7.6 Corolar. (Aria mult imii cuprinse ntre doua arce de curba). Fie f, g : [a, b] R
funct ii continue asa ncat
f(x) g(x) 0, x [a, b].
Atunci mult imea
A =
_
(x, y) R
2
[ x [a, b], g(x) y f(x)
_
,
are arie si avem
Aria(A) =
_
b
a
(f(x) g(x)) dx. (5.7.2)
Demonstrat ie. Reprezentam mult imea An gura 5.7.1
Fig. 5.7.1
Daca consideram trapezele curbilinii
T
1
: =
_
(x, y) R
2
[ x [a, b], 0 y g(x)
_
,
T
2
: =
_
(x, y) R
2
[ x [a, b], 0 y f(x)
_
,
atunci avem ca A = T
2
T
1
si deci
Aria A = Aria(T
2
) Aria(T
1
) =
_
b
a
f(x)dx
_
b
a
g(x)dx =
_
b
a
(f(x) g(x)) dx.
5.7.7 Exemplu. Sa se calculeze aria mult imii E marginite de elipsa
x
2
a
2
+
y
2
b
2
= 1
Solut ie. Explicitam pe y din ecuat ia elipsei si gasim y =
b
a

a
2
x
2
, semnul minus corespunde
arcului de curba de sub axa Ox. Vom avea
Aria(E) = 4
_
a
o
b
a
_
a
2
x
2
dx.
Fig. 5.7.2
Cu schimbarea de variabila x = a sint, 0 t

2
obt inem
Aria(E) = 4
b
a
_
2
0
a
2
cos
2
t dt = 4ab
_
2
0
1 + cos 2t
2
dt = 2ab
_
t +
1
2
sin2t
_

2
0
= ab
5.7.2 Lungimea unui arc de curba
5.7.7 Denit ie. O curba plana este o funct ie continua : [a, b] R
2
, = (f, g), f, g : [a, b] R,
iar ecuat iile
()
_
x = f(t)
y = g(t), t [a, b],
constituie ecuat iile parametrice (reprezentarea parametrica) ale curbei .

In particular, daca f : [a, b] R


este o funct ie continua, atunci gracul sau, adica mult imea punctelor
G
f
=
_
(x, y) R
2
[ y = f(x), x [a, b]
_
,
reprezinta o curba plana data explicit prin ecuat ia y = f(x), x [a, b]. Evident ea poate parametrizata
prin ecuat iile parametrice
_
x = t
y = f(t), t [a, b]
5.7 Aplicat ii geometrice ale integralei Riemann 237
5.7.8 Denit ie.(curba rectiabila, lungimea unei curbe). Fie : [a, b] R
2
o curba plana de
ecuat ii parametrice
_
x = f(t)
y = g(t), t [a, b]
si e = (a t
0
, t
1
, t
2
, ..., t
n
= b) o diviziune a [a, b]. Numarul real
V (, ) =
n

i=1
|(t
i
) (t
i1
)| =
n

i=1
_
[f(t
i
) f(t
i1
)]
2
+ [g(t
i
) g(t
i1
)]
2
, (5.7.3)
se numeste variat ia curbei relativ la diviziunea . Curba se spune ca este rectiabila (sau ca
are lungime) daca mult imea V (, ) : Div[a, b] este marginita.

In acest caz numarul


l() := supV (, ) : Div[a, b] , (5.7.4)
se numeste lungimea curbei .
5.7.9 Observat ie. Daca consideran punctele M
i
(f(t
i
), g(t
i
)) , i = 0, n, ce apart in curbei = (f, g) :
[a, b] R
2
, atunci variat ia V (, ) reprezinta perimetrul (lungimea) liniei poligonale inscrisa n curba ,
iar lungimea curbei este marginea superioara a perimetrelor tuturor liniilor poligonale ce se pot nscrie
n curba .
Fig. 5.7.3
5.7.10 Denit ie. (curba neteda). Spunem ca curba plana : [a, b] R
2
, = (f, g) este neteda
daca funct iile f g : [a, b] R sunt derivabile, cu derivatele f
t
si g
t
continue, si care nu se anuleaza
simultan pe [a, b], adica
f
2
(t) +g
2
(t) ,= 0, t [a, b]
Se verica usor ca o curba neteda este o curba cu variat ia marginita (adica rectiabila).
5.7.11. Teorema. (lungimea unei curbe rectiabile (netede).Fie curba plana : [a, b] R
2
,
neteda, cu reprezentarea parametrica
() :
_
x = f(t)
y = g(t), t [a, b].
Lungimea curbei este data de formula
l() =
_
b
a
_
f
2
(t) +g
2
(t)dt. (5.7.5)
Demonstrat ie. Fie = (t
0
, t
1
, ..., t
n
) Div[a, b] si variat ia curbei relativ la diviziunea .
V (, ) =
n

i=1
_
[f(t
i
) f(t
i1
)]
2
+ [g(t
i
) g(t
i1
)]
2
.

In expresia de sub radical, aplicam formula cresterilor nite pentru f si g pe ecare [t


i1
, t
i
], deci exista

i
]t
i1
, t
i
[ si c
i
]t
i1
, t
i
[ asa ca sa avem
V (, ) =
n

i=1
_
f
2
(
i
) +g
2
(c
i
)(t
i
t
i1
) =
=
n

i=1
_
f
2
(
i
) +g
2
(
i
)(t
i
t
i1
) +
n

i=1
_
_
f
2
(
i
) +g
2
(c
i
)
_
f
2
(
i
) +g
2
(
i
)
_
(t
i
t
i1
)) .
Prima suma din ultima expresie este suma integrala Riemann pentru funct ia continua
_
f
2
(t) +g
2
(t),
diviziunea si sistemul de puncte intermediare = (
1
,
2
, ...,
n
), adica
(F, , ), unde F(t) =
_
f
2
(t) +g
2
(t), t [a, b].
238 5. Funct ii integrabile Riemann si primitive
Putem atunci scrie
[V (, ) (F, , )[ =

i=1
_
_
f
2
(
i
) +g
2
(c
i
)
_
f
2
(
i
) +g
2
(
i
)
_

(t
i
t
i1
)

i=1

_
f
2
(
i
) +g
2
(c
i
)
n

i=1
_
f
2
(
i
) +g
2
(
i
)

(t
i
t
i1
). (5.7.6)
Funct ia g(u, v) =
_
f
2
(u) +g
2
(v), g : [a, b] [a, b] R este continua pe compactul [a, b] [a, b], deci
este uniform continua.

Inseamna ca pentru orice > 0 exista > 0 asa ncat daca (u
t
, v
t
), (u
tt
, v
tt
)
[a, b] [a, b] si
|(u
t
, v
t
) (u
tt
, v
tt
)| = d ((u
t
, v
t
); (u
tt
, v
tt
)) < ,
adica [u
t
v
t
[ < , [u
tt
v
tt
[ < , sa avem
[g(u
t
, v
t
) g(u
tt
, v
tt
)[ <

b a
.

In cazul relat iei (5.7.6) cu u


t
=
i
, v
t
= c
i
; u
tt
=
i
, v
tt
=
i
pentru > 0, exista > 0 asa ncat daca
() < atunci cum [
i
c
i
[ < t
i
t
i1
() < , rezulta ca
[g(
i
, c
i
) g(
i
;
i
)[ =

_
f
2
(
i
) +g
2
(c
i
)
_
f
2
(
i
) +g
2
(
i
)

<

b a
si atunci
[V (, ) (F, , )] <
n

i=1

b a
(t
i
t
i1
) = , Div[a, b]. (5.7.7)
Daca aici n loc de consideram un sir de diviziuni (
j
) cu proprietatea ca (
j
) 0 pentru j ,
din faptul ca l() = supV (, ) : Div[a, b], atunci putem alege sirul (
j
) asa ca
supV (, ) : Div[a, b] = lim
j
V (, ) = l().
Acum relat ia (5.7.7) o scriem
[V (,
j
) (F,
j
, )[ < ,
j
Div[a, b] cu (
j
) 0.
Trecand aici la limita pentru j obt inem

l()
_
b
a
_
f
2
(t) +g
2
(t)dt

< , pentru orice > 0,


adica chiar (5.7.5).
5.7.12 Corolar. Daca curba este data explicit prin ecuat ia y = f(x), x [a, b], unde f este
derivabila si cu f
t
continua, atunci lungimea ei (adica lungimea gracului lui f) este
l() =
_
b
a
_
1 +f
t2
(x)dx. (5.7.8)
5.7.13 Exemplu. Sa se calculeze lungimea unui cerc de raza r. Putem presupune ca cercul este
cu centrul n origine si deci ecuat ia lui carteziana este x
2
+y
2
= r
2
. Daca parametrizam aceasta ecuat ie,
putem deduce ca ecuat iile parametrice ale acestui cerc sunt
_
x = r cos t,
y = r sint, t [0, 2].
5.7 Aplicat ii geometrice ale integralei Riemann 239
Cu formula (5.7.5) avem
l() =
_
2
0
_
r
2
sin
2
t +r
2
cos
2
tdt = r
_
2
0
dt = rt

2
0
= 2r.
5.7.14 Exemplu. Determinat i lungimea gracului funct iei
f :
_

3
,

2
_
R, f(x) = ln(sinx).
Utilizam formula (5.7.8) si obt inem
l() =
_
2

3
_
1 + ctg
2
xdx =
_
2

3
1
sinx
dx.
Cu substitut ia, cos x = t, sinxdx = dt, obt inem
l() =
_
0
1
2
dt
sin
2
x
=
_ 1
2
0
dt
1 t
2
=
_ 1
2
0
dt
t
2
1
=
=
1
2
ln

t 1
t + 1

1
2
0
=
1
2
ln
_
1
2
3
2
_
=
1
2
ln3 = ln

3.
5.7.15 Corolar. Daca curba este reprezentata n cordonate polare, = (), [
1
,
2
], atunci
lungimea ei este
l() =
_

2

1
_

2
() +
t2
()d. (5.7.9)

In adevar relat iile dintre coordonatele carteziene x, y si coordonatele polare , sunt


_
x = () cos ,
y = () sin, [
1
,
2
]
x
t
() =
t
() cos () sin
y
t
() =
t
() sin = () cos
x
t2
() +y
t2
() =
2
() +
t2
()

Inlocuind n (5.7.5), obt inem (5.7.9).


5.7.3 Volumul uni corp de rotat ie
Fie f : [a, b] R o funct ie continua si e f(x) 0, x [a, b]. Daca rotim gracul lui f n jurul axei
0x se obt ine un corp de rotat ie K
f
ca n gura 5.7.4. Are loc urmatoarea
Figura 5.7.4
5.7.16 Teorema. Daca f : [a, b] R e continua si nenegativa atunci
a) Corpul K
f
are volum (este o mult ime masurabila Jordan din R
3
)
b) Volumul sau este
Vol(K
f
) =
_
b
a
f
2
(x)dx.
240 5. Funct ii integrabile Riemann si primitive
Nu vom face n detaliu demonstrat ia teoremei, ci doar vom prezenta idea demonstrat iei.
Daca = (x
0
, x
1
, ..., x
n
) Div[a, b], corpul nostru se descompune n niste trunchiuri de con cu
nat imile x
i
x
i1
si razele bazelor f(x
i
), respectiv f(x
i1
).

Insumand volumele acestor trunchiuri de
con se obt ine

n
=

3
n

i=1
(x
i
x
i1
)
_
f
2
(x
i
) +f
2
(x
i1
) +f(x
i
)f(x
i1
)

. (5.7.11)
Daca alegem un sir de diviziuni (
n
) cu (
n
) 0, cum f
2
e integrabila Riemann, din (5.7.11) pentru
n se obt ine (5.7.10). Rat ionamentul e valabil n ipoteza ca un trunchi de con are volum si el este
V =
h
3
_
R
2
+r
2
+Rr
_
.
5.7.17 Exemplu. Sa se calculeze volumul corpului de rotat ie determinat de funct ia
f : [0, 1] R, f(x) = xe
x
.
Aplicand formula (5.7.10) se obt ine
Vol (K
f
) =
_
1
0
x
2
e
2x
dx =
_
1
0
x
2
_
1
2
e
2x
_
t
dx =
=

2
x
2
e
2x

1
0

_
1
0
xe
2x
dx =

2
e
2

_
1
0
x
_
1
2
e
2x
_
t
dx =
=
e
2
2


2
xe
2x

1
0
+

2
_
1
0
e
2x
dx =
e
2
2

e
2
2
+

4
e
2x

1
0
=

2
_
e
2
1
_
.
5.7.18 Corolar. Daca se dau funct iile continue f, g : [a, b] R, cu proprietatea 0 g(x)f(x),
x [a, b], atunci rotind gracele lor n jurul axei 0x, volumul corpului K
fg
de rotat ie obt inut este
Vol (K
fg
) =
_
b
a
_
f
2
(x) g
2
(x)

dx (5.7.12)
5.7.19 Exemplu. Sa se calculeze volumul torului, corpul obt inut prin rotat ia unui cerc care nu
intersecteaza axa Ox, n jurul axei Ox. (camera de bicicleta). Fie un cerc, putem putem presupune cu
centrul pe axa oy,C(o, b) de raza r, r < b, deci de ecuat ie
Figura 5.7.5
(x 0)
2
+ (y b)
2
= r
2
.
Explicitand aceasta ecuat ie, avem
y = b
_
r
2
x
2
.
Aplicam formula (5.7.12) cu f(x) = b +

r
2
x
2
si g(x) = b

r
2
x
2
si obt inem
Vol (K
f,g
) =
_
r
r
_
_
b +
_
r
2
x
2
_
2

_
b
_
r
2
x
2
_
_
dx =
= 4b
_
r
r
_
r
2
x
2
dx, x = r sint, t
_

2
,

2
_
Vol (K
f,g
) = 4b
_
2

2
r
2
cos
2
tdt = 4br
2
_
2

2
1 + cos 2t
2
dt =
= 2br
2
_
t +
1
2
sin2t
_

2
= 2
2
r
2
b.
5.7 Aplicat ii geometrice ale integralei Riemann 241
5.7.20 Exemplu. Sa se determine volumul corpului obt inut prin rotat ia n jurul axei 0x a mult imii
marginite de cercul x
2
+y
2
= 1 si de parabola y
2
=
3
2
x.
Punctele de intersect ie ale cercului cu parabola sunt A
_
1
2
,

3
2
_
, A
t
_
1
2
,

3
2
_
. Pentru a obt ine corpul
de rotat ie K
f
, se roteste gracul funct iei
f : [0, 1] R, f(x) =
_

_
_
3
2
x, x
_
0,
1
2
_
,

1 x
2
, x
_
1
2
, 1
_
.
Fig. 5.7.6
Acum aplicam formula (5.7.10)
Vol (K
f
) =
_
_ 1
2
0
3
2
xdx +
_
1
1
2
_
1 x
2
_
dx
_
=
3
4
x
2

1
2
0
+
_
x
x
3
3
_

1
1
2
=
=
3
16
+
_
2
3

1
2
+
1
24
_
=
3
16
+
5
24
=
19
48
.
5.7.4 Aria suprafet elor de rotat ie
Fie funct ia continua f : [a, b] R cu f(x) 0, x [a, b]. Rotind gracul lui f n jurul axei 0x se
obt ine corpul de rotat ie K
f
din gura 5.7.4. Dorim sa calculam aria laterala a acestui corp, adica aria
suprafet ei de rotat ie S
f
determinata de funct ia f.
5.7.21 Teorema. Daca f : [a, b] R este derivabila cu derivata f
t
continua, atunci
a) Suprafat a de rotat ie S
f
are arie;
b) Aria sa este data de
Aria(S
f
) = 2
_
b
a
f(x)
_
1 +f
t2
(x)dx. (5.7.12)
Demonstrat ie. Daca consideram o diviziune = (x
0
, x
1
, ..., x
n
) a intervalului [a, b] si pentru ecare
subinterval [x
i1
, x
i
] consideram trunghiul de con cu nalt imea x
i
x
i1
si razele bazelor f(x
i
), f(x
i1
)
respectiv, n ipoteza ca un trunchi de con are arie laterala data de
A
l
= G(R +r) = (f(x
i
) +f(x
i1
))
_
(x
i
x
i1
)
2
+ (f(x
i
) f(x
i1
))
2
,
unde G este generatoarea trunchiului iar R si r razele bazelor. Suma

n
=
n

i=1
(f(x
i
) +f(x
i1
))
_
(x
i
x
i1
)
2
+ (f(x
i
) f(x
i1
))
2
,
reprezinta suma ariilor laterale ale trunchiurilor de con, deci o valoare aproximativa a ariei suprafet ei S
f
.
Daca aplicam formula cresterilor nite sub radical , obt inem

n
=
n

i=1
(f(x
i
) +f(x
i1
))
_
1 +f
t2
(
i
) (x
i
x
i1
) ,
i
(x
i1
, x
i
) .
Sa luam funct ia F : [a, b] R, F(x) = 2f(x)
_
1 +f
t2
(x) si suma sa Riemann
(F, , ) =
n

i=1
F(
i
) (x
i
x
i1
) =
n

i=1
2f(
i
)
_
1 +f
t2
(
i
) (x
i
x
i1
) .
242 5. Funct ii integrabile Riemann si primitive
Atunci
[
n
(F, , )[ =

i=1
[((f(x
i
) f(
i
) +f(x
i1
) f(
i
))]
_
1 +f
t2
(
i
) (x
i
x
i1
)

i=1
([f(x
i
) f(
i
)[ +[f(x
i1
) f(
i
)[)
_
1 +f
t2
(
i
) (x
i
x
i1
) .
Funct ia f e continua pe [x
i1
, x
i
], deci uniform continua si deci > 0, > 0 asa ncat pentru x

, x

[x
i1
, x
i
] cu [x
t
x
tt
[ < , avem [f(x
t
) f(x
tt
[ <

2 (b a)
.
Atunci daca Div[a, b] este asa ca () < , obt inem
[
n
(F, , )[
n

i=1
_

2 (b a)
+

2 (b a)
_
M (x
i
x
i1
) ,
unde
M = sup
x[a,b]
_
1 +f
t2
(x)
(f
t
e continua pe un compact deci marginita).

In nal
[
n
(F, , )[
M
b a
n

i=1
(x
i
x
i1
) = M, > 0.
Alegand un sir de diviziuni (
j
) cu (
j
) 0 deducem ca
lim
j
(
n
(F,
j
, )) = 0.

Insa
lim
j
(F,
j
, ) =
_
b
a
F(x)dx =
_
b
a
2f(x)
_
1 +f
t2
(x)dx
si lim
j

n
=Aria(S
f
) aria laterala a suprafat ei de rotat ie. Asadar am obt inut relat ia (5.7.12)
5.7.22 Exemplu. Sa se deduca aria sferei de raza r.
Suprafat a sferei de raza r se obt ine prin rotat ia n jurul lui 0x a unui cerc cu centrul n origine de raza
r, de ecuat ie x
2
+y
2
= r
2
, adica prin rotat ia gracului funct iei
y = f(x) =
_
r
2
x
2
, x [r, r]
Avem f
t
(x) =
x

r
2
x
2
si cu formula (5.7.12)
Aria (S
f
) = 2
_
r
r
_
r
2
x
2
_
1 +
x
2
r
2
x
2
dx = 2
_
r
r
rdx = 2rx

r
r
= 2r
2
+ 2r
2
= 4r
2
.
5.7.23 Exemplu. Sa se calculeze aria unui elipsoid de rotat ie, adica aria suprafet ei de rotat ie
obt inute prin rotirea elipsei
x
2
a
2
+
y
2
b
2
= 1, cu a > b n jurul lui 0x.
Figura 5.7.7
Din ecuat ia elipsei se obt ine, pentru arcul de elipsa situat deasupra axei Ox,
y = f(x) =
b
a
_
a
2
x
2
, x [a, a],
f
t
(x) =
b
a
x

a
2
x
2
5.7 Aplicat ii geometrice ale integralei Riemann 243
Aria(S
f
) = 2
a
_
a
f(x)
_
1 +f
t2
(x)dx =
2
a
a
_
a

a
2
x
2
_
1 +
b
2
a
2

x
2
a
2
x
2
dx =
=
2b
a
a
_
a
_
a
2
x
2
+
b
2
a
2
x
2
dx =
2b
a
a
_
a
_
a
2

a
2
b
2
a
2
x
2
dx.
Se stie ca la elipsa a
2
b
2
= c
2
si e =
c
a
=

a
2
b
2
a
se numeste excentricitatea elipsei. Deci
Aria(S
f
) =
2b
a
a
_
a
_
a
2
(ex)
2
dx =
2b
ea
ae
_
ae
_
a
2
t
2
dt
=
2b
ae
ae
_
ae
a
2
t
2

a
2
t
2
dt =
2b
ae
_
_
a
2
arcsin
t
a

ae
ae

ae
_
ae
t
_
_
a
2
t
2
_
t
dt
_
_
.

In nal se obt ine


Aria(S
f
) = 2ab
_
arcsine
e
+
_
1 e
2
_
.
Capitolul 6
Probleme de analiza matematica
6.1 Enunt uri
Siruri de numere reale
6.1.1 Folosind denit ia unui sir, aratat i ca sirul cu termenul general
a) a
n
=
n 1
2n + 1
, n 1 are limita a =
1
2
.
b) a
n
=
_
1
5
_
n
, n 1 are limita a = 0.
6.1.2 Folosind criteriul general al lui Cauchy de divergent a a sirurilor, artat i ca sirul cu termenul general
a) a
n
=
1

1
+
1

2
+. . . +
1

n
, n 1 este divergent;
b) a
n
=
1
1 4
+
1
4 7
+. . . +
1
[1 + 3(n 1)](1 + 3n)
, n 1 este convergent.
6.1.3 Folosind teorema de convergent a a sirurilor monotone si marginite sa se studieze convergent a
sirurlui cu termenul general denit prin relat ia de recurent a a
1
=

2, a
n
=

2 +a
n
, n 2.
6.1.4 Se da sirul (x
n
)
nN
denit prin relat ia de recurent a
x
n+1
= x
n
x
2
n
, n 1 x
0
= a, pentru 0 a 1.
Se cere:
a) Sa se arate ca acest sir este convergent;
b) Sa se calculeze limita acestui sir;
c) Sa se calculeze lim
n
n

k=0
x
2
k
.
(I.M. Stancu, GMB, 7892).
6.1.5 Fiind dat sirul cu termenii pozitivi (x
n
)
nN
cu x
0
(0, 2) si x
4
n+1
= 2x
2
n
+ 8 , () n N, sa se
demonstreze ca sirul (x
n
)
nN
este convergent si sa se calculeze limita sa.
6.1.6 Sa se demonstreze ca daca sirul de numere reale (a
n
)
nN
satisface condit iile a
2
n+1
< a
2
n
si
0 < a
n+1
+a
n
oricare ar n N, atunci sirul este convergent.
(D.M.Batinet u, G.M. 16374)
244
6.1 Enunt uri 245
6.1.7 Se dau numerele a
0
, b
0
, c
0
R si se denesc sirurile
a
n+1
=
1
2
(b
n
+c
n
) , b
n+1
=
1
2
(c
n
+a
n
) ; c
n+1
=
1
2
(a
n
+b
n
)
pentru orice n N

. Sa se arate ca sirurile (a
n
)
n0
; (b
n
)
n0
; (c
n
)
n0
sunt convergente si au limita egala
cu
1
3
(a
0
+b
0
+c
0
).
6.1.8 Sa se calculeze limita sirului cu termenul general:
a) a
n
=
1
p
+ 2
p
+. . . +n
p
(n + 1)
p
, n 1, p N;
b) a
n
=
1
n
_
1
ln2
+
1
ln3
+. . . +
1
lnn
_
, n 2.
Serii de numere reale
6.1.9 Utilizand criteriile de comparat ie, stabilit i natura seriilor:
a)

n=2
1
n

lnn
; b)

n=2
a
n
n

n!
c)

n=1
1
n2
n
; d)

n=2
ln n
n
.
6.1.10 Utilizand criteriul de condensare Cauchy, studiat i convergent a seriei

n=2
1
n

(lnn)

.
6.1.11 Folosind criteriul lui Cauchy, studiat i convergent a seriilor:
a)

n=1
sinn
n(n + 1)
; b)

n=1
cos 2n
2
n
.
6.1.12 Studiat i natura seriilor:
a)
(an)
n
n!
, a 0; b)
n!e
n
n
n+p
, p N; c)

n=1
_
3n
3n + 1
_
n
; d)

n=1
2
n
n!
n
n
.
6.1.13 Studiat i seria

n=1
a
n
, cu a
2k1
=
2
k1
3
k1
, a
2k
=
2
k
3
k
pentru k N

folosind criteriul DAlembert si


criteriului radicalului (Cauchy).
6.1.14 Studiat i convergent a seriilor alternate:
a)

n=1
(1)
n+1

1
2n 1
; b)

n=1
(1)
n+1

2n + 1
n(n + 1)
.
6.1.15 Asigurandu-va ca nu se aplica criteriul lui Leibniz la seriile alternante urmatoare, precizat i care
serii sunt divergente, serii convergente si absolut convergente.
a)

a
n
, cu a
2k1
=
1

k + 1 1
, a
2k
=
1

k + 1 1
, k N

;
b)

a
n
, cu a
2k1
=
1
2
k1
, a
2k
=
1
2
k1
, k N

.
6.1.16 Studiat i convergent a absoluta si semiconvergent a seriei

n=1
a(a 1) . . . (a n + 1)
n!
, a R N.
6.1.17 Aratat i ca seria

n=1
n!
( + 1) ( +n 1)
, unde > 0, este convergenta pentru > 2 si diver-
genta pentru 2.
6.1.18 Sa se studieze natura seriilor a)

n=1
a
ln n
, a R

+
.; b)

n=1
ln
_
1 +
1
n
_
.
246 6. Probleme de analiz a matematic a
Funct ii continue
6.1.19 Studiat i continuitatea funct iilor:
a) f : R R, f (x) =
_
_
_
x +e
1
sin x
x
, daca x ,= k, k Z
1, daca x = k, k Z;
b) f : [1, +) R, f (x) =

x
2
4x

arccos
_
1

x
_
;
c) f : R R, f (x) =
_
7x 12, dac a x
x
2
, daca x R .
6.1.20 Stabilit i mult imea de continuitate a funct iei f : R R pentru
a) f (x) = x
2

_
x
2

;
b) f (x) = [x] sinx;
c) f (x) =
_
_
1
x

[x]
+ [x] , x (0, )
1, daca x (, 0] ,
unde [] este funct ia partea ntreaga.
6.1.21 Studiat i continuitatea funct iilor:
a) f : R R, f (x) = lim
n
[sinx[ e
nx
+ cos x
e
nx
+ 1
;
b) f : R R, f (x) = lim
n
n

1 +x
2n
;
c) f : (0, ) R, f (x) = lim
n
ax
2
n
ln x
+x
xn
ln x
+1
, unde a R.
6.1.22 Vericat i daca urmatoarele funct ii pot prelungite prin continuitate n punctul x
0
= 0:
a) f : [1, 0) (0, +) R, f (x) =

1 +x 1
3

1 +x 1
;
b) f : [1, 0) (0, +) R, f (x) =
_
_
_
2tgxarctg
1
x
, x [1, 0)
2

1+x1
x
2
e
x
, x (0, +)
c) f : (0, +) R, f (x) = lnx;
d) f : R 0 R, f (x) = sin
2
x
.
6.1.23 a) Determinat i parametrul a > 0 pentru care funct ia f : [1 +) R,
f (x) =
_

_
_
a
x
+ 4a
x
2
+x
2
, x [1, 2]
ln(x 1)
4 sin(x2)
(x 2)
2
, x (2, +)
este continua.
b) Determinat i a, b R pentru care funct ia f : R R,
f (x) =
_

_
_
x 1
x + 1
_2x
|x|
, daca x (, 1) (1, +)
ax
[x[
sin
_

2
(1 x)
2
_
, daca x [1, 0) (0, 1]
b, daca x = 0
este continua.
6.1.24 Demonstrat i ca daca funct ia f : R R este continua pe R si daca c > 0 este o constanta arbitrara,
atunci funct ia
f
c
: R R, f
c
(x) =
_
_
_
c, daca f (x) < c
f (x) , daca [f (x)[ c
c, daca f (x) > c
6.1 Enunt uri 247
este continua pe R si este marginita.
6.1.25 Demonstrat i ca daca o funct ie monotona f : [a, b] R, unde a, b R, a < b, realizeaza toate
valorile cuprinse ntre f (a) si f (b) , atunci f este continua pe [a, b] .
6.1.26 Fie a R xat. Consideram funct ia f : R [1, 1],
f (x) =
_
sin
1
x a
, daca x ,= a
0, daca x = a.
Demonstrat i ca pentru orice b R, b > a funct ia f pe intervalul [a, b] realizeaza toate valorile cuprinse
ntre f (a) si f (b) , dar funct ia f nu este continua pe [a, b] .
6.1.27 Fie I R un interval nedegenerat. Aratat i ca daca pentru o funct ie f : I R exista o constanta
L (0, ) astfel ncat
[f (x
1
) f (x
2
)[ L [x
1
x
2
[ , x
1
, x
2
I
atunci funct ia f este continua pe I. (Constanta L se numeste constanta Lipschitz.)
6.1.28 Dat i cate un exemplu de funct ie marginita pentru care mult imea punctelor de discontinuitate este
o mult ime
a) nita avand p elemente, unde p N

numar dat;
b) numarabila;
c) nenumarabila.
Funct ii derivabile
6.1.29 Consideram funct ia f : [0, +) R,
f (x) =
_
[xlnx x[ , daca x (0, +)
0, daca x = 0.
a) Studiat i continuitatea si derivabilitatea funct iei.
b) Aratat i ca restrict ia funct iei f la intervalul (e, +) are inversa si studiat i derivabilitatea funct iei
inverse.
6.1.30 Se considera funct ia f : R R,
f (x) =
_
x
n
sin
1
x
, daca x (, 0) (0, +)
0, daca x = 0,
cu n 2, 3, . . .. Aratat i ca:
a) funct ia f este ori de cate ori derivabila pe mult imea (, 0) (0, +);
b) n punctul x
0
= 0 funct ia f este derivabila de
_
n
2

ori dar nu este derivabila de


_
n
2

+ 1 ori;
c) derivata f
([
n
2
])
este continua n punctul x
0
= 0 daca n este impar si este discontinua n x
0
= 0
daca n este par. Prin [] s-a notat funct ia parte ntreaga.
6.1.31. a) Determinat i unghiul dintre axa Ox si dreapta tangenta la gracul funct iei
f : R R, f (x) =
2
3
x
5

1
9
x
3
n punctul de abscisa x = 1. Determinat i unghiul dintre axa Ox si normala la grac n acelasi punct.
Scriet i ecuat iile celor doua drepte ment ionate.
b) Are gracul funct iei f : [0, ] R, f (x) = 1 + sinx puncte n care tangenta dusa la grac este
paralela cu dreapta de ecuat ie x + 2y 5 = 0?
c) Pe arcul AB, unde A(0, 0) ; B(3, 18) , al gracului funct iei f : R R, f (x) = x
3
3x, determinat i
un punct n care tangenta la grac este paralela la coarda AB.
248 6. Probleme de analiz a matematic a
d) Determinat i unghiul sub care se intersecteaza curbele ce reprezinta gracul funct iilor f (x) = 8x
2
si g (x) = x
2
(parabole).
e) Determinat i unghiul sub care se intersecteaza curbele ce reprezinta gracul funct iilor f (x) = x
3
si g (x) =
1
x
2
.
f) Pentru gracul funct iei f : R R, f (x) =

x
3
x

punctul (1, 0) este punct unghiular. Scriet i


ecuat ia semidreptelor tangente n acest punct gracului funct iei si determinat i unghiul format de cele
doua semidrepte.
6.1.32 a) Demonstrat i ca e
x
> 1 x +
1
2
x
2

1
6
x
3
+
1
24
x
4

1
120
x
5
, x (0, +) .
b) Demonstrat i ca funct ia f : (0, ) R, f (x) =
_
1 +
1
x
_
x
este strict crescatoare si g : (0, ) R,
g (x) =
_
1 +
1
x
_
x+1
este strict descrescatoare.
6.1.33 Determinat i punctele de extrem local pentru funct ia f : R R, f (x) = 2 cos x
1
2
cos 2x.
6.1.34 Stabilit i intervalele pe care funct ia f : R R, f (x) =
h

e
h
2
x
2
, h > 0, este convexa respectiv
concava si gasit i punctele de inexiune ale funct iei.
6.1.35 a) Aratat i ca daca radacinile unui polinom P de gard n 2 cu coecient i reali sunt numere reale
si distincte, atunci ecare derivata a polinomului, pana la ordinul (n 1) inclusiv, are radacinile reale si
distincte.
b) Demonstrat i ca daca o funct ie f : [a, b] R este continua pe [a, b] , derivabila pe (a, b) si cu
proprietatea ca exista c (a, b) astfel ncat (f (b) f (c)) (f (c) f (a)) < 0, atunci exista cel put in un
punct d (a, b) astfel ncat f
t
(d) = 0.
6.1.36 a) Fie I R un interval nedegenerat si f : I I o funct ie derivabila pentru care exista M [0, 1)
astfel ncat [f
t
(x)[ M, x I. Aratat i c a funct ia f este o contract ie.
b) Consideram funct ia f : R R, f (x) = axarctgx
a
2
ln
_
x
2
+ 1
_
. Determinat i a R astfel ncat
funct ia f sa e contract ie.
6.1.37. i) Pentru x apropiat de 0, aproximat i f (x) utilizand polinomul MacLaurin de ordinul 4.
a) f :
_

2
,

2
_
R, f (x) = tgx;
b) f : R R, f (x) = a
x
, unde a (0, 1) (1, +);
c) f : (a, a) R, f (x) =
1
a x
, unde a > 0;
d) f : (a, a) R, f (x) =
1
2a
ln
a +x
a x
, unde a > 0;
e) f (x) =
n

a
n
x, unde n 2, 3, . . ., a > 0;
ii) Pentru x apropiat de x
0
aproximat i f (x) utilizand polinomul Taylor de ordinul 3.
a) f : (0, +) R, f (x) =
1
x
, x
0
= 2;
b) f : (0, +) R, f (x) = x
x
, x
0
= 1.
6.1.38 a) Determinat i un interval [a, a] pe care polinomul MacLaurin de ordinul 3 pentru funct ia
f : R R, f (x) = sinx aproximeaza funct ia cu o precizie de 0, 01.
b) Aproximat i numarul sin(0, 6) cu o precizie de 0, 01.
6.1 Enunt uri 249
Funct ii integrabile Riemann si primitive
6.1.39 Scriind sirurile de mai jos ca sume Riemann, sa se calculeze limitele lor
a) a
n
=
n

i=1
1
n +i
; b) a
n
=
n

i=1
i
4
n
5
;
c) a
n
=
n1

i=0
n
n
2
+i
2
; d) a
n
=

n
n1

i=1
sin
i
n
;
e) a
n
=
n

i=1
1

4n
2
i
2
; f) a
n
=
n

i=1
1
n +i
n+1
n
;
g) a
n
=
1

n
n

i=1
1

5n + 2i
; h) a
n
=
n

i=1
1
n
sin
2
i
n
.
6.1.40 Sa se calculeze limita sirului (a
n
) unde
a) a
n
=
n+1
_
n
dx
1 +x
2
; b) a
n
=
1
_
0
x
n
1 +x
.
6.1.41 Fie f : [0, 1] R o funct ie marginita. Sa se calculeze limita sirului cu termenul general
a
n
=
_
1
0
x
n
f(x)dx, n 1.
6.1.42 Daca f : [0, 1] R este o funct ie monoton crescatoare, atunci are loc
f(a)
1
b a
_
b
a
f(x)dx f(b).

In particular, au loc inegalitat ile


1
_
1
0
e
x
2
dx e;

3
_
5
2
_
x 1
x + 1
dx

6.
6.1.43 Sa se arate ca funct iile:
a) f : [0, 2] R, f(x) = min
_
2
x
2
+ 1
, x
_
;
b) g : [0,
5
2
R, g(x) = x [x], unde [x] nseamna partea ntreaga a lui x,
c) h : [0, 1] R,
h(x) =
_

_
x
2
, x [0, 1]
_
1
2
,
1
3
,
1
4
_
1, x = 1/3
1, x
_
1
2
,
1
4
_
sunt integrabile Riemann, iar apoi sa se calculeze integralele lor
6.1.44 Aratat i ca urmatoarele funct ii admit primitive si deducet i primitivele lor:
a) f : R R, f(x) =
_
x
3
, x 0
x, x > 0
b) f : R R, f(x) =
_
e
x
, x < 0
3x + 1, x 0
c) f : [0, +[ R, f(x) =
_
_
_

x +
3

x, x [0, 1)
1

x
+
1
3

x
, x 1
250 6. Probleme de analiz a matematic a
d) f :]

2
],

2
R, f(x) =
_
_
_
sinx + cos x, x
_
0,

2
_
1 + tgx, x
_

2
, 0
_
6.1.45 1) Sa se arate ca funct iile urmatoare nu admit primitive:
a) f : R R, f(x) =
_
0, x 0
1
x
, x > 0
b) f : R R, f(x) =
_
x, x
x
2
, x R
c) f : R R, f(x) =
_
a, x 0, a R
sin
1
x

1
x
cos
1
x
, x > 0
d) f : R R, f(x) =
_
3 x, x
2
x
, x R
e) f : R R, f(x) =
_
arctg
1
x
, x < 0
1, x 0
f) f : R R, f(x) =
_
1, x = 0
sinx
x
, x ,= 0
g) f : R
+
R, f(x) =
_
1, x = 0
xln
1
x
, x > 0
2) Aratat i ca funct iile urmatoare admit primitive:
a) f : [0, +[ R, f(x) = min
_
1, x, x
2
, ...x
n
_
, n N

;
b) f : [0, +[ R, f(x) = max
_
1, x, x
2
, ..., x
n
_
, n N

;
c) f : R R, f(x) =
_
sin
1
x
n
, x ,= 0, n N

0, x = 0
d) f : R R, f(x) =
_
cos
1
x
n
, x ,= 0, n N

0, x = 0
e) f : R R, f(x) =
_
x
n
sin
k
x
, x ,= 0, n N

, k ,= 0
0, x = 0
6.1.46 Folosind integrarea prin part i sa se calculeze:
a)
_
xlnx, x ]0, +[ ; b)
_
lnx
2
dx, x ]0, +[ ;
c)
_
e
3x
_
x
2
+ 3x
_
dx, x R; d)
_
e
3x
cos 2xdx, x R;
e)
_
arcsinxdx, x ]1, 1[ ; f)
_
arctgxdx, x R;
g)
_
9 x
2
dx, x ]3, 3[ ; h)
_
x
2
9dx, x ]3, +[ ;
i)
_
x
2
+ 9dx, x R; j)
_
x

9 x
2
dx, x ]3, 3[
k) I
n
=
_
x
n

x
2
+ 2
dx, x R; n 2. Se cere o formula de recurent a si apoi I
2
si I
3
.
l) I
n
=
_
x
n
sin2xdx, x R; n 2. Se cere o formula de recurent a si apoi I
2
si I
3
.
6.1.47 Utilizand prima sau a doua metod a de schimbare de variabila sa se calculeze primitivele funct iilor
urmatoare:
a) f (x) =
sinx
1 + cos
2
x
, x R; b) f (x) =
arctg
5
x
1 +x
2
, x R;
c) f (x) =
x
2
x
6
+ 1
, x R; d) f (x) =
e
tgx
cos
2
x
, x
_

2
,

2
_
;
6.1 Enunt uri 251
e) f (x) =
x
3

x
8
+ 1
, x R; f) f (x) =
1
x
_
ln
2
x + 2
_, x ]0, +[ ;
g) f (x) =
2
x
4
x
1
, x > 0; h) f (x) = (x + 1)

x
2
+ 2x, x > 0;
l) f (x) =
1
e
x

1 e
2x
, x > 0; m) f (x) =

16 x
2
, x ]4, 4[ ;
n) f (x) =
1

x + 1
3

x + 1
, x > 0; o) f (x) =
1
x
2

9 x
2
, x ]0, 3[ ;
p) f (x) =
1
x

3x
2
2x 1
, x > 1; r) f (x) =
1
_
x(1 x)
, x ]0, 1[ ;
s) f (x) =
ln(cos x)
cos
2
x
, x
_
0,

2
_
; t) f (x) = xe
x
2
_
1 +x
2
_
, x R;
6.1.48 Sa se calculeze primitivele urmatoarelor funct ii rat ionale:
a) f (x) =
x
(2x + 1) (x + 1)
, x >
1
2
; b) f (x) =
x
2x
2
3x 2
, x>2;
c) f (x) =
x 4
x(x 1)
2
, x > 1; d) f (x) =
2x + 1
x
2
+ 5
, x R;
e) f (x) =
x 1
x
2
+ 4x + 5
, x R; f) f (x) =
1
x
3
+ 1
, x > 1;
g) f (x) =
1
x
4
+ 1
, x R; h) f (x) =
x
2
(x
2
+ 9)
2
, x R;
i) f (x) =
1
(x
2
+x + 1)
2
, x R; j) f (x) =
4x
2
8x
(x 1)
2
(x
2
+ 1)
, x>1;
l) f (x) =
x + 4
(x
2
+ 4)
3
, x R; m) f (x) =
x
3
+x 1
(x
2
+ 1)
2
, x R;
6.1.49 Calculat i primitivele urmatoarelor funct ii trigonometrice
a) f (x) =
1
5 + 4 sinx
, x
_

2
,

2
_
; b) f (x) =
sin
2
x
1 + cos
2
x
, x
_

2
,

2
_
;
c) f (x) =
1
2 sinx cos x + 5
, x
_

2
,

2
_
; d) f (x) = sin
2
x cos
3
x, x R;
e) f (x) = sin
5
x, x R; f) f (x) =
1
sin
4
x cos
4
x
, x
_
0,

2
_
;
g) f (x) = cos xcos 3x cos 6x, x R; h) f (x) =
1
tg
8
x
, x
_
0,

2
_
;
i) f (x) =

1 + cos x, x [0, 2] ; j) f (x) =


sin
3
x

cos x
, x
_
0,

2
_
;
6.1.50 Sa se calculeze
a)
_
1
0
dx
(x + 1)

x
2
+x + 1
b)
1
_
1
dx
(x
2
+ 4)

4 x
2
c)
_
dx
x

x
2
x + 1
; x ], 0[ d)
3
_
2
dx
(2x 3)

4x x
2
;
e)
_
dx
x
3

x
2
+ 1
, x > 0; f)
1
_
1
dx
4

x
4
+ 1
;
g)
1
_
1
xdx
_
1 +
3

x
2
; h)
_
x
4
+x
3
dx, x > 0;
i)
_
dx
e
2x
+e
x
2
, x > 0; j)
_

x

1 x
3
dx, x (0, 1) ;
k)
_
2
0
x
2

1 +x
3
dx; l)
_
4
0
ln(1 +tgx) dx.
252 6. Probleme de analiz a matematic a
6.1.51 Calculat i ariile marginite de curbele urmatoare si axa Ox:
a) y = f (x) =
_
x 1
x + 1
, x [1, 2] b) y = f (x) =
x +

x
2
1

[x + 2[
, x [0, 4]
6.1.52 Calculat i aria cuprinsa ntre curbele:
a) y = f (x) =
x
2
2
, y = g (x) =
1
x
2
+ 1
, x [1, 1];
b) y = f (x) = x
2
+ 6x, y = g (x) = x
2
4;
c) y = f (x) = ln
2
x, y = g (x) = lnx, x > 0;
d) Ariile marginite de interiorul cercului x
2
+y
2
= 16 si parabola y
2
= 6x;
e) Aria cuprinsa n interiorul parabolelor y
2
= 8 (2 x) , y
2
= 24 (2 +x).
6.1.53 Sa se calculeze volumele corpurilor de rotat ie obt inute prin rotat ia gracului funct iilor urmatoare,
n jurul axei 0x
a) f :
_
0,
1
2
_
R, f (x) = arcsinx;
b) f : [0, 3] R, f (x) =
_
x(x 3)
x 4
;
c) f : [a, b] R, f (x) = f (x) =
4
_
(x a) (b x), a < b;
d) f : [1, e] R, f (x) = xlnx;
e) Volumul corpului obt inut prin rotat ia n jurul axei 0x a mult imii marginite de cercul x
2
+y
2
= 4
si parabola y
2
= 3x
6.1.54 Calculat i lungimea gracului funct iilor:
a) f : [1, 4] R, f (x) =
x
2
2
ln
4

x;
b) f : [0, 1] R, f (x) =
e
x
+e
x
2
;
c) f :
_
0,
2
3
_
R, f (x) = ln
1
1 x
2
;
d) f :
_

3
,

2
_
R, f (x) = ln(sinx) ;
e) f : [0, 3] R, f (x) =

9 x
2
.
6.2 Solut ii
Siruri de numere reale
6.2.1 Cerint a de a justica ca un sir are o anumita limita data este echivalenta cu: determinat i N()
ncepand de la care are loc inecuat ia (n variabila n): [a
n
a[ < .

In cazul exemplelor date:
a) [a
n
a[ =

n 1
2n + 1

1
2

< implica

2n 2 2n 1
2(2n + 1)

< sau
3
2(2n + 1)
< . Atunci
2(2n + 1)
3
>
1

este echivalenta cu n >


_
3
2
1
_

1
2
. Deci N() =
_
1
2

_
3
2
1
__
.
Spre exemplu: pentru =
1
10
obt inem
N() =
_
1
2

_
3
2
1
10
1
__
=
_
1
2
(15 1)
_
= [7] = 7.
6.2 Solut ii 253
Pentru = 0, 15 rezulta
N() =
_

_
1
2

_
_
_
3
2
15
100
1
_
_
_
_

_ =
_
1
2
(
300
30
1)
_
=
_
9
2
_
= [4, 5] = 4.
b) Avem lim
n
_
1
5
_
n
= 0, deoarece

_
1
5
_
n
0

< sau 5
n
>
1

. Logaritmam ambii membrii ai


inecuat iei si avem n > log
5
1

. Deci N() =
_
log
5
1

_
si am justicat ca lim
n
_
1
5
_
n
= 0.
Exemplu: pentru = 0, 02 =
2
100
=
1
50
avem
N() =
_
log
5
1

_
= [log
5
50] =
_
log
5
5
2
2

= [2 log
5
5 + log
5
2] = [2 + 0, ...] = 2.
6.2.2 a) Divergent a sirului dat se va arata prin negarea criteriului general al lui Cauchy. Aceasta
revine la a gasi un majorant pentru
[a
n+p
a
n
[ =
1

n + 1
+
1

n + 2
+. . . +
1

n +p
.
Deci
1

n + 1
+. . .
1

n +p
> p
1

n +p
aleg
p=n
=
n

2n
=
_
n
2
care nu poate oricat de mic (n 1).

Intr-adevar [a
2n
a
n
[ >
n

n
, deci (a
n
) este divergent.
b) Avem
a
n
=
n

k=1
1
[1 + 3(n 1)](1 + 3n)
=
1
3
n

k=1
_
1
1 + 3(n 1)

1
1 + 3n
_
=
1
3
_
1
1
1 + 3n
_
, n 1.
Atunci
[a
n+p
a
n
[ =
1
3
_
1
1 + 3n

1
1 + 3n + 3p
_
<
1
3

1
1 + 3n
pentru orice p N

, n N. Din inecuat ia
1
3

1
1 + 3n
< rezulta n >
1
3
_
1
3
1
_
. Deci
() > 0 () N() =
_
1
3
_
1
3
1
__
, p N

are loc [a
n+p
a
n
[ < .
Sirul (a
n
) este convergent.
6.2.3 Observam imediat ca a
n
=
_
2 +
_
2 +. . . +

2 > a
n1
si a
n
> 0, n N. Folosind inegali-
tatea

a
1 +a
2
, a > 0 vom avea:
a
n
=

2 +
_
2 +. . . +

2
. .
n ori

1 + 2 +
_
2 +. . .

2
2
=
3
2
+
a
n1
..
_
2 +. . . +

2
2

3
2
+
a
n1
2

3
2
+
3
2
2
+
3
2
3
+. . . +
a
1
2
n1
<
3
2
+
3
2
2
+. . . +
3
2
n
= 3
_
1
2
+
1
2
2
+. . . +
1
2
n
_
= 3
_
_
_
1
2
n+1

1
2
1
2
1
_
_
_ = 3
_
1
1
2
n
_
< 3.
254 6. Probleme de analiz a matematic a
Sirul (a
n
) este marginit:

2 < a
n
< 3 si ind monoton crescator este convergent. Sa aam limita sa.
Din relat ia de recurent a a
n
=

2 +a
n1
notand lim
n
a
n
= lim
n
a
n1
= l, obt inem prin trecerea la limita
pentru n , relat ia l =

2 +l, de unde l
1,2
=
1

1 + 8
2
. Convine l = 2 (

2, 3) care este limita


sirului.
6.2.4 a) Relat ia de recurent a va da:
x
n+1
x
n
= x
2
n
0, pentru orice n N.
deci
x
n+1
< x
n
< . . . < x
0
= a 1.
Sirul este monoton descrescator si are marginea inferioara 1. Pe de alta parte, x
1
= x
0
(1 x
0
), iar
x
0
= a [0, 1] 1 x
0
[0, 1]. Rezulta x
1
[0, 1]. Prin induct ie rezulta x
n
[0, 1] pentru orice n N,
deci este marginit. Sirul este monoton si marginit, deci convergent.
b) Notam lim
n
x
n
= lim
n
x
n+1
= l si trecand la limita n relat ia de recurent a se obt ine ecuat ia
l = l l
2
Limita sirului este l = 0.
c) Avem
n

k=0
x
2
k
=
n

k=0
(x
k
x
k1
) = [x
n
x
0
] = x
n
+x
0
.
Atunci lim
n
n

k=0
x
2
k
= x
0
= a.
6.2.5 Din x
0
(0, 2) va rezulta
x
4
1
= 2x
2
0
+ 8 < 2 2
2
+ 8 = 16, deci x
1
(0, 2).
Dar
x
4
1
x
4
0
= 2x
2
0
+ 8 x
4
0
= x
4
0
+ 2x
2
0
+ 8 = (x
2
0
1)
2
+ 9.
Cum
x
0
< 2 x
2
0
< 4 x
2
0
1 < 4 1 = 3 (x
2
0
1)
2
< 9 si x
4
1
x
4
0
> 0 x
1
> x
0
.
Prin induct ie se demonstreaza ca x
n
< x
n+1
< 2, n 1. Sirul este monoton si x
n
(0, 2) deci
convergent. Limita sa se va aa din relat ia de recurent a care implica ecuat ia l
4
2l
2
8 = 0. Dintre
radacinile l
1
= 2, l
2
= 2, l
3
= i

2, l
4
= i

2. convine l
1
= 2 > 0. Deci lim
n
x
n
= 2.
6.2.7 Adunam relat iile care denesc a
n+1
; b
n+1
; c
n+1
si obt inem
a
n+1
+b
n+1
+c
n+1
= a
n
+b
n
+c
n
= . . . = a
0
+b
0
+c
0
R.
Notam cu a
0
+ b
0
+ c
0
= a. Pe de alta parte b
n
+ c
n
= 2a
n+1
; b
n
+ c
n
= a a
n
= 2a
n+1
. Relat ia
2a
n+1
= a a
n
, va da:
2a
n+1
= a a
n
2a
n
= a a
n1

1
2
_
2a
n1
= a a
n2

1
2
_
2
. . . . . . . . . . . . . . . . . . . . .
2a
1
= a a
0

1
2
_
n

Inmult im relat iile cu factorii din dreapta si adunand pe coloane, obt inem
2a
n+1
= a
_
1 +
_

1
2
_
+
_

1
2
_
2
+. . . +
_

1
2
_
n
_
a
0
_

1
2
_
n
=
6.2 Solut ii 255
= a
1
_

1
2
_
n+1
1 +
1
2
a
0
_

1
2
_
n
=
a
_
1
_

1
2
_
n+1
_
3
2
a
0
_

1
2
_
n
.
Trecand la limita pentru n si observand ca lim
n
_

1
2
_
n
= 0, va rezulta
lim
n
a
n+1
=
1
2
_
2
3
a
_
=
1
3
a =
a
0
+b
0
+c
0
3
.

In acelasi mod se obt ine lim


n
b
n+1
= lim
n
c
n+1
=
a
0
+b
0
+c
0
3
.
Interpretarea geometrica. Fie A
0
(a
0
, x
0
); B
0
(b
0
, y
0
); C
0
(c
0
, z
0
) trei puncte n plan, care formeaza un
triunghi. Punctele
A
1
_
b
0
+c
0
2
,
z
0
+y
0
2
_
; B
1
_
a
0
+c
0
2
,
x
0
+z
0
2
_
; C
1
_
a
0
+b
0
2
,
x
0
+y
0
2
_
formeaza un triunghi cu varfurile n mijloacele laturilor triunghiului A
0
B
0
C
0
.
Continuand procedeul, obt inem un sir de triunghiuri A
n
B
n
C
n
care tind la limita catre un punct, centrul
de greutate G, al triunghiului A
0
B
0
C
0
, G
_
a
0
+b
0
+c
0
3
,
x
0
+y
0
+z
0
3
_
.
6.2.8 a) Se aplica Lema lui Cesaro-Stoltz:
lim
n
1
p
+ 2
p
+. . . +n
p
(n + 1)
p
= lim
n
1
p
+ 2
p
+. . . +n
p
+ (n + 1)
p
[1
p
+ 2
p
+. . . +n
p
]
(n + 1)
p+1
n
p+1
=
= lim
n
(n + 1)
p
(n + 1)
p+1
n
p+1
= lim
n
n
p
+C
1
p
n
p1
+C
2
p
n
p2
+. . . + 1
n
p+1
+C
1
p+1
n
p
+. . . + 1 n
p+1
=
= lim
n
n
p
+pn
p1
+. . . + 1
(p + 1)n
p
+
(p + 1)p
2
n
p1
+. . . + 1
=
1
p + 1
.
b) Se aplica Lema lui Cesaro-Stoltz:
lim
n
1
n
_
1
ln2
+
1
ln3
+. . . +
1
lnn
_
=
= lim
n
1
ln2
+
1
ln3
+. . . +
1
lnn
+
1
ln(n + 1)

_
1
ln2
+. . . +
1
lnn
_
n + 1 n
=
= lim
n
1
ln(n + 1)
1
= 0.
Altfel, avem
a
n
=
1
ln2
+
1
ln3
+. . . +
1
lnn
n
= m
aritm
_
1
ln2
; . . . ;
1
lnn
_

n 1
n
.
Dar m
aritm
(x
1
, x
2
, . . . , x
n
) x
n
pentru x
n
> 0. Va rezulta
lim
n
a
n
= lim
n
1
lnn

n 1
n
= 0 1 = 0.
256 6. Probleme de analiz a matematic a
Serii de numere reale
6.2.9 a) Deoarece lnn n, n 1 avem ca termenul general al seriei a
n
=
1
n

lnn

1
n

n
, n 1.
Deoarece lim
n
n

n = 1, rezulta divergent a seriei

n=2
a
n
(criteriul I de comparat ie).
b) Daca a < 1 Rezulta
a
n
n

n!
< a
n
. Cum seria

n=2
a
n
este convergenta rezulta ca seria

n=2
a
n
n

n!
este
convergenta.
Daca a = 1, atunci avem seria

n=2
1
n

n!
este divergenta deoarece termenul general nu tinde la zero
lim
n
n

n! = 1.
Daca a > 1, atunci
n

n!
n

n
n
= n si a
n
=
a
n
n

n!

a
n
n
, deci seria

n=2
a
n
este divergenta.
c)
1
n2
n

1
2
n
, pentru n 1 si cum seria geometrica

n=1
1
2
n
este convergena, rezulta ca si seria

n=1
1
n2
n
este convergenta.
d) Pentru n 3, lnn > 1 si deci
lnn
n
>
1
n
. Cum seria armonica

n=1
1
n
este divergenta, rezulta ca si
seria data este divergenta.
6.2.10 Seria data are aceeasi natura ca si a cea a seriei

n=2
2
n
1
(2
n
)

(ln2
n
)

n=2
1
(2
n
)
1
(nln2)

=
1
(ln2)

n=2
1
(2
1
)
n
n

n=2
a
n
.
Daca > 1 si > 0, atunci seria

n=2
a
n
este convergenta.

Intr-adevar, folosind criteriul lui DAlambert
avem
a
n+1
a
n
=
(2
1
)
n
n

(2
1
)
n+1
(n + 1)


1
2
1
< 1.
Daca > 1, < 0, = , > 0.

In acelasi mod ca mai sus seria

a
n
=
n

(2
1
)
n
este
convergenta
a
n+1
a
n
=
(2
1
)
n
(n + 1)

(2
1
)
n+1
n


1
2
1
< 1.
Daca = 1, tunci

n=2
1
(lnn)

=
1
(ln2)

1
n

este seria armonica generalizata. Daca > 1 seria


este convergenta si daca 1 seria este divergenta.
Daca < 1, atunci seria

n=2
a
n
=
1
(ln2)

n=2
(2

)
n
n

, unde = 1 , 0 < < 1 este divergenta


oricare ar R, deoarece termenul general
(2

)
n
n

nu tinde la zero.
6.2.11 a) Avem
[S
n+p
S
n
[ =

sin(n + 1)
(n + 1)(n + 2
+... +
sin(n +p)
(n +p)(n +p + 1)

1
(n + 1)(n + 2)
+
1
(n + 2)(n + 3)
+... +
1
(n +p)(n +p + 1)
=
1
n + 1

1
n +p + 1
Am obt inut ca pentru orice > 0 exista N = N(), astfel ncat
1
n + 1

1
n +p + 1
< pentru n N
si p N. Asadar (S
n
) este sir fundamental deci, n virtutea criteriului lui Cauchy este sir convergent.
Seria data este (absolut) convergenta.
6.2 Solut ii 257
b) Analog.
6.2.12 a) Aplicam criteriul raportului pentru serii. Avem
a
n+1
a
n
=
[a(n + 1)]
n+1
(n + 1)!

n!
(an)
n
=
a
n+1
(n + 1)
n
(n + 1)!
a
n
(n + 1)!n
n
= a
_
1 +
1
n
_
n
ae.
Daca ae < 1, adica a <
1
e
, atunci seria

n=1
a
n
este convergenta.
Daca ae > 1, adica a >
1
e
, atunci seria

n=1
a
n
este divergenta.
Daca a =
1
e
, atunci
a
n+1
a
n
=
_
1 +
1
n
_
n
e
. Dar
_
1 +
1
n
_
n
< e <
_
1 +
1
n
_
n+1
si deci
a
n+1
a
n
>
_
1 +
1
n
_
n
_
1 +
1
n
_
n+1
=
1
1 +
1
n
=
n
n + 1
=
1
n + 1
1
n
.
Seria

n=1
b
n
=

n=1
1
n
este divergenta si
a
n+1
a
n
>
b
n+1
b
n
, rezulta ca

n=1
a
n
este divergenta pe baza criteriului
II de comparat ie.
b) Aplicand seriei raportul
a
n+1
a
n
=
(n + 1)! e
n+1
(n + 1)
n+1+p

n
n+p
n! e
n
= e
1
_
1 +
1
n
_
n
_
1 +
1
n
_
p
1,
constatam ca criteriul lui DAlembert nu este concludent. Vom aplica criteriul lui Raabe-Duhamel. Avem
lim
n
n
_
a
n
a
n+1
1
_
= lim n
_

_
_
1 +
1
n
_
n
_
1 +
1
n
_
p
e
1
_

_
> lim
n
n
_

_
_
1 +
1
n
_
n
_
1 +
1
n
_
p
_
1 +
1
n
_
n+1
1
_

_
=
= lim
n
n
_
_
1 +
1
n
_
p1
1
_
= p 1.
Vom avea ca pentru p 1 > 1, adica p > 2, seria este convergenta.
Pentru p < 1 seria este divergenta, deoarece
lim
n
n
_
a
n
a
n+1
1
_
= lim
n
n
_

_
_
1 +
1
n
_
n
_
1 +
1
n
_
p
e
1
_

_
< lim
n
n
__
1 +
1
n
_
p
1
_
= p.
c) Deoarece lim
n
_
3n
3n + 1
_
n
= e

1
3
,= 0, seria este divergenta (nu satisface criteriul necesar de
convergent a.
d) Fie u
n
=
2
n
n!
n
n
, n 1. Avem lim
n
u
n+1
u
n
= 2 lim
n
1
_
1 +
1
n
_
n
=
2
e
< 1, deci seria este convergenta.
6.2.13 Avem
lim
n
a
n+1
a
n
=
_

_
lim
n
a
2k+1
a
2k
= lim
n
2
k
3
k
2
k1
3
k
= lim
n
2
k
2
k1

3
k
3
k
= 2 > 1;
lim
n
a
2k
a
2k1
= lim
n
2
k1
3
k

3
k1
2
k1
=
1
3
< 1.
258 6. Probleme de analiz a matematic a
Constatam ca nu se poate aplica criteriul raportului (DAlembert), deoarece raportul depinde de paritatea
sau imparitatea termenului general a
n
considerat.
Folosind criteriul radacinii obt inem
_

_
lim
n
n

a
n
= lim
n
2k1

a
2k1
= lim
n
_
2
k1
3
k1
_
1
2k 1
= lim
n
2
k 1
2k 1
3
k 1
2k 1
=
2
1
2
3
1
2
=
_
2
3
< 1
convergenta
lim
n
n

a
n
= lim
n
2k

a
2k
= lim
n
_
2
k1
3
k1
_
1
2k
= lim
n
2
k 1
2k
3
k
2k
=
2
1
2
3
1
2
=
_
2
3
< 1
convergenta

In acest caz, oricare ar paritatea termenului general, seria este convergenta.


6.2.15 a) Avem

n=1
(a
2k1
+a
2k
) =

n=1
_
1

k + 1 1

1

k + 1 + 1
_
=

n=1

k + 1 + 1

k + 1 + 1
k
= 2

n=1
1
k
care este o serie divergenta.
6.2.16 Vom studia seria

n=1
[ a
n
[. Avem

a
n+1
a
n

a(a 1) . . . (a n + 1)(a n)
(n + 1)!

n!
a(a 1) . . . (a n + 1)

a n
n + 1

1.
Criteriul DAlembert nu este concludent pentru seria valorilor absolute. Vom aplica criteriul lui Raabe-
Duhamel.
lim
n
n
_
[a
n
[
[a
n+1
[
1
_
= lim
n
n
_
n + 1 [n a[
[n a[
_
=
_

_
lim
n
n + 1 n +a
n a
= a + 1, pentru n > a
lim
n
n
n + 1 +n a
n a
= , pentru n < a
Daca a = 0, seria devine

n=1
0 care este convergenta.
Daca a < 0, seria este divergenta deoarece a + 1 < 1.
Fie a < 0, caz n care
a
n
=
a(a 1) . . . (a n + 1)
n!
= (1)
n
(a)(1 a) . . . (n a + 1)
n!
=
= (1)
n
b
n
, ()n N.
Seria a
n
=

(1)
n
b
n
este o serie alternanta. Studiem monotonia sirului (b
n
), b
n
> 0.
b
n+1
b
n
=
n 1
n + 1
< 1, n a < n + 1.
Pentru a 1, (b
n
) este crescator si lim
n
(b
n
) ,= 0. Rezulta ca pentru a 1, lim
n
a
n
,= 0 si seria
este divergenta. Pentru 1 < a < 0,
b
n+1
b
n
< 1 (b
n
) este descrescator.
Sa aratam ca lim
n
b
n
= 0. Deoarece (b
n
) este descrescator si marginit inferior, este convergent.
Notam lim
n
b
n
= l. Consideram sirul (u
n
)
n1
astfel ncat
b
n
=
u
1
+u
2
+. . . +u
n
n
iar b
n1
=
u
1
+u
2
+. . . +u
n1
n 1
.
6.2 Solut ii 259
Atunci u
n
= nb
n
(n 1)b
n1
.

Inlocuim pe b
n
si b
n1
, avem
u
n
= n
(a)(1 a) . . . (n a 1)
n!
(n 1)
(a)(1 a) . . . (n a 2)
(n 1)!
= (a) b
n1
.
Stim ca limb
n
= limu
n
= l. Am avut u
n
= ab
n1
. Trecand la limita rezulta l = al l = 0.
Pe baza criteriului lui Leibniz, seria este convergenta.
6.2.17 Avem
u
n+1
u
n
=
n + 1
n +
1 cand n . Criteriul raportului nu se aplica (nici criteriul
radicalului). Aplicam Criteriul Raabe-Duhamel. Cum
n
_
u
n
u
n+1
1
_
=
n
n + 1
( 1),
limita sa este 1.Asadar, daca > 2 seria este convergenta, iar daca < 2 seria este divergenta.
Pentru = 2, seria se reduce la

n=1
1
n + 1
, care este tocmai seria armonica (divergenta).
6.2.18 a) Fie x
n
= a
ln n
(n 1). Folosind criteriul lui Raabe-Duhamel, avem
lim
n
n
_
x
n
x
n+1
1
_
= lim
n
_
a
ln
n
n+1
1
_
n = lim
n
a
ln
n
n+1
1
ln
n
n + 1
n ln
n
n + 1
= lna lne
1
= ln
1
a
.
Daca
1
a
> e, adica ln
1
a
> 1, seria este convergenta. Daca
1
a
< e, adica ln
1
a
< 1, seria este divergenta.
b) Criteriul necesar de convergent a, lim
n
ln
_
1 +
1
n
_
= 0, este ndeplinit. Totusi seria este diver-
genta, caci sirul sumelor part iale,
S
n
=
n

k=1
ln
_
1 +
1
k
_
=
n

k=1
[ln(k + 1) lnk] = ln(n + 1) , cand n .
Funct ii continue
6.2.19 a) Pentru x
0
R k[ k Z din
lim
xx
0
f (x) = lim
xx
0
x +e
1
sin x
x
=
x
0
+e
1
sin x
0
x
0

= f (x
0
)
rezulta ca funct ia f este continua n x
0
.
Studiem continuitatea n x
0
= k, k Z, k numar par. Din
lim
xk
x<k
1
sinx
=
1
0

= , respectiv lim
xk
x>k
1
sinx
=
1
0
+
= +
rezulta ca limitele
lim
xk
x<k
f (x) = lim
xk
x<k
x +e
1
sin x
x
=
k +e

(k 1)
=
k + 0
(k 1)
=
k
k 1
,
respectiv
lim
xk
x>k
f (x) = lim
xk
x>k
x +e
1
sinx
x
=
k +e
+
(k 1)
=
_
, daca k 0, k par
+, daca k 2, k par.
sunt diferite, deducem ca lim
xk
f (x) nu exista si prin urmare funct ia f nu este continua n x
0
= k, unde
k este numar par.
260 6. Probleme de analiz a matematic a
Studiem continuitatea n x
0
= k, k Z, k numar impar. Avem
lim
xk
x<k
1
sinx
=
1
0
+
= + si lim
xk
x>k
1
sinx
=
1
0

= .
Pentru k ,= 1, k impar limitele laterale
lim
xk
x<k
f (x) = lim
xk
x<k
x +e
1
sin x
x
=
k +e
+
(k 1)
=
_
, daca k 1, k impar
+, daca k 3, k impar
,
respectiv
lim
xk
x>k
f (x) = lim
xk
x>k
x +e
1
sin x
x
=
k +e

(k 1)
=
k + 0
(k 1)
=
k
k 1
sunt diferite. Atunci lim
xk
f (x) nu exista, prin urmare funct ia f nu este continua n x
0
= k, unde k este
numar impar diferit de 1.
Pentru k = 1 avem x
0
= k = , limitele laterale
lim
x
x<
f (x) = lim
x
x<
x +e
1
sin x
x
=
+e
+
0

= ,
respectiv
lim
x
x>
f (x) = lim
x
x>
x +e
1
sin x
x
=
+e

0
+
=
+ 0
0
+
=

0
+
= +
sunt diferite. Atunci lim
x
f (x) nu exista, prin urmare funct ia f nu este continua n x
0
= .

In concluzie funct ia f este continua n ecare x R k[ k Z , si este discontinua n ecare


x = k, k Z. Ment ionam ca toate punctele de discontinuitate ale funct iei f , x = k, k Z, au spet a
a doua.
b) Funct ia f se obt ine din funct ii continue prin compunere si nmult ire, prin urmare funct ia f este
continua pe mult imea de denit ie [1, +) .
c) Consideram x
0
R si (u
n
)
n1
un sir de numere rat ionale, (v
n
)
n1
un sir de numere irat ionale
ambele convergente catre x
0
. Atunci
lim
n
f (u
n
) = lim
n
(7u
n
12) = 7x
0
12, iar lim
n
f (v
n
) = lim
n
v
2
n
= x
2
0
.
Egalitatea 7x
0
12 = x
2
0
are loc pentru x
0
3, 4 .
Pentru x
0
R 3, 4 sirurile (f (u
n
)), (f (v
n
)) converg catre limite diferite; deducem ca f nu are
limita n punctele x
0
R 3, 4 , si deci funct ia f nu este continua n punctele x
0
R 3, 4 . Aceste
puncte de discontinuitate au spet a a doua.
Funct ia f este continua n punctul x
0
= 3 si n punctul x
0
= 4. Demonstram continuitatea funct iei n
x
0
= 3. Este sucient sa aratam ca pentru ecare vecinatate V a numarului f (3) = 9 exista o vecinatate
U a lui x
0
= 3 astfel ncat f (U) V.
Consideram funct iile f
1
: R R, f
1
(x) = 7x
2
12, f
2
: R R, f
2
(x) = x
2
. Funct iile f
1
si f
2
sunt
continue pe R. Fie V o vecinatate a numarului f (3) = 9. Deoarece V este vecinatate pentru f
1
(3) = 9
si funct ia f
1
este continua n x
0
= 3, exista o vecinatate U
1
a numarului x
0
= 3 astfel ncat f
1
(U
1
) V.
Deoarece V este vecinatate pentru f
2
(3) = 9 si funct ia f
2
este continua n x
0
= 3, exista o vecinatate
U
2
a numarului x
0
= 3 astfel ncat f
2
(U
2
) V. Privind vecinatatea U = U
1
U
2
a lui x
0
= 3 are loc
U U
1
, U U
2
, deducem f
1
(U) V, f
2
(U) V, de unde rezulta f (U) V.
Pentru punctul x
0
= 4 se procedeaza n mod asemanator.
6.2.20 a) Explicitam
_
x
2

.

In acest scop partit ionam domeniul de denit ie astfel
R =
kN
__

k + 1,

k] [

k,

k + 1
__
6.2 Solut ii 261
Pentru x (

k + 1,

k][

k,

k + 1), unde k N, din x


2
[k, k+1),
_
x
2

= k rezulta f (x) = x
2
k,
si deci f este continua pe mult imea (

k + 1,

k) (

k,

k + 1).
Studiem continuitatea funct iei f n punctele x
0
=

k unde k N

respectiv x
0
=

k unde k N.
Avem f(

k) = k k = 0, f(

k) = k k = 0.
Deoarece limitele laterale
lim
x

k
x<

k
f (x) = lim
x

k
x<

k
_
x
2
k
_
= k k = 0, lim
x

k
x>

k
f (x) = lim
x

k
x>

k
_
x
2
(k 1
_
) = k (k 1) = 1
sunt diferite, funct ia f nu are limita n punctul x
0
=

k, si deci nu este continua n acest punct.


Deoarece limitele laterale
lim
x

k
x<

k
f (x) = lim
x

k
x<

k
_
x
2
(k 1
_
) = k (k 1) = 1, lim
x

k
x>

k
f (x) = lim
x

k
x>

k
(x
2
k) = k k = 0
sunt diferite, funct ia f nu are limita n punctul x
0
=

k, si deci nu este continua n acest punct.

In concluzie mult imea de continuitate a funct iei f este


D
0
=
kN
_
(

k + 1,

k) (

k,

k + 1)
_
.
Ment ionam ca toate punctele de discontinuitate ale funct iei f, x =

k, unde k N, au spet a ntai.


b) Explicitam [x] .

In acest scop partit ionam domeniul de denit ie astfel R =
kZ
[k, k + 1) .
Fie k Z arbitrar xat. Obt inem f (x) = k sinx, x [k, k + 1) . Asadar f este continua pe
intervalul (k, k + 1) . Studiem continuitatea funct iei f n punctul x
0
= k. Evident f (k) = k sink =
k 0 = 0. T inand cont de
f (x) =
_
(k 1) sinx, x [k 1, k)
k sinx, x [k, k + 1)
obt inem
lim
xk
x<k
f (x) = lim
xk
x<k
(k 1) sinx = (k 1) sink = (k 1) 0 = 0,
lim
xk
x>k
f (x) = lim
xk
x>k
k sinx = k sink = k 0 = 0
si deci lim
xk
f (x) = 0. Din egalitatea lim
xk
f (x) = f (k) rezulta ca f este continua n x
0
= k.

In concluzie funct ia f este continua pe R.


c) Pentru x (0, 1) avem: [x] = 0;
1
x
> 1 implica
_
1
x
_
1;
_
1
x
_
[x]
=
_
1
x
_
0
= 1; obt inem
f (x) = 1 +0 = 1, iar f (1) =
_
1
1
_
[1]
+[1] = 1
1
+1 = 2. Pentru x (1, +) avem: [x] 1;
1
x
(0, 1) deci
_
1
x
_
= 0;
_
1
x
_
[x]
= 0
[x]
= 0; obt inem f (x) = 0 + [x] = [x] .
Prin urmare f (x) =
_
_
_
1, daca x (, 1)
2, daca x = 1
[x] , daca x (1, )
=
_

_
1, daca x (, 1)
2, daca x = 1
1, daca x (1, 2)
k, daca x [k, k + 1) , k 2, 3, 4, . . .
Din faptul ca funct ia f este constanta pe intervalul (, 1) si pe ecare interval (k, k + 1) unde
k N

, deducem continuitatea funct iei f n punctele x (, 1) si n punctele x (k, k + 1) unde


k N

. Din egalitatea limitelor laterale lim


x1
x<1
f (x) = lim
x1
x<1
1 = 1, lim
x1
x>1
f (x) = lim
x1
x>1
1 = 1 rezulta lim
x1
f (x) = 1.
Dar f (1) = 2. Din lim
x1
f (x) ,= f (1) rezulta ca funct ia f nu este continua n punctul x
0
= 1. Pentru
x
0
= k 2, 3, 4, . . . , din faptul ca limitele laterale lim
xk
x<k
f (x) = lim
xk
x<k
(k 1) = k1, lim
xk
x>k
f (x) = lim
xk
x>k
k = k
262 6. Probleme de analiz a matematic a
sunt diferite deducem ca limita lim
xk
f (x) nu exista. Prin urmare funct ia f nu este continua n punctul
x
0
= k unde k 2, 3, 4, . . . . Toate punctele de discontinuitate ale funct iei f au spet a ntai.
Mult imea de continuitate a funct iei f este (, 1)
_

kN

(k, k + 1)
_
.
6.2.21 Calculam limita data.
Pentru x < 0 din lim
n
e
nx
= 0 rezulta
f (x) = lim
n
[sinx[ e
nx
+ cos x
e
nx
+ 1
=
[sinx[ 0 + cos x
0 + 1
= cos x.
Pentru x = 0 avem
f (0) = lim
n
[sin0[ e
n0
+ cos 0
e
n0
+ 1
=
0 e
0
+ 1
e
0
+ 1
=
1
2
.
Pentru x > 0 din lim
n
e
nx
= + rezulta
f (x) = lim
n
[sinx[ e
nx
+ cos x
e
nx
+ 1
= lim
n
e
nx
_
[sinx[ +
1
e
nx
cos x
_
e
nx
_
1 +
1
e
nx
_ =
[sinx[ + 0 cos x
1 + 0
= [sinx[ .
Asadar
f (x) =
_

_
cos x, daca x < 0
1
2
, daca x = 0
[sinx[ , daca x > 0.
Restrict ia funct iei f la mult imea (, 0) este cos : (, 0) R. Stiind ca funct ia cos este continua
pe mult imea R rezulta ca f este continua pe (, 0) . Restrict ia funct iei f la mult imea (0, +) este
[sin[ : (0, +) R. Stiind ca funct ia sin este continua pe mult imea R deducem ca funct ia [sin[ este
continua pe R si rezulta ca f este continu a pe (0, +) .
Studiem continuitatea funct iei f n punctul x
0
= 0. Deoarece limitele laterale
lim
x0
x<0
f (x) = lim
x0
x<0
cos x = cos 0 = 1, si lim
x0
x>0
f (x) = lim
x0
x>0
[sinx[ = [sin0[ = 0
sunt diferite, rezulta ca funct ia f nu are limita n punctul x
0
= 0. Prin urmare funct ia f nu este continua
n punctul x
0
= 0.

In concluzie funct ia f este continua n ecare punct x (, 0) (0, +) si este discontinua n


x = 0.
b) Calculam limita data.

In acest scop consideram sirul de numere reale a
n
= 1 + x
2n
si studiem
limita
lim
n
a
n+1
a
n
= lim
n
1 +x
2(n+1)
1 +x
2n
= lim
n
1 +x
2

_
x
2
_
n
1 + (x
2
)
n
.
Pentru x (, 1) (1, +) din x
2
(1, +) , lim
n
_
x
2
_
n
= + deducem
lim
n
a
n+1
a
n
= lim
n
1 +x
2

_
x
2
_
n
1 + (x
2
)
n
= lim
n
_
x
2
_
n
_
1
(x
2
)
n
+x
2
_
(x
2
)
n
_
1
(x
2
)
n
+ 1
_ = lim
n
1
(x
2
)
n
+x
2
1
(x
2
)
n
+ 1
=
0 +x
2
0 + 1
= x
2
.
Pentru x 1, 1 cum x
2n
= 1 avem
lim
n
a
n+1
a
n
= lim
n
1 +x
2(n+1)
1 +x
2n
= lim
n
1 + 1
1 + 1
= 1.
6.2 Solut ii 263
Pentru x (1, 1) din lim
n
x
2n
= 0 deducem
lim
n
a
n+1
a
n
= lim
n
1 +x
2(n+1)
1 +x
2n
=
1 + 0
1 + 0
= 1.
Deoarece
lim
n
n
_
1 +x
2n
= lim
n
n

a
n
= lim
n
a
n+1
a
n
,
funct ia f este
f (x) =
_
x
2
, daca x (, 1) (1, +)
1, daca x [1, 1] .
Deoarece f (x) = x
2
, x (, 1) (1, +) rezulta ca funct ia f este continua n ecare
x (, 1) (1, +) . Deoarece f (x) = 1, x (1, 1) rezulta ca funct ia f este continua n
ecare x (1, 1) .

In punctul x
0
= 1, egalitatea limitelor laterale
lim
x1
x<1
f (x) = lim
x1
x<1
x
2
= 1, lim
x1
x>1
f (x) = lim
x1
x>1
1 = 1
implica lim
x1
f (x) = 1. Din egalitatea lim
x1
f (x) = f (1) rezulta ca funct ia f este continua n punctul
x
0
= 1.

In punctul x
0
= 1, egalitatea limitelor laterale
lim
x1
x<1
f (x) = lim
x1
x<1
1 = 1, lim
x1
x>1
f (x) = lim
x1
x>1
x
2
= 1
implica lim
x1
f (x) = 1. Din egalitatea lim
x1
f (x) = f (1) rezulta ca funct ia f este continuan punctul x
0
= 1.

In concluzie funct ia f este continua pe R.


c) Pentru x (0, 1) avem: lnx < 0, deci
lim
n
n
ln x
= 0, f (x) = lim
n
ax
2
n
ln x
+x
xn
ln x
+ 1
=
ax
2
0 +x
x 0 + 1
= x.
Apoi
f (1) = lim
n
a 1
2
n
ln 1
+ 1
1 n
ln 1
+ 1
= lim
n
a n
0
+ 1
n
0
+ 1
= lim
n
a 1 + 1
1 + 1
=
a + 1
2
.
Pentru x (1, +) avem: lnx > 0, lim
n
n
ln x
= +, deci
f (x) = lim
n
ax
2
n
ln x
+x
xn
ln x
+ 1
= lim
n
n
ln x
_
ax
2
+x
1
n
ln x
_
n
ln x
_
x +
1
n
ln x
_ = lim
n
ax
2
+x
1
n
ln x
x +
1
n
ln x
=
ax
2
+x 0
x + 0
= ax.
Asadar
f (x) =
_

_
x, daca x (0, 1)
a + 1
2
, daca x = 1
ax, daca x (1, +) .
Funct ia f este continua pe mult imea (0, 1) si pe mult imea (1, +) . Limitele laterale
lim
x1
x<1
f (x) = lim
x1
x<1
x = 1; lim
x1
x>1
f (x) = lim
x1
x>1
ax = a 1 = a
sunt egale pentru a = 1 si sunt diferite pentru a R 1 . Pentru a = 1 din egalitatea limitelor laterale
rezulta lim
x1
f (x) = 1 si comparand cu f (1) =
1 + 1
2
= 1 din egalitatea lim
x1
f (x) = f (1) deducem ca f
264 6. Probleme de analiz a matematic a
este continua n x
0
= 1. Pentru a R 1 n punctul x
0
= 1 funct ia f are limite laterale diferite, rezulta
ca funct ia f nu are limita n punctul x
0
= 1 si deci nu este continua n acest punct.

In concluzie, pentru a = 1 funct ia f este continua pe (0, +) ; iar pentru a R 1 funct ia f este
continua pe (0, 1) (1, +) si este discontinua n punctul x
0
= 1.
6.2.22 a) Avem
lim
x0
f (x) = lim
x0

1 +x 1
3

1 +x 1
= lim
x0
_
1 +x 1
_ _
1 +x + 1
_

_
3
_
(1 +x)
2
+
3

1 +x + 1
_
_
3

1 +x 1
_
_
3
_
(1 +x)
2
+
3

1 +x + 1
_

_
1 +x + 1
_
= lim
x0
x
_
3
_
(1 +x)
2
+
3

1 +x + 1
_
x
_
1 +x + 1
_
= lim
x0
3
_
(1 +x)
2
+
3

1 +x + 1
_
1 +x + 1
_ =
3
2
.
Denim f prelungirea funct iei f la punctul x
0
= 0, f : [1, +) R,
f (x) =
_
f (x) , daca x [1, 0) (0, +)
3
2
, daca x = 0.
Se arata usor ca funct ia f, si la fel f, sunt continue pe mult imea [1, 0) (0, +) . Deoarece lim
x0
f (x)
= lim
x0
f (x) =
3
2
si f (0) =
3
2
, din lim
x0
f (x) = f (0) rezulta ca funct ia f este continua n punctul x
0
= 0.
Asadar funct ia f poate prelungita prin continuitate la punctul x
0
= 0.
b) Din studiul limitelor laterale
lim
x
x<0
f (x) = lim
x0
x<0
2tgxarctg
1
x
= 2 0

2
= 0, lim
x0
x>0
f (x) = lim
x0
x>0
2

1+x1
x
2
e
x
= 2

= 0
rezulta ca lim
x0
f (x) = 0. Denim f prelungirea funct iei f la punctul x
0
= 0, f : [1, +) R,
f (x) =
_
f (x) , daca x [1, 0) (0, +)
0, daca x = 0.
.
Se arata usor ca funct ia f, si la fel f, sunt continue pe mult imea [1, 0) (0, +) . Deoarece lim
x0
f (x)
= lim
x0
f (x) = 0 si f (0) = 0, din lim
x0
f (x) = f (0) rezulta ca funct ia f este continua n punctul x
0
= 0.
Asadar funct ia f poate prelungita prin continuitate la punctul x
0
= 0.
c) Avemlim
x0
x>0
f (x) = lim
x0
x>0
lnx = . Fie f o prelungire a funct iei f la punctul x
0
= 0, adica
f : [0, +) R, f (x) =
_
f (x) = lnx, daca x (0, +)
y
0
, daca x = 0,
unde y
0
R este xat arbitrar. Avemlim
x0
x>0
f (x) = lim
x0
x>0
lnx = . Pentru orice alegere a lui f (0) = y
0
R
are loc lim
x0
x>0
f (x) ,= f (0) si deci f este discontinuan x
0
= 0. Toate prelungirile funct iei f la punctul x
0
= 0
ind discontinue n punctul x
0
= 0, funct ia f nu poate prelungita prin continuitate la punctul x
0
= 0.
d) Fie f o prelungire a funct iei f la punctul x
0
= 0, adica f : R R,
f (x) =
_
f (x) = sin
2
x
, daca x R 0
y
0
, daca x = 0
,
6.2 Solut ii 265
unde y
0
R este xat arbitrar.
Consideram sirurile de numere reale (u
n
)
n1
, (v
n
)
n1
convergente catre 0, u
n
=
4
1 + 4n
,
v
n
=
4
1 + 4n
, pentru n N. Deoarece limitele
lim
n
f (u
n
) = lim
n
f
_
4
1 + 4n
_
= lim
n
sin
2 (1 + 4n)
4
= lim
n
sin
_

2
+ 2n
_
= 1
si
lim
n
f (v
n
) = lim
n
f
_
4
1 + 4n
_
= lim
n
sin
2 (1 + 4n)
4
= lim
n
sin
_

2
+ 2n
_
= 1
sunt diferite, deducem ca funct ia f nu are limita n punctul x
0
= 0 si deci funct ia f este discontinua n
x
0
= 0, pentru orice alegere a lui f (0) = y
0
R. Asadar funct ia f nu admite prelungire prin continuitate
la punctul x
0
= 0.
6.2.23 a) Pe intervalul [1, 2) funct ia f este rezultatul compunerii unor funct ii continue. La fel pe
intervalul (2, +) . Prin urmare funct ia f este continua pe [1, 2) si pe (2, +) . Avem limitele laterale
lim
x2
x<2
f (x) = lim
x2
x<2
_
a
x
+ 4a
x
2
+x
2
= lim
x2
x<2
_
a
2
+ 4a
2
2
+ 2
2
=
_
(a + 2)
2
= [a + 2[ = a + 2;
lim
x2
x>2
f (x) = lim
x2
x>2
ln(x 1)
4 sin(x2)
(x 2)
2
= lim
x2
x>2
4 sin(x 2) ln(x 1)
(x 2)
2
= lim
x2
x>2
4
sin(x 2)
x 2

ln(1 + (x 2))
x 2
= 4.
Daca a (0, +) 2, atunci a + 2 ,= 4, rezulta ca lim
x2
x<2
f (x) ,= lim
x2
x>2
f (x) si deci funct ia f nu are
limita n punctul x
0
= 2; deducem ca funct ia f nu este continua n punctul x
0
= 2. Daca a = 2, atunci
lim
x2
x<2
f (x) = lim
x2
x>2
f (x) = 4 implica lim
x2
f (x) = 4 si comparand cu f (2) = 4, din egalitatea lim
x2
f (x) = f (2)
deducem ca f este continua n x
0
= 2.

In concluzie funct ia f cu a > 0 este continua pe mult imea de denit ie daca si numai daca a = 2.
b) Pentru x < 0 avem
x
[x[
=
x
x
= 1, iar pentru x > 0 avem
x
[x[
=
x
x
= 1. Asadar
f (x) =
_

_
_
x 1
x + 1
_
2
, daca x (, 1)
a sin
_

2
(1 x)
2
_
, daca x [1, 0)
b, daca x = 0
a sin
_

2
(1 x)
2
_
, daca x (0, 1]
_
x 1
x + 1
_
2
, daca x (1, +) .
Pe ecare interval (, 1) , (1, 0) , (0, 1) , (1, +) funct ia f este rezultat al compunerii unor funct ii
continue, prin urmare funct ia f este continua pe ecare interval ment ionat.
Din
lim
x1
x<1
f (x) = lim
x1
x<1
_
x 1
x + 1
_
2
= lim
x1
x<1
_
x + 1
x 1
_
2
= 0
si
lim
x1
x>1
f (x) = lim
x1
x>1
a sin
_

2
(1 x)
2
_
= a sin2 = 0,
rezulta lim
x1
f (x) = 0; comparam cu f (1) = a sin2 = 0; gasim lim
x1
f (x) = f (1) si deducem ca f
este continua n x
0
= 1. Concluzia ind independenta de valoarea lui a R.
266 6. Probleme de analiz a matematic a
Din
lim
x1
x<1
f (x) = lim
x1
x<1
a sin
_

2
(1 x)
2
_
= a sin0 = 0 si lim
x1
x>1
f (x) = lim
x1
x>1
_
x 1
x + 1
_
2
= 0,
rezulta lim
x1
f (x) = 0; comparam cu f (1) = a sin0 = 0; gasim lim
x1
f (x) = f (1) si deducem ca f este
continua n x
0
= 1. Concluzia ind independenta de valoarea lui a R.
Din
lim
x0
x<0
f (x) = lim
x0
x<0
a sin
_

2
(1 x)
2
_
= a sin

2
= a si lim
x0
x>0
f (x) = lim
x0
x>0
a sin
_

2
(1 x)
2
_
= a sin

2
= a,
cum f (0) = b, deducem ca funct ia f este continua n punctul x
0
= 0 daca si numai daca a = a = b
ceea ce revine la a = b = 0.
Asadar funct ia f este continua pe mult imea de denit ie daca si numai daca a = b = 0.
6.2.24 Consideram funct iile g, h : R R, g (x) = f (x) + c, x R, respectiv h(x) = f (x) c,
x R. Demonstram egalitatea
f
c
(x) =
[g (x)[ [h(x)[
2
, x R.
Daca f (x) < c, atunci f (x) +c < 0, f (x) c < 2c < 0 si prin explicitarea modulelor obt inem
[g (x)[ [h(x)[
2
=
[f (x) +c[ [f (x) c[
2
=
(f (x) c) (f (x) +c)
2
= c = f
c
(x) .
Daca [f (x)[ c, atunci c f (x) c si deci f (x) + c 0, f (x) c 0. Prin explicitarea modulelor
obt inem
[g (x)[ [h(x)[
2
=
[f (x) +c[ [f (x) c[
2
=
(f (x) +c) (f (x) +c)
2
= f (x) = f
c
(x) .
Daca f (x) > c, atunci f (x) +c > 2c > 0, f (x) c > 0 si prin explicitarea modulelor obt inem
[g (x)[ [h(x)[
2
=
[f (x) +c[ [f (x) c[
2
=
(f (x) +c) (f (x) c)
2
= c = f
c
(x) .
Din continuitatea funct iei f pe R rezulta continuitatea funct iilor g si h pe R. Deoarece funct ia modul
[[ : R R pastreaza continuitatea, deducem ca [g[ si [h[ sunt continue pe R. Cum operat iile aritmetice
pastreaza continuitatea, deducem ca funct ia
[g[ [h[
2
= f
c
este continua pe R.
Din [f
c
(x)[ c, x R adica f
c
(x) [c, c] , x R deducem ca funct ia f
c
este marginita.
6.2.25 Utilizam teorema de existent a a limitelor laterale n cazul funct iilor monotone. Consideram
f : [a, b] R crescatoare. Pentru f descrescatoare se va rat iona n mod asemanator. Presupunem ca f
nu este continua pe [a, b] . Atunci exista x
0
[a, b] punct de discontinuitate pentru f.
Daca x
0
(a, b) , atunci pe baza monotoniei funct iei f are loc f (a) f (x
0
0) < f (x
0
+ 0) f (b) .
Din a x < x
0
f (a) f(x) f(x
0
0) si x
0
< x b f (x
0
+ 0) f(x) f (b) deducem ca
pentru y (f(x
0
0), f(x
0
+0)), y ,= f (x
0
) nu exista x [a, b] astfel ncat y = f(x), cu toate ca valoarea
y este cuprinsa ntre f(a) si f(b).
Daca x
0
= a, atunci pe baza monotoniei funct iei f are loc f (a) < f (a + 0) f(b). Deoarece
a < x b f(a + 0) f(x) f(b) deducem ca pentru y (f (a) , f (a + 0)) nu exista x [a, b] astfel
ncat y = f(x), cu toate ca valoarea y este cuprinsa ntre f(a) si f(b).
Daca x
0
= b, atunci pe baza monotoniei funct iei f are loc f (a) f (b 0) < f(b). Deoarece
a x < b f(a) f(x) f(b 0) deducem ca pentru y (f (b 0) , f (b)) nu exista x [a, b] astfel
ncat y = f(x), cu toate ca valoarea y este cuprinsa ntre f(a) si f(b).
Presupunerea noastra duce la contradict ie, deci este incorecta. Asadar f este continua pe [a, b] .
6.2.26 Avem f (a) = 0, f (b) = sin
1
b a
. Notam cu B mult imea numerelor cuprinse ntre 0 si f (b) .
Avem de aratat ca pentru orice y B exista x [a, b] astfel ca y = f (x) .
Evident 0, f (b) [1, 1] si atunci B [1, 1] .
Din y B rezulta y [1, 1] , asadar ecuat ia sin
1
x a
= y, x R are solut ii.
6.2 Solut ii 267
1
x a
= (1)
k
arcsiny +k, k Z
x a =
1
(1)
k
arcsiny +k
, k Z
x = a +
1
(1)
k
arcsiny +k
, k Z
Determinam o valoare k
0
Z pentru care x
k
0
= a +
1
(1)
k
0
arcsiny +k
0

[a, b] . Deoarece arcsiny


_

2
,

2
_
oricare ar y [1, 1] , pentru k
0
numar natural nenul par avem
0 <

2
+k
0
(1)
k
0
arcsiny +k
0


2
+k
0
,
1

2
+k
0

1
(1)
k
0
arcsiny +k
0

2
+k
0

,
a < a +
1

2
+k
0

a +
1
(1)
k
0
arcsiny +k
a +
1

2
+k
0

Inegalitatea a +
1

2
+k
0

b are loc pentru k


0

1
2
+
1
(b a)
. Deci pentru orice y B exista
x [a, b] astfel ncat y = sin
1
x a
, de exemplu x = a +
1
arcsiny +k
0

, unde k
0
este un numar natural
par cu proprietatea k
0

1
2
+
1
(b a)
. Restrict ia funct iei f la intervalul (a, b] este compunere de funct ii
continue, deci funct ia f este continua pe mult imea (a, b] . Studiem funct ia n punctul x
0
= a. Consideram
sirurile de numere reale (u
n
)
n1
, (v
n
)
n1
astfel
u
n
= a +
1

4
+ 2n
si v
n
= a +
1

3
+ 2n
, daca n
1
8
+
1
2 (b a)
;
u
n
= v
n
= a, daca n <
1
8
+
1
2 (b a)
.
Ele sunt convergente catre a si are loc u
n
, v
n
[a, b] , n N. Cum
lim
n
f (u
n
) = sin

4
=

2
2
,= lim
n
f (v
n
) = sin

3
=

3
2
,
rezulta ca limita lim
xa
x>a
f (x) nu exista, si prin urmare f nu este continua n punctul x
0
= a. Din acest motiv
funct ia f nu este continua pe [a, b] .
6.2.27 Fie x
0
I arbitrar. Aratam ca funct ia f este continua n x
0
.
Fie > 0 arbitrar. Denim () =

L
. Din > 0, L > 0 rezulta () > 0. Pentru x I cu
proprietatea [x x
0
[ < () =

L
aplicand inegalitatea din enunt ul problemei luand x
1
= x, x
2
= x
0
obt inem [f (x) f (x
0
)[ L [x x
0
[ si cu inegalitatea [x x
0
[ <

L
deducem
[f (x) f (x
0
)[ L [x x
0
[ < L

L
= .
Deci pentru > 0 arbitrar exista () > 0 (si anume () =

L
) astfel ncat x I, [x x
0
[ < ()
[f (x) f (x
0
)[ < , ceea ce nseamna ca funct ia f este continua n punctul x
0
.
Cum x
0
I a fost arbitrar, deducem ca f este continua pe I.
268 6. Probleme de analiz a matematic a
Alta abordare. Pentru ecare x
0
I din [f (x) f (x
0
)[ L [x x
0
[ , x I si lim
xx
0
L [x x
0
[ = 0,
utilizand criteriul majorarii, rezulta ca lim
xx
0
f (x) = f (x
0
) . Aceasta egalitate ne spune ca funct ia f este
continua n x
0
.
Alta abordare. Pentru > 0 denim () =

L
> 0. Pentru orice x, u I, [x u[ < () , utilizand
inegalitatea din enunt ul problemei, rezult a ca [f (x) f (u)[ L[x u[ < L () = L

L
= . Asadar f
este uniform continua pe I, si deci este continua pe I.
6.2.28 a) Fie p N

numar dat. Fix am p numere reale distincte a


1
< a
2
< . . . < a
p
si consideram
funct ia f : R R,
f (x) =
_
_
_
sin
1
x a
1
+ sin
1
x a
2
+. . . + sin
1
x a
p
, daca x R a
1
, a
2
, . . . , a
p

0, daca x a
1
, a
2
, . . . , a
p
.
Funct ia f este continua pe mult imea (, a
1
) (a
1
, a
2
) (a
2
, a
3
) . . . (a
p1
, a
p
) (a
p
, +) . Funct ia
f nu are limita n punctele a
1
, a
2
, . . . , a
p
si deci este discontinua n aceste puncte. Mult imea punctelor
de discontinuitate a funct iei f este mult imea a
1
, a
2
, . . . , a
p
, nita, avand p elemente.
Din sint [1, 1] , t R deducem f (x) [p, p] , x R, deci funct ia f este marginita.
b) Consideram funct ia f : R R, f (x) = x [x] , unde [] este funct ia partea ntreaga. Pentru
ecare k Z avem f (x) = x k, x [k, k + 1); rezulta ca funct ia f este continua pe ecare interval
(k, k + 1) si deci pe mult imea
kZ
(k, k + 1) . Deoarece limitele laterale lim
xk
x<k
f (x) = lim
xk
x<k
(k 1) = k 1,
lim
xk
x>k
f (x) = lim
xk
x>k
k = k sunt diferite, deducem ca funct ia f nu are limita n punctele k Z, si deci
este discontinua n aceste puncte. Mult imea punctelor de discontinuitate a funct iei f este mult imea Z,
numarabila.
Ment ionam ca x [x] = x este partea fract ionara a numarului real x si x [0, 1), x R, deci
f (x) [0, 1), x R de unde rezulta ca funct ia f este marginita.
c) Fixam a
1
, a
2
R, a
1
< a
2
. Consideram funct ia
f : R R, f (x) =
_
a
1
, daca x
a
2
, daca x R .
Consideram x
0
R arbitrar. Aratam ca f este discontinua n x
0
. Fie (u
n
)
n1
un sir de numere rat ionale
si (v
n
)
n1
un sir de numere irat ionale, ambele convergente catre x
0
. Atunci f (u
n
) = a
1
, n N, f (v
n
) =
a
2
, n N, lim
n
f (u
n
) = lim
n
a
1
= a
1
, lim
n
f (v
n
) = lim
n
a
2
= a
2
. Cum sirurile (f (u
n
))
n1
, (f (v
n
))
n1
converg la limite diferite, deducem ca funct ia f nu are limitan punctul x
0
si prin urmare nu este continua
n x
0
. Mult imea punctelor de discontinuitate a funct iei f este mult imea R, mult ime nenumarabila.
Din f (x) [a
1
, a
2
] , x R rezulta c a funct ia f este marginita.
Funct ii derivabile
6.2.29 a) Pe intervalul (0, +) funct ia f este continua, deoarece este rezultatul compunerii unor
funct ii continue pe (0, +) .
Explicitam modulul. Pentru x (0, e) avem lnx < 1, lnx 1 < 0, [lnx 1[ = lnx + 1. Pentru
x [e, +) avem lnx 1, lnx 1 0, [lnx 1[ = lnx 1. Asadar
f (x) =
_
_
_
0, daca x = 0
xlnx +x, daca x (0, e)
xlnx x, daca x [e, +) .
Studiem continuitatea funct iei n punctul x
0
= 0. Avem lim
x0
x>0
f (x) = lim
x0
x>0
(xlnx +x) = 0. Egalitatea
lim
x0
x>0
f (x) = f (0) ne spune ca funct ia f este continua (la dreapta) n punctul x
0
= 0. Asadar, funct ia f
este continua pe mult imea de denit ie [0, +).
6.2 Solut ii 269
Studiem derivabilitatea funct iei. Aplicand regulile de derivare obisnuite, dar numai n acele puncte
interioare ale mult imii de denit ie care nu sunt puncte de ramicare, obt inem
f
t
(x) =
_
lnx, daca x (0, e)
lnx, daca x (e, +)
.

In punctul x
0
= 0 avem
f
t
d
(e) = lim
x0
x>0
f (x) f (0)
x 0
= lim
x0
x>0
(xlnx +x) 0
x
= lim
x0
x>0
(1 lnx) = 1 () = + / R,
deci f nu este derivabila (la dreapta) n punctul x
0
= 0.

In punctul x
0
= e derivatele laterale f
t
s
(e) = 1, f
t
d
(e) = 1 sunt diferite, din acest motiv funct ia f
nu este derivabila n acest punct.
Mult imea de derivabilitate a funct iei f este (0, e) (e, +). Ment ionam ca punctul (e, f (e)) = (e, 0)
este punct unghiular al gracului funct iei f.
b) Notam cu g restrict ia funct iei f la intervalul (e, +), cu domeniul valorilor redus la (0, +),
corespunzator valorilor realizate de f, pozitive datorita modulului. Deci g : (e, +) (0, +),
g (x) = [xlnx x[ = xlnx x.
Din g
t
(x) = lnx > 0, x (e, +) obt inem ca g este strict monotona (crescatoare) pe intervalul (e, +).
Deci g este injectiva. Funct ia g ind continua, transforma orice interval n interval; ind strict crescatoare
pe (e, +) cu g (e) = 0 si lim
x+
g (x) = +, deducem ca g ((e, +)) = (0, +), egalitate ce ne spune
ca funct ia g este surjectiva.
Funct ia g ind injectiva si surjectiva rezulta ca este bijectiva. Funct ia g ind bijectiva, este si
inversabila. Notam cu g
1
inversa lui g, g
1
: (0, +) (e, +).
Deoarece g este continua si bijectiva, din g
t
(x) = lnx ,= 0 pentru x (e, +) deducem ca funct ia
inversa g
1
este derivabila n ecare punct y = g (x) cu x (e, +), adica n ecare punct y (0, +),
si
_
g
1
_
t
(y) =
1
g
t
(x)
=
1
g
t
((g
1
) (y))
=
1
ln((g
1
) (y))
, y (0, +).
6.2.30 Pe mult imea (, 0) (0, +) funct ia f se obt ine prin compunerea unor funct ii ori de cate
ori derivabile pe aceasta mult ime, prin urmare f este ori de cate ori derivabila pe (, 0) (0, +) .
Pentru x (, 0) (0, +) si i N

avem
f
t
(x) = nx
n1
sin
1
x
x
n2
cos
1
x
f
tt
(x) = n(n 1) x
n2
sin
1
x
2 (n 1) x
n3
cos
1
x
x
n4
sin
1
x

f
(i)
(x) = n(n 1) (n 2) . . . (n i + 1) x
ni
sin
1
x
. . . x
n2i

_
cos
1
x
, daca i impar
sin
1
x
, daca i par
Privind punctul x
0
= 0: deoarece lim
x0
x
n
= 0 si sin
1
x
[1, 1] este funct ie marginita avem
lim
x0
_
x
n
sin
1
x
_
= 0; cum lim
x0
f (x) = f (0) deducem ca n punctul x
0
= 0 funct ia f este continua.
Daca n 2i 1, atunci lim
x0
x
n2i
= 0, lim
x0
_
x
n2i
sin
1
x
_
= 0 si deducem lim
x0
f
(i)
(x) = 0, unde
i
_
0, 1, 2, . . . ,
_
n1
2
_
.
Avand f
(0)
continua n x
0
= 0, unde f
(0)
= f, rat ionam inductiv pentru k
_
1, 2, . . . ,
_
n1
2
_
:
presupunand f
(k1)
continua n x
0
= 0; din existent a limitei lim
x0
_
f
(k1)
(x)
_
t
= lim
x0
f
(k)
(x) = 0, pe
baza consecint ei teoremei lui Lagrange rezulta ca f
(k1)
are derivata n x
0
= 0 si anume
_
f
(k1)
_
t
(0) = lim
x0
_
f
(k1)
(x)
_
t
= 0,
ceea censeamna can x
0
= 0 funt ia f este derivabila de k ori si f
(k)
(0) = 0; comparand cu lim
x0
f
(k)
(x) = 0
deducem ca funct ia f
(k)
este continua n x
0
= 0.
270 6. Probleme de analiz a matematic a

In acest fel rezulta, succesiv pentru ecare i


_
1, 2, . . . ,
_
n1
2
_
, ca funct ia f este de i ori derivabila
n x
0
= 0, f
(i)
(0) = 0 si f
(i)
este continua n x
0
= 0.
Daca n este impar, n = 2p + 1, atunci
_
n1
2

=
_
(2p+1)1
2
_
= p =
_
n
2

. Mai aratam ca n x
0
= 0
funct ia f nu este de p+1 ori derivabila. Ultimul termen al funct iei f
(p)
ind de tipul x
n2p
cos
1
x
= xcos
1
x
deducem ca lim
x0
f
(p)
(x) f
(p)
(0)
x 0
nu exista si deci f
(p)
nu este derivabila n x
0
= 0. Rezulta ca f nu
este de p + 1 =
_
n
2

+ 1 ori derivabila n x
0
= 0 .
Daca n este par, n = 2p, atunci
_
n1
2

=
_
2p1
2

= p 1. Aratam ca n x
0
= 0 funct ia f
(p1)
este derivabila. Ultimul termen al funct iei f
(p1)
ind de tipul x
n2(p1)
cos
1
x
= x
2
cos
1
x
deducem ca
lim
x0
f
(p1)
(x) f
(p1)
(0)
x 0
exista si deci f
(p1)
este derivabila n x
0
= 0, ceea ce nsemna ca f este de
p =
n
2
=
_
n
2

ori derivabila n acest punct. Aratam ca f


(p)
nu este continua n x
0
= 0. Va rezulta ca f
(p)
nu este derivabila n x
0
= 0, si deci f nu este de p +1 =
_
n
2

+1 ori derivabila n x
0
= 0. Ultimul termen
al funct iei f
(p)
ind de tipul x
n2p
cos
1
x
= cos
1
x
, deducem ca lim
x0
f
(p)
(x) nu exista si deci f
(p)
= f
([
n
2
])
nu este continua n x
0
= 0.
6.2.32 a) Unghiul se determina din relat ia tg = f
t
(1) . Cum f
t
(x) =
10
3
x
4

1
3
x
2
, avem f
t
(1) = 3.
Atunci tg = 3 si = arctg3 = 71

43
t
. Ecuat ia dreptei tangente la grac n punctul (1, f (1)) este
y f (1) = f
t
(1) (x 1) ,
deci
y
5
9
= 3 (x 1) .
Unghiul se poate determina din relat ia tg =
1
f
t
(1)
. Obt inem tg =
1
3
, deci = arctg
_

1
3
_
.
Ecuat ia normalei gracului n punctul (1, f (1)) este
y f (1) =
1
f
t
(1)
(x 1) ,
deci
y
5
9
=
1
3
(x 1) .
b) Avem f
t
(x) = cos x, x [0, ] . Tangenta n punctul (x
0
, f (x
0
)) la gracul funct iei f are panta
f
t
(x
0
) = cos x
0
. Dreapta de ecuat ie x+2y 5 = 0 y =
1
2
x+
5
2
are panta
1
2
. Cele doua drepte sunt
paralele atunci cand au pantele egale. Din cos x
0
=
1
2
cu x
0
[0, ] obt inem x
0
=
2
3
. Deci pe gracul
funct iei f exista un punct cu proprietatea ceruta si anume punctul
(x
0
, f (x
0
)) =
_
2
3
, f
_
2
3
__
=
_
2
3
, 1 +

3
2
_
.
c) Funct ia f ind continua pe [0, 3] si derivabila pe (0, 3) ndeplineste condit iile teoremei lui Lagrange
pe intervalul [0, 3] . Pe baza teoremei, exista cel put in un c (0, 3) n care
f
t
(c) =
f (3) f (0)
3 0
;
tangenta dusa la grac n punctul (c, f (c)) este paralela la coarda AB unde A(0, f(0)) = (0, 0) si
B(3, f(3)) = (3, 18). Determinam c.
f
t
(c) =
f (3) f (0)
3 0
3c
2
3 = 6 cu solut iile c
1
=

3 / (0, 3) , c
2
=

3 (0, 3)
6.2 Solut ii 271
Asadar exista un singur punct cu proprietatea ceruta si anume
(c, f (c)) =
_

3, f
_

3
__
=
_

3, 0
_
.
d) Determinam punctele comune ale celor doua curbe (parabole) rezolvand sistemul de ecuat ii
_
y = 8 x
2
y = x
2
.
Gasim
_
x = 2
y = 4
;
_
x = 2
y = 4.
Unghiul sub care se intersecteaza doua curbe ntr-un punct comun este egal cu unghiul format de dreptele
tangente n acel punct la prima si respectiv la a doua curba. Notam cu unghiul sub care se intersecteaza
gracele funct iilor f, g n punctul A(2, 4) si cu unghiul sub care se intersecteaza n punctul B(2, 4) .
Panta dreptei tangente n punctul A(2, 4) la gracul lui f este f
t
(2) = 4, iar panta dreptei
tangente n acelasi punct la gracul lui g este g
t
(2) = 4. Unghiul format de aceste doua drepte este
egal cu unghiul cautat ; avand
tg =
f
t
(2) g
t
(2)
1 +f
t
(2) g
t
(2)
=
4 (4)
1 + 4 (4)
=
8
15
,
obt inem = arctg
_

8
15
_
.
Panta dreptei tangente n punctul B(2, 4) la gracul lui f este f
t
(2) = 4, iar panta dreptei tangente
n acelasi punct la gracul lui g este g
t
(2) = 4. Din
tg =
f
t
(2) g
t
(2)
1 +f
t
(2) g
t
(2)
=
(4) 4
1 + (4) (4)
=
8
15
,
obt inem = arctg
_
8
15
_
.
e) Rezolvand sistemul
_
y = x
3
y =
1
x
2

_
x
5
= 1
y = x
3

_
(x 1)
_
x
4
+x
3
+x
2
+x + 1
_
= 0
y = x
3

_
x = 1
y = 1
gasim ca cele doua curbe au un singur punct comun A(1, 1) . Notam cu unghiul cerut. Avem
tg =
f
t
(1) g
t
(1)
1 +f
t
(1) g
t
(1)
=
3 (2)
1 + 3 (2)
= 1,
deci = arctg (1) =

4
.
f) Explicitam modulul n vecinatatea punctului x
0
= 1. Obt inem
f (x) =
_
x
3
+x, daca x (0, 1)
x
3
x, daca x (1, +)
si f
t
(x) =
_
3x
2
+ 1, daca x (0, 1)
3x
2
1, daca x (1, +) .
Avem
f
t
s
(1) = lim
x1
x<1
f (x) f (1)
x 1
= lim
x1
x<1
_
x
3
+x
_
0
x 1
= 2,
f
t
d
(1) = lim
x1
x>1
f (x) f (1)
x 1
= lim
x1
x>1
_
x
3
x
_
0
x 1
= 2.
272 6. Probleme de analiz a matematic a
Semidreapta tangenta gracului n punctul (1, f(1)) = (1, 0), la stanga acestui punct, are panta (coe-
cientul unghiular) f
t
s
(1) si ecuat ia
y f(1) = f
t
s
(1) (x 1) , adica y = 2 (x 1) , unde x 1.
Semidreapta tangenta gracului n punctul (1, f (1)) = (1, 0), la dreapta acestui punct, are panta (coe-
cientul unghiular) f
t
d
(1) si ecuat ia
y f(1) = f
t
d
(1) (x 1) adica y = 2 (x 1) , unde x 1.
Privind unghiul format de aceste doua semidrepte avem tg =
4
3
si deducem = arctg
_
4
3
_
.
6.2.32 a) Inegalitatea se scrie
e
x
1 +x
1
2
x
2
+
1
6
x
3

1
24
x
4
+
1
120
x
5
> 0, x (0, +) .
Studiem funct ia f : [0, +) R,
f (x) = e
x
1 +x
1
2
x
2
+
1
6
x
3

1
24
x
4
+
1
120
x
5
.
Funct ia f este ori de cate ori derivabila. Avem
f (0) = 0
f
t
(x) = e
x
+ 1 x +
1
2
x
2

1
6
x
3
+
1
24
x
4
, f
t
(0) = 0
f
tt
(x) = e
x
1 +x
1
2
x
2
+
1
6
x
3
, f
tt
(0) = 0
f
(3)
(x) = e
x
+ 1 x +
1
2
x
2
, f
(3)
(0) = 0
f
(4)
(x) = e
x
1 +x, f
(4)
(0) = 0
f
(5)
(x) = e
x
+ 1, f
(5)
(0) = 0
f
(6)
(x) = e
x
, f
(6)
(0) = 1
Succesiv rezulta datele pentru urmatorul tabel:
x 0 +
f
(6)
(x) = e
x
1 + + +
f
(5)
(x) 0
f
(5)
(x) 0 + + +
f
(4)
(x) 0
f
(4)
(x) 0 + + +
f
(3)
(x) 0
f
(3)
(x) 0 + + +
f
tt
(x) 0
f
tt
(x) 0 + + +
f
t
(x) 0
f
t
(x) 0 + + +
Citind din tabel semnul lui f
t
(x) rezulta ca f este strict crescatoare pe [0, +) si cum f (0) = 0
deducem ca f (x) > 0, x (0, +) adic a inegalitatea ceruta. b) Aratam ca f
t
(x) > 0, x (0, +) .
De aici rezulta ca f este strict crescatoare pe (0, +) . Avem
f
t
(x) =
__
1 +
1
x
_
x
_
t
=
_
1 +
1
x
_
x
_

1
x + 1
+ ln
_
1 +
1
x
__
.
Deoarece
_
1 +
1
x
_
x
> 0, x (0, +) ramane sa aratam ca
u(x) =
1
x + 1
+ ln
_
1 +
1
x
_
=
1
x + 1
+ ln(x + 1) lnx > 0, x (0, +) .
6.2 Solut ii 273
Din
u
t
(x) =
1
(x + 1)
2
+
1
x + 1

1
x
=
1
x(x + 1)
2
< 0, x (0, +) ,
rezulta ca u este strict descrescatoare pe (0, +), si t inand cont de
lim
x0
x>0
u(x) = lim
x0
x>0
_

1
x + 1
+ ln(x + 1) lnx
_
= +, lim
x+
u(x) = lim
x+
_

1
x + 1
+ ln
x + 1
x
_
= 0,
rezulta ca u(x) > 0 , x (0, +).
Aratam ca g
t
(x) < 0, x (0, +) de unde rezulta ca g este strict descrescatoare pe (0, +) . Avem
g
t
(x) =
_
_
1 +
1
x
_
x+1
_
t
=
_
1 +
1
x
_
x+1
_

1
x
+ ln
_
1 +
1
x
__
.
Deoarece
_
1 +
1
x
_
x+1
> 0, x (0, +) ramane sa aratam ca
v (x) =
1
x
+ ln
_
1 +
1
x
_
=
1
x
+ ln(x + 1) lnx < 0, x (0, +) .
Din
v
t
(x) =
1
x
2
+
1
x + 1

1
x
=
1
x
2
(x + 1)
> 0, x (0, +)
rezulta ca v este strict crescatoare pe (0, +) , si t inand cont de
lim
x0
x>0
v (x) = lim
x0
x>0
_

1
x
+ ln(x + 1) lnx
_
= lim
x0
x>0
_

1 +xlnx
x
+ ln(x + 1)
_
= ,
lim
x+
v (x) = lim
x+
_

1
x
+ ln
x + 1
x
_
= 0 + ln 1 = 0
rezulta ca v (x) < 0 , x (0, +) .
6.2.33 Deoarece f este funct ie periodica cu perioada T = 2, vom studia funct ia pe intervalul
[0, 2). Punctele de extrem local ale unei funct ii f se aa printre punctele sale stat ionare. Din acest
motiv determinam punctele stat ionare ale funct iei date. Avem f
t
(x) = 2 sinx + sin2x, f
t
(x) = 0
2 sinx + sin2x = 0 2 sinx + 2 sinxcos x = 0 sinx(1 + cos x) = 0 sinx = 0 sau cos x = 1.
Din sinx = 0, x [0, 2) obt inem x = 0 sau x = . Din cos x = 1, x [0, 2) obt inem x = 0. Asadar
n intervalul [0, 2) funct ia f are doua puncte stat ionare x
1
= 0 si x
2
= . Studiem pe rand cele doua
puncte.
Privind x
1
= 0 avem: f
tt
(x) = 2 cos x + 2 cos 2x, f
tt
(0) = 0; f
(3)
(x) = 2 sinx 4 sin2x,
f
(3)
(0) = 0; f
(4)
(x) = 2 cos x8 cos 2x, f
(4)
(0) = 6. Din f
t
(0) = 0, f
tt
(0) = 0, f
(3)
(0) = 0, f
(4)
(0) ,= 0,
numarul de derivari n = 4 ind numar par si f
(4)
(0) < 0 deducem ca n x
1
= 0 funct ia f are maxim
local.
Privind x
2
= avem: f
t
() = 0, f
tt
() = 4 ,= 0, numarul de derivari n = 2 ind numar par si
f
tt
() > 0 deducem ca n x
2
= funct ia f are minim local.

In nal, t inand cont ca funct ia f este periodica cu T = 2, obt inem ca toate punctele x = x
1
+kT =
0 +k 2 = 2k sunt puncte de maxim local si toate punctele x = x
2
+kT = +k 2 = (2k + 1) sunt
puncte de minim local ale funct iei f, unde k Z.
6.2.34 Avem f
t
(x) =
2

h
3
xe
h
2
x
2
, f
tt
(x) =
2

h
3
e
h
2
x
2
_
2h
2
x
2
1
_
, x R.
Funct ia f
tt
(x) se anuleaza pentru x
1
=
1

2h
, x
2
=
1

2h
. Avem f
tt
(x) > 0, x
_
,
1

2h
_

_
1

2h
, +
_
, respectiv f
tt
(x) < 0, x
_

2h
,
1

2h
_
. Rezulta ca funct ia f este convexa pe intervalul
_
,
1

2h
_
, este concava pe intervalul
_

2h
,
1

2h
_
si este convexa pe intervalul
_
1

2h
, +
_
.
274 6. Probleme de analiz a matematic a
Din faptul ca funct ia f este continua n x
1
=
1

2h
, are derivata n x
1
, f este convexa pe
_
,
1

2h
_
si concava pe
_

2h
,
1

2h
_
, rezulta ca x
1
=
1

2h
este un punct de inexiune al
funct iei; similar, f este continua n x
2
=
1

2h
, are derivata n x
2
, f este concava pe
_

2h
,
1

2h
_
si
convexa pe
_
1

2h
, +
_
, deci x
2
=
1

2h
este un punct de inexiune al lui f.
6.2.35 a) Consideram un polinom Q de grad k 2, cu coecient i reali, avand ca radacini numere
reale distincte si aratam ca radacinile polinomului Q
t
sunt de asemnea reale si distincte.
Notam x
1
< x
2
< . . . < x
k
radacinile lui Q. Avem Q(x
1
) = 0, Q(x
2
) = 0, . . . , Q(x
k
) = 0. Aplicam
teorema lui Rolle funct iei Q pe ecare interval [x
i
, x
i+1
] cu i 1, 2, . . . , k 1. Funct ia polinomiala
Q este continua pe intervalul [x
i
, x
i+1
] , este derivabila pe (x
i
, x
i+1
) , Q(x
i
) = Q(x
i+1
), deci, pe baza
teoremei, exista c
i
(x
i
, x
i+1
) cu proprietatea Q
t
(c
i
) = 0. Cele k1 numere reale c
1
, c
2
, . . . , c
k1
obt inute
sunt distincte dupa cum rezulta din x
1
< c
1
< x
2
< c
2
< x
3
< . . . < x
k1
< c
k1
< x
k
si sunt radacini
ale lui Q
t
. Deoarece Q
t
este polinom de grad k 1, conform teoremei fundamentale a algebrei, Q
t
are
un total de k 1 radacini n mult imea numerelor complexe. Asadar toate radacinile lui Q
t
sunt numere
reale si distincte.
Mai departe rat ionam inductiv asupra polinomului P din enunt , aplicand succesiv rezulatul demon-
strat mai sus. Polinomul P are radacinile reale si distincte P
t
are radacinile reale si distincte P
tt
are radacinile reale si distincte . . . P
(n3)
are radacinile reale si distincte P
(n2)
are radacinile
reale si distincte. Derivata P
(n1)
este polinom de gradul 1, deci are o singura radacina, care evident
este numar real.
b) Aplicam teorema lui Lagrange funct iei f pe intervalul [a, c] . Rezulta ca exista d
1
(a, c) astfel
ncat
f (c) f (a)
c a
= f
t
(d
1
) .
Aplicam teorema lui Lagrange funct iei f pe intervalul [c, b] . Rezulta ca exista d
2
(c, b) astfel ncat
f (b) f (c)
b c
= f
t
(d
2
) .
Deducem ca
f
t
(d
1
) f
t
(d
2
) =
(f (c) f (a)) (f (b) f (c))
(c a) (b c)
< 0.
Cum f
t
are proprietatea lui Darboux pe intervalul [d
1
, d
2
] , din inegalitatea f
t
(d
1
) f
t
(d
2
) < 0 rezulta ca
exista d (d
1
, d
2
) astfel ncat f
t
(d) = 0.

In mod evident d (a, b) are loc.
6.2.36 a) Funct ia f : I I este contract ie daca exista [0, 1), numit coecient de contract ie,
astfel ncat
[f (x
1
) f (x
2
)[ [x
1
x
2
[ , x
1
, x
2
I.
Fie x
1
, x
2
I arbitrare, x
1
< x
2
. Funct ia f ind derivabila pe I, ndeplineste condit iile teoremei lui
Lagrange pe intervalul [x
1
, x
2
] , adica f este continua pe [x
1
, x
2
] si este derivabila pe (x
1
, x
2
) . Con-
form teoremei exista (x
1
, x
2
) asfel ncat
f (x
1
) f (x
2
)
x
1
x
2
= f
t
() . Din

f (x
1
) f (x
2
)
x
1
x
2

= [f
t
()[ si
[f
t
()[ M, deducem

f (x
1
) f (x
2
)
x
1
x
2

M si [f (x
1
) f (x
2
)[ M[x
1
x
2
[ . Prin ipoteza M [0, 1).
Deci funct ia f este contract ie cu coecientul de contract ie = M.
b) Funct ia f este derivabila pe R. Deoarece funct ia f
t
(x) = a arctgx, x R este continua, arctgx
_

2
,

2
_
, x R si lim
x
arctgx =

2
, lim
x+
arctgx =

2
, deducem ca sup[f
t
(x)[[ x R = [a[

2
.
Evident
[f
t
(x)[ sup[f
t
(x)[[ x R = [a[

2
, x R.
6.2 Solut ii 275
Determinam a R astfel ncat sup[f
t
(x)[[ x R = [a[

2
< 1; atunci din [f
t
(x)[ [a[

2
< 1, x R,
utilizand a), rezulta ca funct ia f este contract ie avand coecientul de contract ie
= sup[f
t
(x)[[ x R = [a[

2
.
Obt inem a
_

,
2

_
.
6.2.37 i) Formula de aproximare este
f (x) f (0) +
f
t
(0)
1!
x +
f
tt
(0)
2!
x
2
+
f
ttt
(0)
3!
x
3
+
f
(4)
(0)
4!
x
4
= T
4
(x) ,
cu eroarea
f (x) T
4
(x) = R
4
(x) =
f
(5)
()
5!
x
5
,
unde este un numar cuprins ntre 0 si x.
a) Pentru f (x) = tgx avem f (0) = 0;
f
t
(x) =
1
cos
2
x
, f
t
(0) = 1; f
tt
(x) =
2 sinx
cos
3
x
, f
tt
(0) = 0;
f
ttt
(x) =
2 + 4 sin
2
x
cos
4
x
, f
ttt
(0) = 2; f
(4)
(x) =
16 sinx + 8 sin
3
x
cos
5
x
, f
(4)
(0) = 0;
f
(5)
(x) =
16 + 88 sin
2
x + 16 sin
4
x
cos
6
x
;
f (x) 0 +
1
1!
x +
0
2!
x
2
+
2
3!
x
3
+
0
4!
x
4
, deci tgx x +
1
3
x
3
,
eroarea R
4
(x) =
16 + 88 sin
2
+ 16 sin
4

120 cos
6

x
5
, situat ntre 0 si x.
b) Pentru f (x) = a
x
avem f (0) = 1;
f
t
(x) = a
x
lna, f
t
(0) = lna; f
tt
(x) = a
x
ln
2
a, f
tt
(0) = ln
2
a;
f
ttt
(x) = a
x
ln
3
a, f
ttt
(0) = ln
3
a; f
(4)
(x) = a
x
ln
4
a, f
(4)
(0) = ln
4
a;
f
(5)
(x) = a
x
ln
5
a;
f (x) 1 +
lna
1!
x +
ln
2
a
2!
x
2
+
ln
3
a
3!
x
3
+
ln
4
a
4!
x
4
, deci a
x
1 +lna x +
ln
2
a
2
x
2
+
ln
3
a
6
x
3
+
ln
4
a
24
x
4
,
eroarea R
4
(x) =
a

ln
5
a
120
x
5
, situat ntre 0 si x.
c) Pentru funct ia f : (a, a) R, f (x) =
1
a x
avem f (0) =
1
a
;
f
t
(x) =
1
(a x)
2
, f
t
(0) =
1
a
2
; f
tt
(x) =
2
(a x)
3
, f
tt
(0) =
2
a
3
;
f
ttt
(x) =
6
(a x)
4
, f
ttt
(0) =
6
a
4
; f
(4)
(x) =
24
(a x)
5
, f
(4)
(0) =
24
a
5
;
f
(5)
(x) =
120
(a x)
6
;
f(x)
1
a
+
1
a
2
1!
x +
2
a
3
2!
x
2
+
6
a
4
3!
x
3
+
24
a
5
4!
x
4
, deci
1
a x

1
a
+
1
a
2
x +
1
a
3
x
2
+
1
a
4
x
3
+
1
a
5
x
4
,
eroarea R
4
(x) =
1
(a )
6
x
5
, situat ntre 0 si x.
d) Avem f (0) = 0;
f
t
(x) =
1
2a
_
1
a +x
+
1
a x
_
, f
t
(0) =
1
a
2
; f
tt
(x) =
1
2a
_

1
(a +x)
2
+
1
(a x)
2
_
, f
tt
(0) = 0;
f
ttt
(x) =
1
2a
_
2
(a +x)
3
+
2
(a x)
3
_
, f
ttt
(0) =
2
a
4
;
276 6. Probleme de analiz a matematic a
f
(4)
(x) =
1
2a
_

6
(a +x)
4
+
6
(a x)
4
_
, f
(4)
(0) = 0;
f
(5)
(x) =
12
a
_
1
(a +x)
5
+
1
(a x)
5
_
;
f (x) 0 +
1
a
2
1!
x +
0
2!
x
2
+
2
a
4
3!
x
3
+
0
4!
x
4
, deci
1
2a
ln
a +x
a x

1
a
2
x +
1
3a
4
x
3
,
eroarea R
4
(x) =
1
10a
_
1
(a +)
5
+
1
(a )
5
_
x
5
, situat ntre 0 si x.
e) Avem f (0) = a;
f
t
(x) =
_
_
(a
n
x)
1
n
_
_
t
=
1
n
(a
n
x)
1
n
1
=
1
n
(a
n
x)
1 n
n
, f
t
(0) =
1
n

1
a
n1
;
f
tt
(x) =
n 1
n
2
(a
n
x)
1 2n
n
, f
tt
(0) =
n 1
n
2

1
a
2n1
;
f
ttt
(x) =
(n 1) (2n 1)
n
3
(a
n
x)
1 3n
n
, f
ttt
(0) =
(n 1) (2n 1)
n
3

1
a
3n1
;
f
(4)
(x) =
(n 1) (2n 1) (3n 1)
n
4
(a
n
x)
1 4n
n
, f
(4)
(0) =
(n 1) (2n 1) (3n 1)
n
4

1
a
4n1
;
f
(5)
(x) =
(n 1) (2n 1) (3n 1) (4n 1)
n
5
(a
n
x)
1 5n
n
;
n

a
n
x a
1
na
n1
x
n 1
2n
2
a
2n1
x
2

(n 1) (2n 1)
6n
3
a
3n1
x
3

(n 1) (2n 1) (3n 1)
24n
4
a
4n1
x
4
,
eroarea R
4
(x) =
(n 1) (2n 1) (3n 1) (4n 1)
120n
5
(a
n
)
1 5n
n
x
5
, situat ntre 0 si x.
ii) Formula de aproximare este
f (x) f (x
0
) +
f
t
(x
0
)
1!
(x x
0
) +
f
tt
(x
0
)
2!
(x x
0
)
2
+
f
ttt
(x
0
)
3!
(x x
0
)
3
= T
3
(x) ,
cu eroarea f (x) T
3
(x) = R
3
(x) =
f
(4)
()
4!
(x x
0
)
4
, unde este un numar cuprins ntre x
0
si x.
a) Avem
f (x) f (2) +
f
t
(2)
1!
(x 2) +
f
tt
(2)
2!
(x 2)
2
+
f
ttt
(2)
3!
(x 2)
3
,
cu eroarea R
3
(x) =
f
(4)
()
4!
(x 2)
4
, unde este un numar cuprins ntre 2 si x. Se obt ine
f(2) =
1
2
; f
t
(x) =
1
x
2
, f
t
(2) =
1
4
; f
tt
(x) =
2
x
3
, f
tt
(2) =
1
4
; f
ttt
(x) =
6
x
4
, f
ttt
(2) =
3
8
;
f
(4)
(x) =
24
x
5
f (x)
1
2
+

1
4
1!
(x 2) +
1
4
2!
(x 2)
2
+

3
8
3!
(x 2)
3
, deci
1
x

1
2

1
4
(x 2) +
1
8
(x 2)
2

1
16
(x 2)
3
,
cu eroarea R
3
(x) =
1

5
(x 2)
4
, unde este un numar cuprins ntre 2 si x.
b) Avem
f (x) f (1) +
f
t
(1)
1!
(x 1) +
f
tt
(1)
2!
(x 1)
2
+
f
ttt
(1)
3!
(x 1)
3
,
cu eroarea R
3
(x) =
f
(4)
()
4!
(x 1)
4
, unde este un numar cuprins ntre 1 si x. Se obt ine
6.2 Solut ii 277
f(1) = 1
1
= 1; f
t
(x) = xx
x1
+x
x
lnx = x
x
(lnx + 1), f
t
(1) = 1; f
tt
(x) = x
x
(lnx + 1)
2
+x
x1
,
f
tt
(1) = 2; f
ttt
(x) = x
x
ln
3
x + 3x
x
ln
2
x + 3x
x1
lnx + 3x
x
lnx + 3x
x1
+x
x
x
x2
, f
ttt
(1) = 3;
f
(4)
(x) =
x
x
ln
4
x+4x
x
ln
3
x+6x
x
ln
2
x+4x
x
lnx+x
x
+6x
x1
ln
2
x+12x
x1
lnx+6x
x1
x
x2
4x
x2
lnx+2x
x3
f (x) 1 +
1
1!
(x 1) +
2
2!
(x 1)
2
+
3
3!
(x 1)
3
, deci x
x
1 + (x 1) + (x 1)
2
+
1
2
(x 1)
3
,
cu eroarea R
3
(x) =
f
(4)
()
24
(x 1)
4
, unde este un numar cuprins ntre 1 si x.
6.2.38 a) Polinomul MacLaurin de ordinul 3 pentru funct ia f (x) = sinx este
T
3
(x) = f (0) +
f
t
(0)
1!
x +
f
tt
(0)
2!
x
2
+
f
ttt
(0)
3!
x
3
= x
1
6
x
3
.
Eroarea aproximarii f (x) T
3
(x) se reprezinta
f (x) T
3
(x) = R
3
(x) =
f
(4)
()
4!
x
4
,
unde este un numar ntre 0 si x, deci la aproximarea sinx x
1
6
x
3
eroarea se reprezinta
sin
24
x
4
cu
un numar cuprins ntre 0 si x.
Avem de determinat a > 0 astfel ncat [f (x) T
3
(x)[ 0.01, x [a, a], adica

sin
24
x
4

0, 01, x [a, a] .
Din

sin
24
x
4

= [sin[
[x[
4
24

[x[
4
24
, x R, t inand cont de
[x[
4
24
0, 01 x
_

0, 24,
4

0, 24

deducem
ca numarul a =
4

0, 24 corespunde cerint ei.


b) Utilizand a) o aproximare este sin (0, 6) 0, 6
1
6
(0, 6)
3
, adica sin(0, 6) 0, 564. Deoarece
x = 0, 6
_

0, 24,
4

0, 24

, privind eroarea aproximarii are loc


[sin(0, 6) 0, 564[ 0, 01.
Funct ii integrabile Riemann si primitive
6.2.39 a) Daca scriem termenul general a
n
sub forma
a
n
=
1
n
n

i=1
1
1 +
i
n
=
1
n
_
_
_
1
1 +
1
n
+
1
1 +
2
n
+... +
1
1 +
n
n
_
_
_,
observam ca termenii din paranteza reprezinta valorile funct iei f(x) =
1
1 +x
, f : [0, 1] R, calculata n
punctele intermediare ale diviziunii
=
_
0,
1
n
,
2
n
,
3
n
, ..,
n 1
n
,
n
n
= 1
_
intervalului [0, 1] , respectiv n punctele
i
=
i
n

_
i
n
,
i + 1
n
_
, i = 1, 2, ..., n, =
1
,
2
, ...,
n
, x
i
=

i
=
i
n
, i = 1, 2, ..., n, x
0
= 0. Factorul
1
n
din fat a sumei este chiar norma acestei diviziuni cu puncte
echidistante
x
i
x
i1
= () =
1
n
0, n .
278 6. Probleme de analiz a matematic a
Deci a
n
se poate scrie ca o suma Riemann
a
n
= (f, , ) =
n

i=1
f (
i
) (x
i
x
i1
) =
n

i=1
1
n
f
_
i
n
_
, n 1.
Cum f este integrabila, ind continua, deducem ca
lim
n
a
n
= lim
n
(f, , ) =
_
1
0
1
1 +x
dx = ln(1 +x)

1
0
= ln2.
b) Avem a
n
=
1
n
n

i=1
_
i
n
_
4
, n 1 si e clar ca suma reprezinta o suma Riemann pentru funct ia
f (x) = x
4
, f : [0, 1] R, relativ la diviziunea = (x
0
, x
1
, x
2
, ..., x
n
) Div [0, 1] , cu x
0
= 0,
x
1
=
1
n
, x
2
=
2
n
, ..., x
i
=
i
n
, ..., x
n
=
n
n
= 1,
echidistante si cu sistemul de puncte intermediare
=
1
,
2
, ...,
n
,
i
= x
i
=
i
n
, i = 1, 2, ..., n, x
i
x
i1
=
1
n
= () = norma diviziunii
Deci
a
n
= (f, , ) =
1
n
n

i=1
_
i
n
_
4

_
1
0
x
4
dx =
1
5
,
deoarece f (x) = x
4
este continua, deci integrabila Riemann.
c) Se procedeaza analog si se obt ine
lim
n
a
n
= lim
n
(f, , ) , unde f : [0, 1] R, f (x) =
1
1 +x
2
Deci
lim
n
a
n
=
_
1
0
1
1 +x
2
dx = arctgx

1
0
=

4
.
d)
lim
n
a
n
=
_

0
sinxdx = cos x

1
0
= 2.
e)
lim
n
a
n
=
_
1
0
1

4 x
2
dx = arcsin
x
2

1
0
=

6
.
f) Scriem sirul sub forma a
n
=
1
n
n

i=1
1
1 +
i
n
+
i
n
2
, n 1. Consideram funct ia f : [0, 1] R,
f (x) =
1
1 +x
, diviziunea
=
_
0,
1
n
,
2
n
, ...,
n
n
_
Div [0, 1]
si punctele intermediare

i
=
i
n
+
i
n
2

_
i
n
,
i + 1
n
_
, i = 1, 2, ..., n
Deci
a
n
= (f, , ) si () = x
i
x
i1
=
1
n
0,
6.2 Solut ii 279
astfel ca
lim
n
a
n
=
_
1
0
1
1 +x
dx = ln 2.
Analog se trateaza exemplele g) si h). Pentru g) avem
a
n
=
1

n
n

i=1
1

n
_
5 + 2
i
n
=
1
n
n

i=1
1
_
5 + 2
i
n
, n 1.
Se considera funct ia f : [0 , 1] R, f (x) =
1

5 + 2x
, continua pe [0, 1] si diviziunea =
_
0,
1
n
,
2
n
, ...,
n
n
_
cu () = max (x
i
x
i1
) = max
1
n
=
1
n
0. Deci a
n
= (f, , ) cu =
_
1
n
,
2
n
, ...,
n
n
_
si
lim
n
a
n
=
_
1
0
1

5 + 2x
dx =

5 + 2x

1
0
=

5.
6.2.40 a) Pentru n x n + 1, avem
0 <
1

1 +x
2

1 +n
2
,
0
n+1
_
n
dx

1 +x
2

1 +n
2
n+1
_
n
dx =
1

1 +n
2
,
adica
0 a
n

1

1 +n
2
.
Cu teorema clestelui se obt ine lim
n
a
n
= 0.
b) Pentru 0 x 1, avem
1
2
< 1 x + 1 2, atunci
1
x + 1
< 1, si
0
1
_
0
x
n
1 +x
dx
1
_
0
x
n
dx =
1
n + 1
Cu teorema clestelui, rezulta ca lim
n
a
n
= 0.
6.2.41 Din inegalitatea [f(x)[ M, x [0, 1], avem
0 < [a
n
[ =

_
1
0
x
n
f(x)dx

_
1
0
x
n
[f(x)[ dx M
_
1
0
x
n
dx =
M
n + 1
.
Asadar
lim
n
[a
n
[ = 0 si lim
n
a
n
= 0.
6.2.42 Daca f e crescatoare, atunci
f(a) f(x) f(b), x [a, b].
Integrand inegalitatea de mai sus, membru cu membru pe intervalul [a, b], obt inem
_
b
a
f(a)dx
_
b
a
f(x)dx
_
b
a
f(b)dx,
280 6. Probleme de analiz a matematic a
(b a) f(a)
_
b
a
f(x)dx (b a) f(b),
adica chiar inegalitatea ceruta.

In cazurile particulare, se calculeaza f
t
(x) si se arata ca f
t
(x) 0.
6.2.43 a) Explicitam funct ia f cu ajutorul semnului diferent ei
F(x) =
2
x
2
+ 1
x, F : [0, 2] R
Din faptul ca
F
t
(x) =
4x
(x
2
+ 1)
2
1 =
x
4
+ 2x
2
+ 4x + 1
(x
2
+ 1)
2
< 0, x [0, 2]
rezulta F este strict descrescatoare pe [0, 2] si F(1) = 0. Din tabelul de variat ie
x 0 1 2
F
t
(x)
F(x) F(0) 0 F(2)
rezulta ca pentru x [0, 1], F(x) 0, deci f(x) = x, iar pentru x [1, 2], F(x) 0, deci f(x) =
2
x
2
+ 1
,
f(x) =
_
x, x [0, 1]
2
x
2
+ 1
, x (1, 2].
Pe [0, 1] , f e integrabila Riemann si
_
1
0
f(x)dx =
_
1
0
xdx =
1
2
. Pe (1, 2], f diferita de funct ia f
1
(x) =
2
x
2
+ 1
, f
1
: [1, 2] R doar n punctul x = 1 si atunci f e integrabila si
_
2
1
f(x)dx =
_
2
1
f
1
(x)dx =
_
2
1
2
x
2
+ 1
dx = 2arctgx

2
1
= 2arctg2

2
Deducem ca
_
2
0
f(x)dx =
1
2
+ 2arctg2

2
.
b) Explicitam funct ia g si obt inem
g(x) =
_

_
x, x [0, 1)
x 1, x [1, 2)
x 2, x
_
2,
5
2
_
Funct iile
g
1
: [0, 1] R, g
1
(x) = x,
g
2
: [1, 2] R, g
2
(x) = x 1,
sunt continue si deci integrabile pe intervalele de denit ie. Aceste funct ii difera de restrict iile lui g: g

[0,1)
,
g

[1,2)
n cate un punct, iar pe
_
2,
5
2
_
funct ia g e continua, deci integrabila. Cu teorema 5.2.17 avem ca
g e integrabila pe
_
0,
5
2
_
si avem
_
5/2
0
g(x)dx =
_
1
0
g
1
(x) dx +
_
2
1
g
2
(x) dx +
_
5/2
2
g (x) dx =
=
_
1
0
xdx +
_
1
1
(x 1) dx +
_
5/2
2
(x 2) dx =
6.2 Solut ii 281
=
x
2
2

1
0
+
_
x
2
2
x
_

2
1
+
_
x
2
2
2x
_

5/2
2
=
1
8
.
c) Se considera funct ia h
1
: [0, 1] R, h
1
(x) = x
2
, x [0, 1] care e continua, deci integrabila
Riemann pe [0, 1] si
1
_
0
h
1
(x) dx =
x
3
3

1
0
=
1
3
.

Insa conform Teoremei 5.2.17, deoarece funct ia h difera de funct ia h


1
ntr-un numar nit de puncte
_
1
2
,
1
3
,
1
4
_
, deducem ca si h este integrabila Riemann si
1
_
0
h(x) dx =
1
_
0
h
1
(x) dx =
1
3
.
6.2.44 a) Se constata ca f e continua n x = 0, deci pe R si atunci f admite primitive. Primitivele sale
vor avea forma
f(x) =
_ _
x
3
dx, x 0
_
xdx, x > 0
=
_

_
x
4
4
+C, x 0
x
2
2
+C
1
, x > 0
Constantele C si C
1
se determina din condit ia ca F sa e continua n x = 0. Deci
F (0 0) = F(0) = lim
x0
_
x
4
4
+C
_
= C si F (0 + 0) = lim
x0
_
x
2
2
+C
1
_
= C
1
.
Deducem ca C
1
= C si atunci primitivele lui f vor :
F(x) =
_

_
x
4
4
+C, x 0
x
2
2
+C, x > 0
, C R.

In mod analog se trateaza exemplele b), c), d).


6.2.45 1. a) Daca f ar admite primitive sa presupunem ca F : R R este o primitiva. Atunci F este
derivabila n x = 0 si F
t
(0) = f(0) = 0.

Insa
F
t
s
(0) = lim
x,0
F (x) F (0)
x 0
= lim
x0
(x 0) F
t
(c)
x
unde 0 < c < x.(cu teorema lui Lagrange)
Acum se deduce
F
t
s
(0) = lim
c0
F
t
(c) = lim
c0
f (c) = lim
c0
0 = 0.
De asemenea
F
t
d
(0) = lim
x0
xF
t
(c)
x
= lim
c0
f (c) = lim
c0
1
c
= +, unde 0 < c < x.
Deci F nu e derivabila n x = 0, astfel ca F nu poate o primitiva a lui f.
b) Vom arata ca f nu are proprietatea lui Darboux. Pentru aceasta e sucient sa aratam ca f nu
transforma un interval tot ntr-un interval.
Fie I = [2, 3] R si sa observam ca
f (I) = f ([2, 3]) = f ([2, 3] ) f ([2, 3] [R ]) .

Insa f (2) = 2, f (3) = 3, si atunci f(I) = ([2, 3] ) A, unde A ]4, 9[ . Este deci evident ca mult imea
f (I) nu este un interval si atunci f nu are proprietatea lui Darboux, deci nu admite primitive.
282 6. Probleme de analiz a matematic a
c) Daca f ar admite primitive, o primitiva a sa ar avea forma
F(x) =
_
C
1
, x 0
xsin
1
x
+C
2
, x > 0
Din continuitatea lui F n x = 0 rezulta ca C
1
= C
2
= C. Se poate lua C = 0 si atunci
F(x) =
_
0, x 0
xsin
1
x
, x > 0
trebuie sa e o primitiva a lui f, deci F
t
(x) = f(x) pentru orice x R. Sa cercetam daca F
t
(0) =
f (0) = a. Avem
F
t
d
(0) = lim
x0
F (x) F (0)
x 0
= lim
x0
xsin
1
x
x
= lim
x0
sin
1
x
,
limita care se stie ca nu exista. Deci F nu e derivabila n x = 0, deci nu poate primitiva a lui f.
d) Se trateaza analog cu exercit iul b), de exemplu se arata ca intervalul I = [1, 2] nu are ca imagine
f (I) tot un interval.
e) Pentru x < 0 putem scrie
_
xarctg
1
x
_
t
= arctg
1
x

x
1 +x
2
,
iar de aici
_
arctg
1
x
dx = xarctg
1
x
+
1
2
ln
_
1 +x
2
_
+C.
Sa presupunem ca f admite primitive. Atunci o primitiva F ar avea forma
F (x) =
_
xarctg
1
2
+
1
2
ln
_
1 +x
2
_
+C, x < 0
x +C
1
, x 0.
Din continuitatea lui F rezulta ca C
1
= C.
Sa studiem acum derivabilitatea lui F n x = 0
F
t
s
= lim
x,0
xarctg
1
x
+
1
2
ln
_
1 +x
2
_
+C C
x
= lim
x,0
arctg
1
x
+
1
2
lim
x0
ln
_
1 +x
2
_
x
=

2
+ 0 =

2
;
F
t
d
= lim
x0
F(x) F(0)
x 0
= lim
x0
x +C C
x
= lim
x0
1 = 1.
Deci F
t
s
(0) ,= F
t
d
(0), adica F nu e derivabila n 0 si astfel F nu poate primitiva a lui f.
g) Funct ia
f(x) =
_
sinx
x
, x ,= 0
1, x = 0
difera de funct ia continua
g : R R, g(x) =
_
sinx
x
, x ,= 0
1, x = 0,
ntr-un singur punct x = 0.
Daca f ar admite primitive atunci si f g ar admite, ceea ce este fals, caci
(f g) (x) = f (x) g (x) =
_
0, x ,= 0
2, x = 0
6.2 Solut ii 283
nu are proprietatea lui Darboux.
h) Funct ia data difera de funct ia continua g : ]0, +[ R, g (x) =
_
0, x = 0
xln
1
x
, x > 0
,
doar n punctul x = 0. Daca f ar admite primitive, atunci si f g ar admite.

Insa
(f g) (x) = f (x) g (x) =
_
1, x = 0
0, x > 0
si aceasta funct ie nu are proprietatea lui Darboux, deci nu admite primitive.
2) a) Explicitam funct ia f si obt inem
f (x) =
_
x
n
, x [0, 1]
1, x ]1, +[
Funct ia f e continua, deci admite primitive.
b) Avem
f (x) =
_
1, x [0, 1]
x
n
, x ]1, +[
Cum f e continua avem ca f admite primitive.
c) Se considera funct ia g : R R
g (x) =
_
_
_
x
n+1
n
cos
1
x
n
, x ,= 0, n N

0, x = 0
Funct ia g este derivabila pe R, caci
g
t
(0) = lim
x0
g (x) g (0)
x 0
= lim
x0
x
n
n
cos
1
x
n
= 0,
si nca
g
t
(x) =
_
n + 1
n
cos
1
x
n
+ sin
1
x
n
, x ,= 0
0, x = 0
=
_
n + 1
n
cos
1
x
n
, x ,= 0
0, x = 0
+
_
1
n
n
, x ,= 0
0, x = 0
= H (x) +f (x) ,
unde funct ia H : R R este o funct ie continua. Atunci f = g
t
H si deci f admite primitive (ca
diferent a a doua funct ii care admit primitive).
d) Se trateaza analog, considerand funct ia derivabila g : R R
g (x) =
_
_
_
x
n+1
n
sin
1
x
n
, x ,= 0, n N

0, x = 0
e) Se arata ca f este continua.
6.2.46

In toate cazurile se foloseste integrarea prin part i. Sa rezolvam efectiv cateva dintre exemplele
propuse:
b)
_
ln
2
xdx =
_
x
t
ln
2
xdx = xln
2
x
_
x 2 lnx
1
x
dx = xln
2
x 2
_
x
t
lnxdx
= xln
2
x 2xlnx + 2
_
dx = xln
2
x 2xlnx + 2x +C.
284 6. Probleme de analiz a matematic a
d)
I =
_
e
3x
cos 2xdx =
_ _
1
3
e
3x
_
t
cos 2xdx =
1
3
e
3x
cos 2x +
1
3
2
_
e
3x
sin2xdx
=
1
3
e
3x
cos 2x +
2
3
_ _
1
3
e
3x
_
t
sin2xdx =
1
3
e
3x
sin2x +
2
9
e
3x
sin2x
4
9
_
e
3x
cos 2xdx.
Deci
I +
4
9
I = e
3x
_
1
3
cos 2x +
2
9
sin2x
_

13
9
I = e
3x
_
1
3
cos 2x +
2
9
sin2x
_
,
I =
1
13
e
3x
(3 cos 2x + 2 sin2x) +C.
f)
_
arctgxdx =
_
x
t
arctg xdx = xarctgx
_
x
1
1 +x
2
dx =
= xarctgx
1
2
_
_
1 +x
2
_
t
1 +x
2
dx = xarctgx
1
2
ln
_
1 +x
2
_
+C.
h)
_
_
x
2
9dx =
_
x
2
9

x
2
9
dx =
_
x
_
_
x
2
9
_
t
dx 9
_
dx

x
2
9
= x
_
x
2
9
_
_
x
2
9dx 9 ln

x +
_
x
2
9

.
Rezolvand aceasta ecuat ie n raport cu
_
x
2
9dx gasim
_
_
x
2
9dx =
1
2
x
_
x
2
9
9
2
ln
_
x +
_
x
2
9
_
+C, x ]3, +[.
n)
I
n
=
_
x
n

x
2
+ 2
dx =
_
x
n1
_
_
x
2
+ 2
_
t
dx
= x
n1
_
x
2
+ 2 (n 1)
_
x
n2
_
x
2
+ 2 = x
n1
_
x
2
+ 2 (n 1)
_
x
n2
x
2
+ 2

x
2
+ 2
dx
= x
n1
_
x
2
+ 22 (n 1)
_
x
n

x
2
+ 2
dx 2 (n 1)
_
x
n2

x
2
+ 2
= x
n1
_
x
2
+ 2 (n 1) I
n
2 (n 1) I
n2
.
Rezolvand n raport cu I
n,
obt inem
I
n
=
1
n
x
n1
_
x
2
+ 2
2 (n 1)
n
I
n2
, n 2.
Pentru n = 2
I
2
=
1
2
x
_
x
2
+ 2 I
0
=
1
2
x
_
x
2
+ 2
_
1

x
2
+ 2
dx;
I
2
=
1
2
x
_
x
2
+ 2 ln
_
x +
_
x
2
+ 2
_
+C.
Pentru n = 3
I
3
=
1
3
x
2
_
x
2
+ 2
4
3
I
1
,
I
1
=
_
x

x
2
+ 2
dx =
_
x
2
+ 2,
I
3
=
1
3
x
2
_
x
2
+ 2
4
3
_
x
2
+ 2 +C.
6.2 Solut ii 285
6.2.47 a) Cu schimbarea de variabila cos x = t, avem
_
sinx
1 + cos
2
x
dx =
_
dt
1 +t
2
= arctg t +C = arctg (cos x) +C
b) Cu schimbarea de variabila arctg x = t,
1
1 +x
2
dx = dt, avem
_
arctg
5
x
1 +x
2
dx =
_
t
5
dt =
t
6
6
+C =
1
6
arctg
6
x +C.
c) Cu schimbarea de variabila x
3
= t, 3x
2
dx = dt avem
_
x
2
x
6
+ 1
dx =
_
1
3
dt
t
2
+ 1
=
1
3
arctgt +C =
1
3
arctg
_
x
3
_
+C.
g) Cu schimbarea de variabila 2
x
= t, 2xln2dx = dt avem
_
2
x
4
x
1
dx =
1
ln2
_
dt
t
2
1
=
1
2 ln 2
ln
_
t 1
t + 1
_
+C =
=
1
ln4
ln
_
2
x
1
2
x
+ 1
_
+C.
h) Cu schimbarea de variabila x
2
+ 2x = t, 2 (x + 1) dx = dt obt inem
_
(x + 1)
_
x
2
+ 2xdx =
1
2
_

tdt =
1
2
_
t
1
2
dt =
1
2
t
3
2
3
2
=
1
3

t
3
+C =
1
3
_
(x
2
+ 2x)
3
+C.
l) Cu schimbarea de variabila e
x
= t, e
x
dx = dt avem
_
1
e
x

1 e
2x
dx =
_
dt

1 t
2
= arcsin
_
e
x
_
+C.
m) Cu schimbarea de variabila x = 4 sint, dx = 4 cos dt, t = arcsin
x
4
avem
_
_
16 x
2
dx =
_
_
16
_
1 sin
2
t
_
4 cos dt = 16
_
cos
2
t dt =
= 16
_
1 + cos 2t
2
dt = 8
_
t +
1
2
sin2t
_
+C =
= 8 (t + sint cos t) +C = 8
_
arcsin
x
4
+
x
16
_
16 x
2
_
+C.
n) Cu schimbarea de variabila x + 1 = t
6
, t =
6

x + 1, dx = 6t
5
dt avem
_
1

x + 1
3

x + 1
dx =
_
6t
5
dt
t
3
t
2
= 6
_
t
3
1 + 1
t 1
dt
= 6
_
t
3
3
+
t
2
2
+t + ln(t 1)
_
= C 2
6
_
(x + 1) + 3
3

x + 1 + 6
6

x + 1 + 6 ln
_
6

x + 1 1
_
+C.
p) Cu schimbarea de variabila x =
1
t
, dx =
1
t
2
dt avem
_
dx
x

3x
2
2x 1
=
_
dt

3 2t t
2
=
_
dt
_
4 (t + 1)
2
= arcsin
t + 1
2
+C = arcsin
_
1
x
+ 1
2
_
+C = arcsin
_
x + 1
2x
_
+C.
286 6. Probleme de analiz a matematic a
r) Putem scrie
I =
_
dx
_
x(1 x)
=
_
dx

x x
2
=
_
dx

1
4

_
1
4
2
1
2
x +x
2
_
=
_
dx

1
4

_
1
2
x
_
2
Facem schimbarea de variabila
1
2
x =
1
2
cos t, dx =
1
2
sint dt, t = arccos (1 2x),
I =
1
2
_
sint dt
_
1
4

1
4
cos
2
t
=
_
sint
sint
dt = t +C = arccos (1 2x) +C.
s) Avand n vedere ca (tgx)
t
=
1
cos
2
x
, e convenabil sa facem substitut ia tgx = t, cos x =
1

1 +t
2
_
ln(cos x)
cos
2
x
dx =
_
ln
_
1

1 +t
2
_
dt =
1
2
_
ln
_
1 +t
2
_
dt =
1
2
_
t
t
ln
_
1 +t
2
_
dt
=
1
2
t ln
_
1 +t
2
_
+
1
2
_
t
2t
1 +t
2
dt
=
1
2
t ln
_
1 +t
2
_
+
_
t
2
+ 1 1
t
2
+ 1
dt =
1
2
t ln
_
1 +t
2
_
+t arctgt +C
= t ln
_
1

1 +t
2
_
+t arctg t +C
= tgxln(cos x) + tgx x +C.
t) Facem schimbarea de variabila x
2
= u, 2xdx = du
_
xe
x
2 _
x
2
+ 1
_
dx =
1
2
_
e
u
(u + 1) du =
1
2
_
(e
u
)
t
(u + 1) du
=
1
2
e
u
(u + 1)
1
2
_
e
u
du =
1
2
e
u
(u + 1)
1
2
e
u
+C
=
1
2
ue
u
+C =
1
2
x
2
e
x
2
+C.
6.2.48 a) Descompunem n fract ii simple
f (x)
A
2x + 1
+
B
x + 1
, x A(x + 1) +B(2x + 1) , x (A+ 2B) x +A+B.
Constantele A si B se obt in din sistemul
A+ 2B = 1, A+B = 0 A = 1, B = 1.
Rezulta
_
x
(2x + 1) (x + 1)
dx =
_

1
2x + 1
dx +
_
1
x + 1
dx
=
1
2
ln(2x + 1) + ln(x + 1) +C = ln
x + 1

2x + 1
+C.
b) Avem
f (x) =
x
2
_
x +
1
2
_
(x 2)
=
x
(x 2) (2x + 1)
.
6.2 Solut ii 287
Se descompune n fract ii simple si se continua ca su n exemplul a)
c) Avem
x 4
x(x 1)
2
=
A
x
+
B
(x 1)
2
+
C
x 1
Se obt ine A = 4, B = 3, C = 4, de unde
_
x 4
x(x 1)
2
dx = 4
_
dx
x
3
_
dx
(x 1)
2
+ 4
_
dx
x 1
= 4 lnx +
3
x 1
+ 4 ln (x 1) +C =
3
x 1
+ ln
_
x 1
x
_
4
+C.
d)
_
2x + 1
x
2
+ 5
dx =
_
2x
x
2
+ 5
dx +
_
1
x
2
+ 5
dx =
= ln
_
x
2
+ 5
_
+
1

5
arctg
x

5
+C.
e)
_
x 1
x
2
+ 4x + 5
dx =
_
x 1
(x + 2)
2
+ 1
dx
x+2=t
=
_
t 3
t
2
+ 1
dt
=
_
t
t
2
+ 1
dt 3
_
dt
t
2
+ 1
=
1
2
ln
_
t
2
+ 1
_
3arctg t +C =
=
1
2
ln
_
x
2
+ 4x + 5
_
3arctg (x + 2) +C.
f) Avem
1
x
3
+ 1
=
1
(x + 1) (x
2
x + 1)
=
A
x + 1
+
Bx +C
x
2
x + 1
.
Prin identicare, se obt ine sistemul
A+B = 0, A+B +C = 0, A+C = 1,
cu solut ia
A =
1
3
, B =
1
3
, C =
2
3
.
Atunci
I =
_
1
x
3
+ 1
dx =
1
3
_
dx
x + 1

1
3
_
x 2
x
2
x + 1
=
1
3
ln(x + 1)

1
3
_
x 2
x
2
2
1
2
x +
1
4
+
3
4
dx =
1
3
ln(x + 1)
1
3
_
x 2
_
x
1
2
_
2
+
3
4
dx.
Facem substitut ia x
1
2
= t, rezulta
I =
1
3
ln(x + 1)
1
3
_
t +
1
2
2
t
2
+
3
4
dt =
1
3
ln(x + 1)
1
3
_
t
t
2
+
3
4
dt+

1
2
_
dt
t
2
+
3
4
=
1
3
ln(x + 1)
1
6
ln
_
t
2
+
3
4
_
+
1
2
2

3
arctg
2t

3
+C =
288 6. Probleme de analiz a matematic a
=
1
3
ln(x + 1)
1
6
ln
_
x
2
x + 1
_
+
1

3
arctg
_
2x 1

3
_
+C.
g)
f (x) =
1
x
4
+ 1
=
1
x
4
+ 2x
2
+ 1 2x
2
=
1
(x
2
+ 1)
2
2x
2
=
=
1
_
x
2

2x + 1
_ _
x
2
+

2x + 1
_ =
Ax +B
x
2

2x + 1
+
Cx +D
x
2
+

2x + 1
.
Eliminand numitorii si identicand numaratorii se obt ine sistemul liniar
A+C = 0, (AC)

2 +B +D = 0, A+C + (B D)

2 = 0, B +D = 1,
cu solut ia
A =

2
4
, B =
1
2
, C =

2
4
, D =
1
2
.
Atunci
I =
_
dx
x
4
+ 1
=
_

2
4
x +
1
2
x
2

2x + 1
dx +
_

2
4
x +
1
2
x
2

2x + 1
dx =
=
1
4
_

2x 2
x
2
2

2
2
x +
2
4
+
1
2
dx +
1
4
_

2x + 2
x
2
+ 2

2
2
x +
2
4
+
1
2
dx
=
1
4
_

2x 2
_
x

2
2
_
2
+
1
2
dx +
1
4
_

2x + 2
_
x
2
+

2
2
_
2
+
1
2
dx.
La prima integrala punem x

2
2
= t, iar la a doua x +

2
2
= u si obt inem
_
dx
x
4
+ 1
=
1
4
_

2
_
t +

2
2
_
2
t
2
+
1
2
dt +
1
4
_

2
_
u

2
2
_
+ 2
u
2
+
1
2
du =
=

2
4
_
t
t
2
+
1
2
dt +
1
4
_
1
t
2
+
1
2
dt +

2
4
_
u du
u
2
+
1
2
+
1
4
_
du
u
2
+
1
2
=

2
8
ln
_
t
2
+
1
2
_
+
+
1
4

2arctg
_

2t
_
+

2
8
ln
_
u
2
+
1
2
_
+
1
4

2arctg
_

2u
_
+C =
=

2
8
ln
x
2
+

2x + 1
x
2

2x + 1
+

2
4
arctg
_

2x 1
_
+

2
4
arctg
_

2x + 1
_
+C.
h) Funct ia f (x) =
x
2
(x
2
+ 9)
2
, o descompunem n fract ii simple
x
2
(x
2
+ 9)
2
=
Ax +B
(x
2
+ 9)
2
+
Cx +D
x
2
+ 9
.
Din identicare se obt in A = 0, B = 9, C = 0, D = 1. Rezulta
_
x
2
(x
2
+ 9)
2
dx = 9
_
dx
(x
2
+ 9)
2
+
_
dx
x
2
+ 9
.
6.2 Solut ii 289
Daca notam I
n
=
_
dx
(x
2
+ 9)
n
, atunci
_
x
2
(x
2
+ 9)
2
dx = 9I
2
+I
1

Insa I
1
=
1
3
arctg
x
3
, iar pentru a ajunge la I
2
integram prin part i pe I
1
(n general pentru a obt ine I
n
,
integram prin part i pe I
n1
). Avem
I
1
=
_
dx
x
2
+ 9
=
_
x
t
1
x
2
+ 9
dx =
x
x
2
+ 9

_
x
2x
(x
2
+ 9)
2
dx
=
x
x
2
+ 9
+ 2
_
1
x
2
+ 9
dx 18
_
dx
(x
2
+ 9)
2
.
Rezulta
18I
2
=
x
x
2
+ 9
+ 2I
1
I
1
sau I
2
=
1
18
x
x
2
+ 9
+
1
18
1
3
arctg
x
3
.

In nal
_
x
2
(x
2
+ 9)
2
dx =
1
2
x
x
2
+ 9

1
6
arctg
x
3
+
1
3
arctg
x
3
+C =
=
x
2 (x
2
+ 9)
+
1
6
arctg
x
3
+C.
l) Avem
_
x + 4
(x
2
+ 4)
3
dx =
_
x
(x
2
+ 4)
3
dx + 4
_
1
(x
2
+ 4)
3
dx (1)
La prima integrala punem x
2
+ 4 = t, 2xdx = dt
_
x
(x
2
+ 4)
3
dx =
1
2
_
dt
t
3
=
1
2
t
2
2
=
1
4
1
t
2
=
1
4 (x
2
+ 4)
2
.
Integrala a doua din (1) o notam cu
I
3
=
_
dx
(x
2
+ 4)
3
.
Vom considera si pe I
2
=
_
dx
(x
2
+ 4)
2
, pe care o integram prin part i
I
2
=
_
x
t
1
(x
2
+ 4)
2
dx =
x
(x
2
+ 4)
2

_
x
2
_
x
2
+ 4
_
2x
(x
2
+ 4)
4
dx =
=
x
(x
2
+ 4)
2
+ 4
_
x
2
+ 4 4
(x
2
+ 4)
3
dx =
x
(x
2
+ 4)
2
+ 4
_
dx
(x
2
+ 4)
2
16I
3
.
De aici se obt ine
16I
3
=
x
(x
2
+ 4)
2
+ 4I
2
I
2
=
x
(x
2
+ 4)
2
+ 3I
2
;
I
3
=
1
16
x
(x
2
+ 4)
2
+
3
16
I
2
. (2)
290 6. Probleme de analiz a matematic a
Acum pentru a obt ine pe I
2
, plecam de la I
1
I
1
=
_
dx
x
2
+ 4
=
1
2
arctg
x
2
=
_
x
t
1
x
2
+ 4
dx
=
x
x
2
+ 4

_
x
2x
(x
2
+ 4)
2
dx =
x
x
2
+ 4
+ 2
_
x
2
+ 4 4
(x
2
+ 4)
2
dx
=
x
x
2
+ 4
+ 2
_
dx
x
2
+ 4
8
_
dx
(x
2
+ 4)
2
.
Atunci
8I
2
=
x
(x
2
+ 4)
2
+I
1
, sau I
2
=
1
8
x
x
2
+ 4
+
1
16
arctg
x
2
Acum din (2), obt inem
I
3
=
1
16
x
(x
2
+ 4)
2
+
3
128
x
x
3
+ 4
+
3
256
arctg
x
2
Revenind la relat ia (1)
_
x + 4
(x
2
+ 4)
2
dx =
1
4 (x
3
+ 4)
2
+
1
4
x
(x
2
+ 4)
2
+
3
32
x
x
2
+ 4
+
3
64
arctg
x
2
+C
=
x 1
4 (x
2
+ 4)
2
+
3
32
x
x
2
+ 4
+
3
64
arctg
x
2
+C.
6.2.49 a) Se face schimbarea de variabile
tg
x
2
= t, x = 2 arctg t, dx =
2
1 +t
2
dt, sinx =
2t
1 +t
2
si se obt ine
_
dx
5 + 4 sinx
=
_
2dt
1 +t
2
5 +
8t
1 +t
2
= 2
_
dt
5t
2
+ 8t + 5
.
Trinomul 5t
2
+ 8t + 5 are < 0 si atunci se poate scrie ca o suma de patrate
5t
2
+ 8t + 5 = 5
_
t
2
+
8
5
t + 1
_
= 5
_
t
2
+ 2
4
5
t +
16
25
+
9
25
_
=
= 5
_
_
t +
4
5
_
2
+
_
3
5
_
2
_
.
Revenind la integrala avem
2
_
dt
5
_
_
t +
4
5
_
2
+
_
3
5
_
2
_
t+
4
5
=u
=
2
5
_
du
u
2
+
_
3
5
_
2
=
2
5
5
3
arctg
5u
3
+C =
2
3
arctg
_
5t + 4
3
_
+C
=
2
3
arctg
_
_
5tg
x
2
+ 4
3
_
_
+C.
b) Funct ia f (x) =
sin
2
x
1 + cos
2
x
este para simultan n sinx si cos x si atunci se face substitut ia
tgx = t, x = arctg t, cos x =
1

1 +t
2
, sinx =
t

1 +t
2
6.2 Solut ii 291
Obt inem
_
sin
2
x
1 + cos
2
x
dx =
_
t
2
1 +t
2
1 +
1
1 +t
2
dt
1 +t
2
=
_
t
2
(t
2
+ 1) (t
2
+ 2)
dt.
Descompunem n fract ii simple
t
2
(t
2
+ 1) (t
2
+ 2)
=
At +B
t
2
+ 1
+
Ct +D
t
2
+ 2
.
Se obt ine A = 0, B = 1, C = 0, D = 2 si
_
t
2
(t
2
+ 1) (t
2
+ 2)
dt =
_
dt
t
2
+ 1
+ 2
_
dt
t
2
+ 2
= arctgt +
2

2
arctg
t

2
+C = x +

2arctg
_
tgx

2
_
+C.
c) Cu substitut ia tg
x
2
= t se obt ine
_
dx
2 sinx cos x + 5
=
1

5
arctg
_
_
3tg
x
2
+ 1

5
_
_
+C.
d) Funct ia este impara n raport cu cos x si se face substitut ia sinx = t , cos xdx = dt
_
sin
2
xcos
3
xdx =
_
t
2
_
1 t
2
_
dt =
t
3
3

t
5
3
+C
=
sin
3
x
3

sin
5
x
5
+C.
e) Funct ia e impara n raport cu sinx si atunci e convenabila substitut ia cos x = t, sinx = dt
_
sin
5
xdx =
_
_
1 t
2
_
2
dt =
_
_
1 2t
2
+t
4
_
dt
= 7 t + 2
t
3
3

t
5
5
+C = cos x +
2
3
cos
3
x
1
5
cos
5
x +C.
f) Funct ia e para simultan n raport cu sinx si cos x si se pune tgx = t, sinx =
t

1 +t
2
_
dx
sin
4
x cos
4
x
=
_
dt
1 +t
2
t
4
(1 +t
2
)
2

1
(1 +t
2
)
2
=
_
_
1 +t
2
_
3
t
4
dt =
=
_
1 + 3t
2
+ 3t
4
+t
6
t
4
dt =
_
_
t
4
+ 3t
2
+ 3 +t
2
_
dt =
=
t
3
3

3
t
+ 3t +
t
3
3
+C =
1
3tg
3
x

3
tgx
+ 3tgx +
1
3
tg
3
x +C.
g) Se transforma produsul n suma
cos xcos 3xcos 6x = (cos 4x + cos 2x) cos 6x =
= cos 10x + cos 2x + cos 8x + cos 4x.
292 6. Probleme de analiz a matematic a
Atunci
_
cos xcos 3xcos 6x dx =
1
10
sin10x +
1
8
sin8x +
1
4
sin4x +
1
2
sin2x +C.
h) Facem substitut ia tgx = t
_
dx
tg
8
x
=
_
dt
t
8
(t
2
+ 1)
.
Descompunem funct ia rat ionala de sub integrala n fract ii simple
1
t
8
(t
2
+ 1)
=
A
8
t
8
+
A
7
t
7
+
A
6
t
6
+... +
A
1
t
+
Bt +C
t
2
+ 1
. (1)
Determinarea constantelor A
1
, A
2
, ..., A
8
, B, C o vom face n felul urmator: Determinam pe A
8
, nmult ind
relat ia (1) n ambii membri cu t
8
si apoi facem t = 0. Obt inem A
8
= 1 si trecemn membrul stang termenul
A
8
t
8
=
1
t
8
1
t
8
(t
2
+ 1)

1
t
8
=
t
2
t
8
(t
2
+ 1)
=
1
t
6
(t
2
+ 1)
=
A
7
t
7
+
A
6
t
6
+...
Acum din identitatea

1
t
6
(t
2
+ 1)
=
A
7
t
7
+
A
6
t
6
+... +
A
1
t
+
Bt +C
t
2
+ 1
,
determinam pe A
7
ca si pe A
8,
adica nmult im egalitatea n ambii membri cu t
7
si apoi facem t = 0. Vom
obt ine A
7
= 0 si identitatea devine

1
t
6
(t
2
+ 1)
=
A
6
t
6
+
A
5
t
5
+... +
A
1
t
+
Bt +C
t
2
+ 1
.
Procedand analog, gasim
A
6
= 1, A
5
= 0, A
4
= 1, A
3
= 0, A
2
= 1, A
1
= 0, B = 0, C = 1.
Asadar
1
t
8
(t
2
+ 1)
=
1
t
8

1
t
6
+
1
t
4

1
t
2
+
1
t
2
+ 1
_
dx
tg
8
x
=
t
7
7
+
t
5
5

t
3
3
+
1
t
+ arctg t +C =
=
1
7
1
tg
7
x
+
1
5
1
tg
5
x

1
3
1
tg
3
x
+
1
tgx
+x +C.
6.2.50 Folosim substitut ia lui Euler
_
x
2
+x + 1 = x +t,
x
2
+x + 1 = x
2
+ 2tx +t
2
, x(1 2t) = t
2
1, x =
t
2
1
1 2t
dx = 2
t
2
t + 1
(1 2t)
2
dt, x = 0 t = 1, x = 1 t =

3 1
_
1
0
dx
(x + 1)

x
2
+x + 1
=
_

31
1
2
t
2
t + 1
(1 2t)
2
dt
t
2
2t
1 2t
_
t
2
1
1 2t
+t
_ = +2
_

31
1
t
2
t + 1
t (t 2) (t
2
t + 1)
dt =
= 2
_

31
1
dt
t (t 1)
,
1
t (t 2)
=
A
t
+
B
t 2
, A =
1
2
, B =
1
2
6.2 Solut ii 293
= 2
_
1
2
_
1
t 2

1
t
_
dt = (ln[t 2[ ln[t[)

31
1
=
= ln

t 2
t

31
1
= ln

3
b) Facem substiut ia x = 2 sint, dx = 2 cos t dt si obt iem
I =
_
1
1
dx
(x
2
+ 4)

4 x
2
=
_
6

6
dt
sin
2
t + 1
.
Facem o noua substitut ie tg t = u
I =
1
2
_

3
3

3
3
du
u
2
+
1
2
=
1
2

2arctg

2u

3
3

3
3
=

2arctg

6
3
.
c) Facem substiut ia
_
x
2
x + 1 = x t, x =
t
2
1
2t 1
, dx = 2
t
2
t + 1
(2t 1)
2
dt.
Obt inem
I =
_
du
x

x
2
x + 1
= 2
_ t
2
t+1
(2t1)
2
dt
t
= 2
_
t
2
t + 1
t (2t 1)
2
dt
Se descompune funct ia n fract ii simple si se obt ine
I = 2
_
dt
t
+ 3
_
dt
(2t 1)
2
3
_
dt
2t 1
= 2 ln[t[
3
2
1
2t 1

3
2
ln[2t 1[ +C =
= 2 ln

x
_
x
2
x + 1


3
2
_
2x 2

x
2
x + 1 1
_
3
2
ln

2x 2
_
x
2
x + 1 1

+C.
d) Avem
I =
_
3
2
dx
(2x 3)

4x x
2
.
Vom face cea de a treia substitut ie a lui Euler
_
4x x
2
= tx, x = 2 t = 1, x = 3 t =

3
3
,
x =
4
t
2
+ 1
, dx =
8t
(t
2
+ 1)
2
dt.
Integrala devine
I =
2
3
_

3
3
1
dt
t
2
5/3
=
2
3
1
2
_
5
3
ln

t
_
5/3
t +
_
5/3

3
3
1
=
1

15
ln
_
3

5
_ _
4 +

15
_
2
.
e) Avem
_
dx
x
3

x
2
+ 1
=
_
x
1
_
x
2
+ 1
_

1
3
dx
Este o integrala binoama
_
x
m
(ax
n
+b)
p
dx cu m, n, p .

In cazul nostru m = 1,n = 2, p =
1
3
.
Cum p ,= Z dar
m+ 1
n
=
1 + 1
2
= 0 Z, se face schimbarea de variabile ax
n
+ b = t
r
unde r este
numitorul lui p,
x
2
+ 1 = t
3
, x =
_
t
3
1
_
1/2
, dx =
3
2
t
2
_
t
3
1
_

1
2
dt.
294 6. Probleme de analiz a matematic a
Atunci
I =
_
x
1
_
x
2
+ 1
_

1
3
dx =
3
2
_
t
t
3
1
dt =
3
2
_
t
(t 1) (t
2
+t + 1)
Se descompune funct ia rat ionala n fract ii simple si se obt ine
I =
1
2
_
dt
t 1

1
2
_
t 1
t
2
+

t + 1
dt =
1
2
ln[t 1[
1
2
_
t 1
_
t +
1
2
_
2
+
3
4
dt
t+
1
2
=u
=
1
2
ln[t 1[
1
2
_
u du
u
2
+
3
4
+
3
4
_
du
u
2
+
3
4
=
1
2
ln[t 1[
1
4
ln
_
u
2
+
3
4
_
+
3
4
2

3
arctg
2u

3
+C
=
1
2
ln[t 1[
1
4
ln
_
t
2
+t + 1
_
+

3
2
arctg
2t + 1

3
+C
=
1
2
ln
_
3
_
x
2
+ 1 1
_

1
4
ln
_
3
_
(x
2
+ 1)
2
+
3
_
x
2
+ 1 + 1
_
+
+

3
2
arctg
_
2
3

x
2
+ 1 1

3
_
+C.
f) Avem
_
1
1
dx
4

x
4
+ 1
=
_
1
1
_
x
4
+ 1
_

1
4
dx, unde m = 0, n = 4, p =
1
4
.
Deoarece
p / Z,
m+ 1
n
=
0 + 1
4
=
1
4
/ Z,
m+ 1
n
+p =
1
4

1
4
= 0 Z
facem cea dea treia substitut ie a lui Cebsev
a +bx
n
= t

, = numitorul lui p.

In cazul nostru
1 +x
4
= t
4
, x
4
= t
4
1,
1
x
4
= t
4
1, x
4
=
1
t
4
1
,
x = 1, t =
4

2, x = 1, t =
4

2, x =
_
t
4
1
_

1
4
, dx =
_
t
4
1
_

5
4
t
3
dt.
Atunci
_
1
1
_
x
4
+ 1
_

1
4
dx =
_ 4

2
4

t
4
1
t
t
3
4
_
(t
4
1)
5
dt =
_ 4

2
t
2
1
t
4
1
dt =
=
_

2

2
t
2
(t 1) (t + 1) (t
2
+ 1)
dt =
1
4
_

2

2
dt
t 1
+
1
4
_

2

2
dt
t + 1

1
2
_

2

2
dt
t
2
+ 1
=
=
1
4
ln

t + 1
t 1

1
2
arctg t

2
=
1
4
ln
_

2 + 1

2 1
_
2
arctg

2 = ln
_

2 + 1
_
arctg

2.
h) Avem ca
_
_
x
4
+x
3
dx =
_
_
x
4
+x
3
_1
2
dx =
_
_
x
3
(x + 1)
1
2
dx =
_
x
3
2
(x + 1)
1
2
dx,
este o integrala binoama cu m =
3
2
, n = 1, p =
1
2
, unde
m+ 1
n
=
3
2
+ 1
1
=
5
2
/ Z,
m+ 1
n
+p =
5
2
+
1
2
= 3 Z.
6.2 Solut ii 295
Facem a treia substitut ie a lui Cebsev
1 +x
1
= t
2
, x =
1
t
2
1
=
_
t
2
1
_
1
, dx = 2t
_
t
2
1
_
2
dt
Atunci
_
_
x
4
+x
3
dx = 2
_
_
t
2
1
_

3
2
t
_
t
2
1
_

1
2
t
_
t
2
1
_
2
dt =
= 2
_
t
2
_
t
2
1
_
4
dt = 2
_
t
2
(t
2
1)
4
dt.
Facem descompunerea n fract ii simple
t
2
(t 1)
4
(t + 1)
4
=
A
1
(t 1)
4
+
A
2
(t 1)
3
+
A
3
(t 1)
2
+
A
4
t 1
+
+
B
1
(t + 1)
4
+
B
2
(t + 1)
3
+
B
3
(t + 1)
2
+
B
4
t + 1
.
Dupa calcule lungi se obt ine
A
1
=
1
16
, B
1
=
1
16
, A
2
= B
2
= 0, A
3
= B
3
=
1
32
, A
4
=
1
32
, B
4
=
1
32
si
_
_
x
4
+x
3
dx =
1
8
_
dt
(t 1)
4

1
8
_
dt
(t + 1)
4
+
1
16
_
dt
(t 1)
2
dt+
+
1
16
_
dt
(t + 1)
2

1
16
_
dt
t 1
+
1
16
_
dt
t + 1
=
1
24
1
(t 1)
3
+
+
1
24
1
(t + 1)
3

1
16
1
t 1

1
16
1
t + 1
+ 16 ln

t + 1
t 1

+C.
Aici se nlocuieste t =
_
x + 1
x
. i) e
x
= t, x = lnt, dx =
dt
t
.
_
dx
e
2x
+e
x
2
=
_
dt
t (t
2
t 2)
=
_
dt
t (t + 1) (t + 2)
=
1
2
_
dt
t

_
dt
t 1
+
1
2
_
dt
t + 2
=
1
2
ln[t (t + 2)[ ln(t 1)
= ln
_
t (t + 2)
(t 1)
+C.
j) Integrala
_
x

1 x
3
dx =
_
x
1
2
_
x
3
+ 1
_

1
2
dx,
este o integrala binoama cu m =
1
2
, n = 3, p =
1
2
, p =
1
2
/ Z,
m+ 1
n
=
1
2
/ Z,
m+ 1
n
+p =
1
2

1
2
=
0 Z. Facem a treia substitut ie a lui Cebsev
1 +x
3
= t
2
, x =
_
t
2
+ 1
_

1
3
, x
3
+ 1 = t
2
_
t
2
+ 1
_
1
, dx =
2
3
+
_
t
2
+ 1
_

4
5
dt,
Atunci
_
x

1 x
3
dx =
2
3
_
_
t
2
+ 1
_

1
6
t
1
_
t
2
+ 1
_1
2
t
_
t
2
+ 1
_

4
3
dt =
296 6. Probleme de analiz a matematic a
=
2
3
_
dt
t
2
+ 1
=
2
3
arctg t +C =
2
3
arctg
_

1 x
3
x

x
_
+C.
k) Se face schimbarea 1 +x
3
= t
2
_
2
0
x
2
_
1 +x
3
dx =
2
3
_
3
1
t
2
dt =
2
3
t
3
3

3
1
=
52
9
.
l) Pentru calculul integralei
I =
_
4
0
ln(1 + tg x) dx
n prealabil este util sa stabilim relat ia
_
b
a
f (x) dx =
_
b
a
f (a +b x) dx, (1)
pentru orice funct ie f : [a, b] R, integrabila Riemann. Justicarea este imediata. Sa aplicam relat ia
(1) n cazul integralei noastre
I =
_
4
0
ln(1 + tg x) dx =
_
4
0
ln
_
1 + tg
_

4
x
__
dx =
=
_
4
0
ln
_
1 +
1 tg x
1 + tg x
_
dx =
_
4
0
ln
_
2
1 + tg x
_
dx =

4
_
0
ln2dx I.
Din aceasta egalitate rezulta
_
4
0
ln(1 +tg x) dx = I
si gasim
I =
_
4
0
ln(1 +tg x) dx =
1
2
ln2
_
4
0
dx =

8
ln2.
6.2.51 1) a) Se constata ca f (x) 0 si atunci
Aria (D) =
_
2
1
_
x 1
x + 1
dx.
Se face substitut ia
x 1
x + 1
= t
2
, x =
1 +t
2
1 t
2
, dx =
4t
(1 t
2
)
2
dt. atunci
Aria (D) = 4
_

3
3
0
t
2
dt
(1 t
2
)
2
.
Avem
t
2
(t
2
1)
2
=
A
(t 1)
2
+
B
t 1
+
C
(t + 1)
2
+
D
t + 1
.
Se obt ine
Aria (D) =

3
3
_
0
1
4
(t 1)
2
+
1
4
t 1
+
1
4
(t + 1)
2

1
4
t + 1
dt =
1
4
ln

t 1
t + 1

3
3
0

1
t 1

3
3
0

1
t + 1

3
3
0
=
1
4
ln
_
2

3
_
+

3.
6.2 Solut ii 297
b) Avem
Aria (D) = 4
_
4
0
x +

x
2
1

[x + 2[
Explicitand funct ia de sub integrala, avem
x +

x
2
1

[x + 2[
=
_

_
x + 1 x
2
x + 2
, x [0, 1]
x +x
2
1
x + 2
, x [1, 4]
atunci
Aria (D) =
_
1
0
1 +x x
2
x + 2
dx+
_
4
1
x
2
+x 1
x + 2
dx =
_
1
0
_
x + 3
5
x + 2
_
dx+
_
4
1
_
x 1 +
1
x + 2
_
dx =
=
_

x
2
2
+ 3x 5 ln (x + 2)
_

1
0
+
_
x
2
2
x + ln(x + 2)
_

4
1
= 7 5 ln 3 + 6 ln2
c) Avem
Figura 5.8.1
A =
_
1
1
_
1
x
2
+ 1

x
2
2
_
dx =
_
arctg x
x
3
6
_

1
1
=

2

1
3
d) Sa aam punctele de intersect ie A si B ale gracelor celor doua funct ii (doua parabole) y =
x
2
+ 6x, y = x
2
4. Obt inem A(2, 8) , B(1, 5) Atunci
Aria (D) =
_
1
2
_
x
2
4 x
2
6x
_
dx =
_
1
2
_
2x
2
6x 4
_
dx =
_

2
3
x
3
3x
2
4x
_

1
2
=
1
3
Figura 5.8.2
e) Gracul celor doua funct ii se intersecteaza n punctele de abscise x =
1
e
si x = 1 si n intervalul
x
_
1
e
, 1
_
avem, lnx < 0 si ln
2
x + lnx < 0, astfel ca
Aria (D) =
_
1
1
e

ln
2
x + lnx

dx =
_
1
1
e
_
ln
2
x lnx
_
dx =
=
_
1
1
e
x
t
ln
2
xdx
_
1
1
e
lnxdx = x ln
2
x

1
1
e
+ 2
_
1
1
e
lnxdx
_
1
1
e
lnxdx
=
1
e
+
_
1
1
e
.
x
t
lnxdx =
1
e
+xlnx

1
1
e

_
1
1
e
dx =
3
e
1.
f) Parabola si cercul se intersecteaza n punctele A
_
2, 2

3
_
, B
_
2, 2

3
_
.
Aria (D
1
) = 2
_
2
0

6xdx + 2
_
4
2
_
16 x
2
dx =
= 2

6
2
3

x
3

2
0
+ 2
_
2

6
4 cos
2
t dt
Figura 5.8.3
298 6. Probleme de analiz a matematic a
La a doua integrala se pune x = 4 sint.
Aria (D
1
) =
16

3
3
+ 4
_
2

6
(1 + cos 2t) dt =
16

3
3
+ 4
_
t +
1
2
sin2t
_

6
=
=
16

3
3
+ 4
_

3
4
_
=
13

3
3
+
4
3
.
Aria (D
2
) = 16 Aria (D
1
) = 16
13

3
3

4
3
=
44
3

13

3
3
.
g) Sa trasam cele doua parabole cu axele de simetrie paralele cu 0x si sa aam punctele lor de
intersect ie. Punctele de intersect ie sunt A
_
1, 2

6
_
, B
_
1, 2

6
_
Aria(D) = 2
_
1
2
_
24 (2 +x) dx + 2
_
2
1
_
8 (2 x) dx = 4

6
_
1
2

2 +xdx + 4

2
_
2
1

2 xdx =
32

6
3
.
Figura 5.8.4
6.2.52 a)
V ol (K
f
) =
_ 1
2
0
f
2
(x) dx =
_ 1
2
0
arcsin
2
xdx =
_ 1
2
0
x
t
arcsin
2
xdx
= x arcsin
2
x

1
2
0
2
_ 1
2
0
x
arcsinx

1 x
2
dx
=

2
_

6
_
2
2
_ 1
2
0
_

_
1 x
2
_
t
arcsinxdx =

3
72
+ 2
_
1 x
2
arcsinx

1
2
0
2
_ 1
2
0
dx
=

3
72
+ 2

3
2

6
2
1
2
=

3
72
+

2

3
6

b) Avem
V ol (K
f
) =
_
3
0
x(x 3)
x 4
dx =
_
3
0
x
2
4x +x
x 4
dx =
_
3
0
xdx +
_
3
0
x 4 + 4
x 4
dx
=
x
2
2

3
0
+x

3
0
+ 4 ln[x 4[

3
0
=
9
2
+ 3 4 ln4 =
_
15
2
8 ln 2
_
.
c) Avem
V ol (K
f
) =
_
b
a
_
(x a) (b x)dx =
_
b
a
_
ab + (a +b) x x
2
dx
=
_
b
a

_
(a +b)
2
4
ab
_
(a +b)
2
4

2 (a +b)
2
x +x
2
_
dx
=
_
b
a

(b a)
2
4

_
a +b
2
x
_
2
dx
a+b
2
x=u
=
_

ba
2
ba
2

(b a)
2
4
u
2
du =
_ ba
2

ba
2

(b a)
2
4
u
2
du.
Se face substitut ia u =
b a
2
sint si se obt ine
V ol (K
f
) =
_
2

2
(b a)
2
4
cos
2
t dt =
(b a)
2
8
_
2

2
(1 + cos 2t) dt =
(b a)
2
8
=

2
(b a)
2
8
.
6.2 Solut ii 299
e) Cercul x
2
+ y
2
= 4 se intersecteaza cu parabola y
2
= 3x n punctele A
_
1,

3
_
, B
_
1,

3
_
. Volumul
corpului obt inut K
fg
va
V ol (K
fg
) =
_
1
0
3xdx +
_
2
1
_
4 x
2
_
dx =
3x
2
2

1
0
+
_
4x
x
3
3
_

2
1
=
3
2
+
5
3
=
19
6
.
Figura 5.8.5
6.2.53 a) Avem
l (G
f
) =
_
4
1
_
1 +f

2
(x)dx =
_
4
1

1 +
_
x
1
4x
_
2
dx =
_
4
1
_
1 +x
2
+
1
16x
2

1
2
dx
=
_
4
1
_
x
2
+
1
16x
2
+
1
2
dx =
_
4
1

16x
2
+ 8x
2
+ 1
4x
dx =
_
4
1
4x
2
+ 1
4x
dx =
1
4
_
4
1
_
4x +
1
x
_
dx =
=
1
4
_
4
x
2
2
+ lnx
_

4
1
=
15 + ln2
2
.
b) Avem
l (G
f
) =
_
1
0
_
1 +
1
4
(e
x
e
x
)
2
dx =
1
2
_
1
0
_
e
2x
+e
2x
+ 2dx =
1
2
_
1
0
_
(e
x
+e
x
)
2
dx =
=
1
2
_
1
0
_
e
x
+e
x
_
dx =
1
2
_
e
x
e
x
_

1
0
=
1
2
_
e
1
e
1 + 1
_
=
1
2
_
e
1
e
_
.
c) Avem
l (G
f
) =
_ 2
3
0

1 +
_
2x
1 x
2
_
2
dx =
_ 2
3
0

(x
2
+ 1)
(1 x
2
)
2
2
dx =
_ 2
3
0
x
2
+ 1
1 x
2
dx =
_ 2
3
0
x
2
1 + 2
x
2
1
dx =
=
_ 2
3
0
dx 2
_ 2
3
0
dx
x
2
1
= x

2
3
0
2
1
2
ln

x 1
x + 1

2
3
0
=
2
3
ln
1
3
5
3
=
2
3
ln
1
5
= ln5
2
3
d) Avem
l (G
f
) =
_
2

3
_
1 + ctg
2
xdx =
_
2

3
1
sinx
dx.
Punem tg
x
2
= t, x = 2arctgt, dx =
2dt
1 +t
2
, sinx =
2t
1 +t
2
. Atunci
=
_
1

3
3
dt
t
= lnt

3
3
= ln
1

3
= ln

3.
a) Fie I =
b
_
a
f (a +b x) dx; cu schimbarea de variabila t = a +b x se ajunge la rezultatul dorit.
b) Fie I =

2
_
0
sin
n
x
sin
n
x + cos
n
x
dx; folosind punctul a) rezulta I =

2
_
0
cos
n
x
sin
n
x + cos
n
x
dx. Deci 2I =

2
,
de unde I =

4
.
300 6. Probleme de analiz a matematic a
6.3 Probleme propuse
6.3.1 Se considera sirurile de numere reale (x
n
)
n1
si (y
n
)
n1
denite prin
e x
2
n
= x
n1
, n 1, unde x
1
= e
2
;
2y
n
= lnx
n
+ 1, n 1.
a) Sa se arate ca sirul (y
n
)
n1
formeaza o progresie geometrica;
b) Sa se determine limita sirului (x
n
)
n1
;
c) Sa se calculeze lim
n
n

k=1
x
k
.
6.3.2 Se considera sirul de numere reale (a
n
)
n1
dat de termenul general
a
n
=
1
2

3
4
. . .
2n 1
2n
, n 1.
a) Sa se enunt e Criteriul lui Weierstrass de convergent a a sirurilor numerice;
b) Sa se demonstreze ca sirul (a
n
)
n1
este convergent;
c) Sa se demonstreze inegalitatea
1
2

3
4
. . .
2n 1
2n
<
_
1
2n + 1
, n 1,
iar apoi sa se calculeze lim
n
a
n
.
6.3.3 Se considera sirul lui Euler (c
n
)
n1
, unde
c
n
= 1 +
1
2
+
1
3
+. . . +
1
n
lnn, n 1.
a) Folosind inegalitatea
1
n + 1
< ln(n + 1) lnn <
1
n
, n 1,
sa se arate ca sirul (c
n
)
n1
este marginit;
b) Sa se calculeze lim
n
n

k=1
1
n +k
.
6.3.4 Se considera sirul numeric cu termenul general x
n
= ne
an
, n 1, a R si sirul de funct ii (f
n
)
n1
,
unde f
n
: [0, 1] R, f
n
(x) = nxe
nx
2
, n 1.
a) Sa se calculeze lim
n
x
n
, n funct ie de valorile parametrului real a R;
b) Sa se calculeze limita sirului de numere reale (f
n
(2))
n1
;
c) Sa se determine lim
n
1
_
0
f
n
(x) dx;
d) Sa se precizeze natura convergent ei sirului (f
n
)
n1
.
6.3.5 Se considera sirul de numere reale (x
n
)
n1
denit prin
x
n
=
1 +

2 +. . . +

n
1 +
3

2 +. . . +
3

n
, n 1.
6.3 Probleme propuse 301
a) Sa se determine valoarea limitei lim
n
n

n raport cu valorile parametrilor , R;


b) Sa se calculeze lim
n
x
n
;
c) Sa se calculeze lim
n
x
n
6

n
.
6.3.6 Fie p N

si sirul de numere reale (x


n
)
n2
denit prin
x
n
=
_
n

1 +
n

2 +. . . +
n

p
p
_
n
, n 2.
a) Sa se calculeze lim
n
_
n

p +
n

n
_
, pentru p N

;
b) Sa se arate ca lim
n
p
1
n
1
1
n
= lnp pentru orice p 2.
c) Sa se determine limita sirului (x
n
)
n2
.
6.3.7 Se considera sirurile de numere reale (x
n
)
n1
si (y
n
)
n1
denite respectiv prin
a
n
=
1
n
_
1
n+1
arcsin(nx) dx, b
n
=
1
n
_
1
n+1
arctan(nx) dx, n 1.
a) Sa se enunt e teorema de medie pentru integrala denita;
b) Sa se arate ca lim
n
a
n
= lim
n
b
n
= 0;
c) Sa se calculeze lim
n
a
n
b
n
;
6.3.8 Se considera sirurile de numere reale (x
n
)
n1
si (y
n
)
n1
denite prin
x
n+1
=
x
n
+ 1
2
, y
n
= x
n
1, n 1, x
0
= 2;
a) Sa se arate ca sirul (y
n
)
n1
este o progresie geometrica;
b) Sa se determine termenul general al sirului (x
n
)
n1
;
c) Sa se calculeze lim
n
(x
n
)

k=0
y
k

n
.
6.3.9 Se considera sirul de numere reale (x
n
)
n1
denit prin
x
1
= 1 si x
n+1
=
x
n
+ 2
x
n
+ 1
, x 1.
a) Sa se arate ca
x
n+1

2
x
n

2
=
1

2
x
n
+ 1
, n 1;
b) Sa se demonstreze inegalitatea

x
n

<
_
2 1
_
n
, n 1;
c) Sa se determine limita sirului (x
n
)
n1
.
6.3.10 Se considera sirul de numere reale (x
n
)
n1
denit prin x
n
=
n

k=1
2
k
2
, n 1.
302 6. Probleme de analiz a matematic a
a) Sa se enunt e Critetiul general de convergent a a lui Cauchy pentru siruri;
b) Sa se arate ca sirul (x
n
)
n1
este convergent;
c) Sa se arate ca lim
n
x
n
R.
6.3.11 Se considera sirul (x
n
)
n1
, unde x
n
=
1
n(4n
2
1)
, n 1.
a) Sa se arate ca x
n
=
1
2n + 1

1
n
+
1
2n 1
pentru orice n 1;
b) Sa se studieze natura seriei

n=1
x
n
;
c) Sa se calculeze suma seriei

n=1
x
n
.
Indicat ie: Pentru calcul sumei part iale avem
n

k=1
x
k
=
n

k=1
_
1
2k + 1
+
1
2k 1

1
k
_
= 1 +
1
3
+
1
5
+. . . +
1
2n 1
+
1
3
+
1
5
+. . . +
1
2n 1
+
1
2n + 1

_
1
1
+
1
2
+. . . +
1
n
_
= 1 +
1
2n + 1
+ 2
_
1
3
+
1
5
+. . . +
1
2n 1
_

_
1
1
+
1
2
+. . . +
1
n
_
= 2
_
1 +
1
3
+
1
5
+. . . +
1
2n + 1
_
1
1
2n + 1
+
_
1 +
1
2
+. . . +
1
n
_
2
_
1 +
1
2
+. . . +
1
n
_
= 2
_
1 +
1
3
+
1
5
+. . . +
1
2n + 1
_
1
1
2n + 1
+ 2
_
1
2
+
1
4
+. . . +
1
2n
_
2
_
1 +
1
2
+. . . +
1
n
_
= 2
_
1 +
1
2
+
1
3
+. . . +
1
2n + 1
_
2
_
1 +
1
2
+. . . +
1
n
_
1
1
2n + 1
= 2
_
1
n + 1
+
1
n + 2
+. . . +
1
2n + 1
_
1
1
2n + 1
.
6.3.12 Se considera sirul (a
n
)
n1
denit prin relat ia de recurent a a
n+1
= a
n
+r, n 1, unde a
1
, r (0, )
sunt numere reale cunoscute.
a) Sa se arate ca
r
a
n
a
n+1
=
1
a
n

1
a
n+1
, n 1;
b) Sa se studieze convergent a seriei

n=1
1
a
n
a
n+1
;
c) Sa se calculeze suma seriei

n=1
1
4n
2
+ 8n + 3
.
6.3.13 Se considera sirurile (x
n
)
n1
si (y
n
)
n1
denite prin
6.3 Probleme propuse 303
x
n
= lim
x0
n

k=1
e
kx
1
n
2
x
, y
n
=
n
_
x
n

1
2
pentru orice n 1.
a) Sa se arate ca x
n
=
n + 1
2n
pentru n 1;
b) Sa se determine natura seriei

n=1
y
n
;
c) Sa se studieze natura seriei

n=1
(y
n
)
n
6.3.14 Se considera funct ia f : R R, f(x) = x[x a[ +[x b[ . Sa se determine parametrii reali a si b
astfel ncat funct ia f sa e derivabila pe R.
6.3.15 Fie polinomul P(X) = X
2n
X
n
+X+1, n N

. Sa se calculeze
2n

k=1
1
x
k
4
, unde x
1
, x
2
, . . . , x
2n
sunt radacinile polinomului.
6.3.16 Sa se demonstreze ca x (1 +x) ln(1 +x) < 0 , () x > 0 si sa se arate ca funct ia h : (0, ) R,
h(x) = (1 +x)
1
x
este monoton descrescatoare.
6.3.17 Se considera funct ia f : R R, f(x) = x + arctgx.
a) Sa se calculeze lim
x
f(x);
b) Sa se calculeze f
t
(x);
c) Sa se calculeze lim
x
f(x) f(0)
x
;
d) Sa se determine asimptota catre +;
e) Notam cu f
(n)
(x) derivata de ordinul n. Claculat i lim
n
f
(2n+1)
(0)
(2n + 1)!
.
f) Sa se arate ca f este inversabila si notam cu g inversa sa. Sa se calculeze g
_
1 +

4
_
si g
t
_
1 +

4
_
;
g) Sa se calculeze
1+

4
_
0
g(x) si g
/
(0);
6.3.18 Sa se demonstreze inegalitatea
ln(1 +x) >
arctgx
1 +x
2
, x > 0.
6.3.19 Se considera funct iile f : R R, f (x) = lim
n
cos x +[x 1[ e
nx
1 +e
nx
si g :
_

2
,

2
_
R,
g (x) = f (x) pentru orice x
_

2
,

2
_
.
a) Sa se arate ca f (x) =
_
cos x, x < 0
[x 1[ , x 0;
b) Sa se studieze continuitatea pe R a fuct iei f;
c) Sa se studieze derivabilitatea fucnt iei g;
d) Sa se determine punctele de extrem global ale funct iei g.
304 6. Probleme de analiz a matematic a
6.3.20 Sa se arate ca funct ia f : [0, 1] R,
f(x) =
_
2xsin
1
x
2

2
x
cos
1
x
2
, x (0, 1]
0, x = 0
admite primitive pe [0, 1], dar nu este integrabila pe [0, 1].
Indicat ie: Funct ia f nu este integrabila pe [0, 1] deoarece nu este marginita. De exemplu, pentru
sirul (x
n
)
nR
denit prin x
n
=
1

+ 2
care tinde la 0, sirul valorilor f(x
n
) tinde la +.
6.3.21 Sa se arate ca funct ia f : [0, 1] R, f(x) =
1
x

_
1
x
_
, pentru x (0, 1] si f(0) = 0, este integrabila
Riemann pe [0, 1].
6.3.22 Sa se arate ca funct ia f : [0, 1] R data prin f(x) = x
2
, daca x[
_
1
n
/n N

_
si f
_
1
n
_
= n,
n N, nu este integrabila Riemann.
6.3.23 Calculat i urmatoarele integrale:
a)
1
_
0
e
x
sinxdx; b)
1
_
0
e
x
sin
2
xdx; c)
1
_
0
dx
x
3
+ 1
;
d)
1
_
0
dx
x
2
+ 2x + 1
, 0 1; e)

_
0

sinx
1

dx; f)
1
_
0
ln(1 +x)
1 +x
2
dx;
g)

_
0
xsinx
1 + cos
2
x
dx; h)
2
_
0
dx
3 + cos x
; i)
5
_
2
dx

5 + 4x x
2
;
j)

4
_
0
tg
n
xdx(n N

).
6.3.24 Sa se calculeze integrala
_
dx
3 + sinx + cos x
, x (0, 2).
6.3.25 Calculat i, folosind denit ia integralei denite, urmatoarele limite:
a) lim
n
1
n
n

k=0
e
ka+(nk)b
n
, a, b R, a ,= b;
b) lim
n
n
_
(n + 1)(n + 2)...(3n)
n
2
.
6.3.26 Sa se calculeze lim
n
1
_
0
x
n
cos(x
n
)dx
1
_
0
x
n
ln(1 +x) dx
.
Indicat ie: Aplicam ecarei integrale metoda de integrare prin part i si obt inem
1
_
0
x
n
cos x
n
dx =
1
n
1
_
0
x(sinx
n
)
t
dx =
1
n
_
sin1
1
_
0
sinx
n
dx
_
,
1
_
0
x
n
ln(1 +x)dx =
1
n + 1
1
_
0
(x
n+1
)
t
ln(1 +x)dx =
1
n + 1
_
ln2
1
_
0
x
n+1
1 +x
dx
_
.
6.3 Probleme propuse 305
Atunci
lim
n
1
_
0
x
n
cos x
n
dx
1
_
0
x
n
ln(1 +x)dx
=
sin1
ln2
.
6.3.27 Sa se calculeze lim
x0
x
3
_
0
sint
2
dt
x
9
.
6.3.28 Aat i valoarea minima a funct iei
F : R R, F(a) =
1
_
0

x
2
+a

dx, a R.
6.3.29 Fie f : R R, f(x) = e
x
2
si F : R R o primitiva a sa. Sa se arate ca
lim
x+
xF(x)
f(x)
=
1
2
.
6.3.30 Fie funct ia f :]0, +[ R, data prin f(x) =
2x
x + 2
2 ln(x + 2). Sa se determine a, b, c R asa
fel, ncat funct ia G :]0, +[R,
G(x) =
a ln(x + 2) +bx +c
x
sa e o primitiva a funct iei g :]0, +[R, g(x) =
f(x)
x
2
si nca lim
x0
G(x) = 1.
6.3.31 Calculat i perechile de integrale
a) I =
_
xdx
x
8
+ 6x
4
+ 1
, J =
_
x
5
dx
x
8
+ 6x
4
+ 1
, x ]0, +[,
b) I =
_
e
x
cos x
e
x
cos x sinx
dx, J =
_
sinx
e
x
cos x sinx
dx,
c) I =
_
x
2
1 +x
12
dx, J =
_
x
8
1 +x
12
dx, x R.
6.3.32 Sa se calculeze
I =
_
4
0
dx
a
2
cos
2
x +b
2
sin
2
x
, unde a, b > 0.
6.3.33 Calculat i derivatele funct iilor
1
0
) F :]0, +[R, F (x) =
x
4
_
0
dt
1 +t
4
2
0
) F : R R F (x) =
arctgx
_
0
e
tg
2
t
dt,
3
0
) F :] 1, 1[R F (x) =
arcsin x
_

4
ln
_
1 + sin
2
t
_
dt.,
306 6. Probleme de analiz a matematic a
6.3.34 a) Daca f : [a, b] R este o funct ie integrabila pe [a, b], aratat i ca
b
_
a
f (x) dx =
b
_
a
f (a +b x) dx;
b) Calculat i

2
_
0
sin
n
x
sin
n
x + cos
n
x
dx, n N

;
c) Calculat i lim
n

3
_
0
sin
n
x
sin
n
x + cos
n
x
dx.
6.3.35 a) Sa se demonstreze identitatea:
1
1 +

x
= 1

x +
_
x
_
2

_
x
_
3
+. . . + (1)
n
_
x
_
n
+
(1)
n+1
(

x)
n+1
1 +

x
,
oricare ar x [0, 1], n N

.
b) Sa se arate ca 0
(

x)
n+1
1 +

x
(

x)
n+1
, oricare ar x [0, 1], n N

.
c) Sa se deduca egalitatea lim
n
1
_
0
(

x)
n+1
1 +

x
dx = 0.
d) Sa se arate ca
lim
n
_
x +
(1)
1
x
1
2
+1
1
2
+ 1
+
(1)
2
x
2
2
+1
2
2
+ 1
+. . . +
(1)
n
x
n
2
+1
n
2
+ 1
_
=
x
_
0
1
1 +

t
dt, pentu orice x [0, 1].
6.3.36 a) Aratat i ca funct ia continua f : R R este periodica de perioada T daca si numai daca
x+T
_
x
f (t) dt = constant, pentru orice x R.
b) Daca f : R R este continua si periodica de perioada T, atunci
a+T
_
a
f (x) dx =
T
_
0
f (x) dx, oricare ar a R.
c) Daca f : R R este continua si periodica de periodaa T, atunci
b
_
a
f (x) dx =
b+nT
_
a+nT
f (x) dx, pentru orice a, b R, n Z.
6.3.37 a) Fie u : [0, a] [0, b] o funct ie strict monotona si derivabila. Aratat i ca
a
_
0
u(t) dt +
b
_
0
u
1
(t) dt = ab,
unde u
1
: [0, b] [0, a] este inversa funct iei u.
6.3 Probleme propuse 307
b) Demonstrat i egalitatea

2
_
0
arctg (sinx) dx +

4
_
0
arcsin (tgx) dx =

2
8
.
6.3.38 Fie a, b R, a < b
a) Sa se calculeze
I
n
=
b
_
a
(x a)
n
(b x)
n
dx, n N

.
b) Sa se calculeze
lim
n
n
_
I
n
.
Bibliograe
[1] Andrica D., Duca I.D., Purdea I., Pop Ioana, Matematica de baza, Editura Studium, Cluj Napoca,
2000
[2] Andrica D., si colectiv, Manual pentru clasa a XII, Editura Plus, Bucuresti, 2003
[3] Arama Lia, Morozan T., Culegere de probleme de calcul diferent ial si integral, Vol. I, Editura Tehnica
Bucuresti, 1964
[4] Batinet u D.M., Probleme de matematica-Siruri, Editura Albatros, 1979
[5] Chirit a Stan, Probleme de matematici superioare, Editura Didactica si Pedagogica, Bucuresti, 1989
[6] Cobzas S., Analiza matematica (Calcul diferent ial), Presa Universitara Clujeana, 1977
[7] Coroian I., Analiza matematica. Integrarea, Editura Risoprint, Cluj Napoca, 2004
[8] Demidovici B.P., Problems in Mathematical Analysis, Mir Publishers, Moscow, 1976
[9] Danciu N., Flondor D., Algebra si Analiza matematica. Culegere de probleme, Editura Didactica si
Pedagogica, Bucuresti, 1979
[10] Duca D., Duca Eugenia, Culegere de probleme de analiza matematica, Editura Gil, Zalau, 1997
[11] Ganga M., Matematica, manual pentru clasa a XII-a, Vol.I, Elemente de analiza matematica, Edi-
tura Mathpress, Ploiesti, 2003
[12] Megan M., si colectiv, Analiza matematica, Editura Amarcord, Timisoara, 1995
[13] Precupanu Anca, Bazele analizei matematice, Editura POLIROM, Iasi, 1998
[14] Rosculet M., Analiza matematica, Editura Didactica si Pedagogica, Bucuresti, 1980
[15] Siretchi Gh., Calcul diferent ial si integral, Vol. 1 si 2, Editura Stiint ica si Enciclopedica, Bucuresti,
1985
308

You might also like